Academia.eduAcademia.edu
7th EDITION NEW SYLLABUS MATHEMATICS TEACHER’S RESOURCE BOOK 1 1 CONTENTS 1 CONTENTS Syllabus Matching Grid . . . . . . . . . . . . . . . . . . . . . . . . . . . . . . . . . . . . . . . . . . . . . . . . 1 Scheme of Work . . . . . . . . . . . . . . . . . . . . . . . . . . . . . . . . . . . . . . . . . . . . . . . . . . . . . . 7 Chapter 1: Primes, Highest Common Factor and Lowest Common Multiple Teaching Notes . . . . . . . . . . . . . . . . . . . . . . . . . . . . . . . . . . . . . . . . . . . . . . . . . . . . 29 Worked Solutions . . . . . . . . . . . . . . . . . . . . . . . . . . . . . . . . . . . . . . . . . . . . . . . . . . 31 Chapter 2: Integers, Rational Numbers and Real Numbers Teaching Notes . . . . . . . . . . . . . . . . . . . . . . . . . . . . . . . . . . . . . . . . . . . . . . . . . . . . 39 Worked Solutions . . . . . . . . . . . . . . . . . . . . . . . . . . . . . . . . . . . . . . . . . . . . . . . . . . 41 Chapter 3: Approximation and Estimation Teaching Notes . . . . . . . . . . . . . . . . . . . . . . . . . . . . . . . . . . . . . . . . . . . . . . . . . . . . 55 Worked Solutions . . . . . . . . . . . . . . . . . . . . . . . . . . . . . . . . . . . . . . . . . . . . . . . . . . 56 Chapter 4: Basic Algebra and Algebraic Manipulation Teaching Notes . . . . . . . . . . . . . . . . . . . . . . . . . . . . . . . . . . . . . . . . . . . . . . . . . . . . 61 Worked Solutions . . . . . . . . . . . . . . . . . . . . . . . . . . . . . . . . . . . . . . . . . . . . . . . . . . 63 Revision Exercise A1 . . . . . . . . . . . . . . . . . . . . . . . . . . . . . . . . . . . . . . . . . . . . . . . . . 75 Revision Exercise A2 . . . . . . . . . . . . . . . . . . . . . . . . . . . . . . . . . . . . . . . . . . . . . . . . . 76 Chapter 5: Linear Equations and Simple Inequalities Teaching Notes . . . . . . . . . . . . . . . . . . . . . . . . . . . . . . . . . . . . . . . . . . . . . . . . . . . . 77 Worked Solutions . . . . . . . . . . . . . . . . . . . . . . . . . . . . . . . . . . . . . . . . . . . . . . . . . . 78 Chapter 6: Functions and Linear Graphs Teaching Notes . . . . . . . . . . . . . . . . . . . . . . . . . . . . . . . . . . . . . . . . . . . . . . . . . . . . 99 Worked Solutions . . . . . . . . . . . . . . . . . . . . . . . . . . . . . . . . . . . . . . . . . . . . . . . . . 100 Chapter 7: Number Patterns Teaching Notes . . . . . . . . . . . . . . . . . . . . . . . . . . . . . . . . . . . . . . . . . . . . . . . . . . . 111 Worked Solutions . . . . . . . . . . . . . . . . . . . . . . . . . . . . . . . . . . . . . . . . . . . . . . . . . 113 Revision Exercise B1 . . . . . . . . . . . . . . . . . . . . . . . . . . . . . . . . . . . . . . . . . . . . . . . . 121 Revision Exercise B2 . . . . . . . . . . . . . . . . . . . . . . . . . . . . . . . . . . . . . . . . . . . . . . . . 122 Chapter 8: Percentage Teaching Notes . . . . . . . . . . . . . . . . . . . . . . . . . . . . . . . . . . . . . . . . . . . . . . . . . . . 123 Worked Solutions . . . . . . . . . . . . . . . . . . . . . . . . . . . . . . . . . . . . . . . . . . . . . . . . . 124 Chapter 9: Ratio, Rate, Time and Speed Teaching Notes . . . . . . . . . . . . . . . . . . . . . . . . . . . . . . . . . . . . . . . . . . . . . . . . . . . 134 Worked Solutions . . . . . . . . . . . . . . . . . . . . . . . . . . . . . . . . . . . . . . . . . . . . . . . . . 136 2 1 Chapter 10: Basic Geometry Teaching Notes . . . . . . . . . . . . . . . . . . . . . . . . . . . . . . . . . . . . . . . . . . . . . . . . . . . 150 Worked Solutions . . . . . . . . . . . . . . . . . . . . . . . . . . . . . . . . . . . . . . . . . . . . . . . . . 151 Chapter 11: Triangles, Quadrilaterals and Polygons Teaching Notes . . . . . . . . . . . . . . . . . . . . . . . . . . . . . . . . . . . . . . . . . . . . . . . . . . . 160 Worked Solutions . . . . . . . . . . . . . . . . . . . . . . . . . . . . . . . . . . . . . . . . . . . . . . . . . 162 Chapter 12: Geometrical Constructions Teaching Notes . . . . . . . . . . . . . . . . . . . . . . . . . . . . . . . . . . . . . . . . . . . . . . . . . . . 182 Worked Solutions . . . . . . . . . . . . . . . . . . . . . . . . . . . . . . . . . . . . . . . . . . . . . . . . . 183 Revision Exercise C1 . . . . . . . . . . . . . . . . . . . . . . . . . . . . . . . . . . . . . . . . . . . . . . . . 203 Revision Exercise C2 . . . . . . . . . . . . . . . . . . . . . . . . . . . . . . . . . . . . . . . . . . . . . . . . 204 Chapter 13: Perimeter and Area of Plane Figures Teaching Notes . . . . . . . . . . . . . . . . . . . . . . . . . . . . . . . . . . . . . . . . . . . . . . . . . . . 205 Worked Solutions . . . . . . . . . . . . . . . . . . . . . . . . . . . . . . . . . . . . . . . . . . . . . . . . . 206 Chapter 14: Volume and Surface Area of Prisms and Cylinders Teaching Notes . . . . . . . . . . . . . . . . . . . . . . . . . . . . . . . . . . . . . . . . . . . . . . . . . . . 216 Worked Solutions . . . . . . . . . . . . . . . . . . . . . . . . . . . . . . . . . . . . . . . . . . . . . . . . . 218 Chapter 15: Statistical Data Handling Teaching Notes . . . . . . . . . . . . . . . . . . . . . . . . . . . . . . . . . . . . . . . . . . . . . . . . . . . 230 Worked Solutions . . . . . . . . . . . . . . . . . . . . . . . . . . . . . . . . . . . . . . . . . . . . . . . . . 231 Revision Exercise D1 . . . . . . . . . . . . . . . . . . . . . . . . . . . . . . . . . . . . . . . . . . . . . . . . 239 Revision Exercise D2 . . . . . . . . . . . . . . . . . . . . . . . . . . . . . . . . . . . . . . . . . . . . . . . . 240 Problems in Real-World Contexts . . . . . . . . . . . . . . . . . . . . . . . . . . . . . . . . . . . . . . 241 1 3 Syllabus Matching Grid Cambridge O Level Mathematics (Syllabus D) 4024/4029. Syllabus for examination in 2018, 2019 and 2020. Theme or Topic 1. Number 2. Set language and notation Subject Content Identify and use: • Natural numbers • Integers (positive, negative and zero) • Prime numbers • Square numbers • Cube numbers • Common factors and common multiples • Rational and irrational numbers (e.g. p, 2 ) • Real numbers • Use set language, set notation and Venn diagrams to describe sets and represent relationships between sets • Definition of sets: e.g. A = {x : x is a natural number}, B = {(x, y): y = mx + c}, C = {x : a < x < b}, D = {a, b, c, …} 2. Squares, square roots, cubes and cube roots 4. Directed numbers 5. Vulgar and decimal fractions and percentages 6. Ordering • Order quantities by magnitude and demonstrate familiarity with the symbols =, ≠, <, >, <, >. 7. Standard form • Use the standard form A × 10n, where n is a positive or negative integer, and 1 < A < 10. 8. The four operations 9. Estimation 10 Limits of accuracy Calculate • Squares • Square roots • Cubes and cube roots of numbers • Use directed numbers in practical situations • Use the language and notation of simple vulgar and decimal fractions and percentages in appropriate contexts • Recognise equivalence and convert between these forms Use the four operations for calculations with: • Whole numbers • Decimals • Vulgar (and mixed) fractions including correct ordering of operations and use of brackets. • Make estimates of numbers, quantities and lengths • Give approximations to specified numbers of significant figures and decimal places • Round off answers to reasonable accuracy in the context of a given problem • Give appropriate upper and lower bounds for data given to a specified accuracy • Obtain appropriate upper and lower bounds to solutions of simple problems given to a specified accuracy 1 Reference Book 1: Chapter 1 Chapter 2 Book 2: Chapter 14 Book 4: Chapter 2 Book 1: Chapter 1 Chapter 2 Book 1: Chapter 2 Book 1: Chapter 2 Book 1: Chapter 2 Chapter 5 Book 3: Chapter 4 Book 1: Chapter 2 Book 1: Chapter 3 Book 3: Chapter 3 1 11. Ratio, proportion, rate 12. Percentages 13. Use of an electronic calculator • • • • Demonstrate an understanding of ratio and proportion Increase and decrease a quantity by a given ratio Use common measures of rate Solve problems involving average speed • • • • Calculate a given percentage of a quantity Express one quantity as a percentage of another Calculate percentage increase or decrease Carry out calculations involving reverse percentages • • • • Use an electronic calculator efficiently Apply appropriate checks of accuracy Enter a range of measures including ‘time’ Interpret the calculator display appropriately Book 1: Chapter 9 Book 2: Chapter 1 Book 1: Chapter 8 Book 3: Chapter 5 Book 1: Chapter 2 Chapter 4 Book 2: Chapter 11 Book 3: Chapter 10 Book 4: Chapter 4 14. Time • Calculate times in terms of the 24-hour and 12-hour clock • Read clocks, dials and timetables 15. Money • Solve problems involving money and convert from one currency to another Book 1: Chapter 9 Book 3: Chapter 5 16. Personal and small business finance • Use given data to solve problems on personal and small business finance involving earnings, simple interest and compound interest • Extract data from tables and charts Book 3: Chapter 5 17. Algebraic representation and formulae • Use letters to express generalised numbers and express arithmetic processes algebraically • Substitute numbers for words and letters in formulae • Construct and transform formulae and equations Book 1: Chapter 4 Chapter 5 Book 2: Chapter 2 Book 3: Chapter 1 18. Algebraic manipulation 19. Indices 1 • • • • Manipulate directed numbers Use brackets and extract common factors Expand product of algebraic expressions Factorise where possible expressions of the form: ax + bx + kay + kby a2x2 – b2y2 a2 + 2ab + b2 ax2 + bx + c • Manipulate algebraic fractions • Factorise and simplify rational expressions • Understand and use the rules of indices • Use and interpret positive, negative, fractional and zero indices 2 Book 1: Chapter 4 Book 2: Chapter 3 Chapter 4 Chapter 6 Book 3: Chapter 4 20. Solutions of equations and inequalities • • • • Solve simple linear equations in one unknown Solve fractional equations with numerical and linear algebraic denominators Solve simultaneous linear equations in two unknowns Solve quadratic equations by factorisation, completing the square or by use of the formula • Solve simple linear inequalities Book 1: Chapter 5 Book 2: Chapter 2 Chapter 5 Book 3: Chapter 1 Chapter 3 21. Graphical representation of inequalities • Represent linear inequalities graphically 22. Sequences • Continue a given number sequence • Recognise patterns in sequences and relationships between different sequences • Generalise sequences as simple algebraic statements Book 1: Chapter 7 23. Variation • Express direct and inverse variation in algebraic terms and use this form of expression to find unknown quantities Book 2: Chapter 1 24. Graphs in practical situations • Interpret and use graphs in practical situations including travel graphs and conversion graphs • Draw graphs from given data • Apply the idea of rate of change to easy kinematics involving distance-time and speed-time graphs, acceleration and deceleration • Calculate distance travelled as area under a linear speed-time graph Book 4: Chapter 1 Book 1: Chapter 6 Book 2: Chapter 2 Book 3: Chapter 7 25. Graphs in practical situations • Construct tables of values and draw graphs for functions of the form axn where a is a rational constant, n = –2, –1, 0, 1, 2, 3, and simple sums of not more than three of these and for functions of the form kax where a is a positive integer • Interpret graphs of linear, quadratic, cubic, reciprocal and exponential functions • Solve associated equations approximately by graphical methods • Estimate gradients of curve by drawing tangents Book 1: Chapter 6 Book 2: Chapter 1 Chapter 2 Chapter 5 Book 3: Chapter 1 Chapter 7 26. Function notation • Use function notation, e.g. f(x) = 3x – 5; f : x  3x – 5, to describe simple functions • Find inverse functions f–1(x) Book 2: Chapter 7 Book 3: Chapter 2 27. Coordinate geometry • Demonstrate familiarity with Cartesian coordinates in two dimensions • Find the gradient of a straight line • Calculate the gradient of a straight line from the coordinates of two points on it • Calculate the length and the coordinates of the midpoint of a line segment from the coordinates of its end points • Interpret and obtain the equation of a straight line graph in the form y = mx + c • Determine the equation of a straight line parallel to a given line • Find the gradient of parallel and perpendicular lines 3 Book 1: Chapter 6 Book 2: Chapter 2 Book 3: Chapter 6 1 28. Geometrical terms • Use and interpret the geometrical terms: point; line; plane; parallel; perpendicular; bearing; right angle, acute, obtuse and reflex angles; interior and exterior angles; similarity and congruence • Use and interpret vocabulary of triangles, special quadrilaterals, circles, polygons and simple solid figures • Understand and use the terms: centre, radius, chord, diameter, circumference, tangent, arc, sector and segment Book 1: Chapter 10 Chapter 11 Book 2: Chapter 8 Book 3: Chapter 9 to Chapter 13 29. Geometrical constructions 30. Similarity and congruence 31. Symmetry 32. Angles 33. Loci 34. Measures 1 • Measure lines and angles • Construct a triangle, given the three sides, using a ruler and a pair of compasses only • Construct other simple geometrical figures from given data, using a ruler and protractor as necessary • Construct angle bisectors and perpendicular bisectors using a pair of compasses as necessary • Read and make scale drawings • Use and interpret nets • Solve problems and give simple explanations involving similarity and congruence • Calculate lengths of similar figures • Use the relationships between areas of similar triangles, with corresponding results for similar figures, and extension to volumes and surface areas of similar solids • Recognise rotational and line symmetry (including order of rotational symmetry) in two dimensions • Recognise symmetry properties of the prism (including cylinder) and the pyramid (including cone) • Use the following symmetry properties of circles: (a) equal chords are equidistant from the centre (b) the perpendicular bisector of a chord passes through the centre (c) tangents from an external point are equal in length • Calculate unknown angles and give simple explanations using the following geometrical properties: (a) angles at a point (b) angles at a point on a straight line and intersecting straight lines (c) angles formed within parallel lines (d) angle properties of triangles and quadrilaterals (e) angle properties of regular and irregular polygons (f) angle in a semi-circle (g) angle between tangent and radius of a circle (h) angle at the centre of a circle is twice the angle at the circumference (i) angles in the same segment are equal (j) angles in opposite segments are supplementary • Use the following loci and the method of intersecting loci for sets of points in two dimensions which are: (a) at a given distance from a given point (b) at a given distance from a given straight line (c) equidistant from two given points (d) equidistant from two given intersecting straight line • Use current units of mass, length, area, volume and capacity in practical situations and express quantities in terms of larger or smaller units 4 Book 1: Chapter 12 Chapter 14 Book 2: Chapter 8 Book 4: Chapter 8 Book 2: Chapter 8 Book 3: Chapter 11 Chapter 12 Book 2: Chapter 13 Book 3: Chapter 13 Book 1: Chapter 10 Chapter 11 Book 3: Chapter 13 Book 4: Chapter 8 Book 1: Chapter 13 Chapter 14 35. Mensuration 36. Trigonometry 37. Vectors in two dimensions 38. Matrices 39. Transformations 40. Probability • Solve problems involving: (a) the perimeter and area of a rectangle and triangle (b) the perimeter and area of a parallelogram and a trapezium (c) the circumference and area of a circle (d) arc length and sector area as fractions of the circumference and area of a circle (e) the surface area and volume of a cuboid, cylinder, prism, sphere, pyramid and cone (f) the areas and volumes of compound shapes • Interpret and use three-figure bearings • Apply Pythagoras’ theorem and the sine, cosine and tangent ratios for acute angles to the calculation of a side or an angle of a right-angled triangles • Solve trigonometrical problems in two dimensions involving angles of elevation and depression • Extend sine and cosine functions to angles between 90° and 180° • Solve problems using the sine and cosine rules for any triangle and the 1 formula area of triangle = ab sin C 2 • Solve simple trigonometrical problems in three dimensions ⎛ x⎞ → • Describe a translation by using a vector represented by ⎜⎝ y ⎟⎠ , AB or a • Add and subtract vectors • Multiple a vector by a scalar ⎛ x⎞ • Calculate the magnitude of a vector ⎜⎝ y ⎟⎠ as x 2 + y 2 • Represent vectors by directed line segments • Use the sum and difference of two vectors to express given vectors in terms of two coplanar vectors • Use position vectors • Display information in the form of a matrix of any order • Solve problems involving the calculation of the sum and product (where appropriate) of two matrices, and interpret the results • Calculate the product of a matrix and a scalar quantity • Use the algebra of 2 × 2 matrices including the zero and identity 2 × 2 matrices • Calculate the determinant |A| and inverse A–1 of a non-singular matrix A • Use the following transformations of the plane: reflection (M), rotation (R), translation (T), enlargement (E) and their combinations • Identify and give precise descriptions of transformations connecting given figures • Describe transformations using coordinates and matrices • Calculate the probability of a single event as either a fraction or a decimal • Understand that the probability of an event occurring = 1 – the probability of the event not occurring • Understand relative frequency as an estimate of probability • Calculate the probability of simple combined events using possibility diagrams and tree diagrams where appropriate 5 Book 1: Chapter 13 Chapter 14 Book 2: Chapter 12 Book 3: Chapter 10 Book 2: Chapter 10 Chapter 11 Book 3: Chapter 8 Chapter 9 Book 4: Chapter 7 Book 4: Chapter 5 Book 2: Chapter 9 Book 4: Chapter 6 Book 2: Chapter 15 Book 4: Chapter 3 1 41. Categorical, numerical and grouped data 42. Statistical diagrams 1 • Collect, classify and tabulate statistical data • Read, interpret and draw simple inferences from tables and statistical diagrams • Calculate the mean, median, mode and range for individual and discrete data and distinguish between the purposes for which they are used • Calculate an estimate of the mean for grouped and continuous data • Identify the modal class from a grouped frequency distribution • Construct and interpret bar charts, pie charts, pictograms, simple frequency distributions, frequency polygons, histograms with equal and unequal intervals and scatter diagrams • Construct and use cumulative frequency diagrams • Estimate and interpret the median, percentiles, quartiles and interquartile range for cumulative frequency diagrams • Calculate with frequency density • Understand what is meant by positive, negative and zero correlation with reference to a scatter diagram • Draw a straight line of best fit by eye 6 Book 1: Chapter 15 Book 2: Chapter 17 Book 4: Chapter 4 Book 1: Chapter 15 Book 2: Chapter 16 Book 4: Chapter 4 Secondary 1 Mathematics Scheme of Work Week (5 classes × 45 min) 1 Chapter Section 1.1 Prime 1 Numbers Primes, Highest (pp. 3 – 9) Common Factor and Lowest Common Multiple Specific Instructional Objectives (SIOs) • Explain what a prime number is • Determine whether a whole number is prime • Express a composite number as a product of its prime factors Syllabus Subject Content Identity and use prime numbers Activity Investigation – Classification of Whole Numbers (pp. 3 – 4) Thinking Time (p. 4) Investigation – Sieve of Eratosthenes (p. 5) ICT Investigation – Interesting Facts about Prime Numbers (p. 8) Additional Resources Reasoning, Communication and Connection Thinking Time (p. 4) Investigation – Sieve of Eratosthenes (p. 5) Journal Writing (p. 5) Worked Example 2 (p. 7) 7 Journal Writing (p. 5) Practise Now 2 Q 1 – 2 (p. 7) Investigation – Interesting Facts about Prime Numbers (p. 7) Thinking Time (p. 8) Thinking Time (p. 8) 1 1 Week (5 classes × 45 min) 1 Chapter Section 1.2 Square Roots and Cube Roots (pp. 9 – 14) Specific Instructional Objectives (SIOs) • Find square roots and cube roots using prime factorisation, mental estimation and calculators Syllabus Subject Content Identify and use square numbers and cube numbers Activity ICT Additional Resources Thinking Time (p. 12) Reasoning, Communication and Connection Attention (p. 10) Attention (p. 11) Thinking Time (p. 12) Calculate squares, square roots, cubes and cube roots of numbers 8 Main Text – ‘Since √ 997 = 31.6 (to 1 d.p.), the largest prime less than or equal to √ 997 is 31. To determine whether 997 is a prime, it is enough to test whether 997 is divisible by 2, 3, 5, 7, ... or 31 (only 11 prime numbers to test). We do not have to test all the 167 prime numbers. Why?’ (p. 13) Ex 1A Q 11 – 12 (p. 14) 2 2 1.3 Highest Common Factor and Lowest Common Multiple (pp. 14 – 24) Miscellaneous • Find the highest common factor (HCF) and lowest common multiple (LCM) of two or more numbers • Solve problems involving HCF and LCM in real-world contexts Practise Now 11 Q 3 (p. 18) Identify and use common factors and common multiples Ex 1B Q6, 9(a) – (e), 10, 11(a) – (d), 13(i), 14(ii) (pp. 21 – 22) Solutions for Challenge Yourself Week (5 classes × 45 min) 2 Chapter 2 Integers, Rational Numbers and Real Numbers Section 2.1 Negative Numbers (pp. 27 – 30) Specific Instructional Objectives (SIOs) • Use negative numbers, rational numbers and real numbers in a realworld context • Represent real numbers on a number line and order the numbers 9 3 2.2 Addition and Subtraction involving Negative Numbers (pp. 30 – 37) • Perform operations in real numbers, including using the calculator Syllabus Subject Content Identify and use natural numbers and integers (positive, negative and zero) Activity ICT Thinking Time (p. 28) Use directed numbers in Main Text practical situations (pp. 28 – 29) Thinking Time (p. 28) Use the four operations for calculations with whole numbers including correct ordering of operations and use of brackets. Main Text (pp. 31 – 32) Class Discussion – Addition involving Negative Numbers (p. 33) Main Text (p. 34) Class Discussion – Subtraction involving Negative Numbers (p. 35) Reasoning, Communication and Connection Class Discussion – Use of Negative Numbers in the Real World (p. 27) Class Discussion – Use of Negative Numbers in the Real World (p. 27) Order quantities by magnitude and demonstrate familiarity with the symbols =, ≠, <, >, ≤, ≥. Additional Resources Main Text – ‘The number is marked out on the number line. Explain how the point on the number line is obtained.’ (p. 29) Main Text – ‘Alternatively, you may visit http://www. shinglee.com.sg/ Student Resources/ to access the AlgeToolTM software.’ (p. 30) Class Discussion – Addition involving Negative Numbers (p. 33) Class Discussion – Subtraction involving Negative Numbers (p. 35) 1 1 Week (5 classes × 45 min) 3 Chapter Section Specific Instructional Objectives (SIOs) Syllabus Subject Content 2.3 Multiplication and Division involving Negative Numbers (pp. 37 – 43) Activity ICT Additional Resources Reasoning, Communication and Connection Class Discussion – Multiplication involving Negative Numbers (p. 39) Main Text (pp. 38 – 39) Class Discussion – Multiplication involving Negative Numbers (p. 39) Thinking Time (p. 41) Thinking Time (p. 41) 4 10 2.4 Rational Numbers and Real Numbers (pp. 44 – 54) Use of an electronic calculator efficiently Main Text (p. 42) Apply appropriate checks of accuracy Practise Now 4b Use the four operations for calculations with decimals, vulgar (and mixed) fractions including correct ordering of operations and use of brackets. Identify and use rational Investigation – Terminating, and irrational numbers Recurring and (e.g. π, √2) Non-Recurring Decimals (p. 50) Thinking Time (p. 49) Investigation – Some Interesting Facts about the Irrational Number p (p. 51) Thinking Time (p. 49) Investigation – Some Interesting Facts about the Irrational Number p (p. 51) 4 Miscellaneous Solutions for Challenge Yourself Week (5 classes × 45 min) 4 Chapter Section 3 Approximation and Estimation 3.1 Approximation (pp. 59 – 62) Specific Instructional Objectives (SIOs) Syllabus Subject Content Make estimates of numbers, quantities and lengths Activity Class Discussion – Actual and Approximated Values (p. 59) ICT Additional Resources Reasoning, Communication and Connection Class Discussion – Actual and Approximated Values (p. 59) Practise Now 1 Q 2 (p. 60) Practise Now 2 Q 2 (p. 61) Ex 3A Q 5 – 7 (p. 62) 5 3.2 Significant Figures (pp. 63 – 67) • Round off numbers to a required number of decimal places and significant figures Give approximations to specified numbers of significant figures and decimal places Investigation – Rounding in Real Life (p. 67) Journal Writing (p. 67) Practise Now Q 2 (p. 64) Practise Now 4 Q 2 (p. 66) 11 Investigation – Rounding in Real Life (p. 67) Journal Writing (p. 67) 5 3.3 Rounding and Truncation Errors (pp. 68 – 71) • Explain the problem of rounding and truncation errors Investigation – The Missing 0.1% Votes (p. 68) Investigation – The Missing 0.1% Votes (p. 68) Thinking Time (p. 70) Ex 3B Q 3, 8 – 9, 10(iii) (pp. 70 – 71) Thinking Time (p. 69) 1 Investigation – Rounding and Truncation Errors in Calculators (p. 70) 1 Week (5 classes × 45 min) Chapter 5 Section 3.4 Estimation (pp. 71 – 77) Specific Instructional Objectives (SIOs) • Estimate the results of computations • Apply estimation in real-world contexts Syllabus Subject Content Round off answers to reasonable accuracy in the context of a given problem Activity ICT Additional Resources Reasoning, Communication and Connection Worked Example 6 (p. 73) Investigation – Use of a Smaller Quantity to Estimate a Larger Quantity (p. 75) Performance Task (p. 76) Miscellaneous 5 6 12 4 Basic Algebra and Algebraic Manipulation 4.1 Fundamental Algebra (pp. 81 – 91) Solutions for Challenge Yourself • Use letters to represent numbers • Express basic arithmetical processes algebraically • Evaluate algebraic expressions • Add and subtract linear expressions Use letters to express generalised numbers and express arithmetic processes algebraically Substitute numbers for words and letters in formulae Class Discussion – Expressing Mathematical Relationships using Algebra (p. 83) Investigation – Comparison between Pairs of Expressions (p. 84) Construct and transform Main Text formulae and equations (pp. 85 – 88) Manipulate directed numbers Practise Now (p. 89) Investigation –Comparison between Pairs of Expressions (p. 84) Journal Writing (p. 85) Practise Now (p. 89) Investigation – Comparison between Pairs of Expressions (p. 84) Journal Writing (p. 85) Week (5 classes × 45 min) 6 Chapter Section 4.2 Expansion and Simplification of Linear Expressions (pp. 91 – 97) Specific Instructional Objectives (SIOs) • Simplify linear expressions Syllabus Subject Content Expand product of algebraic expressions Activity ICT Main Text (pp. 91 – 95) Practise Now (p. 92) Practise Now (p. 92) Practise Now (p. 92) Practise Now (p. 92) Practise Now (p. 95) Additional Resources Reasoning, Communication and Connection Practise Now (p. 95) Class Discussion – The Distributive Law (p. 94) Thinking Time (p. 96) Thinking Time (p. 96) 13 7 7 7 4.3 Simplification of Linear Expressions with Fractional Coefficients (pp. 98 – 99) 4.4 Factorisation (pp. 100 – 102) Miscellaneous • Factorise algebraic expressions by extracting common factors Use brackets and extract Class Discussion common factors – Equivalent Expressions (p. 101) Solutions for Challenge Yourself 1 1 Week (5 classes × 45 min) 8 Chapter Section 5 Linear Equations and Simple Inequalities 5.1 Linear Equations (pp. 109 – 119) Specific Instructional Objectives (SIOs) Syllabus Subject Content • Explore the concepts Solve simple linear equations in one of equation and unknown inequality • Solve linear equations in one variable • Solve fractional equations that can be reduced to linear equations Activity ICT Main Text (pp. 110 – 113) Practise Now (p. 110) Practise Now (p. 110) Practise Now (p. 111) Practise Now (p. 111) Practise Now (p. 112) Practise Now (p. 112) Practise Now (p. 113) Practise Now (p. 113) Additional Resources Reasoning, Communication and Connection Main Text – ‘From Table 5.1, discuss with your classmate what a linear equation is.’ (p. 109) Journal Writing (p. 113) Thinking Time (p. 115) Journal Writing (p. 113) Thinking Time (p. 115) 14 Solve fractional equations with numerical and linear algebraic denominators 8 9 5.2 Formulae (pp. 118 – 121) 5.3 Applications of Linear Equations in Real-World Contexts (pp. 122 – 125) Worked Example 2 (pp. 115 - 116) Worked Example 3 (pp. 116 - 117) • Evaluate an unknown in a formula • Formulate linear equations to solve word problems Ex 5B Q 17(ii) (p. 121) Internet Resources (p. 122) Internet Resources (p. 122) Week (5 classes × 45 min) Chapter 9 Section 5.4 Simple Inequalities (pp. 125 – 129) Specific Instructional Objectives (SIOs) • Solve simple linear inequalities Syllabus Subject Content Solve simple linear inequalities Activity ICT Additional Resources Journal Writing (p. 128) Reasoning, Communication and Connection Investigation – Properties of Inequalities (p. 126) Ex 5D Q 7 (p. 129) Investigation – Properties of Inequalities (p. 126) 9 Miscellaneous 10 6 Functions and Linear Graphs 6.1 Cartesian Coordinates (pp. 135 – 138) Solutions for Challenge Yourself • State the coordinates of a point • Plot a point in a Cartesian plane Demonstrate familiarity with Cartesian coordinates in two dimensions 15 Class Discussion – Internet Resources Battleship Game (p. 135) (Two Players) (p. 135) Story Time Internet Resources (p. 138) (p. 135) Class Discussion – Ordered Pairs (p. 136) Class Discussion – Ordered Pairs (p. 136) Journal Writing (p. 137) 10 6.2 Functions (pp. 139 – 145) Investigation – Function Machine (pp. 139 – 142) Internet Resources (p. 139) Thinking Time (p. 143) Thinking Time (p. 143) 10 6.3 Graphs of Linear Functions (pp. 145 – 148) • Draw the graph of a linear function Draw graphs from given Class Discussion – data Equation of a Function (p. 147) 1 Thinking Time (p. 147) Class Discussion – Equation of a Function (p. 147) Thinking Time (p. 147) 1 Week (5 classes × 45 min) Chapter 11 Section 6.4 Applications of Linear Graphs in Real-World Contexts (pp. 149 – 153) Specific Instructional Objectives (SIOs) • Solve problems involving linear graphs in real-world contexts Syllabus Subject Content Interpret and use graphs in practical situations including travel graphs and conversion graphs Activity ICT Additional Resources Worked Example 2 (p. 149) Thinking Time (p. 151) Ex 6C Q 3(ii) (p. 153) Thinking Time (p. 151) Interpret graphs of linear functions Miscellaneous 11 11 12 16 12 12 7 Number Patterns 7.1 Number Sequences (pp. 159 – 161) 7.2 General Term of a Number Sequence (pp. 161 – 165) 7.3 Number Patterns (pp. 165 – 167) 7.4 Number Patterns in Real-World Contexts (pp. 168 – 177) Solutions for Challenge Yourself Recognise simple patterns from various number sequences and determine the next few terms Continue a given number sequence Class Discussion – Number Sequences (p. 159) Determine the next few terms and find a formula for the general term of a number sequence Recognise patterns in sequences and relationships between different sequences Class Discussion – Generalising Simple Sequences (p. 162) Solve problems involving number squences and number patterns Generalise sequences as simple algebraic statements Class Discussion – The Triangular Number Sequence (p. 167) Worked Example 5 Journal Writing (p. 169) (p. 170) Investigation – Fibonacci Sequence (pp. 168 – 169) Journal Writing (p. 169) 12 Miscellaneous Reasoning, Communication and Connection Solutions for Challenge Yourself Week (5 classes × 45 min) 13 13 17 13 Chapter Section 8 Percentage 8.1 Introduction to Percentage (pp. 185 – 192) 8.2 Percentage Change and Reverse Percentage (pp. 193 – 200) Miscellaneous Specific Instructional Objectives (SIOs) • Express a percentage as a fraction and vice versa • Express a percentage as a decimal and vice versa • Express one quantity as a percentage of another • Compare two quantities by percentage • Solve problems involving percentage change and reverse percentage Syllabus Subject Content Calculate a given percentage of a quantity Express one quantity as a percentage of another Activity ICT Additional Resources Reasoning, Communication and Connection Class Discussion – Percentage in Real Life (p. 185) Class Discussion – Percentage in Real Life (p. 185) Class Discussion – Expressing Two Quantities in Equivalent Forms (p. 189) Calculate percentage increase or decrease Thinking Time (p. 198) Carry out calculations involving reverse percentages Internet Resources (p. 198) Just for Fun (p. 195) Internet Resources (p. 198) Attention (p. 195) Thinking Time (p. 198) Solutions for Challenge Yourself 1 1 Week (5 classes × 45 min) 14 Chapter Section 9.1 Ratio 9 (pp. 205 – 213) Ratio, Rate, Time and Speed Specific Instructional Objectives (SIOs) • Find ratios involving rational numbers • Find ratios involving three quantities • Solve problems involving ratio Syllabus Subject Content Activity Increase and decrease a Journal Writing quantity by a given ratio (p. 208) ICT Additional Resources Reasoning, Communication and Connection Journal Writing (p. 208) Worked Example 5 Internet Resources (p. 210) (p. 208) Class Discussion – Performance Task Making Sense of (p. 210) the Relationship between Ratios and Fractions (p. 207) Worked Example 4 (p. 208) Worked Example 6 (p. 211) 18 Internet Resources (p. 208) Investigation – Golden Ratio (p. 207) Performance Task (p. 210) 15 9.2 Rate (pp. 214 – 217) • Distinguish between constant and average rates • Solve problems involving rate Use common measures of rate Investigation – Average Pulse Rate (p. 215) Investigation – Average Pulse Rate (p. 215) Thinking Time (p. 216) Thinking Time (p. 216) Week (5 classes × 45 min) 15 Chapter Section 9.3 Time (pp. 218 – 220) Specific Instructional Objectives (SIOs) • Solve problems involving time Syllabus Subject Content Calculate times in terms of the 24-hour and 12hour clock Activity ICT Additional Resources Reasoning, Communication and Connection Main Text (pp. 218 – 219) Read clocks, dials and timetables 16 9.4 Speed (pp. 221 – 227) • Discuss special types of rates such as speed and rate of rotation • Solve problems involving speed Solve problems Main Text involving average speed (pp. 221 – 226) Performance Task (p. 225) Just for Fun (p. 221) Internet Resources (p. 222) Thinking Time (p. 224) Performance Task (p. 225) Thinking Time (p. 224) 16 Miscellaneous 10 10.1 Points, Lines Basic Geometry and Planes (pp. 233 – 234) 17 10.2 Angles (pp. 235 – 266) 19 16 Solutions for Challenge Yourself Use and interpret the geometrical terms: point; line; plane • Identify various types of angles • Solve problems involving angles on a straight line, angles at a point and vertically opposite angles Use and interpret the geometrical terms: parallel; perpendicular; right angle, acute, obtuse and reflex angles Calculate unknown angles and give simple explanations using the following geometrical properties: (a) angles at a point (b) angles at a point on a straight line and intersecting straight lines Thinking Time (p. 234) Internet Resources (p. 234) Thinking Time (p. 234) Just for Fun (p. 235) 1 1 Week (5 classes × 45 min) Chapter Section Specific Instructional Objectives (SIOs) 10.3 Angles formed • Solve problems involving angles by Two Parallel formed by two Lines and a parallel lines and Transversal a transversal, i.e. (pp. 244 – 252) corresponding angles, alternate angles and interior angles 17 Syllabus Subject Content Calculate unknown angles and give simple explanations using angles formed within parallel lines Activity Investigation – Corresponding Angles, Alternate Angles and Interior Angles (pp. 245 – 246) ICT Additional Resources Reasoning, Communication and Connection Investigation – Corresponding Angles, Alternate Angles and Interior Angles (pp. 245 – 246) Investigation – Corresponding Angles, Alternate Angles and Interior Angles (pp. 245 – 246) Practise Now (p. 246) Ex 10B Q 1(b) – (c) (p. 250) Miscellaneous 17 18 11 Triangles, Quadrilaterals and Polygons 11.1 Triangles (pp. 259 – 268) Solutions for Challenge Yourself 20 • Identify different types of triangles and state their properties • Solve problems involving the properties of triangles Use and interpret the geometrical terms: interior and exterior angles Investigation – Basic Properties of a Triangle (pp. 262 – 263) Use and interpret vocabulary of triangles Calculate unknown angles and give simple explanations using angle properties of triangles Thinking Time (p. 260) Investigation – Basic Properties of a Triangle (pp. 262 – 263) Thinking Time (p. 260) Investigation – Basic Properties of a Triangle (pp. 262 – 263) Week (5 classes × 45 min) 18 Chapter Section 11.2 Quadrilaterals (pp. 268 – 276) Specific Instructional Objectives (SIOs) • Identify different types of special quadrilaterals and state their properties • Solve problems involving the properties of special quadrilaterals Syllabus Subject Content Calculate unknown angles and give simple explanations using angle properties of quadrilaterals Activity Investigation – Properties of Special Quadrilaterals (pp. 269) ICT Additional Resources Reasoning, Communication and Connection Investigation – Properties of Special Quadrilaterals (p. 269) Thinking Time (p. 271) Class Discussion – Naming of Polygons (p. 277) Main Text – ‘The shapes shown in Fig. 11.13 are not polygons. Why?’ (p. 276) Just for Fun (p. 271) Use and interpret vocabulary of quadrilaterals Thinking Time (p. 271) 19 11.3 Polygons (pp. 276 – 290) 21 • Identify different types of polygons and state their properties • Solve problems involving the properties of polygons Use and interpret vocabulary of polygons Calculate unknown angles and give simple explanations using angle properties of regular and irregular polygons Investigation – Sum of Interior Angles of a Polygon (pp. 279 – 280) Investigation – Tessellation (pp. 282 - 283) Investigation – Sum of Exterior Angles of a Pentagon (pp. 284 – 285) Main Text (p. 285) Internet Resources (p. 277) Investigation – Sum of Exterior Angles of a Pentagon (pp. 284 – 285) Thinking Time (p. 277) Journal Writing (p. 278) Investigation – Sum of Exterior Angles of a Pentagon (p. 284 – 285) 1 1 Week (5 classes × 45 min) Chapter Section Specific Instructional Objectives (SIOs) Syllabus Subject Content Activity ICT Additional Resources Reasoning, Communication and Connection Thinking Time (p. 285) Thinking Time (p. 285) Class Discussion – Naming of Polygons (p. 277) Ex 11C Q 19(ii) – (iv) (p. 290) Thinking Time (p. 277) Investigation – Properties of a Regular Polygon (p. 278) Journal Writing (p. 278) Miscellaneous 19 22 20 20 20 12 Geometrical Constructions Solutions for Challenge Yourself 12.1 Introduction to Geometrical Constructions (pp. 297 – 298) 12.2 Perpendicular • Construct perpendicular Bisectors and bisectors and angle Angle Bisectors bisectors (pp. 299 – 301) • Apply properties of perpendicular bisectors and angle bisectors 12.3 Construction of Triangles (pp. 301 – 306) • Construct triangles and solve related problems Measure lines and angles Construct angle bisectors and perpendicular bisectors using a pair of compasses as necessary Construct a triangle, given the three sides, using a ruler and a pair of compasses only Investigation – Property of a Perpendicular Bisector (p. 300) Investigation – Property of a Perpendicular Bisector (p. 300) Investigation – Property of an Angle Bisector (p. 301) Internet Resources (p. 300) Investigation – Property of an Angle Bisector (p. 301) Just for Fun (p. 303) Ex 12A Q 15 – 16 (p. 306) Week (5 classes × 45 min) Chapter Section Specific Instructional Objectives (SIOs) 12.4 Construction of • Construct quadrilaterals Quadrilaterals and solve related (pp. 306 – 311) problems 20 Syllabus Subject Content Activity ICT Additional Resources Worked Example 6 (pp. 306 – 307) Internet Resources (p. 309) Construct other simple geometrical figures from given data, using a ruler and protractor as necessary Reasoning, Communication and Connection Practise Now 6 Q 1–2 (p. 307) Practise Now 7 Q 2 (p. 308) Ex 12B Q 1 – 3, 5, 8 – 9, 11(ii), 15 (pp. 310 – 311) Miscellaneous 20 21 23 21 13 Perimeter and Area of Plane Figures 13.1 Conversion of Units (p. 317) 13.2 Perimeter and Area of Basic Plane Figures (pp. 318 – 323) Solutions for Challenge Yourself • Convert between cm2 and m2 • Find the perimeter and area of squares, rectangles, triangles and circles • Solve problems involving the perimeter and area of composite figures Use current units of mass, length and area in practical situations and express quantities in terms of larger or smaller units Class Discussion – International System of Units (p. 317) Solve problems involving the perimeter and area of a rectangle and triangle, and the circumference and area of a circle Practise Now (p. 318) Class Discussion – International System of Units (p. 317) 1 1 Week (5 classes × 45 min) 22 Chapter Section 13.3 Perimeter and Area of Parallelograms (pp. 324 – 327) Specific Instructional Objectives (SIOs) • Find the perimeter and area of parallelograms • Solve problems involving the perimeter and area of composite figures Syllabus Subject Content Activity Solve problems involving the perimeter and area of a parallelogram Investigation – Formula for Area of a Parallelogram (p. 325) ICT Additional Resources Thinking Time (p. 325) Reasoning, Communication and Connection Investigation – Formula for Area of a Parallelogram (p. 325) Thinking Time (p. 325) Practise Now (p. 324) Thinking Time (p. 325) 22 13.4 Perimeter and Area of Trapeziums (pp. 328 – 333) • Find the perimeter and area of trapeziums • Solve problems involving the perimeter and area of composite figures Solve problems involving the perimeter and area of a trapezium Investigation – Formula for Area of a Trapezium (pp. 328 – 329) Investigation – Formula for Area of a Trapezium (pp. 328 – 329) Thinking Time (p. 329) Practise Now (p. 328) 24 Thinking Time (p. 329) 22 Miscellaneous Solutions for Challenge Yourself Week (5 classes × 45 min) 23 23 23 25 24 Chapter Section 14 Volume and Surface Area of Prisms and Cylinders 14.1 Conversion of Units (pp. 339 – 340) Specific Instructional Objectives (SIOs) • Convert between cm3 and m3 14.2 Nets (pp. 341 – 342) 14.3 Volume and Surface Area of Cubes and Cuboids (pp. 343 – 347) • Find the volume and surface area of cubes and cuboids • Find the volume and 14.4 Volume and surface area of prisms Surface Area of Prisms (pp. 348 – 353) Syllabus Subject Content Activity ICT Additional Resources Reasoning, Communication and Connection Class Discussion – Measurements in Daily Lives (p. 339) Use current units of volume and capacity in practical situations and express quantities in terms of larger or smaller units Class Discussion – Measurements in Daily Lives (p. 329) Use and interpret nets Investigation – Cubes, Cuboids, Prisms and Cylinders (pp. 341 – 342) Solve problems involving the surface area and volume of a cuboid Class Discussion – Surface Area of Cubes and Cuboids (p. 345) Class Discussion – Surface Area of Cubes and Cuboids (p. 345) Thinking Time (p. 349) Thinking Time (p. 349) Solve problems involving the surface area and volume of a prism Ex 14A Q 18(ii) (p. 347) Main Text – ‘Can you find a relationship between the volume of a prism and the area of its cross section?’ (p. 349) 1 1 Week (5 classes × 45 min) 24 Chapter Section 14.5 Volume and Surface Area of Cylinders (pp. 354 – 360) Specific Instructional Objectives (SIOs) • Find the volume and surface area of cylinders Syllabus Subject Content Solve problems involving the surface area and volume of a cylinder Activity ICT Additional Resources Thinking Time (p. 354) Thinking Time (p. 354) Investigation – Comparison between a Cylinder and a Prism (p. 355) Just for Fun (p. 356) Thinking Time (p. 358) Class Discussion – Total Surface Area of Other Types of Cylinders (p. 358) Thinking Time (p. 358) Class Discussion – Total Surface Area of Other Types of Cylinders (p. 358) 24 26 24 14.6 Volume and Surface Area of Composite Solids (pp. 361 – 363) Miscellaneous • Solve problems involving the volume and surface area of composite solids Reasoning, Communication and Connection Solve problems involving the areas and volumes of compound shapes Solutions for Challenge Yourself Week (5 classes × 45 min) 25 25 Chapter Section 15 Statistical Data Handling 15.1 Introduction to Statistics (p. 369) Specific Instructional Objectives (SIOs) 15.2 Pictograms and • Collect, classify and tabulate data Bar Graphs (pp. 369 – 374) • Construct and interpret data from pictograms and bar graphs Syllabus Subject Content Activity ICT Additional Resources Reasoning, Communication and Connection Story Time (p. 369) Collect, classify and tabulate statistical data Main Text (p. 370) Read, interpret and draw simple inferences from tables and statistical diagrams Construct and interpret pictograms and bar charts Main Text – ‘Two levels in the school are selected as the sample group for the survey conducted by the school canteen vendor. Are they representative of the entire school? Explain your answer.’ (p. 369) 27 Main Text – ‘If the canteen vendor decides to sell three types of fruits to the students, which three should he choose? Explain your answer.’ (p. 370) Thinking Time (p. 371) Practise Now Q 2(d) (ii), (e) (p. 372) Ex 15A Q 4(e), 5(iv), 6(iii) (p. 374) 1 Thinking Time (p. 371) 1 Week (5 classes × 45 min) Chapter Section 25 15.3 Pie Charts (pp. 375 – 377) 25 15.4 Line Graphs (pp. 377 – 379) 25 28 26 Specific Instructional Objectives (SIOs) • Construct and interpret data from pie charts • Construct and interpret data from line graphs • Evaluate the purposes and appropriateness of the use of different statistical diagrams Activity Miscellaneous Reasoning, Communication and Connection Practise Now 1 Q 2(iii) (p. 377) Read, interpret and draw simple inferences from tables and statistical diagrams Worked Example 2 Q (ii) (p. 378) Worked Example 2 Q (iv) (p. 378) Class Discussion – Comparison of Various Statistical Diagrams (p. 379) Practise Now 2 Q (iv) (p. 379) Class Discussion – Evaluation of Statistics (pp. 382 – 383) Ex 15B Q 10 – 13 (pp. 385 – 386) 26 Additional Resources Main Text (pp. 375 – 376) Performance Task (p. 381) Explain why some statistical information or diagrams can lead to a misinterpretation of data ICT Construct and interpret pie charts Main Text (pp. 380 – 381) 15.5 Statistics in Real-World Contexts (pp. 380 – 381) 15.6 Evaluation of Statistics (pp. 382 – 386) Syllabus Subject Content Class Discussion – Comparison of Various Statistical Diagrams (p. 379) Performance Task (p. 381) Internet Resources (p. 381) Performance Task (p. 381) Class Discussion – Evaluation of Statistics (pp. 382 – 383) Class Discussion – Evaluation of Statistics (pp. 382 – 383) Ex 15B Q 8(iv), 10 – 11, 12(iii), 13 (pp. 385 – 386) Solutions for Challenge Yourself Chapter 1 Primes, Highest Common Factor and Lowest Common Multiple TEACHING NOTES Suggested Approach Students have learnt only whole numbers in primary school (they will only learn negative numbers and integers in Chapter 2). They have also learnt how to classify whole numbers into two groups, i.e. odd and even numbers. Teachers can introduce prime numbers as another way in which whole numbers can be classified (see Section 1.1). Traditionally, prime numbers apply to positive integers only, but the syllabus specifies whole numbers, which is not an issue since 0 is not a prime number. Teachers can also arouse students’ interest in this topic by bringing in real-life applications (see chapter opener on page 2 of the textbook). Section 1.1: Prime Numbers Teachers can build upon prerequisites, namely, factors, to introduce prime numbers by classifying whole numbers according to the number of factors they have (see Investigation: Classification of Whole Numbers). Since the concept of 0 may not be easily understood, it is dealt with separately in the last question of the investigation. Regardless of whether 0 is classified in the same group as 1 or in a new fourth group, 0 and 1 are neither prime nor composite. Teachers are to take note that 1 is not a prime number ‘by choice’, or else the uniqueness of prime factorisation will fail (see Information on page 8 of the textbook). Also, 0 is not a composite number because it cannot be expressed as a product of prime factors unlike e.g. 40 = 23 × 5. To make practice more interesting, a game is designed in Question 2 of Practise Now 1. Teachers can also tell students about the largest known prime number (there is no largest prime number since there are infinitely many primes) and an important real-life application of prime numbers in the encryption of computer data (see chapter opener and Investigation: Interesting Facts about Prime Numbers) in order to arouse their interest in this topic. Section 1.2: Square Roots and Cube Roots Teachers can build upon what students have learnt about squares, square roots, cubes and cube roots in primary school. Perfect squares are also called square numbers and perfect cubes are also called cube numbers. Perfect numbers are not the same as perfect squares or perfect cubes. Perfect numbers are numbers which are equal to the sum of its proper factors, where proper factors are factors that are less than the number itself, e.g. 6 = 1 + 2 + 3 and 28 = 1 + 2 + 4 + 7 + 14 are the only two perfect numbers less than 100 (perfect numbers are not in the syllabus). After students have learnt negative numbers in Chapter 2, there is a need to revisit square roots and cube roots to discuss negative square roots and negative cube roots (see page 40 of the textbook). Teachers can impress upon students that the square root symbol √ refers to the positive square root only. A common debate among some teachers is whether 0 is a perfect square. There is an argument that 0 is not a perfect square because 0 can multiply by any number (not necessarily itself) to give 0. However, this is not the definition of a perfect square. Since 0 is equal to 0 multiplied by itself, then 0 (the first 0, not the second 0, in this sentence) is a perfect square. Compare this with why 4 is a perfect square (4 is equal to the integer 2 multiplied by itself). Similarly, 0 is a perfect cube. Section 1.3: Highest Common Factor and Lowest Common Multiple Teachers can build upon prerequisites, namely, common factors and common multiples, to develop the concepts of Highest Common Factor (HCF) and Lowest Common Multiple (LCM) respectively (HCF and LCM are no longer in the primary school syllabus although some primary school teachers teach their students HCF and LCM). Since the listing method (see pages 15 and 18 of the textbook) is not an efficient method to find the HCF and the LCM of two or more numbers, there is a need to learn the prime factorisation method and the ladder method (see Methods 1 and 2 in Worked Example 9 and in Worked Example 11). However, when using the ladder method to find the LCM of two or three numbers (see Worked Examples 11 and 12), we stop dividing when there are no common prime factors between any two numbers. The GCE O-level examinations emphasise on the use of the prime factorisation method. 29 1 Challenge Yourself Some of the questions (e.g. Questions 4 and 5) are not easy for average students while others (e.g. Question 2) should be manageable if teachers guide them as follows: Question 2: The figure consists of 3 identical squares but students are to divide it into 4 identical parts. Teachers can guide students by asking them to find the LCM of 3 and 4, which is 12. Thus students have to divide the figure into 12 equal parts before trying to regroup 3 equal parts to form each of the 4 identical parts. Questions 4 and 5: Teachers can get students to try different numerical examples before looking for a pattern in order to generalise. In both questions, it is important that students know whether m and n are co-primes, m i.e. HCF(m, n) = 1. If m and n are not co-primes, they can be built from the ‘basic block’ of HCF( m , n ) and n HCF( m , n ) , which are co-primes. 1 30 WORKED SOLUTIONS Investigation (Sieve of Eratosthenes) Investigation (Classification of Whole Numbers) 1. 1. Number Working Factors 1 1 is divisible by 1 only . 1 2 2=1×2 1, 2 3 3=1×3 1, 3 4 4=1×4=2×2 1, 2, 4 5 5=1×5 1, 5 6 6=1×6=2×3 1, 2, 3, 6 7 7=1×7 1, 7 8 8=1×8=2×4 1, 2, 4, 8 9 9=1×9=3×3 1, 3, 9 10 10 = 1 × 10 = 2 × 5 1, 2, 5, 10 11 11 = 1 × 11 1, 11 12 12 = 1 × 12 = 2 × 6 = 3 × 4 1, 2, 3, 4, 6, 12 13 13 = 1 × 13 1, 13 14 14 = 1 × 14 = 2 × 7 1, 2, 7, 14 15 15 = 1 × 15 = 3 × 5 1, 3, 5, 15 16 16 = 1 × 16 = 2 × 8 = 4 × 4 1, 2, 4, 8, 16 17 17 = 1 × 17 1, 17 18 18 = 1 × 18 = 2 × 9 = 3 × 6 1, 2, 3, 6, 9, 18 19 19 = 1 × 19 1, 19 20 20 = 1 × 20 = 2 × 10 = 4 × 5 1, 2, 4, 5, 10, 20 1 2 3 4 5 6 7 8 9 10 11 12 13 14 15 16 17 18 19 20 21 22 23 24 25 26 27 28 29 30 31 32 33 34 35 36 37 38 39 40 41 42 43 44 45 46 47 48 49 50 51 52 53 54 55 56 57 58 59 60 61 62 63 64 65 66 67 68 69 70 71 72 73 74 75 76 77 78 79 80 81 82 83 84 85 86 87 88 89 90 91 92 93 94 95 96 97 98 99 100 2. (a) (b) (c) (d) The smallest prime number is 2. The largest prime number less than or equal to 100 is 97. There are 25 prime numbers which are less than or equal to 100. No, not every odd number is a prime number, e.g. the number 9 is an odd number but it is a composite number. (e) No, not every even number is a composite number, e.g. the number 0 is an even number but it is neither a prime nor a composite number. (f) For a number greater than 5, if its last digit is 0, 2, 4, 6 or 8, then the number is a multiple of 2, thus it is a composite number; if its last digit is 0 or 5, then the number is a multiple of 5, thus it is a composite number. Hence, for a prime number greater than 5, its last digit can only be 1, 3, 7 or 9. Journal Writing (Page 5) Table 1.1 2. Group A: 1 Group B: 2, 3, 5, 7, 11, 13, 17, 19 Group C: 4, 6, 8, 9, 10, 12, 14, 15, 16, 18, 20 3. 0 is divisible by 1, 2, 3, 4, … 0 has an infinite number of factors. 1. Yes, the product of two prime numbers can be an odd number, e.g. the product of the two prime numbers 3 and 5 is the odd number 15. 2. Yes, the product of two prime numbers can be an even number, e.g. the product of the two prime numbers 2 and 3 is the even number 6. 3. No, the product of two prime numbers P1 and P2 cannot be a prime number since P1P2 has at least 3 distinct factors, i.e. 1, P1 and P1 P2 . Thinking Time (Page 4) Investigation (Interesting Facts about Prime Numbers) 1. A prime number is a whole number that has exactly 2 different factors, 1 and itself. A composite number is a whole number that has more than 2 different factors. A composite number has a finite number of factors. Since 0 has an infinite number of factors, it is neither a prime nor a composite number. Since 1 has exactly 1 factor, it is also neither a prime nor a composite number. 2. No, I do not agree with Michael. Consider the numbers 0 and 1. They are neither prime numbers nor composite numbers. The 1 000 000th prime number is 15 485 863. The last digit of the largest known prime number is 1. Thinking Time (Page 8) The index notation is useful for writing complicated and repetitive expressions in a more compact form, e.g. we can write 3 × 3 × 3 × 3 × 3 × 3 × 3 × 3 as 38 . 31 1 Thinking Time (Page 12) Practise Now 3 If no brackets are used in pressing the sequence of calculator keys in Worked Example 7, the value obtained would be 60.0416 (to 4 d.p.). The mathematical statement that would have been evaluated is 1. 126 = 2 × 32 × 7 2. 539 = 72 × 11 82 + 50 – 73 3 Practise Now 4 63 . 1. 784 = 2 × 2 × 2 × 2 × 7 × 7 = (2 × 2 × 7) × (2 × 2 × 7) = (2 × 2 × 7)2 Practise Now 1 1. 537 is an odd number, so it is not divisible by 2. Since the sum of the digits of 537 is 5 + 3 + 7 = 15 which is divisible by 3, therefore 537 is divisible by 3 (divisibility test for 3). \ 537 is a composite number. 784 = 2 × 2 × 7 = 28 Alternatively, 784 = 2 × 2 × 2 × 2 × 7 × 7 = 24 × 72 \ 59 is an odd number, so it is not divisible by 2. Since the sum of the digits of 59 is 5 + 9 = 14 which is not divisible by 3, then 59 is not divisible by 3. The last digit of 59 is neither 0 nor 5, so 59 is not divisible by 5. A calculator may be used to test whether 59 is divisible by prime numbers more than 5. Since 59 is not divisible by any prime numbers less than 59, then 59 is a prime number. 49 183 147 93 121 236 201 261 150 11 131 5 89 291 117 153 End 57 0 61 192 231 27 1 111 100 149 17 103 43 7 127 51 53 83 33 32 105 29 71 37 7056 = 2 4 × 32 × 7 2 = 22 × 3 × 7 = 84 2. Practise Now 5 2. 135 784 = 2 4 × 7 2 = 22 × 7 = 28 \ 1. 2744 = 2 × 2 × 2 × 7 × 7 × 7 = (2 × 7) × (2 × 7) × (2 × 7) = (2 × 7)3 \ 3 \ 3 2744 = 2 × 7 = 14 Alternatively, 2744 = 2 × 2 × 2 × 7 × 7 × 7 = 23 × 73 2. Start Practise Now 2 1. Since 31 is a prime number, then 1 and 31 are its only two factors. It does not matter whether p or q is 1 or 31 as we only want to find the value of p + q . \ p + q = 1 + 31 = 32 2. Since n × (n + 28) is a prime number, then n and n + 28 are its only two factors. Since 1 has to be one of its two factors, then n = 1 . \ n × (n + 28) = 1 × (1 + 28) = 1 × 29 = 29 3 3 9261 = 33 × 7 3 =3×7 = 21 3 Practise Now 6 123 ≈ 121 = 11 3 123 ≈ 3 125 =5 Practise Now 7 1. (a) 232 + (b) 1 2744 = 2 3 × 7 3 =2×7 = 14 32 2025 – 73 = 231 32 × 53 – 3 20 2013 = 0.3582 (to 4 d.p.) Practise Now 11 2. Length of each side of poster = 987 = 31.42 cm (to 2 d.p.) Perimeter of poster = 4 × 31 .42 = 125.7 cm (to 1 d.p.) 1. Method 1: 24 = 23 × 3 90 = 2 × 32 × 5 LCM of 24 and 90 = 23 × 32 × 5 = 360 Method 2: Practise Now 8 2013 = 44.9 (to 1 d.p.), so the largest prime number less than or equal to 2013 is 43 . 2013 is an odd number, so it is not divisible by 2. Since the sum of the digits of 2013 is 2 + 1 + 3 = 6 which is divisible by 3, therefore 2013 is divisible by 3 (divisibility test for 3). \ 2013 is a composite number. 2 24, 90 3 12, 45 4, 15 HCF of 24 and 90 = 2 × 3 × 4 × 15 = 360 2. Smallest whole number that is divisible by both 120 and 126 = LCM of 120 and 126 = 23 × 32 × 5 × 7 = 2520 3. 6 = 2 × 3 24 = 23 × 3 Smallest value of n = 23 =8 2017 = 44.9 (to 1 d.p.), so the largest prime number less than or equal to 2017 is 43 . Since 2017 is not divisible by any of the prime numbers 2, 3, 5, 7, …, 43, then 2017 is a prime number. Practise Now 9 1. Method 1: 56 = 23 × 7 84 = 22 × 3 × 7 HCF of 56 and 84 = 22 × 7 = 28 Method 2: Practise Now 12 9 = 32 30 = 2 × 3 × 5 108 = 22 × 33 LCM of 9, 30 and 108 = 22 × 33 × 5 = 540 2 56, 84 2 28, 42 Practise Now 13 7 14, 21 2, 3 1. 15 = 3 × 5 16 = 24 36 = 22 × 32 LCM of 15, 16 and 36 = 24 × 32 × 5 = 720 720 minutes = 12 hours \ The three bells will next toll together at 2.00 a.m. 2. (i) 140 = 22 × 5 × 7 168 = 23 × 3 × 7 210 = 2 × 3 × 5 × 7 HCF of 140, 168 and 210 = 2 × 7 = 14 Greatest possible length of each of the smaller pieces of rope = 14 cm (ii) Number of smallest pieces of rope he can get altogether HCF of 56 and 84 = 2 × 2 × 7 = 28 2. 28 = 22 × 7 70 = 2 × 5 × 7 Largest whole number which is a factor of both 28 and 70 = HCF of 28 and 70 =2×7 = 14 3. Greatest whole number that will divide both 504 and 588 exactly = HCF of 504 and 588 = 22 × 3 × 7 = 84 Practise Now 10 90 = 2 × 32 × 5 135 = 33 × 5 270 = 2 × 33 × 5 HCF of 90, 135 and 270 = 32 × 5 = 45 140 168 210 + + 14 14 14 = 10 + 12 + 15 = 37 = 33 1 Exercise 1A Alternatively, 576 = 2 × 2 × 2 × 2 × 2 × 2 × 3 × 3 = 26 × 32 1. (a) 87 is an odd number, so it is not divisible by 2. Since the sum of the digits of 87 is 8 + 7 = 15 which is divisible by 3, therefore 87 is divisible by 3 (divisibility test for 3). \ 87 is a composite number. (b) 67 is an odd number, so it is not divisible by 2. Since the sum of the digits of 67 is 6 + 7 = 13 which is not divisible by 3, then 67 is not divisible by 3. The last digit of 67 is neither 0 nor 5, so 67 is not divisible by 5 . A calculator may be used to test whether 67 is divisible by prime numbers more than 5. Since 67 is not divisible by any prime numbers less than 67, then 67 is a prime number. (c) 73 is an odd number, so it is not divisible by 2. Since the sum of the digits of 73 is 7 + 3 = 10 which is not divisible by 3, then 73 is not divisible by 3. The last digit of 73 is neither 0 nor 5, so 73 is not divisible by 5 . A calculator may be used to test whether 73 is divisible by prime numbers more than 5. Since 73 is not divisible by any prime numbers less than 73, then 73 is a prime number. (d) 91 is an odd number, so it is not divisible by 2. Since the sum of the digits of 91 is 9 + 1 = 10 which is not divisible by 3, then 91 is not divisible by 3. The last digit of 91 is neither 0 nor 5, so 91 is not divisible by 5 . A calculator may be used to test whether 91 is divisible by prime numbers more than 5. Since 91 is divisible by 7, therefore 91 is a composite number. 2. (a) 72 = 23 × 32 (b) 187 = 11 × 17 4 (c) 336 = 2 × 3 × 7 (d) 630 = 2 × 32 × 5 × 7 3. (a) 1764 = 2 × 2 × 3 × 3 × 7 × 7 = (2 × 3 × 7) × (2 × 3 × 7) = (2 × 3 × 7)2 576 = 2 6 × 32 = 23 × 3 = 24 (c) 3375 = 3 × 3 × 3 × 5 × 5 × 5 = (3 × 5) × (3 × 5) × (3 × 5) = (3 × 5)3 \ \ 3375 = 3 × 5 = 15 Alternatively, 3375 = 3 × 3 × 3 × 5 × 5 × 5 = 33 × 53 3375 = 3 33 × 5 3 =3×5 = 15 (d) 1728 = 2 × 2 × 2 × 2 × 2 × 2 × 3 × 3 × 3 = (2 × 2 × 3) × (2 × 2 × 3) × (2 × 2 × 3) = (2 × 2 × 3)3 4. 5. 1764 = 2 × 3 × 7 = 42 Alternatively, 1764 = 2 × 2 × 3 × 3 × 7 × 7 = 22 × 32 × 72 \ 1764 = 2 2 × 32 × 7 2 =2×3×7 = 42 (b) 576 = 2 × 2 × 2 × 2 × 2 × 2 × 3 × 3 = (2 × 2 × 2 × 3) × (2 × 2 × 2 × 3) = (2 × 2 × 2 × 3)2 3 \ 3 \ 3 1728 = 3 2 6 × 33 = 22 × 3 = 12 9801 = 34 × 112 = 32 × 11 = 99 3 3 6 3 21 952 = 2 × 7 2 =2 ×7 = 28 6. (a) 66 ≈ 64 =8 (b) 80 ≈ 81 =9 (c) 3 218 ≈ 3 216 =6 (d) 3 730 ≈ 3 729 =9 7. (a) 72 – (b) 576 = 2 × 2 × 2 × 3 = 24 1 \ 1728 = 2 × 2 × 3 = 12 Alternatively, 1728 = 2 × 2 × 2 × 2 × 2 × 2 × 3 × 3 × 3 = 26 × 33 \ \ 3 (c) 34 361 + 213 = 9291 555 + 5 2 2 × 3 3 43 + 222 3 = 1.0024 (to 4 d.p.) 4913 = 9 Exercise 1B 8. Length of each side of photo frame = 250 = 15.81 cm (to 2 d.p.) Perimeter of photo frame = 4 × 15 .81 = 63.2 cm (to 1 d.p.) 1. (a) 12 = 22 × 3 30 = 2 × 3 × 5 HCF of 12 and 30 = 2 × 3 =6 (b) 84 = 22 × 3 × 7 156 = 22 × 3 × 13 HCF of 84 and 156 = 22 × 3 = 12 (c) 15 = 3 × 5 60 = 22 × 3 × 5 75 = 3 × 52 HCF of 15, 60 and 75 = 3 × 5 = 15 (d) 77 = 7 × 11 91 = 7 × 13 143 = 11 × 13 HCF of 77, 91 and 143 = 1 2. (a) 24 = 23 × 3 30 = 2 × 3 × 5 LCM of 24 and 30 = 23 × 3 × 5 = 120 (b) 42 = 2 × 3 × 7 462 = 2 × 3 × 7 × 11 LCM of 42 and 462 = 2 × 3 × 7 × 11 = 462 (c) 12 = 22 × 3 18 = 2 × 32 81 = 34 LCM of 12, 18 and 81 = 22 × 34 = 324 (d) 63 = 32 × 7 80 = 24 × 5 102 = 2 × 3 × 17 LCM of 63, 80 and 102 = 24 × 32 × 5 × 7 × 17 = 85 680 3. 42 = 2 × 3 × 7 98 = 2 × 72 Largest whole number which is a factor of both 42 and 98 = HCF of 42 and 98 =2×7 = 14 4. Greatest whole number that will divide both 792 and 990 exactly = HCF of 792 and 990 = 2 × 32 × 11 = 198 5. Smallest whole number that is divisble by both 176 and 342 = LCM of 176 and 342 = 24 × 32 × 11 × 19 = 30 096 9. Length of each side of box = 3 2197 = 13 cm Area of one side of box = 132 = 169 cm2 10. (a) 667 = 25.8 (to 1 d.p.), so the largest prime number less than or equal to 667 is 23 . . 667 is an odd number, so it is not divisible by 2. Since the sum of the digits of 667 is 6 + 6 + 7 = 19 which is not divisible by 3, then 667 is not divisible by 3. The last digit of 667 is neither 0 nor 5, so 667 is not divisible by 5 . A calculator may be used to test whether 667 is divisible by prime numbers more than 5. Since 667 is divisible by 23, therefore 667 is a composite number. (b) 677 = 26.0 (to 1 d.p.), so the largest prime number less than or equal to 677 is 23 . Since 677 is not divisible by any of the prime numbers 2, 3, 5, 7, …, 23, then 677 is a prime number. (c) 2021 = 45.0 (to 1 d.p.), so the largest prime number less than or equal to 2021 is 43 . 2021 is an odd number, so it is not divisible by 2. Since the sum of the digits of 2021 is 2 + 0 + 2 + 1 = 5 which is not divisible by 3, then 2021 is not divisible by 3. The last digit of 2021 is neither 0 nor 5, so 2021 is not divisible by 5 . A calculator may be used to test whether 2021 is divisible by prime numbers more than 5. Since 2021 is divisible by 43, therefore 2021 is a composite number. (d) 2027 = 45.0 (to 1 d.p.), so the largest prime number less than or equal to 2027 is 43 . Since 2027 is not divisible by any of the prime numbers 2, 3, 5, 7, …, 43, then 2027 is a prime number. 11. Since 37 is a prime number, then 1 and 37 are its only two factors. It does not matter whether p or q is 1 or 37 as we only want to find the value of p + q . \ p + q = 1 + 37 = 38 12. Since n × (n + 42) is a prime number, then n and n + 42 are its only two factors. Since 1 has to be one of its two factors, then n = 1 . \ n × (n + 42) = 1 × (1 + 42) = 1 × 43 = 43 35 1 (c) True. If 18 is a multiple of a whole number n, then 18 = nk for 18 some whole number k. Thus = k is a whole number, i.e. 18 n is divisible by n . (d) True. Since m is a multiple of p, by the same reasoning as in (c), then m is divisible by p. Similarly, m is divisible by q . 12. (i) 64 = 26 48 = 24 × 3 HCF of 64 and 48 = 24 = 16 Length of each square = 16 cm 64 48 (ii) Number of squares that can be cut altogether = × 16 16 =4×3 = 12 13. (i) Let the number of boys in the class be n . Then 15 × n = 3 × 5 × n is divisible by 21 = 3 × 7 . Thus the possible values of n are multiples of 7. Hence, n = 14 since 14 + 20 = 34 students is the only possibility where the number of students in the class is between 30 and 40. \ Number of students in the class = 34 (ii) Number of chocolate bars their form teacher receive 6. 15 = 3 × 5 45 = 32 × 5 Smallest value of n = 32 =9 7. (i) 171 = 32 × 19 63 = 32 × 7 27 = 33 HCF of 171, 63 and 27 = 32 =9 Largest number of gift bags that can be packed = 9 (ii) Number of pens in a gift bag = 171 ÷ 9 = 19 Number of pencils in a gift bag = 63 ÷ 9 =7 Number of erasers in a gift bag = 27 ÷ 9 =3 8. (i) 60 = 22 × 3 × 5 80 = 24 × 5 LCM of 60 and 80 = 24 × 3 × 5 = 240 It will take 240 s for both cars to be back at the starting point at the same time. (ii) 5 × 240 s = 1200 s = 20 minutes It will take 20 minutes for the faster car to be 5 laps ahead of the slower car. 9. (a) True. If 6 is a factor of a whole number n, then n = 6k for some whole number k . We have n = 6k = 2(3k). Since 3k is a whole number, then 2 is a factor of n . We also have n = 6k = 3(2k). Since 2k is a whole number, then 3 is a factor of n . (b) True. Since 2 and 3 are distinct prime factors of the whole number, then the prime factorisation of the whole number will contain both of these prime factors. (c) False, e.g. 2 and 4 are factors of 4, but 8 is not a factor of 4. (d) True. If f is a factor of n, then n = fk for some whole number n k. Thus = k is a whole number. Since n can be written as f n n a product of the whole numbers and f, then is a factor f f of n . (e) True. Since h is a factor of both p and q, then both p and q are divisible by h . 10. 9 = 32 12 = 22 × 3 252 = 22 × 32 × 7 Possible values of n = 7, 3 × 7 or 32 × 7 = 7, 21 or 63 11. (a) True. If 6 is a multiple of a whole number n, then 6 = nk for some whole number k . We have 12 = 2nk = n(2k). Since 2k is a whole number, then 12 is a multiple of n . (b) False, e.g. 12 is a multiple of 4, but 6 is not a multiple of 4. 1 15 × 14 21 = 10 14. (i) 126 = 2 × 32 × 7 108 = 22 × 33 HCF of 126 and 108 = 2 × 33 = 18 Length of each square = 18 cm Least number of square patterns that could be formed on the sheet of paper = 126 108 × 18 18 =7×6 = 42 (ii) To fit the sheet of paper perfectly, the patterns can be rectangular, triangular or trapeziums with two right angles, etc. 15. (i) 45 = 32 × 5 42 = 2 × 3 × 7 LCM of 45 and 42 = 2 × 32 × 5 × 7 = 630 Number of patterns needed to form the smallest square = 630 630 × 45 42 = 14 × 15 = 210 (ii) 630 mm = 0.63 m Area of smallest square that can be formed = 0.632 = 0.3969 m2 By trial and error, Area of largest square that can be formed = 0 .3969 × 22 = 1.5876 m2 < 1.6 m2 = \ Length of largest square that can be formed = 1.5876 = 1.26 m 36 5. 6 = 2 × 3 Review Exercise 1 12 = 22 × 3 660 = 22 × 3 × 5 × 11 Possible values of n = 5 × 11, 2 × 5 × 11, 3 × 5 × 11, 22 × 5 × 11, 2 × 3 × 5 × 11 or 22 × 3 × 5 × 11 = 55, 110, 165, 220, 330 or 660 6. (i) 108 = 22 × 33 81 = 34 54 = 2 × 33 HCF of 108, 81 and 54 = 33 = 27 Largest number of baskets that can be packed = 27 (ii) Number of stalks of roses in a basket = 108 ÷ 27 =4 Number of stalks of lilies in a basket = 81 ÷ 27 =3 Number of stalks of orchids in a basket = 54 ÷ 27 =2 7. Time taken for Khairul to run 1 round = 360 s = 6 minutes Time taken for Devi to cycle 1 round = 4 ÷ 2 = 2 minutes 18 = 2 × 32 6=2×3 2=2 LCM of 18, 6 and 2 = 2 × 32 = 18 All three of them will next meet at 6.03 p.m. 8. (i) By counting, they will next have the same day off on 7 May. (ii) 4 = 22 6=2×3 LCM of 4 and 6 = 22 × 3 = 12 Subsequently, they will have the same day off every 12 days. 1. (a) 1225 = 5 × 5 × 7 × 7 = (5 × 7) × (5 × 7) = (5 × 7)2 1225 = 5 × 7 = 35 Alternatively, 1225 = 5 × 5 × 7 × 7 = 52 × 72 \ \ 1225 = 5 2 × 7 2 =5×7 = 35 (b) 13 824 = 2 × 2 × 2 × 2 × 2 × 2 × 2 × 2 × 2 × 3 × 3 × 3 = (2 × 2 × 2 × 3) × (2 × 2 × 2 × 3) × (2 × 2 × 2 × 3) = (2 × 2 × 2 × 3)3 \ 3 \ 3 13 824 = 2 × 2 × 2 × 3 = 24 Alternatively, 13 824 = 2 × 2 × 2 × 2 × 2 × 2 × 2 × 2 × 2 × 3 × 3 × 3 = 29 × 33 2. (a) (b) 3. (a) 13 824 = 3 2 9 × 33 = 23 × 3 = 24 63 ≈ 64 =8 3 345 ≈ 3 343 =7 753 = 27.4 (to 1 d.p.), so the largest prime number less than or equal to 753 is 23 . 753 is an odd number, so it is not divisible by 2. Since the sum of the digits of 753 is 7 + 5 + 3 = 15 which is divisible by 3, therefore 753 is divisible by 3 (divisibility test for 3). \ 753 is a composite number. (b) 757 = 27.5 (to 1 d.p.), so the largest prime number less Challenge Yourself than or equal to 757 is 23 . Since 757 is not divisible by any of the prime numbers 2, 3, 5, 7, …, 23, then 757 is a prime number. 4. (i) Greatest whole number that will divide both 840 and 8316 exactly = HCF of 840 and 8316 = 22 × 3 × 7 = 84 (ii) Smallest whole number that is divisible by both 840 and 8316 = LCM of 840 and 8316 = 23 × 33 × 5 × 7 × 11 = 83 160 1. (i) The six adjacent numbers are 11, 12, 1, 2, 3, 4. (ii) The other six numbers are 5, 6, 7, 8, 9, 10. Make a list where the sum of each of the pairs of numbers is a prime number: • 4 + 7 = 11; 4 + 9 = 13 • 5 + 6 = 11; 5 + 8 = 13 • 6 + 5 = 11; 6 + 7 = 13; 6 + 11 = 17 • 7 + 4 = 11; 7 + 6 = 13; 7 + 10 = 17 • 8 + 5 = 13; 8 + 9 = 17; 8 + 11 = 19 • 9 + 4 = 13; 9 + 8 = 17; 9 + 10 = 19 • 10 + 7 = 17; 10 + 9 = 19 • 11 + 6 = 17; 11 + 8 = 19 37 1 (iii) Yes, the result in (ii) can be generalised for any two numbers. Proof: Consider two numbers x and y . Then x = HCF × p, — (1) y = HCF × q, — (2) where the HCF of p and q is 1 . (1) × q: x × q = HCF × p × q — (3) (2) × p: y × p = HCF × p × q — (4) (3) × (4): x × y × p × q = HCF × p × q × HCF × p × q x × y = HCF × HCF × p × q Since the HCF of p and q is 1, we cannot take out a factor greater than 1 in the product p × q, thus HCF × p × q = LCM. \ x × y = HCF × LCM (iv) No, the result in (ii) cannot be generalised for any three numbers. For example, consider the numbers 10, 20 and 25. 10 = 2 × 5 20 = 22 × 5 25 = 52 HCF of 10, 20 and 25 = 5 LCM of 10, 20 and 25 = 22 × 52 = 100 HCF × LCM = 5 × 100 = 500 ≠ 10 × 20 × 25 4. Number of squares passed through by a diagonal of a m-by-n rectangle = m + n – HCF(m, n) Notice that 5 and 10 are the only numbers that can be adjacent to two numbers only: • 5 can be adjacent to 6 and 8 only, i.e. 6 – 5 – 8 or 8 – 5 – 6; • 10 can be adjacent to 7 and 9 only, i.e. 7 – 10 – 9 or 9 – 10 – 7 . Since 4 and 11 are adjacent to another number on one side, then • the only two possibilities for the other side of 4 are 7 and 9; • the only two possibilities for the other side of 11 are 6 and 8 . With the above information, we can narrow down the possible arrangements to only two ways: 11 12 11 12 1 2 8 3 5 6 4 7 10 9 1 2 6 3 5 8 4 9 10 7 2. LCM of 3 and 4 = 12 \ We divide the 3 identical squares into 12 equal parts. Thus we have: 5. (i) Fraction of a sausage each person gets = 12 18 2 = 3 \ Least number of cuts required = 12 (ii) Least number of cuts required = n – HCF(m, n) 3. (i) 120 = 23 × 3 × 5 126 = 2 × 32 × 7 HCF of 120 and 126 = 2 × 3 =6 LCM of 120 and 126 = 23 × 32 × 5 × 7 = 2520 (ii) HCF × LCM = 6 × 2520 = 15 120 = 120 × 126 (Shown) 120 = 23 × 3 × 5 126 = 2 × 32 × 7 To obtain the HCF of 120 and 126, we choose the power of each of the common prime factors with the smaller index, i .e . 2 and 3 . On the other hand, to obtain the LCM of 120 and 126, we choose the power of each of the common prime factors with the higher index, i.e. 23 and 32, and the remaining factors, i.e. 5 and 7 . Since each term in the prime factorisation of 120 and 126 is used to find either their HCF or their LCM, the product of the HCF and LCM of 120 and 126 is equal to the product of 120 and 126 . 1 38 Chapter 2 Integers, Rational Numbers and Real Numbers TEACHING NOTES Suggested Approach Although the concept of negative numbers is new to most students as they have not learnt this in primary school, they do encounter negative numbers in their daily lives, e.g. in weather forecasts. Therefore, teachers can get students to discuss examples of the use of negative numbers in the real world to bring across the idea of negative numbers (see Class Discussion: Uses of Negative Numbers in the Real World). The learning experiences in the new syllabus specify the use of algebra discs. In this chapter, only number discs (or counters) showing the numbers 1 and –1 are needed. Since many Secondary 1 students are still in the concrete operational stage (according to Piaget), the use of algebra discs can help them to learn the concepts more easily. However, there is still a need to guide students to move from the ‘concrete’ to the ‘abstract’, partly because they cannot use algebra discs in examinations, and partly because they cannot use algebra discs to add or subtract large negative integers, and decimals (see Section 2.2). Section 2.1: Negative Numbers Teachers should teach students to read the negative number –2 as negative 2, not minus 2 (‘negative’ is a state while ‘minus’ is an operation). For example, if you have $5 and you owe your friend $2, how much do you have left? Since nothing is mentioned about you returning money to your friend, you have $5 left. Thus $2 is a state of owing money. However, if you return $2 to your friend, you have $5 + (–$2) = $5 – $2 = $3 left, i.e. 5 minus 2 is an operation of returning money. Students should also learn about the absolute value of a negative number (see page 29 of the textbook) because they will need it in Section 2.2. In primary school, students have only learnt the terms ‘less than’ and ‘more than’, so there is a need to teach them how to use the symbols ‘<’ and ‘>’ when comparing numbers. It is not necessary to teach them about ‘less than or equal to’ and ‘more than or equal to’ now. Section 2.2: Addition and Subtraction involving Negative Numbers Algebra discs cannot be used to add or subtract large negative integers, and decimals, so there is a need to help students consolidate what they have learnt in the class discussions on pages 33 and 35 of the textbook by moving away from the ‘concrete’ to the following two key ‘abstract’ concepts: Key Concept 1: Adding a negative number is the same as subtracting the absolute value of the number, e.g. 5 + (−2) = 5 − 2. Key Concept 2: Subtracting a negative number is the same as adding the absolute value of the number, e.g. 5 − (−2) = 5 + 2. To make the key concepts less abstract, numerical examples are used. Do not use algebra now because students are still unfamiliar with algebra even though they have learnt some basic algebra in primary school. Avoid teaching students ‘− × − = +’ now because the idea behind 5 − (−2) is subtraction, not multiplication. To make practice more interesting, a puzzle is designed on page 36 of the textbook. Section 2.3: Multiplication and Division involving Negative Numbers The idea of flipping over a disc to obtain the negative of a number, e.g. –(−3) = 3, is important in teaching multiplication involving negative numbers. Since algebra discs cannot be used to teach division involving negative numbers, another method is adopted (see page 40 of the textbook). There is a need to revisit square roots and cube roots in this section to discuss negative square roots and negative cube roots (see page 40 of the textbook). Teachers can impress upon students that the square root symbol √ refers to the positive square root only. 39 1 Section 2.4: 1 Rational Numbers and Real Numbers Traditionally, real numbers are classified as either rational or irrational numbers. Another way to classify real numbers is according to whether their decimal forms are terminating, recurring, or non-recurring (see page 50 of the textbook). If teachers show students the first million digits of p (see page 51 of the textbook), many students may be surprised that p has so many digits! This suggests that students do not know that p has an infinite number of decimal places. Teachers may wish to celebrate Pi Day with students on March 14 by talking about p or singing the Pi song. 40 WORKED SOLUTIONS Thinking Time (Page 28) Class Discussion (Uses of Negative Numbers in the Real World) • • • • • –5 One of the most common uses of negative numbers is in the measurement of temperature, where negative numbers are used to show temperatures below the freezing point of water, i.e. 0 °C. Absolute zero, defined as 0 Kelvin, is the theoretical lowest possible temperature. 0 Kelvin is equivalent to a temperature of –273.15 °C, therefore the theoretical lowest possible temperature is 273.15 °C below 0 °C. The elevation of a location commonly refers to its height with reference to Earth’s sea level and can be represented by a positive or a negative number. Given a point with an elevation of –200 m, we can deduce that the point is 200 m below sea level. The lowest elevation on Earth that is on dry land is the Dead Sea shore in Asia with an elevation of – 423 m, i.e. the shore of the Dead Sea is 423 m below sea level. Negative numbers are also used to tell time zones, which are based on Greenwich Mean Time (GMT). A country which is in the time zone of GMT –2 means that the time in that country is 2 hours behind the GMT. For example, Honolulu, Hawaii is in the time zone of GMT –10, while Liverpool, United Kingdom is in the time zone of GMT 0, therefore when it is 10 a.m. in Liverpool, it is 12 midnight in Honolulu. Latitude and longitude are a set of coordinates that allow for the specification of a geographical location on the Earth’s surface and can be represented by positive and/or negative numbers. The latitude of a point is determined with reference to the equatorial plane; the North Pole has a latitude of +90°, which means that it is 90° north of the equator while the South Pole has a latitude of –90°, which means that it is 90° south of the equator. The longitude of a point gives its east-west position relative to the Prime Meridian (longitude 0°); a location with a longitude of +50° means that it is 50° east of the Prime Meridian while a location with a longitude of –50° means that it is 50° west of the Prime Meridian. The latitude and longitude of Rio Grande, Mexico are approximately –32° and –52° respectively, which means that it is 32° south of the equator and 52° west of the Prime Meridian. The use of negative numbers can also be seen in scoring systems, such as in golf. Each hole has a par score, which indicates the number of strokes required and a golfer’s score for that hole is calculated based on the number of strokes played. A score of +3 on a hole shows that the golfer played three strokes above par, while a score of –3 on a hole shows that the golfer played three strokes under par. –4 –3 –2 –1 0 1 2 3 4 5 (a) Since –3 is on the left of 2, we say ‘–3 is less than 2’ and we write ‘–3 < 2’. (b) Since –3 is on the right of –5, we say ‘–3 is more than –5’ and we write ‘–3 > –5’. Class Discussion (Addition involving Negative Numbers) Part I 1. (a) (b) 2. (a) (b) 3. (a) (b) 7 + (–3) = 4 6 + (– 4) = 2 (–7) + 3 = – 4 (–6) + 4 = –2 (–7) + (–3) = –10 (– 6) + (– 4) = –10 Note: • If we add a positive number and a negative number, (i) we take the difference between the absolute values of the two numbers, and (ii) the sign of the answer follows the sign of the number with the greater absolute value, e.g. 5 + (–2) = 3 and (–5) + 2 = –3. • If we add two negative numbers, (i) we take the sum of the absolute values of the two numbers, and (ii) the answer is negative, e.g. (–5) + (–2) = –7. Class Discussion (Subtraction involving Negative Numbers) Part I 1. (a) 7 – (–3) = 7 + 3 = 10 (b) 6 – (– 4) = 6 + 4 = 10 2. (a) (–7) – 3 = (–7) + (–3) = –10 (b) (– 6) – 4 = (– 6) + (– 4) = –10 3. (a) (–7) – (–3) = (–7) + 3 = –4 (b) (– 4) – (– 6) = (– 4) + 6 = 2 4. (a) 3 – 7 = 3 + (–7) = –4 (b) 4 – 6 = 4 + (– 6) = –2 Teachers may wish to note that the list is not exhaustive. 41 1 Note: • If we take the difference of a positive number and a negative number, (i) we add the absolute values of the two numbers, and (ii) the sign of the answer follows the sign of the number with the greater absolute value, e.g. 5 – (–2) = 7 and (–5) – 2 = –7. • If we take the difference of two negative numbers or two positive numbers, (i) we take the difference between the absolute values of the two numbers, and (ii) the sign of the answer depends on whether the first number is greater than or smaller than the second number, e.g. (–5) – (–2) = –3 but (–2) – (–5) = 3; 2 – 5 = –3 but 5 – 2 = 3. Investigation (Terminating, Recurring and Non-Recurring Decimals) Group 1 1 = –3.125 8 63 = 0.984 375 64 – 123 = –1.242 424 242 – 3 5 = –1.709 975 947 99 22 = 3.142 857 143 7 8 –√ 5 –3 –2 –3 –4 3 –1 – Note: In general, p = 3.141 592 654 123 99 0 64 1 4 2 1 √2 p 3 4 22 7 Fig. 2.7 positive number × negative number = negative number, negative number × positive number = negative number, negative number × negative number = positive number. 5. 9 63 1 1 123 22 1 , ,– , p, , , , – 3 5 , –3 4 64 99 7 8 2 3 Investigation (Some Interesting Facts about the Irrational Number p) 1. The 1 000 000th digit of p is 1. 2. The 5 000 000 000 000th digit of p is 2. 3. Lu Chao, a graduate student from China, took 24 hours and 4 minutes to recite p to 67 890 decimal places in 2005. Thinking Time (Page 41) It is not possible to obtain the square roots of a negative number, e.g. ± –16 , because the square of any number is more than or equal to 0. Teachers may wish to take this opportunity to highlight to higher-ability students that even though ± –16 is not defined in the set of real numbers, it is defined in the set of complex numbers. Practise Now (Page 27) 1. (i) 2013, 6 (ii) –5, –17 Thinking Time (Page 49) (iii) 2013, 1.666, 2 m (a) Any integer m can be expressed in the form , e.g. 2 = and 1 1 –3 –3 = . 1 3 ,6 4 1 2 (iv) –5, – , –3.8, –17, – 2 3 2. (a) – 43.6 °C (b) – 423 m (c) –1 (d) –$10 000 0 In particular, the integer 0 can be expressed in the form , where n n is any integer except 0. 1 = 0.707 106 781 2 22 . 7 2. For each of the numbers in Group 2, some digits after the decimal point repeat themselves indefinitely. The numbers in Group 2 are rational numbers. 3. For each of the numbers in Group 1, the digits after the decimal point terminate. The numbers in Group 1 are rational numbers. For each of the numbers in Group 3, the digits after the decimal point do not repeat but they continue indefinitely. The numbers in Group 3 are irrational numbers. 9 1 63 4. 1 3 1 × (– 4) = – 4 2 × (– 4) = –8 3 × (– 4) = –12 (–1) × 4 = – 4 (–2) × 4 = –8 (–3) × 4 = –12 (–1) × (– 4) = 4 (–2) × (– 4) = 8 (–3) × (– 4) = 12 e.g. 0.5 = 2 1. Based on the calculator values, p is not equal to Part I (b) There is more than one way to express a decimal in the form 1 Table 2.1 Class Discussion (Multiplication involving Negative Numbers) 1. (a) (b) (c) 2. (a) (b) (c) 3. (a) (b) (c) Group 3 1 = 0.333 333 333 3 3 9 = 2.25 4 –3 Group 2 a , b 2 3 1 2 3 1 = = … and 0.333… = = = … 6 9 2 4 6 3 42 Practise Now (Page 39) Practise Now (Page 29) 1. (b) < 2. –5 –5 (c) > –1 –3.8 –4 –3 \ –5, –3.8, –1 1 2 –2 (d) > 0 –1 0 3 4 1.666 1 (a) (b) (c) (d) (e) (f) (g) (h) (i) (j) 4 2 3 4 5 1 3 , 0, , 1.666, 4 2 4 Practise Now (Page 33) (a) (b) (c) (d) (e) (f) (g) (h) 9 + (–2) = 7 –7 + 4 = –3 3 + (–5) = –2 – 6 + (–8) = –14 27 + (–13) = 14 –25 + 11 = –14 14 + (–16) = –2 –12 + (–15) = –27 2 × (–6) = –12 –5 × 4 = –20 –1 × (–8) = 8 –3 × (–7) = 21 –(–10) = 10 –9(–2) = 18 15 × (–2) = –30 –3 × 12 = –36 – 4 × (–10) = 40 –2(–100) = 200 Practise Now (Page 40) (a) –8 ÷ 2 = – 4 (b) 15 ÷ (–3) = –5 (c) –8 ÷ (– 4) = 2 –6 = –2 3 20 (e) = –4 –5 –12 (f) =4 –3 (d) Practise Now 1 Temperature in the morning = –8 °C + 2 °C = – 6 °C Practise Now 3a Practise Now (Page 35) (a) Square roots of 64 = ± 64 = ±8 (a) 9 – (–2) = 9 + 2 = 11 (b) –7 – 4 = –11 (c) –3 – (–5) = –3 + 5 = 2 (d) –8 – (–6) = –8 + 6 = –2 (e) 4 – 8 = – 4 (f) 27 – (–13) = 27 + 13 = 40 (g) –25 – 11 = –36 (h) –14 – (–16) = –14 + 16 = 2 (i) –15 – (–12) = –15 + 12 = –3 (j) 10 – 28 = –18 (b) Negative square root of 9 = – 9 = –3 36 = 6 (c) Practise Now 3b (a) (–3)3 = –27 (c) 3 216 = 6 (b) (– 4)3 = – 64 (d) 3 –8 = –2 Practise Now 4a (a) –3 × (15 – 7 + 2) = –3 × (8 + 2) = –3 × 10 = –30 (b) 43 – 7 × [16 – ( 3 64 – 5)] = 64 – 7 × [16 – (4 – 5)] = 64 – 7 × [16 – (–1)] = 64 – 7 × (16 + 1) = 64 – 7 × 17 = 64 – 119 = –55 Practise Now 2 1. Point A shows –5 °C. Point B shows 23 °C. Difference in temperature = 23 °C – (–5 °C) = 23 °C + 5 °C = 28 °C 2. Altitude at D = –165 m Difference in altitude = 314 m – (–165 m) = 314 m + 165 m = 479 m Practise Now 4b (a) –3 × (15 – 7 + 2) = –30 (b) 43 – 7 × [16 – ( 3 64 – 5)] = –55 43 1 Practise Now 5 (b) 1 (a) 7 3  1 1 3 +  –3  = 7 – 3 5  2 2 5 5 6 =7 –3 10 10 5 6 10   =  6 + 10  + 10 – 3 10 9 =3 10 (b) –2  5  2 3 3 5 2 +  –  –  –  = –2 – +  6  3 4 4 6 3 11 5 2 =– – + 4 6 3 33 10 8 =– – + 12 12 12 –33 – 10 + 8 = 12 – 43 + 8 = 12 Practise Now 7b Practise Now 5 (a) 7 –35 12 11 = –2 12 (a) 2 2 93 8 2 9 × = × 41 3 4 1 3 = 2×3 = 6 1 5 25 5 ÷ = ÷ 6 2 6 2 (a) 5  4 1 7 ÷  –2  = –1  5 4 8 (b) 1 6 1  3 9 ×  +  –   = 1 5 2 4 40     Practise Now 8 1 25 2 = × 51 3 6 (a) 13 . 5 6 2.4 5 424 + 27 12 3 2. 5 4 4 × 5 = 3 2 = 1 3 \ 13.56 × 2.4 = 32.544 Practise Now 7a (b)  14  4 1 21 ÷  –2  = ÷ –   5  4 4  5 3  5  21 = × –  4 14  2  13 7 . 8 × 0 . 35 6 890 + 41 34 4 8. 2 3 0 \ 137.8 × 0.35 = 48.23 15 =– 8 7 = –1 8 1 2 9 × =6 3 4 Practise Now 7a 5 (a) 5 5 3 11 +  –  –  – 2  = –2 4 12  6  3 Practise Now 6 Practise Now 6 (b) 4 9 3 1 +  –3  = 3 10 2  5 (b) –2 = (a) 2 6 1  7  6 1 3 ×  +  –   = × – 5 2   4  5 2  4   7  12 5  = ×  – 4  10 10  7 7 = × 4 10 49 = 40 9 =1 40 44 (b) 4 1 5 2 ÷ =1 6 2 3 Practise Now 9 Exercise 2A  0.92 (a) 0.92 ÷ 0.4 = 0.4 1. (i) 10 001, 4 (ii) –12, –2017 1 5 1 (iii) , 4.33, 10 001, 4 (iv) – 0.3, – , –12, –1 , –2017 5 7 2 2. (a) 30 m above sea level (b) –35 (c) An anticlockwise rotation of 30° (d) A speed of 45 km/h of a car travelling West 3. (a) < (b) < (c) < (d) > (e) < (f) > 4. (a)  9.2 = 4 2.3 4 ) 9.2 –8 12 –12 0 \ 0.92 ÷ 0.4 = 2.3  1.845 (b) 1.845 ÷ 0.15 = 0.15  184.5 = 15 12 . 3 15 ) 1 8 4 . 5 –15 34 –30 45 –45 0 –4 – 2.8 –4 –3 (b) 2 0 –2 0 –1 – –2 1 10 –1 2 1 2 5 6 3 2 5 4 4 0 1 2 3 4 –3 –2 –1 0 1 2 3 –3 –2 –1 0 1 2 3 –2 6 – 0.55 (c) \ 1.845 ÷ 0.15 = 12.3 –4 Practise Now 10 (d) 1 (a) 32 – (–1.6) = 32 + 1.6 = 33.6 (b) 1.3 + (–3.5) = –2.2   –0.23  0.12 1.2  –0.23  (c) ×  = ×  0.6   0.6  0.4 4 0 1 –13   –20 –0.23  = 1 3 ×   6 2  3 4 5 6 7 8 9 2 3 4 5 6 7 8 9 10 0 20 40 80 100 120 140 160 180 200 220 240 \ –13, –3, 23, 30, 230 – (b) 3 20 –10 – 0.65 60 30  5 45 – 27 230 23 –3 = –0.115  45  4.5  (d) – 0.32 ×  – 0.65 – 0.65 = – 0.32 × –27  –2.7  0.03 2 5. (a)   –0.23  = 0.3 ×   0.6  = – 0.09 × 4 15 150 31 = 0.15 – 0.65 = – 0.5 –20 0 20 40 60 80 100 120 140 160 – 0.5 \ –10, – 0.5, – Practise Now 11 π × 0.7 2 = 0.583 (to 3 d.p.) 3 3 2.4 + 1 10 6. (a) (b) 7. (a) (c) 45 –273.15 °C –86 m > < 3 , 15, 150 20 (b) > (d) > 1 8. (a) – 1 3 0.11 1 1 3 8 2.5 4. – 0.5 0 0.5 1 1.5 2.5 2 – 0.2 (b) 1 0 3 1 2 3 4 (c) 5 7 5 6 7 2 3 2 3 (d) 0 9 8 1 2 1 2 15 17 19 9 10 11 12 13 14 15 16 17 18 19 20 5 4 13 11 5 5. 7 6 7 8 9 10 4 3 4 5 Exercise 2B 1. (a) (b) (c) (d) (e) (f) (g) (h) 2. (a) (b) (c) (d) (e) (f) (g) (h) 3. (a) (b) (c) (d) (e) (f) (g) 6 + (–2) = 4 –5 + 8 = 3 4 + (–10) = – 6 –1 + (–7) = –8 9 + (–3) = 6 –11 + (–5) = –16 –10 + 2 = –8 1 + (–8) = –7 –(–7) = 7 5 – (–3) = 5 + 3 = 8 – 4 – 7 = –11 –8 – (–2) = –8 + 2 = –6 –1 – (–10) = –1 + 10 = 9 6 – 9 = –3 –8 – 3 = –11 2 – (–7) = 2 + 7 = 9 4 + (–7) – (–3) = 4 + (–7) + 3 = 0 –3 – 5 + (–9) = –17 1 – 8 – (–8) = 1 – 8 + 8 = 1 –2 + (–1) – 6 = –9 8 – (–9) + 1 = 8 + 9 + 1 = 18 –5 + (–3) + (–2) = –10 6 + (–5) – (–8) = 6 + (–5) + 8 = 9 1 6. 7. 8. 9. (h) 2 – (–7) – 8 = 2 + 7 – 8 = 1 (a) 23 + (–11) = 12 (b) –19 + 12 = –7 (c) 17 + (–29) = –12 (d) –21 + (–25) = – 46 (e) –13 + 18 = 5 (f) –24 + (–13) = –37 (g) 16 + (–27) = –11 (h) –26 + 14 = –12 (a) 22 – (–13) = 22 + 13 = 35 (b) –14 – 16 = –30 (c) –19 – (–11) = –19 + 11 = –8 (d) –18 – (–22) = –18 + 22 = 4 (e) 17 – 23 = –6 (f) –20 – 15 = –35 (g) 12 – (–17) = 12 + 17 = 29 (h) –21 – 17 = –38 Temperature in the morning = –11 °C + 7 °C = – 4 °C Point A shows –7 °C. Point B shows 16 °C. Difference in temperature = 16 °C – (–7 °C) = 16 °C + 7 °C = 23 °C Altitude of town = –51 m Difference in altitude = 138 m – (–51 m) = 138 m + 51 m = 189 m (i) Difference between –2 and 3 = 3 – (–2) =3+2 =5 (ii) The timeline for BC and AD does not have a zero while the number line has a zero. (iii) There are 4 years between 2 BC and 3 AD. Note: As there is no zero on the timeline, we cannot use 3 – (–2) to find the difference between 2 BC and 3 AD. In fact, the calculation should be 3 – (–2) – 1, provided one year is in BC and the other year is in AD. If both are in BC, or both are in AD, the calculation is the same as that in (i). (iv) A real-life example is the floors in a building, i.e. we can consider B1 (Basement 1) as –1 but there is no floor with the number 0. Exercise 2C 1. (a) (b) (c) (d) 46 3 × (–9) = –27 –8 × 4 = –32 –7 × (–5) = 35 –1 × (–6) = 6 (e) (f) 2. (a) (b) (c) (g) 3 × (–3)2 – (7 – 2)2 = 3 × (–3)2 – 52 = 3 × 9 – 25 = 27 – 25 =2 (h) 5 × [3 × (–2) – 10] = 5 × (–6 – 10) = 5 × (–16) = –80 2 (i) –12 ÷ [2 – (–2)] = –12 ÷ [4 – (–2)] = –12 ÷ (4 + 2) = –12 ÷ 6 = –2 –2(–7) = 14 –6 × 0 = 0 –21 ÷ 7 = –3 16 ÷ (–2) = –8 –8 ÷ (–2) = 4 –14 = –7 2 15 (e) = –3 –5 –18 (f) =6 –3 (d) 3. (a) Square roots of 81 = ± 81 = ±9 (j) = (b) Square roots of 16 = ± 16 = ±4 (c) Square roots of 25 = ± 25 = ±5 8. (a) (b) (c) (d) (e) (f) (g) (h) (i) (d) Square roots of 100 = ± 100 = ±10 81 = 9 4. (a) 4 =2 (b) (c) – 9 = –3 (d) 5. (a) (b) (c) (d) 6. (a) Not possible (–2)3 = –8 (–5)3 = –125 (–10)3 = –1000 (– 6)3 = –216 3 27 = 3 (b) – 64 = – 4 3 10 – (–6) 10 + 6 = 16 =4 –55 + (–10) – 10 = –75 –12 – [(–8) – (–2)] + 3 = –3 –100 + (– 45) + (–5) + 20 = –130 –2 + 3 × 15 = 43 (–5 – 2) × (–3) = 21 –25 × (– 4) ÷ (–12 + 32) = 5 3 × (–3)2 – (7 – 2)2 = 2 5 × [3 × (–2) – 10] = –80 –12 ÷ [22 – (–2)] = –2 (j) 10 – 3 × (–2) = 4 9. (a) 24 × (–2) × 5 ÷ (– 6) = – 48 × 5 ÷ (–6) = –240 ÷ (–6) = 40 (b) 4 × 10 – 13 × (–5) = 40 – (–65) = 40 + 65 = 105 (c) (16 – 24) – (57 – 77) ÷ (–2) = (–8) – (–20) ÷ (–2) = (–8) – 10 = –18 (d) 160 ÷ (– 40) – 20 ÷ (–5) = – 4 – (– 4) = –4 + 4 =0 (e) [(12 – 18) ÷ 3 – 5] × (– 4) = (–6 ÷ 3 – 5) × (– 4) = (–2 – 5) × (– 4) = (–7) × (– 4) = 28 (f) {[(–15 + 5) × 2 + 8] – 32 ÷ 8} – (–7) = {[(–10) × 2 + 8] – 32 ÷ 8} – (–7) = [(–20 + 8) – 32 ÷ 8] – (–7) = [(–12) – 32 ÷ 8] – (–7) = [(–12) – 4] – (–7) = (–16) – (–7) = (–16) + 7 = –9 3 (c) 10 – 3 × (–2) = 8 =2 (d) –216 = –6 7. (a) –55 + (–10) – 10 = –65 – 10 = –75 (b) –12 – [(–8) – (–2)] + 3 = –12 – [(–8) + 2] + 3 = –12 – (–6) + 3 = –12 + 6 + 3 = –6 + 3 = –3 (c) –100 + (– 45) + (–5) + 20 = –145 + (–5) + 20 = –150 + 20 = –130 (d) –2 + 3 × 15 = –2 + 45 = 43 (e) (–5 – 2) × (–3) = (–7) × (–3) = 21 (f) –25 × (– 4) ÷ (–12 + 32) = –25 × (– 4) ÷ 20 = 100 ÷ 20 =5 3 47 1 (g) (5 – 2)3 × 2 + [– 4 + (–7)] ÷ (–2 + 4)2 = 33 × 2 + (–11) ÷ 22 = 27 × 2 + (–11) ÷ 4 3  = 54 +  –2   4 = 51 (b) 3 1 2 8  =  2 + 8  + 8 – 8 7 = 2 8 1 1 2 5 (c) 5 – 4 = 5 –4 5 2 10 10 2 5 10   + = –4 4+ 10 10  10  7 = 10 2 1 1 2 (d) –3 +  – 4  = –3 – 4 3 6 6 3  1 4 = –3 – 4 6 6 5 = –7 6 3 1 1  2. (a) – +  –  = –1 2 4  4 1 4 (h) {–10 – [12 + (–3)2] + 33} ÷ (–3) = [–10 – (12 + 9) + 33] ÷ (–3) = (–10 – 21 + 33) ÷ (–3) = (–10 – 21 + 27) ÷ (–3) = (–31 + 27) ÷ (–3) = (– 4) ÷ (–3) 10. (a) (b) (c) (d) (e) (f) 1 = 1 3 24 × (–2) × 5 ÷ (–6) = 40 4 × 10 – 13 × (–5) = 105 (16 – 24) – (57 – 77) ÷ (–2) = –18 160 ÷ (– 40) – 20 ÷ (–5) = 0 [(12 – 18) ÷ 3 – 5] × (– 4) = 28 {[(–15 + 5) × 2 + 8] – 32 ÷ 8} – (–7) = –9 (g) (5 – 2)3 × 2 + [– 4 + (–7)] ÷ (–2 + 4)2 = 51 (h) {–10 – [12 + (–3)2] + 33} ÷ (–3) = 1 11. 3 1  1 1 1 + – = 3 – 8  4  8 4 1 2 = 3 – 8 8 1 4 1 3 –2 × (–6.5) – [–2 × (–3) + 8 × (–2) – 8 × 2] + 5 2 3 = –2 × (–6.5) – [6 + (–16) – 16] + 5 2 (b) 3  1 1 7 + –  =2  4 8 8 (c) 5 1 1 7 –4 = 5 2 10 2 1 5 +  – 4  = –7 3 6 6  5 15 41 5 3. (a) × = 31 2 2 8 (d) –3 3 –2 × (–6.5) – (–10 – 16) + 5 2 = 3 –2 × (–6.5) – (–26) + 5 2 = 1 = 2 2 3 –2 × (–6.5) – (–26) + 25 = 3 = 13 – (–26) + 25 (b) 2 3 = 13 + 26 + 25 3 15 × = 5 26 3 = 39 + 25 3 = 64 = 4 (c) 15 5 ÷ 4 2 Exercise 2D 1. (a) – 1  3 1 3 = – – + – 2  4  2 4 2 3 = – – 4 4 –2 – 3 = 4 –5 = 4 1 = –1 4 1 (d) 1 7 4 ÷ 9 3 15 13 × 5 26 2 1 3 = 2 1 = 1 2 3 15 = 2 4 3 = 2 1 = 1 2 16 = ÷ 9 4 16 = 3 9 4 = 3 1 = 1 3 48 3 1 × 21 51 4 3 × 31 41 8 5. (a) 5 7. (a) 4 1 =2 3 2 15 1 × =1 26 2 5 1 ÷ =1 2 2 4 1 ÷ =1 3 3 × \ 14.72 × 1.2 = 17.664 (b)  3  64 8 × –  =– 5  81  15 3 5 \ 130.4 × 0.15 = 19.56  31  4 × –   10 5  5 15 2 = – 25 7 3 49 1 3 (c) – 6 × =– × 8 14 8 14 2 21 = – 16 5 = –1 16 4 ÷  – 10  = 15  3  2 (c) × (d) 11 (d) –2 1 22 5 1 2 ×4 =– × 51 2 5 1 2 (e) –1 1 3 5 3 ÷ =– ÷ 4 8 4 8 5 82 = – × 3 1 4 10 = – 3 1 = –3 3 2 8 8  5  (f) – ÷  –1  = – ÷  –  3  3  9 9  2.7 3 ) 8.1 –6 21 –2 1 0 (e) (f) \ 0.81 ÷ 0.3 = 2.7  1.32 (b) 1.32 ÷ 0.12 = 0.12  (d) \ 0.27 × 0.08 = 0.0216 0 . 25 × 1 . 96 150 225 + 25 0 . 4900 8.1 = 3  31  8 = – × –  9  5  3 8 = 15 64 3  3 6. (a) ×  –  = –1  8 15 5 (c) 0 . 27 0 . 08 0. 0 2 1 6 \ 0.25 × 1.96 = 0.49  0.81 8. (a) 0.81 ÷ 0.3 = 0.3 = –11 (b) 13 0 . 4 0 . 15 6 520 +13 0 4 1 9. 5 6 0 × 1 = –1 (b) 14 . 72 1.2 2 944 + 14 72 17.664 × 132 = 12 = 11 4 2 ÷  – 10  = – 15 25  3  1 3 5 –6 × = –1 8 14 16 1 2 –2 × 4 = –11 2 5 1 3 1 –1 ÷ = –3 4 8 3 8 8 2 – ÷  –1  =  3  9 15 (c) 3.426 ÷ 0.06 =  3.426 0.06  15 8 3 (b) 2 5 15 (c) 4 7 (d) 1 9 4. (a) 342.6 = 6 6 5 7.1 ) 3 4 2.6 –30 42 –42 6 – 6 0 49 \ 3.426 ÷ 0.06 = 57.1 1  4.35 1.5  4 1 4  1 (d) – 4 +  –3  +  –  = – 4 – 3 –  3 8 3  8 1 1 = –4 – 3 – 1 8 3 3 8 = –4 – 3 –1 24 24 11 = –8 24 7 1 1 1 1 7 (e) – + 2 +  –  = – + 2 – 5 4 5 4 2  2 1 9 7 =– + – 5 4 2 4 45 70 =– + – 20 20 20 – 4 + 45 – 70 = 20 –29 = 20 9 = –1 20  (d) 4.35 ÷ 1.5 = 43.5 = 15 2.9 15 ) 4 3.5 –30 135 –135 0 \ 4.35 ÷ 1.5 = 2.9 9. (a) 4.3 – (–3.9) = 4.3 + 3.9 = 8.2 (b) 2.8 + (–1.5) = 1.3 (c) –5.9 + 2.7 = –3.2 (d) – 6.7 – 5.4 = –12.1 10. (a) – (b) 6 (c) 4 1 8 1 8 1 1 –  –2  – = – +2 – 5 2 5 4 2  4 8 9 1 =– + – 5 4 2 32 45 10 =– + – 20 20 20 –32 + 45 – 10 = 20 3 = 20 11. (a) – 1  3 1 1 3 1 –  –  +  – 4  = 6 + –4  4 5 5 4 10  10  31 3 41 = + – 5 4 10 124 15 82 = + – 20 20 20 124 + 15 – 82 = 20 57 = 20 17 =2 20 1    3 1 17 –  –  +  –4  = 2  10   4 5 20 (c) 4 1   4  2 6 +  –6 3  –  –  = –1  21  7 7 (e) – 12. (a) –  7 1 1 9 + 2 +  –  = –1  2 5 4 20  28 5  2 5 5  28 × – + +1  = – ×  –  15 3  3 7 7  15 =–  28 25  5 + × –  15 15  7 =– 5 3 ×  –  7  15  =– 1 =  1  5 × –  7  51  1 7  1 1   (b)  – 1 –  – 1   ÷  1 – 1  =  – +  ÷  4 3 4 3 4 3       3 4  =  – +  12 12  90 – 133 + 4 21 = –39 21 –13 = 7 6 = –1 7 1 (b) 6 1 11  4 (d) – 4 +  – 3  +  –  = –8 24  8  3  4  1 2 2 1 4  +  –6  –  –  = 4 – 6 +  21  3  7 7 3 21 30 19 4 = – + 7 3 21 90 133 4 = – + 21 21 21 = 1 8 1 3 –  –2  – = 4 5 2 20  = =  1 1  4 – 3  ÷  3 – 4   12 12  1  1  ÷ –   12  12 1 1 × (– 12 ) 1 12 = –1 50 (c) 10 – 3 1 15  1 ×  ÷ 4  +  –  2  4 8 2 = 10 –  3 9 15  1 ×  ÷  + –   2 2  4 8 = 10 –  13 21  15  1 × ×  + –  9 3   4 8  12   (b)  – 1 –  – 1   ÷  1 – 1  = –1 4 3      4 3 5 2 (e) 2   ×  –0.16  = 0.3  1.2    –0.16  = 13 ×    12 4  = – 0.04  0.027  1.4   1.4  2.7 = (b) ×  ×  0.03  –0.18   –0.18  3   ×  1.4  = 0.9  –0.18      –1.3  (c) – 0.42 ×  0.8     1 4 1 1 1 4 1 + ×  –  = + × 9 3 9 3 41  2 1 1 = + 3 9 3 1 = + 9 9 4 = 9  14  = 19 ×    – 18 2  = –7  –13  – 0.62 = – 0.42 ×  8  – 0.62   0.02  –13  = – 0.16 ×  – 0.62  81  = 0.26 – 0.62 = – 0.36  27 270 (d) (– 0.2)3 × + 0.105 = (– 0.2)3 × + 0.105 1.6 16  2 1 1 4 4 + ×  –  = 3 9 9  2 3 1 1 1 (f)   ×  – 2  = –5 3 10  2  15   –0.16  1.5  –0.16  = 14. (a) 0.15 ×  ×   1.2   1.2  5 0.5 2 (e) 2 3 3 3 1 3 (d)   –   +  –  = –1 16  4  4  2 15 1  1 + –  × 8  4 31 1 5   + –  = 10 –  4 8 5 1 – = 10 – 8 4 5 2 = 10 – – 8 8 8 5 2  =  9 + 8  – 8 – 8 1 = 9 8 3 2  1  3  3  3 1 9 (d)   –   +  –  = – + –   2  4  4  4 8 16 1 9 3 = – – 8 16 4 2 9 12 = – – 16 16 16 2 – 9 – 12 = 16 –19 = 16 3 = –1 16 = 10 – 1  1 3 15 1 ×  ÷ 4  + –  =9 2  4 2 8 8 (c) 10 – 0.001 = – 0.008 × 2 = – 0.135 + 0.105 = – 0.03 1  3 7  1  1  3 –2  =   ×  – (f)   ×  3  2   15  15 3   2 35   1  3 – =   ×   15 15   2 2  1  π + 52  15. (a)   = 16.934 (to 3 d.p.)  –2.1  2  3  34  =   × –   2  15  17 9  34 ×–  15 5 2 4 51 =– 10 1 = –5 10 5 1 13. (a) – ×  – 28 + 1 2  = 7 7 3  15 = 3 135 270 + 0.105 16 8 1 2    (b) – π2 + 7– 3 (c) (d) 51 2 3 4 = –2.085 (to 3 d.p.) 14 2 + 19 2 = –5.842 (to 3 d.p.) π – 4.55  1 4.6 2 + 8.32 –  6   2 2 × 4.6 – 8.3 2 = 7.288 (to 3 d.p.) 1 (b) 2.36 – 10.58 = –8.22 –11.97 – (–2.69) = –11.97 + 2.69 = –9.28 \ 2.36 – 10.58 > –11.97 – (–2.69) (c) –5 × 1.5 = –7.5 50 ÷ (–8) = – 6.25 \ –5 × 1.5 < 50 ÷ (–8) 3   1 1 3 (d) 7 –  –3  = 7 + 3  10  5 5 10 2 3 = 7 +3 10 10 5 = 10 10 1 = 10 2 1   2 2 1 19 +  –8  = 19 – 8 10   5 5 10 4 1 = 19 –8 10 10 3 = 11 10 1  3   1 2  \ 7 –  –3  < 19 +  –8  10    10  5 5 29 2. (a) 3 16. Amount of time Nora spent on visiting old folks’ homes 4 1 =× 8 7 16 1 4 = × 129 7 16 4 129 = 28 17 = 4 hours 28 7    23  3 5 3 5 23 7 17. 5 – 2 +  –  –  – 4  = 5 – 2 – +4  10   15  4 6 4 6 15 10 23 17 23 47 = – – + 4 6 15 10 345 170 92 282 = – – + 60 60 60 60 345 – 170 – 92 + 282 = 60 365 = 60 73 = 12 1 =6 12 18. Fraction of sum of money left after Farhan has taken his share = 1– 4 = 5 Fraction of sum of money left after Khairul has taken his share 1  4 =  1 – 3  × 5 2 4 =× 3 5 8 = 15 Fraction of sum of money left after Huixian has taken her share –3 –2 \ 5.5, 4, –1 33 4 0 –8 –2p –8 –6 3. (a) 3 82 = × 4 15 5 1 (b) 2 = 5 (c) 1  2 Fraction of sum of money taken by Jun Wei =  1 –  × 7  5 6 2 = × 7 5 12 = 35 (d) 4. (a) (b) Review Exercise 2 (c) 1. (a) –7 – 38 = – 45 8 + (–55) = – 47 \ –7 – 38 > 8 + (–55) 52 2 5 8 –4 3 4 5 –2 0 10 5.855 2 1 5 , 5.855, , –2p, –8 2 8 13 – (–54) = 13 + 54 = 67 (–74) – (– 46) = –74 + 46 = –28 11 + (–33) – (–7) = –22 – (–7) = –22 + 7 = –15 –13 + (–15) + (–8) = –28 + (–8) = –36 –12 × 7 = –84 4 × (–5) × (– 6) = –20 × (– 6) = 120 – 600 ÷ 15 = – 40 \ 10 1 1 5.5 4 6 29 3 , – , –2.365 33 4 (b) 1 8  =  1 – 4  × 15 1 – –2.365 1 5 4 6 8 10 1 2 12 50 ÷ (–5) 8 25 =– ÷ (–5) 4 (d) 50 ÷ (–8) ÷ (–5) = – 6. –18 –  3 –(33 × 5 3 ) – (– 6)2  –18 –  3 –3375 – (– 6)2  = 4+9 4+9 –18 – [–(3 × 5) – 36] 2+9 –18 – (–15 – 36) = 11 –18 – (–51) = 11 –18 + 51 = 11 33 = 11 =3 = 25  1  =– ×–  4  51  5 = 4 1 =1 4 (–3 – 5) × (–3 – 4) = (–8) × (–7) = 56 4 × (–5) ÷ (–2) = –20 ÷ (–2) = 10 –5 × 6 – 18 ÷ (–3) = –30 – (– 6) = –30 + 6 = –24 2 × (–3)2 – 3 × 4 = 2 × 9 – 3 × 4 = 18 – 12 =6 –3 × (–2) × (2 – 5)2 = –3 × (–2) × (–3)2 = –3 × (–2) × 9 =6×9 = 54 (–2)2 – (–2) × 3 + 2 × 32 = 4 – (–2) × 3 + 2 × 9 = 4 – (– 6) + 18 = 4 + 6 + 18 = 10 + 18 = 28 (– 4)2 ÷ (–8) + 3 × (–2)3 = 16 ÷ (–8) + 3 × (–8) = (–2) + (–24) = –26 4 × 32 ÷ (–6) – (–1)3 × (–3)2 = 4 × 9 ÷ (–6) – (–1) × 9 = 36 ÷ (–6) – (–1) × 9 = – 6 – (–9) = –6 + 9 =3 –2 × (–2)3 × (–2) × 3 + (–2) × 3 × (–1)2 = –2 × (–8) × (–2) × 3 + (–2) × 3 × 1 = 16 × (–2) × 3 + (– 6) = –32 × 3 + (– 6) = –96 + (– 6) = –102 5 – {12 × [(–5)2 – 7] ÷ 3} = 5 – [12 × (25 – 7) ÷ 3] = 5 – (12 × 18 ÷ 3) = 5 – (216 ÷ 3) = 5 –72 = – 67 5 5. (a) (b) (c) (d) (e) (f) (g) (h) (i) (j) 4 2 4 2 3  3 +1 – –  =3 +1 +  7 7 5 7 5 7 25 7 3 = + + 7 5 7 125 49 15 = + + 35 35 35 125 + 49 + 15 = 35 189 = 35 27 = 5 2 =5 5 2 3   2 2 3 2   (b) – –3 +3 – + –  = 20   5 3  3 20 5 2 63 2 = + – 3 20 5 40 189 24 = + – 60 60 60 40 + 189 – 24 = 60 205 = 60 41 = 12 5 =3 12 4 3 5 16 27 20 (c) – 6 – 3 – 3 = – 6 –3 –3 9 4 9 36 36 36 63 = –12 36 7 = –12 4 3 = –13 4 7. (a) 3 53 1 = –12 14 1 5 5 + ÷ (– 4) – ×  –  2 3 7  5  = –12 1 5  1 + × –  – 2 3  4 1 5 +  –  – (–2) 2  12  1 5 = –12 – +2 2 12 6 5 = –12 – +2 12 12 11 = –10 12 2 3 1 2   13 8 2  15  1 3 × 1 ×  –1  = – × ×–  13   4 5 51  13 1  1 4 =6 1 1   1 1 3 ×  – –  ÷  –2 + 1   4 6 4 5  3 8.  4  2  – 7  –  – 5  – 64 ÷ 625 3 – 3 8 125 = 598 1225 9. (a) –12.8 – 88.2 = –101 (b) 500.3 – (–200.2) – 210.1 = 500.3 + 200.2 – 210.1 = 700.5 – 210.1 = 490.4  1.44 (c) 1.44 ÷ 1.2 × (– 0.4) = × (– 0.4) 1.2 2  4 3   3  3 – ÷  –2 + 1  =×  – 12   12 12   12 5 1 3  5  = ×  –  ÷  –1  12   12  5   = 14.4 × (– 0.4) 12 = 1.2 × (– 0.4) = – 0.48 (d) (– 0.3)2 ÷ (– 0.2) + (–2.56) = 0.09 ÷ (– 0.2) + (–2.56)  0.09 = + (–2.56) –0.2 3  5  13 = ×  –  ÷  –   12  5  12  1 1  5   12  3 = ×–   ×– 13   12 1   1 5 3 = 13 3   7 9 3 1 57 19 1 (g) –3 ÷1 – ×  –1  = – ÷ – × –  4   4 16 16 3 16 16 3 3  =– = 1 0.9 + (–2.56) –2 = – 0.45 + (–2.56) = – 0.45 – 2.56 = –3.01 57 16 1  7 × – ×–  19 1 3  4  1 16 7 = –3 –  –   12  7 = –3 + 12 5 = –2 12 Challenge Yourself 1. Since x – 3 and (y + 2)2 cannot be negative, x – 3 = 0 and x – 3 = 0 and \ x = 3 and 2. (a) 324 × 57 2268 +1620 18468 (y + 2)2 = 0 y+2 =0 y = –2 (b) 3. (b) (3 + 3) ÷ 3 + 3 – 3 = 2 (d) (3 + 3 + 3 + 3) ÷ 3 = 4 (f) 3 + 3 + (3 – 3) × 3 = 6 1  14 2  5 × –   51  1 7 1 = –12 (e) –3 (f)  4 1 2 5 + 1 ÷ (– 4) – ×  –2   5 2 3 7 (h) –12 1  1   1 1  1  (d)  – +  +  +  –   +  –  4 3  2 3    20    1 1 1 1 1 = – + + – – 2 3 4 3 20 1 1 1 1 1 = – + – + – 2 3 3 4 20 1 1 1 = – + – 2 4 20 10 5 1 = – + – 20 20 20 –10 + 5 – 1 = 20 –6 = 20 3 = – 10 54 48 28 ) 1 3 4 4 –112 224 –224 0 (c) 3 + 3 – 3 – 3 + 3 = 3 (e) 3 + 3 ÷ 3 + 3 ÷ 3 = 5 Chapter 3 Approximation and Estimation TEACHING NOTES Suggested Approach Teachers can give students a real-life example when an approximated or estimated value is used before getting them to discuss occasions when they use approximation and estimation in their daily lives. In this chapter, they will first learn the five rules to identify the digits which are significant in a number before learning how to round off numbers to a specified number of significant figures. Students will also learn how to carry out estimation through worked examples that involve situations in real-world contexts. Section 3.1: Approximation To make learning of mathematics relevant, students should know some reasons why they need to use approximations in their daily lives (see Class Discussion: Actual and Approximated Values). Teachers should do a recap with students on what they have learnt in primary school, i.e. how to round off numbers to the nearest tenth, whole number and 10 etc. Section 3.2: Significant Figures Through the example on measuring cylinders on page 63 of the textbook, students will learn that a number is more accurate when it is given to a greater number of significant figures. After learning how to round off numbers to a specified number of significant figures, teachers can arouse students’ interest in this topic by bringing in real-life situations where they cannot just round off a number using the rules they have learnt (see Investigation: Rounding in Real Life). The journal writing on page 67 of the textbook requires students to cite examples of such situations. Section 3.3: Rounding and Truncation Errors Teachers should tell students that the general instructions for O-level Mathematics examinations state, ‘If the degree of accuracy is not specified in the question, and if the answer is not exact, give the answer to three significant figures. Give answers in degrees to one decimal place.’ The investigation on page 68 of the textbook highlights the importance of giving intermediate values correct to four significant figures if we want the final answer to be accurate to three significant figures. Otherwise, a rounding error may occur. Students should also learn that there is a difference between ‘approximately 2.5 million’ and ‘equal to 2.5 million (to 2 s.f.)’ (see the thinking time on page 69 of the textbook). Teachers should tell students the difference between rounding off a number to, say, 3 significant figures and truncating the same number to 3 significant figures. The investigation on page 70 of the textbook enables students to find out more about rounding and truncation errors in calculators. Section 3.4: Estimation Teachers can impress upon students that there are differences between approximation and estimation. Since students need to be aware when an answer is obviously wrong, estimation allows them to check the reasonableness of an answer obtained from a calculator (see Worked Example 6). Students will also learn an important estimation strategy: use a smaller quantity to estimate a larger quantity (see Investigation: Use of a Smaller Quantity to Estimate a Larger Quantity). Teachers should get students to work in groups to estimate quantities in a variety of contexts, compare their estimates and share their estimation strategies with one another. (see the performance task on page 76 of the textbook). 55 1 WORKED SOLUTIONS Journal Writing (Page 67) Class Discussion (Actual and Approximated Values) • 1. The actual values indicated in the article include ‘42 038 777 passengers’, ‘13 .0%’, ‘24 awards’ and ‘four terminals’ while approximated values include ‘over 360 awards’ and ‘73 million passengers’ . Actual values are exact numbers while approximated values are values which are usually rounded off . 2. (a) It is not necessary to specify the actual number of awards won, as an approximation is sufficient to show that Changi Airport has won many awards . (b) A headline serves as a brief summary of the article to draw readers’ attentions, thus it is more appropriate to use an approximated value instead of the actual value . • Investigation (Rounding in Real Life) Teachers may wish to note that the list is not exhaustive. Scenario 1 Total number of passengers = 215 + 5 = 220 Investigation (The Missing 0.1% Votes) 1. The percentage of votes for each candidate given is correct to 3 significant figures. Due to rounding errors in the intermediate steps, there is a follow-through error, resulting in the missing 0 .1% of the votes. If the final answer is correct to 2 significant figures, we will obtain 100%. Hence, the final answer can only be accurate to 2 significant figures. Number of buses required = 220 ÷ 30 =7 1 3 1 is 7 . However, 7 buses are not enough 3 to carry 220 passengers, thus we round up to find the number of buses required to carry all the passengers . \ The number of buses required is 8 . The nearest whole number to 7 188 × 100% 301 = 62 .5% (to 3 s .f .) 2. Percentage of votes for Vishal = 52 × 100% 301 = 17 .3% (to 3 s .f .) Scenario 2 Maximum mass of lift = 897 kg = 900 kg (to the nearest 100 kg) Percentage of votes for Rui Feng = 61 × 100% 301 = 20 .3% (to 3 s .f .) Total percentage of votes = 62 .5% + 17 .3% + 20 .3% = 100 .1% Percentage of votes for Huixian = If the maximum mass of the lift is given as 900 kg, it means that the lift is able to carry a mass of  900 kg . However, the maximum mass allowed is only 897 kg . \ The maximum mass of the lift should be given as 800 kg . The percentage of votes for each candidate given is correct to 3 significant figures. Due to rounding errors in the intermediate steps, which results in a follow through error, the total percentage of votes is 100.1%. If the final answer is correct to 2 significant figures, we will obtain 100%. Hence, the final answer can only be accurate to 2 significant figures. Scenario 3 In Singapore, the issue of 1-cent coins has ceased since 2002; while the coins are legal tender and are still in circulation, most shops have stopped accepting 1-cent coins . As such, when people wish to pay for their purchases in cash, the prices of their purchases have to be rounded off to the nearest 5 cents which is now considered to be the smallest denomination of currency in Singapore . Thinking Time (Page 69) Teachers may wish to ask students to explain why when other methods of payment are used, it is not necessary to round off the prices of their purchases to the nearest 5 cents. 1 A developer wants to build a house on a plot of land that has a height restriction of 10 m. The height from the floor to the ceiling of each level is about 2 .6 m . Number of levels the developer can build = 10 m ÷ 2 .6 m = 3 .85 (to 3 s .f .) The nearest whole number to 3 .85 is 4 . However, a house with 4 levels will be taller than 10 m, and thus will go against the height restrictions . Hence, the maximum number of levels that the developer can build is 3 . The boiling point of oxygen, i.e. the temperature at which liquid oxygen boils to form gaseous oxygen, is –183 °C. The maximum temperature, correct to the nearest 10 °C, at which liquid oxygen can be stored is –190 °C as oxygen will be in its gaseous state at a temperature of –180 °C. 1. (i) When the population of City A is approximately 2 .5 million, it is possible for the exact population size to be 2 .47 million . (ii) When the population of City A is approximately 2 .5 million, it is possible for the exact population size to be 2 .6 million . 56 2. (i) When the population of City B is equal to 2 .5 million (to 2 s .f .), it is possible for the exact population size to be 2 .47 million as it is equal to 2.5 million when rounded off to 2 significant figures. (ii) When the population of City B is equal to 2 .5 million (to 2 s .f .), it is not possible for the exact population size to be 2 .6 million as it is still equal to 2.6 million when rounded off to 2 significant figures. Performance Task (Estimation in Our Daily Lives) 1. Use surveys, questionnaires or verbal questioning to find out the number of hours spent surfing the Internet by each student in the class on a weekday and on a Saturday or Sunday . Ensure that students have a common understanding of the phrase ‘surfing the Internet’. Calculate the total number of hours spent surfing the Internet by all the students in the class on a weekday and on a Saturday or Sunday . Note: There is a difference between ‘approximately 2 .5 million’ and ‘equal to 2 .5 million (to 2 s .f .)’ . Total amount of time spent surfing the Internet by all the students in the class on a weekday = x hours Investigation (Rounding and Truncation Errors in Calculators) Total amount of time spent surfing the Internet by all the students in the class on a Saturday or Sunday = y hours For this activity, the calculator model used is SHARP EL-509VM. (a) 1. 0 .727 922 061 2. 7 .27 922 061 3 3. 2 .7 922 061 3 The calculator stores 12 digits . Estimate the total number of hours spent surfing the Internet by all the students in the class in a month . Assume that the average number of weekdays and the average number of Saturdays and Sundays in a month are 22 and 8 respectively . Total amount of time spent surfing the Internet by all the students in the class in a month ≈ (22x + 8y) hours The calculator truncates the value of 162 at the 12th digit to give 12 .727 922 061 3, instead of rounding 162 to 12 .727 922 061 4 . (b) 5. 6 .999 999 999 2. Assume that there are 8 slices in a large pizza . Use verbal questioning to find out the number of slices needed to feed one class (e .g . about 40 students) in the school when they go for an excursion . Investigation (Use of a Smaller Quantity to Estimate a Larger Quantity) Number of slices needed to feed one class in the school = x For this investigation, the smaller box used is of length 9.2 cm, width 5.6 cm and height 2.7 cm. x 8 Find out the number of classes in the school . Ensure that there is approximately the same number of students in each class, e .g . 40 students . Number of pizzas needed to feed one class in the school = Three trials are carried out to find the average number of 10¢ coins that can fill the box. The result of each trial is shown in the table. Trial Number of 10¢ coins 1 294 2 280 3 284 Number of classes in the school = y Estimate the amount of pizza needed to feed all the students in the school during an excursion . Total number of pizzas needed to feed all the students in the school xy ≈ 8 Average number of 10¢ coins that can fill the smaller box 294 + 280 + 284 3 858 = 3 = 286 = 3. Find out the opening hours of the drinks stall on a weekday and determine the durations of the peak (e .g . recess and lunchtime) and non-peak periods respectively . Duration of peak period = x hours Volume of smaller box = 9 .2 × 5 .6 × 2 .7 = 139 .104 cm3 Duration of non-peak period = y hours Volume of tank = 50 × 23 × 13 = 14 950 cm3 Find out the amount of money collected by the drinks stall in half an hour during the peak period and half an hour during the non-peak period . Number of 10¢ coins that can fill the tank = 286 × 14 950 139.104 = 30 737 (to the nearest whole number) \ Amount of money in the tank = 30 737 × 10¢ = $3073 .70 Amount of money collected by drinks stall in half an hour during peak period = $p Amount of money collected by drinks stall in half an hour during non-peak period = $q 57 1 Estimate the total amount of money collected for both the peak and non-peak periods . (c) The number 3.0021 has 5 significant figures. (d) The number 70.8001 has 6 significant figures. Total amount of money collected by drinks stall during peak period ≈ $2px Practise Now (Page 64) 1. (a) The number 0.10 has 2 significant figures. (b) The number 0.500 has 3 significant figures. (c) The number 41.0320 has 6 significant figures. (d) The number 6.090 has 4 significant figures. 2. 4.10 cm is more accurate because 4.10 cm is measured to 3 significant figures, while 4.1 cm is measured to 2 significant figures. Total amount of money collected by drinks stall during non-peak period ≈ $2qy Hence, Total amount of money collected by drinks stall on a weekday ≈ $(2px + 2qy) Practise Now (Page 65) (a) The number 0.021 has 2 significant figures. (b) The number 0.603 has 3 significant figures. (c) The number 0.001 73 has 3 significant figures. (d) The number 0.1090 has 4 significant figures. Practise Now 1 1. (a) 3 409 725 = 3 409 730 (to the nearest 10) (b) 3 409 725 = 3 409 700 (to the nearest 100) (c) 3 409 725 = 3 410 000 (to the nearest 1000) (d) 3 409 725 = 3 410 000 (to the nearest 10 000) 2. Largest possible number of overseas visitors = 11 649 999 Smallest possible number of overseas visitors = 11 550 000 Practise Now (Page 65) (a) 3800 m, which is corrected to the nearest 10 m, has 3 significant figures. (b) 25 000 km, which is corrected to the nearest km, has 5 significant figures. (c) 100 000 g, which is corrected to the nearest 10 000 g, has 2 significant figures. Practise Now 2 1. (a) 78 .4695 = 78 .5 (to 1 d .p .) (b) 78 .4695 = 78 (to the nearest whole number) (c) 78 .4695 = 78 .47 (to the nearest hundredth) (d) 78 .4695 = 78 .470 (to the nearest 0 .001) 2. No, I do not agree with Jun Wei . 8 .40 is rounded off to 2 decimal places which is more accurate than 8 .4 which is rounded off to 1 decimal place . Practise Now 4 3748 = 3750 (to 3 s .f .) 0 .004 709 89 = 0 .004 710 (to 4 s .f .) 4971 = 5000 (to 2 s .f .) 0 .099 99 = 0 .10 (to 2 s .f .) 0 .099 99 = 0 .100 (to 3 s .f .) 2. Since 67 0X1 (to 3 s .f .), then the possible values of X are 5, 6, 7, 8 or 9 . If 67 0X1 is a perfect square, then by trial and error, X = 8 . 1. (a) (b) (c) (d) Practise Now 3 Cost of 450 kWh of electricity = 450 × $0 .29 = $130 .50 Cost of 38 m3 of water = 38 × $1 .17 = $44 .46 Practise Now 5 Total amount of money the household has to pay = $130 .50 + $44 .46 = $174 .96 = $175 (to the nearest dollar) (i) Length of square = 105 = 10 .2 m (to 3 s .f .) (ii) Perimeter of square = 10 .25 × 4 = 41 .0 m (to 3 s .f .) Practise Now (Page 64) (a) (b) (c) (d) The number 192 has 3 significant figures. The number 83.76 has 4 significant figures. The number 3 has 1 significant figure. The number 4.5 has 2 significant figures. Practise Now 6 1. 798 × 195 ≈ 800 × 200 = 160 000 \ Nora’s answer is not reasonable . 2. (a) 5712 ÷ 297 ≈ 5700 ÷ 300 = 19 Using a calculator, 5712 ÷ 297 = 19 .2 (to 3 s .f .) . \ The estimated value is close to the actual value . Practise Now (Page 64) (a) The number 506 has 3 significant figures. (b) The number 1.099 has 4 significant figures. 1 58 (b) 63 × 3 129 ≈ 64 × =8×5 = 40 3 5. No, I do not agree with Kate . She needs to put a ‘0’ in the ones place as a place holder after dropping the digit ‘2’, i .e . 5192 .3 = 5190 (to the nearest 10) . 6. Largest possible value of Singapore’s population = 5 077 499 Smallest possible value of Singapore’s population = 5 076 500 7. No, I do not agree with Farhan . 27 .0 is rounded off to 1 decimal place which is more accurate than 27 which is rounded off to the nearest whole number . 125 Using a calculator, 63 × 3 129 = 40 .1 (to 3 s .f .) . \ The estimated value is close to the actual value . 250 80 240 ≈ hours 80 3. Time taken to drive from Singapore to Malacca = Exercise 3B 1. (a) The number 39 018 has 5 significant figures. (b) The number 0.028 030 has 5 significant figures. (c) 2900, which is corrected to the nearest 10, has 3 significant figures. 2. (a) 728 = 730 (to 2 s .f .) (b) 503 .88 = 503 .9 (to 4 s .f .) (c) 0 .003 018 5 = 0 .003 019 (to 4 s .f .) (d) 6396 = 6400 (to 2 s .f .) 6396 = 6400 (to 3 s .f .) (e) 9 .9999 = 10 .0 (to 3 s .f .) (f) 8 .076 = 8 .08 (to 3 s .f .) 3. Possible values of x = 4, 5 or 6 Practise Now 7 Rp 10 000 ≈ S$1 .50, so Rp 20 000 ≈ S$3, Rp 5000 ≈ S$0 .75 \ The price of the pair of earrings is Rp 25 000 ≈ S$3 .75 . Practise Now 8 For option A, 300 ml costs about $9 . Thus 100 ml will cost about $3, and 50 ml will cost about $1 .50 . \ For option A, 350 ml will cost about $9 + $1 .50 = $10 .50 . For option B, 350 ml costs $10 .40 which is $0 .10 cheaper than option A . However, for option A, 300 ml actually costs $8 .80 which is less than $9 . Thus for option A, 350 ml will cost at least $0 .20 less than the estimated $10 .50 . \ Option A is better value for money . 1 = 0 .010 10 (to 4 s .f .) 99 (b) 871 × 234 = 203 814 = 200 000 (to 2 s .f .) 4. (a) Practise Now 9 2 × 100% 3 2 = 66 % 3 Percentage of shaded region = (c) 212 = 2013 .698 63 0.219 = 2013 .7 (to 5 s .f .) 3.913 – 2.1 = 9 .0 (to 2 s .f .) 6.41 5. Greatest number of sweets that can be bought $2 = $0.30 = 6 (to the nearest whole number) (d) Exercise 3A 1. (a) (b) (c) 2. (a) (b) (c) 3. (i) (ii) 4. (a) (b) (c) 698 352 = 698 400 (to the nearest 100) 698 352 = 698 000 (to the nearest 1000) 698 352 = 700 000 (to the nearest 10 000) 45 .7395 = 45 .7 (to 1 d .p .) 45 .7395 = 46 (to the nearest whole number) 45 .7395 = 45 .740 (to 3 d .p .) Perimeter of land = 2(28 .3 + 53 .7) = 2(82) = 160 m (to the nearest 10 m) Area of grass needed to fill up the entire plot of land = 28 .3 × 53 .7 = 1519 .71 m2 = 1500 m2 (to the nearest 100 m2) 4 .918 m = 4 .9 m (to the nearest 0 .1 m) 9 .71 cm = 10 cm (to the nearest cm) $10 .982 = $11 .00 (to the nearest ten cents) (d) 6 .489 kg = 6 .49 kg (to the nearest 6. (i) Length of square = 264 = 16 .2 cm (to 3 s .f .) (ii) Perimeter of square = 16 .25 × 4 = 65 .0 cm (to 3 s .f .) 136 2p = 21 .6 m (to 3 s .f .) (ii) Area of circle = p(21 .65)2 = 1470 m2 (to 3 s .f .) 8. Since 21 X09 = 22 000 (to 2 s .f .), then the possible values of X are 5, 6, 7, 8 or 9 . If 21 X09 is a perfect square, then by trial and error, X = 6 . 9. Largest possible number of people at the concert = 21 249 Smallest possible number of people at the concert = 21 150 10. (i) 987 654 321 + 0 .000 007 – 987 654 321 = 0 .000 007 (ii) 987 654 321 + 0 .000 007 – 987 654 321 = 0 7. (i) Radius of circle = 1 kg) 100 59 1 10. KRW 900 ≈ S$1 \ Price of handbag = KRW 26 700 ≈ KRW 27 000 = 30 × KRW 900 ≈ 30 × S$1 = S$30 (iii) No, the answers for (i) and (ii) are different . This is because the calculator truncates the value of 987 654 321 + 0 .000 007 to give 987 654 321 . Hence, the answer for (ii) is 0 . Exercise 3C 1. 218 ÷ 31 ≈ 210 ÷ 30 =7 \ Priya’s answer is not reasonable . Using a calculator, 218 ÷ 31 = 7 .03 (to 3 s .f .) . \ The estimated value is close to the actual value . 2. (a) 2013 × 39 ≈ 2000 × 40 = 80 000 Using a calculator, 2013 × 39 = 78 507 . \ The estimated value is close to the actual value . (b) 145.6 ÷ 3 65.4 ≈ 144 ÷ = 12 ÷ 4 =3 3 Review Exercise 3 6479 .952 = 6500 (to the nearest 100) 6479 .952 = 6000 (to the nearest 1000) 6479 .952 = 6480 .0 (to the nearest tenth) 4 .793 = 4 .8 (to 2 s .f .) 39 .51 = 40 (to 2 s .f .) (ii) 4 .793 ÷ 39 .51 ≈ 4 .8 ÷ 40 = 0 .12 (to 2 s .f .) 3. Smallest possible mass of chocolate truffle = 0.0245 kg 4. Rp 10 000 ≈ S$1 .50, so Rp 30 000 ≈ S$4 .50, Rp 5000 ≈ S$0 .75 . \ The price of the toy is Rp 35 000 ≈ S$5 .25 . 5. Total mass = 3 × 109 + 2 × 148 + 5 × 84 ≈ (3 × 110 + 2 × 150 + 5 × 80) g 1. (a) (b) (c) 2. (i) 64 Using a calculator, 145.6 ÷ 3 65.4 = 2 .99 (to 3 s .f .) . \ The estimated value is close to the actual value . 3. (i) 3 .612 = 3 .6 (to 2 s .f .) 29 .87 = 30 (to 2 s .f .) (ii) 3 .612 ÷ 29 .87 ≈ 3 .6 ÷ 30 = 0 .12 (to 2 s .f .) 28.2 4.03 28 ≈ 4 6. Number of batteries required = 274 9.1 270 ≈ l 9 Ratio of area of shaded region to that of unshaded region = 1 : 2 Total amount of money that the shopkeeper has to pay = 32 × $18 + 18 × $8 + 47 × $26 + 63 × $23 + 52 × $9 ≈ 30 × $20 + 20 × $10 + 50 × $30 + 60 × $20 + 50 × $10 = $600 + $200 + $1500 + $1200 + $500 = $4000 (to the nearest hundred dollars) RM10 ≈ S$4, so RM20 ≈ S$8, RM5 ≈ S$2 . \ The price of the bag is RM25 ≈ S$10 . For option A, 300 g costs about $6 . Thus 100 g will cost about $2 . \ For option A, 500 g will cost about 5 × $2 = $10 . For option B, 500 g costs $9 .90 which is $0 .10 cheaper than option A . However, for option A, 300 g actually costs $5 .80 which is $0 .20 less than $6 . Thus for option A, 500 g will cost at least $0 .20 less than the estimated $10 . \ Option A is better value for money . 80 Price of dress in Shop A after a 20% discount = × $79 .50 100 80 ≈ × $80 100 80 × $85 .05 100 80 ≈ × $85 100 4. Amount of petrol used = 5. 6. 7. 8. 9. 7. Price of hard disk in Store A after a 20% discount = 90 × $76 .05 100 90 ≈ × $76 100 Price of hard disk in Store B after a 10% discount = 8. For option A, 250 ml costs about $15 . Thus 50 ml will cost about $3, and 100 ml will cost about $6 . \ For option A, 300 ml will cost about 3 × $6 = $18 . Furthermore, for option A, 250 ml actually costs $15 .20 which is $0 .20 more than $15 . Thus for option A, 300 ml will cost at least $0 .20 more than the estimated $18 . \ Option B is better value for money . Challenge Yourself 1. 987 × 123 is more than 988 × 122 because 987 × 123 = 987 × (122 + 1), i .e . there is an additional 987 × 1; but 988 × 122 = (987 + 1) × 122, i .e . there is only an additional 1 × 122 . In fact, 987 × 123 – 988 × 122 = 987 – 122 = 865 . 2. This question tests students’ sense of mass . The mass of an ordinary car is likely to be 2000 kg . Teachers may wish to get students to give examples of objects with masses of 20 kg, 200 kg and 20 000 kg, e.g. 2 10-kg bags of rice have a total mass of 20 kg, 5 Secondary 1 students have a total mass of about 200 kg and a rocket has a mass of about 20 000 kg. 90 × $69 .50 100 90 ≈ × $70 100 Price of dress in Shop B after a 10% discount = 1 60 Chapter 4 Basic Algebra and Algebraic Manipulation TEACHING NOTES Suggested Approach Some students are still unfamiliar with algebra even though they have learnt some basic algebra in primary school. Thus for the lower ability students, teachers should teach this chapter as though they do not know algebra at all. The learning experiences in the new syllabus specify the use of algebra discs. In addition to the algebra discs showing the numbers 1 and –1 which students have encountered in Chapter 2, algebra discs showing x, –x, y and –y are needed. Since many Secondary 1 students are still in the concrete operational stage (according to Piaget), the use of algebra discs can help them to learn the concepts more easily. However, there is still a need to guide students to move from the ‘concrete’ to the ‘abstract’, partly because they cannot use algebra discs in examinations, and partly because they cannot use algebra discs to manipulate algebraic expressions which consist of algebraic terms that have large or fractional coefficients (see Section 4.1, 4.2 and 4.3). Section 4.1: Fundamental Algebra Teachers should teach students how to use letters to represent numbers and interpret basic algebraic notations such as ab = a × b. Teachers should illustrate the definitions of mathematical terms such as ‘algebraic term’, ‘coefficient’, ‘algebraic expression’ and ‘linear expression’ using appropriate examples. In the class discussion on page 83 of the textbook, students are required to use algebraic expressions to express mathematical relationships. To make learning more interactive, students are given the opportunity to use a spreadsheet to explore the concept of variables (see Investigation: Comparison between Pairs of Expressions). Through this investigation, students should be able to observe that evaluating an algebraic expression means finding the value of the expression when the variables take on certain values. This investigation also provides students with an intuitive sense of the difference between pairs of expressions such as 2n and 2 + n, n2 and 2n, and 2n2 and (2n)2. Students are expected to give a more rigorous mathematical explanation for the difference between such a pair of expressions in the journal writing on page 85 of the textbook. Algebra discs cannot be used to add or subtract algebraic terms with large coefficients, so there is a need to help students consolidate what they have learnt in Worked Example 2. For the lower ability students, before going through Worked Example 2(d) and (e), teachers should revisit the procedure for simplifying ordinary numerical fractions, e.g. 1 + 1 . 2 Section 4.2: 3 Expansion and Simplification of Linear Expressions The idea of flipping over a disc to obtain the negative of a number or variable, e.g. –(−x) = x, is needed to teach students how to obtain the negative of a linear expression. Algebra discs cannot be used to manipulate algebraic expressions which consist of algebraic terms that have large coefficients, so there is a need to help students consolidate what they have learnt in the class discussion on page 94 of the textbook by moving away from the ‘concrete’ to the following ‘abstract’ concept: Distributive Law: a(b + c) = ab + ac Teachers should emphasise the importance of the rules by which operations are performed when an algebraic expression involves brackets by using the thinking time on page 96 of the textbook. Section 4.3: Simplification of Linear Expressions with Fractional Coefficients After going through Worked Example 5 and 6, students should observe that the procedure for simplifying linear expressions with fractional coefficients is similar to that of simplifying ordinary numerical fractions. 61 1 Section 4.4: Factorisation Students should learn how to appreciate the factorisation process, i.e. it is the reverse of expansion. Teachers should tell students the difference between ‘complete’ and ‘incomplete’ factorisation. In Secondary 1, students only need to know how to factorise algebraic expressions by extracting the common factors. The class discussion on page 101 of the textbook requires students to work in pairs to select and justify pairs of equivalent expressions. Teachers should make use ofthis opportunity to highlight some common errors made by students when manipulating algebraic expressions. 1 62 (iii) When n = 8, 2n = 2 × 8 = 16 When n = 9, 2n = 2 × 9 = 18 When n = 10, 2n = 2 × 10 = 20 WORKED SOLUTIONS Class Discussion (Expressing Mathematical Relationships using Algebra) 1. In words Algebraic expression (a) Sum of 2x and 3z 2x + 3z (b) Product of x and 7y 7xy 3ab 2c (c) Divide 3ab by 2c 3. (d) Subtract 6q from 10z 10z – 6q 1 (e) Subtract the product of x and y from the sum of p and q (p + q) – xy 2 (f) Divide the sum of 3 and y by 5 (g) Subtract the product of 2 and c from the positive square root of b (h) There are three times as many girls as boys in a school . Find an expression, in terms of x, for the total number of students in the school, where x represents the number of boys in the school . 3+ y 5 b – 2c It is given that x represents the number of boys . \ 3x represents the number of girls . Total number of students = x + 3x = 4x (i) The age of Nora’s father is thrice hers . The age of Nora’s brother is 5 years more than hers . Find an expression, in terms of y, for the sum of their ages, where y represents Nora’s age . It is given that y represents Nora’s age . \ Nora’s father is 3y years old . Nora’s brother is (y + 5) years old . Sum of their ages = y + 3y + y + 5 = (5y + 5) years (j) The length is three times as long as the breadth of the rectangle . Find an expression, in terms of b, for the perimeter and the area of the rectangle, where b represents the breadth of the rectangle . It is given that b represents the breadth of the rectangle in m . 3b represents the length of the rectangle in m . Perimeter of rectangle = 2(3b + b) = 2(4b) = 8b m Area of rectangle = 3b × b = 3b2 m2 A B C D E F n 2n 2+n n2 2n2 (2n)2 3 1 2 3 1 2 4 4 2 4 4 4 8 16 5 3 6 5 9 18 36 6 4 8 6 16 32 64 7 5 10 7 25 50 100 4. • 2n and 2 + n Referring to columns B and C on the spreadsheet, the expressions 2n and 2 + n are equal only when n = 2 . When n < 2, 2n < 2 + n . When n > 2, 2n > 2 + n . n2 and 2n Referring to columns B and D on the spreadsheet, the expressions n2 and 2n are equal when n = 2 . By observation, they are also equal when n = 0 . When n < 0 or n > 2, n2 > 2n . When 0 < x < 2, n2 < 2n . 2n2 and (2n)2 By observation, the expressions 2n2 and (2n)2 are equal when n = 0 . For any value of n ≠ 0, (2n)2 > 2n2 . • • Journal Writing (Page 85) By observation, the expressions 5 + n and 5n are equal only when Table 4.3 n=1 1 1 1 . When n < 1 , 5n < 5 + n . When n > 1 , 5n > 5 + n . 4 4 4 Investigation (Comparison between Pairs of Expressions) A 2. B C D E Class Discussion (The Distributive Law) F 1. (a) (b) (c) (d) 1 2 n 2n 3 1 2 4 2 4 5 3 6 6 4 8 7 5 10 2+n n 2 2n 2 2 (2n) 2(–x – 4) = –2x – 8 –2(–x – 4) = 2x + 8 3(y – 2x) = 3y – 6x –3(y – 2x) = –3y + 6x 2. a(b + c) = ab + ac (i) The value of 2n changes as n changes . (ii) We multiply the given value of n by 2 to obtain the corresponding value of 2n . 63 1 3. G and N Thinking Time (Page 96) 3( x + 3) 4(2 x + 3) 9( x + 3) – 16(2 x + 3) – = 3 4 12 9 x + 27 – 32 x – 48 = 12 9 x – 32 x + 27 – 48 = 12 –23 x – 21 = 12 Students may mistakenly match B and G due to an error in their working as shown: –(x – 5) + 6x – (7x – 2) + 12 = –x + 5 + 6x – 7x + 2 + 12 = –x + 6x – 7x + 5 + 2 + 12 = –2x + 19 Possible ways: • –(x – 5) + 6x – 7x – (2 + 12) = –(x – 5) + 6x – 7x – 14 = –x + 5 + 6x – 7x – 14 = –x + 6x – 7x + 5 – 14 = –2x– 9 • –x – (5 + 6x) – (7x – 2) + 12 = –x – 5 – 6x – 7x + 2 + 12 = –x – 6x – 7x – 5 + 2 + 12 = –14x + 9 • –x – (5 + 6x) – 7x – (2 + 12) = –x – (5 + 6x) – 7x – 14 = –x – 5 – 6x – 7x – 14 = –x – 6x – 7x – 5 – 14 = –14x – 19 3( x + 3) 4(2 x + 3) 9( x + 3) – 16(2 x + 3) – = 3 4 12 9 x + 27 – 32 x + 48 = 12 9 x – 32 x + 27 + 48 = 12 –23 x + 75 = 12 4. I and M 2x – 3[5x – y – 2(7x – y)] = 2x – 3(5x – y – 14x + 2y) = 2x – 3(5x – 14x – y + 2y) = 2x – 3(–9x + y) = 2x + 27x – 3y = 29x – 3y Students may mistakenly match L and M due to errors in their working as shown: 2x – 3[5x – y – 2(7x – y)] = 2x – 3(5x – y – 14x – 2y) = 2x – 3(5x – 14x – y – 2y) = 2x – 3(–9x – 3y) = 2x – 27x – 9y = –25x – 9y 5. C and J, C and K or J and K 7ay – 49y = 7(ay – 7y) = 7y(a – 7) Class Discussion (Equivalent Expressions) The five pairs of equivalent expressions are as follows: 1. D and F 3(x – 2y) – 2(3x – y) = 3x – 6y – 6x + 2y = 3x – 6x – 6y + 2y = –3x – 4y Students may mistakenly match D and O due to an error in their working as shown: 3(x – 2y) – 2(3x – y) = 3x – 6y – 6x – 2y = 3x – 6x – 6y – 2y = –3x – 8y 2. A and E x–3 2x – 5 3( x – 3) – 2(2 x – 5) – = 6 2 3 3 x – 9 – 4 x + 10 = 6 3 x – 4 x – 9 + 10 = 6 –x + 1 = 6 1– x = 6 Students may mistakenly match E and H due to an error in their working as shown: Teachers may wish to get students to indicate the expression which is obtained when the expression 7ay – 49y is factorised completely. Practise Now 1 1. (a) 5y – 4x = 5(4) – 4(–2) = 20 + 8 = 28 x–3 2x – 5 3( x – 3) – 2(2 x – 5) – = 6 2 3 3 x – 9 – 4 x – 10 = 6 (b) 3 x – 4 x – 9 – 10 6 – x – 19 = 6 = 1 64 1 1 –y+3= –4+3 x −2 1 =– –4+3 2 1 = –4 + 3 2 1 = –1 2 2. (i) 2p – 5q + 7r – 4p + 2q – 3r = 2p – 4p – 5q + 2q + 7r – 3r = –2p – 3q + 4r 2  1 2. p2 + 3q2 =  –  + 3(–2)2  2 1 1 , q = – , r = 4, 2 3  1  1 –2p – 3q + 4r = –2   – 3  –  + 4(4)  2  3 = –1 + 1 + 16 = 0 + 16 = 16 1 + 3(4) 4 1 = + 12 4 1 = 12 4 (ii) When p = = Practise Now (Page 87) (a) (b) (c) (d) Practise Now (Page 92) 3x + 4x = 7x 3x + (– 4x) = –x –3x + 4x = x –3x + (– 4x) = –7x (a) (b) (c) (d) –(3x + 2) = –3x – 2 –(3x – 2) = –3x + 2 –(–3x – 2) = 3x + 2 –(2x + y – 4) = –2x – y + 4 Practise Now (Page 88) Practise Now (Page 92) (a) 4x – 3x = x (b) 4x – (–3x) = 4x + 3x = 7x (c) – 4x – 3x = –7x (d) – 4x – (–3x) = – 4x + 3x = –x (a) x + 1 + [–(3x – 1)] = x + 1 – 3x + 1 = x – 3x + 1 + 1 = –2x + 2 (b) 5x – 3 + [–(4x + 1)] = 5x – 3 – 4x – 1 = 5x – 4x – 3 – 1 =x–4 (c) 3x + 2y + [–(–y + 2x)] = 3x + 2y + y – 2x = 3x – 2x + 2y + y = x + 3y (d) – 4x + 2y + [–(–x – 5y)] = – 4x + 2y + x + 5y = – 4x + x + 2y + 5y = –3x + 7y Practise Now (Page 89) (a) x + 2 + 5x – 4 = x + 5x + 2 – 4 = 6x – 2 (b) 2x + (–3) – 3x + 5 = 2x – 3x + (–3) + 5 = –x + 2 (c) –x – y – (–2x) + 4y = –x – y + 2x + 4y = –x + 2x – y + 4y = x + 3y (d) –3x – 7y + (–2y) – (– 4x) = –3x – 7y + (–2y) + 4x = –3x + 4x – 7y + (–2y) = x – 9y Practise Now (Page 93) (a) (b) (c) (d) 3(5x) = 15x 3(–5x) = –15x –3(5x) = –15x –3(–5x) = 15x Practise Now 2 Practise Now 3 1. (a) 2x – 5y + 4y + 8x = 2x + 8x – 5y + 4y = 10x – y (b) 11x – (–5y) – 14x – 2y = 11x + 5y – 14x – 2y = 11x – 14x + 5y – 2y = –3x + 3y (c) –9x – (–y) + (–3x) – 7y = –9x + y – 3x – 7y = –9x – 3x + y – 7y = –12x – 6y (a) 3(x + 2) = 3x + 6 (b) –5(x – 4y) = –5x + 20y (c) –a(x + 2y) = –ax – a(2y) = –ax – 2ay Practise Now (Page 95) (a) x + 7 + 3(x – 2) = x + 7 + 3x – 6 = x + 3x + 7 – 6 = 4x + 1 (b) 3(x + 2) + 2(–2x + 1) = 3x + 6 – 4x + 2 = 3x – 4x + 6 + 2 = –x + 8 (c) 2(–x – y) – (2x – y) = –2x – 2y – 2x + y = –2x – 2x – 2y + y = – 4x – y 1 1 3 2 (d) x– x= x– x 2 3 6 6 1 = x 6 14 7 5 5 (e) y– y= y– y 4 8 8 8 9 = y 8 65 1 (d) –(x + 4y) – 2(3x – y) = –x – 4y – 6x + 2y = –x – 6x – 4y + 2y = –7x – 2y Practise Now 6 1. (a) Practise Now 4 1. (a) 6(4x + y) + 2(x – y) = 24x + 6y + 2x – 2y = 24x + 2x + 6y – 2y = 26x + 4y (b) x – [y –3(2x – y)] = x – (y – 6x + 3y) = x – (–6x + y + 3y) = x – (–6x + 4y) = x + 6x – 4y = 7x – 4y (c) 7x – 2[3(x – 2) – 2(x – 5)] = 7x – 2(3x – 6 – 2x + 10) = 7x – 2(3x – 2x – 6 + 10) = 7x – 2(x + 4) = 7x – 2x – 8 = 5x – 8 2. (i) Michael’s present age = (p + 5) years (ii) Vishal’s present age = 3(p + 5) = (3p + 15) years (iii) Sum of their ages in 6 years’ time = p + p + 5 + 3p + 15 + 3 × 6 = p + p + 5 + 3p + 15 + 18 = p + p + 3p + 5 + 15 + 18 = (5p + 38) years (iv) Sum of their ages 3 years ago = p + p + 5 + 3p + 15 – 3 × 3 = p + p + 5 + 3p + 15 – 9 = p + p + 3p + 5 + 15 – 9 = (5p + 11) years Alternatively, Sum of their ages 3 years ago = 5p + 38 – 3 × 9 = 5p + 38 – 27 = (5p + 11) years (b) 1 1 2 1 1 x+ y– y– x = x– 2 4 5 3 2 3 = x– 6 1 = x– 6 1 x+ 3 2 x+ 6 3 y 20 x–2 2x – 7 3( x – 2) 4(2 x – 7) – = – 12 4 3 12 3( x – 2) – 4(2 x – 7) = 12 3 x – 6 – 8 x + 28 = 12 3 x – 8 x – 6 + 28 = 12 –5 x + 22 = 12 6 x –1 2 x – 3 4( x – 1) 3(2 x – 3) 1 + – = + – 12 12 2 3 4 12 4( x – 1) + 6 – 3(2 x – 3) = 12 4x – 4 + 6 – 6x + 9 = 12 4x – 6x – 4 + 6 + 9 = 12 –2 x + 11 = 12 x–4 2 x – 5 9(2 x ) x–4 3(2 x – 5) (b) 2x + – = + – 9 3 9 9 9 9(2 x ) + x – 4 – 3(2 x – 5) = 9 18 x + x – 4 – 6 x + 15 = 9 18 x + x – 6 x – 4 + 15 = 9 13 x + 11 = 9 2. (a) Practise Now 5 (a) x–3 2 x – 5 3( x – 3) 2(2 x – 5) + = + 6 2 3 6 3( x – 3) + 2(2 x – 5) = 6 3 x – 9 + 4 x – 10 = 6 3 x + 4 x – 9 – 10 = 6 7 x – 19 = 6 1 2 y– y 4 5 5 8 y– y 20 20 Practise Now 7 (a) –10x + 25 = –5(2x – 5) (b) 18a – 54ay + 36az = 9a(2 – 6y + 4z) 1 1 (b) [–y – 3(16x – 3y)] = (–y – 48x + 9y) 8 8 1 = (–y + 9y – 48x) 8 1 = (8y – 48x) 8 = y – 6x Exercise 4A 1. (a) ab + 5y (c) 6kq (e) 3x – 4 z 1 66 (b) f 3 – 3 2w (d) 3 xy 2p (f) 5q (d) 6x – 20y + 7z – 8x + 25y – 11z = 6x – 8x – 20y + 25y + 7z – 11z = –2x + 5y – 4z 5. (a) Required answer = 2x + 4y + (–5y) = 2x + 4y – 5y = 2x – y (b) Required answer = –b – 4a + 7b – 6a = – 4a – 6a – b + 7b = –10a + 6b (c) Required answer = 6d – 4c + (–7c + 6d) = 6d – 4c – 7c + 6d = – 4c – 7c + 6d + 6d = –11c + 12d (d) Required answer = 3pq – 6hk + (–3qp + 14kh) = 3pq – 6hk – 3qp + 14kh = 3pq – 3qp – 6hk + 14kh = 8hk 2. (a) 4x – 7y = 4(6) – 7(–4) = 24 + 28 = 52 (b) 5x 5(6) +x= +6 3y 3(– 4) 30 +6 –12 1 = –2 + 6 2 1 =3 2 = (c) 2x2 – y3 = 2(6)2 – (– 4)3 = 72 – (–64) = 72 + 64 = 136 x 6 (d) 3x + – y2 = 3(6) + – (– 4)2 y –4 = 18 – 1 = 16 1 – 16 2 6. (a) (a + b)2 – 3 3 xy (b) Total value = (20x + 500y) cents 3a – b 3(3) – (–2) 3a – c 3(3) – (– 4) 7. (a) + = + 2(–2) c–b 2c –2 – (– 4) 9+4 9+2 = + –4 –2 + 4 13 11 + = –4 2 1 1 = –3 + 5 4 2 1 =2 4 5(3) + 4(–2) 2(–2) – 3 5a + 4c 2c – a (b) – = – –2 – 3 3(–2) + (– 4) c–a 3c + b –4 – 3 15 – 8 = – –6 – 4 –5 −7 7 = – –10 –5 7 2 = +1 10 5 1 =2 10 1 – 16 2 1 2 3. (a) a(3c – b) = 3[3(6) – (–5)] = 3(18 + 5) = 3(23) = 69 (b) ab2 – ac = 3(–5)2 – 3(6) = 3(25) – 18 = 75 – 18 = 57 = (c) (d) 4. (a) (b) (c) b c –5 6 – = – a b 3 –5 2 1 = –1 + 1 3 5 7 =– 15 b+c a+c –5 + 6 3+ 6 + = + a b 3 −5 9 1 = + 3 –5 1 4 = –1 3 5 7 = –1 15 5x + 22 – 6x – 23 = 5x – 6x + 22 – 23 = –x – 1 x + 3y + 6x + 4y = x + 6x + 3y + 4y = 7x + 7y 6xy + 13x – 2yx – 5x = 6xy – 2yx + 13x – 5x = 4xy + 8x (c) 67 3 + (– 4) + 2(–2) 5(–2) a + b + 2c 5c – = – 4(– 4) 3(–2) – 3 – (– 4) 3c – a – b 4b –10 3– 4 – 4 = – –16 –6 – 3 + 4 –5 5 = – 8 –5 5 =1– 8 3 = 8 1 (d) 11. (i) Raj’s age 5 years later = (12m + 5) years (ii) Present age of Raj’s son = 12m – 9m = 3m years Age of Raj’s son 5 years later = (3m + 5) years Sum of their ages in 5 years’ time = 12m + 5 + 3m + 5 = 12m + 3m + 5 + 5 = (15m + 10) years 12. Amount of money Huixian had at first = 8 × $w + 7 × $m + $(3w + 5m) = $8w + $7m + $(3w + 5m) = $(8w + 3w + 7m + 5m) = $(11w + 12m) 5 13. (a) Number of people who order plain prata = a 2 b–c  bc ac  ÷  + b   a 3c + 4 b = – 4 – (–2)  (– 4)(–2) 3(–2)  + ÷  3 – 4  3(–2) + 4(– 4)  = –4 + 2  8 –6  ÷  3 + –4  –6 – 16   2 1 –2 ÷  2 + 1  2 –22  3 1 1 = ÷4 6 11 6 = 275 15x + (–7y) + (–18x) + 4y = 15x – 7y – 18x + 4y = 15x – 18x – 7y + 4y = –3x – 3y –3x + (–5y) – (–10y) – 7x = –3x – 5y + 10y – 7x = –3x – 7x – 5y + 10y = –10x + 5y 9x – (–2y) – 8x – (–12y) = 9x + 2y – 8x + 12y = 9x – 8x + 2y + 12y = x + 14y –7x – (–15y) – (–2x) + (–6y) = –7x + 15y + 2x – 6y = –7x + 2x + 15y – 6y = –5x + 9y = 8. (a) (b) (c) (d) 9. (a) (b) (c) (d) 10. (i) 2 b 5 2 (c) Number of people who order egg prata = c 7 (b) Number of people who order egg prata = Exercise 4B 3 4 1 1 x+ x = x+ x 4 3 12 12 7 = x 12 6 5 2 1 y– y = y– y 5 3 15 15 1 = y 15 15 21 3 3 – a+ a =– a+ a 7 5 35 35 6 = a 35 27 16 9 4 b– b= b– b 4 3 12 12 11 = b 12 3p + (–q) – 7r – (–8p) – q + 2r = 3p – q – 7r + 8p – q + 2r = 3p + 8p – q – q – 7r + 2r = 11p – 2q – 5r 3. 4. 1 , r = –5, 2 1  11p – 2q – 5r = 11(2) – 2  –1  – 5(–5) 2  = 22 + 3 + 25 = 25 + 25 = 50 5. (ii) When p = 2, q = –1 6. 1 –(x + 5) = –x – 5 –(4 – x) = – 4 + x 2(3y + 7) = 6y + 14 8(2y – 5) = 16y – 40 8(3a – 4b) = 24a – 32b –3(c + 6) = –3c – 18 – 4(d – 6) = – 4d + 24 2a(x – y) = 2ax – 2ay 5(a + 2b) – 3b = 5a + 10b – 3b = 5a + 7b (b) 7(p + 10q) + 2(6p + 7q) = 7p + 70q + 12p + 14q = 7p + 12p + 70q + 14q = 19p + 84q (c) a + 3b – (5a – 4b) = a + 3b – 5a + 4b = a – 5a + 3b + 4b = – 4a + 7b (d) x + 3(2x – 3y + z) + 7z = x + 6x – 9y + 3z + 7z = 7x – 9y + 10z Present age of Khairul’s uncle = 4(x + 5) = (4x + 20) years Total cost = 4x + 6(x – y) = 4x + 6x – 6y = (10x – 6y) cents Total cost of skirts Devi bought = 7 × $x + n × $12 + (2n + 1) × $15 + 4 × $3x = $7x + $12n + $15(2n + 1) + $12x = $7x + $12n + $(30n + 15) + $12x = $(7x + 12n + 30n + 15 + 12x) = $(7x + 12x + 12n + 30n + 15) = $(19x + 42n + 15) (a) 4u – 3(2u – 5v) = 4u – 6u + 15v = –2u + 15v 1. (a) (b) (c) (d) (e) (f) (g) (h) 2. (a) 68 9. Average monthly salary of the female employees (b) –2a – 3(a – b) = –2a – 3a + 3b = –5a + 3b (c) 7m – 2n – 2(3n – 2m) = 7m – 2n – 6n + 4m = 7m + 4m – 2n – 6n = 11m – 8n (d) 5(2x + 4) – 3(– 6 – x) = 10x + 20 + 18 + 3x = 10x + 3x + 20 + 18 = 13x + 38 (e) – 4(a – 3b) – 5(a – 3b) = – 4a + 12b – 5a + 15b = – 4a – 5a + 12b + 15b = –9a + 27b (f) 5(3p – 2q) – 2(3p + 2q) = 15p – 10q – 6p – 4q = 15p – 6p – 10q – 4q = 9p – 14q (g) x + y – 2(3x – 4y + 3) = x + y – 6x + 8y – 6 = x – 6x + y + 8y – 6 = –5x + 9y – 6 (h) 3(p – 2q) – 4(2p – 3q – 5) = 3p – 6q – 8p + 12q + 20 = 3p – 8p – 6q + 12q + 20 = –5p + 6q + 20 (i) 9(2a + 4b – 7c) – 4(b – c) – 7(–c – 4b) = 18a + 36b – 63c – 4b + 4c + 7c + 28b = 18a + 36b – 4b + 28b – 63c + 4c + 7c = 18a + 60b – 52c (j) – 4[5(2x + 3y) – 4(x + 2y)] = – 4(10x + 15y – 4x – 8y) = – 4(10x – 4x + 15y – 8y) = – 4(6x + 7y) = –24x – 28y 7. (a) Required answer = 2x – 5 – (–6x – 3) = 2x – 5 + 6x + 3 = 2x + 6x – 5 + 3 = 8x – 2 (b) Required answer = 10x – 2y + z – (6x – y + 5z) = 10x – 2y + z – 6x + y – 5z = 10x – 6x – 2y + y + z – 5z = 4x – y – 4z (c) Required answer = – 4p – 4q + 15sr – (8p + 9q – 5rs) = – 4p – 4q + 15sr – 8p – 9q + 5rs = – 4p – 8p – 4q – 9q + 15sr + 5rs = –12p – 13q + 20rs (d) Required answer = 10a – b – 4c – 8d – (8a – 3b + 5c – 4d) = 10a – b – 4c – 8d – 8a + 3b – 5c + 4d = 10a – 8a – b + 3b – 4c – 5c – 8d + 4d = 2a + 2b – 9c – 4d 8. (a) –2{3a – 4[a – (2 + a)]} = –2[3a – 4(a – 2 – a)] = –2[3a – 4(–2)] = –2(3a + 8) = –6a – 16 (b) 5{3c – [d – 2(c + d)]} = 5[3c – (d – 2c – 2d)] = 5[3c – (–2c – d)] = 5(3c + 2c + d) = 5(5c + d) = 25c + 5d  2000( m + f ) – m (b + 200)  =$  f   2000 m + 2000 f – mb – 200 m  = $   f  2000 m – 200 m – mb + 2000 f  = $   f  1800 m – mb + 2000 f  = $   f Exercise 4C 1. (a) (b) (c) (d) 2. (a) (b) 69 1 1 1 1 1 1 1 1 x+ y– x– y = x– x+ y– y 4 5 6 10 4 6 5 10 3 2 2 1 = x– x+ y– y 10 10 12 12 1 1 = x+ y 12 10 2 1 3 2 1 3 a – b + 2a – b = a + 2a – b – b 3 7 5 3 7 5 6 5 21 2 = a+ a– b– b 3 3 35 35 8 26 = a– b 3 35 5 3 7 4 5 7 3 4 c+ d– c– d = c– c+ d– d 9 4 8 3 9 8 4 3 40 63 9 16 = c– c+ d– d 72 72 12 12 23 7 =– c– d 72 12 28 5 9 1 5 2f – h + k – f – k+ h 3 4 2 4 5 28 1 5 5 9 = 2f – f – h + h + k – k 2 3 4 4 5 4 20 15 45 112 1 = f– f– h+ h+ k– k 2 2 12 12 20 20 67 3 5 = f– h– k 2 12 20 3 3   5a + 4b – 3c –  2 a – b + c  2 2   3 3 = 5a + 4b – 3c – 2a + b – c 2 2 3 3 = 5a – 2a + 4b + b – 3c – c 2 2 8 6 3 3 = 3a + b + b – c – c 2 2 2 2 11 9 = 3a + b– c 2 2 1 1 [2x + 2(x – 3)] = (2x + 2x – 6) 2 2 1 = (4x – 6) 2 = 2x – 3 1 (c) (d) 3. (a) (b) (c) (d) (e) 2 2 [12p – (5 + 2p)] = (12p – 5 – 2p) 5 5 2 = (12p – 2p – 5) 5 2 = (10p – 5) 5 = 4p – 2 (f) 1 1 [8x + 10 – 6(1 – 4x)] = (8x + 10 – 6 + 24x) 2 2 1 = (8x + 24x + 10 – 6) 2 1 = (32x + 4) 2 = 16x + 2 (g) x 2x 5x 4x + = + 2 5 10 10 9 = x 10 a a 4a 3a – = – 3 4 12 12 1 = a 12 2h h +1 10 h 7( h + 1) + = + 7 5 35 35 10 h + 7( h + 1) = 35 10 h + 7 h + 7 = 35 17 h + 7 = 35 3x x+2 3x 2( x + 2) – = – 8 4 8 8 3 x – 2( x + 2) = 8 3x – 2 x – 4 = 8 x–4 = 8 4x + 1 2(4 x + 1) 5(3 x – 1) 3x – 1 + = + 2 5 10 10 2(4 x + 1) + 5(3 x – 1) = 10 8 x + 2 + 15 x – 5 = 10 8 x + 15 x + 2 – 5 = 10 23 x – 3 = 10 1 (h) 4. (a) (b) (c) (d) (e) 3y – 1 2 y – 3 3(3 y – 1) 2(2 y – 3) – = – 12 4 6 12 3(3 y – 1) – 2(2 y – 3) = 12 9y – 3 – 4y + 6 = 12 9y – 4y – 3 + 6 = 12 5y + 3 = 12 a–2 a+7 2( a – 2) a+7 – = – 4 8 8 8 2( a – 2) – ( a + 7) = 8 2a – 4 – a – 7 = 8 2a – a – 4 – 7 = 8 a – 11 = 8 3 p – 2q 4 p – 5q 4(3 p – 2 q ) 3(4 p – 5 q ) – = – 3 4 12 12 4(3 p – 2 q ) – 3(4 p – 5 q ) = 12 12 p – 8 q – 12 p + 15 q = 12 12 p – 12 p – 8 q + 15 q = 12 7 = q 12 12x – 9 = 3(4x – 3) –25y – 35 = –5(5y + 7) 27b – 36by = 9b(3 – 4y) 8ax + 12a – 4az = 4a(2x + 3 – z) 4m – 6my – 18mz = 2m(2 – 3y – 9z) 5. (a) y – 2 (9x – 3y) = y – 2(3x – y) 3 = y – 6x + 2y = – 6x + y + 2y = – 6x + 3y 1 {6(p + q) – 3[p – 2(p – 3q)]} 3 1 = – [6(p + q) – 3(p – 2p + 6q)] 3 1 = – [6(p + q) – 3(–p + 6q)] 3 1 = – (6p + 6q + 3p – 18q) 3 1 = – (6p + 3p + 6q – 18q) 3 1 = – (9p – 12q) 3 = –3p + 4q (b) – 70 6. (a) (b) (c) (d) (e) 7( x + 3) 5(2 x – 5) 21( x + 3) 10(2 x – 5) + = + 2 3 6 6 21( x + 3) + 10(2 x – 5) = 6 21x + 63 + 20 x – 50 = 6 21x + 20 x + 63 – 50 = 6 41x + 13 = 6 3x – 4 3( x – 1) 2(3 x – 4) 15( x – 1) – = – 5 2 10 10 2(3 x – 4) – 15( x – 1) = 10 6 x – 8 – 15 x + 15 = 10 6 x – 15 x – 8 + 15 = 10 –9 x + 7 = 10 3( z – 2) 4(2 z – 3) 15( z – 2) 16(2 z – 3) – = – 4 5 20 20 15( z – 2) – 16(2 z – 3) = 20 15 z – 30 – 32 z + 48 = 20 15 z – 32 z – 30 + 48 = 20 –17 z + 18 = 20 2( p – 4 q ) 3(2 p + q ) 4( p – 4 q ) 9(2 p + q ) – = – 3 2 6 6 4( p – 4 q ) – 9(2 p + q ) = 6 4 p – 16 q – 18 p – 9 q = 6 4 p – 18 p – 16 q – 9 q = 6 –14 p – 25 q = 6 2b 3( a – 2 b ) 10 b 9( a – 2 b ) – – =– – 3 5 15 15 –10 b – 9( a – 2 b ) = 15 –10 b – 9 a + 18 b = 15 –9 a – 10 b + 18 b = 15 –9 a + 8 b = 15 (f) (g) (h) (i) (j) 71 2( x + 3) 3x – 4 8( x + 3) 10 5(3 x – 4) 1 – + = – + 2 5 4 20 20 20 8( x + 3) – 10 + 5(3 x – 4) = 20 8 x + 24 – 10 + 15 x – 20 = 20 8 x + 15 x + 24 – 10 – 20 = 20 23 x – 6 = 20 a +1 a+3 5a – 2 – – 2 3 4 6( a + 1) 4( a + 3) 3(5 a – 2) = – – 12 12 12 6( a + 1) – 4( a + 3) – 3(5 a – 2) = 12 6 a + 6 – 4 a – 12 – 15 a + 6 = 12 6 a – 4 a – 15 a + 6 – 12 + 6 = 12 13 =– a 12 x +1 x+3 5x – 1 3( x + 1) 2( x + 3) 5x – 1 + – = + – 2 3 6 6 6 6 3( x + 1) + 2( x + 3) – (5 x – 1) = 6 3x + 3 + 2 x + 6 – 5 x + 1 = 6 3x + 2 x – 5 x + 3 + 6 + 1 = 6 10 = 6 5 = 3 2 =1 3 2( a – b ) 2 a + 3b a+b – + 7 14 2 4( a – b ) 2 a + 3b 7( a + b ) = – + 14 14 14 4( a – b ) – (2 a + 3b ) + 7( a + b ) = 14 4 a – 4 b – 2 a – 3b + 7 a + 7 b = 14 4 a – 2 a + 7 a – 4 b – 3b + 7 b = 14 9 = a 14 x+3 5(3 x + 4) 2( x + 3) 5(3 x + 4) 6 + +1 = + + 3 6 6 6 6 2( x + 3) + 5(3 x + 4) + 6 = 6 2 x + 6 + 15 x + 20 + 6 = 6 2 x + 15 x + 6 + 20 + 6 = 6 17 x + 32 = 6 1 7. (a) –39b2 – 13ab = –13b(3b + a) (b) 5x + 10x(b + c) = 5x[1 + 2(b + c)] = 5x(1 + 2b + 2c) (c) 3xy – 6x(y – z) = 3x[y – 2(y – z)] = 3x(y – 2y + 2z) = 3x(–y + 2z) (d) 2x(7 + y) – 14x(y + 2) = 2x[7 + y – 7(y + 2)] = 2x(7 + y – 7y – 14) = 2x(y – 7y + 7 – 14) = 2x(–6y – 7) (e) –3a (2 + b) + 18a(b – 1) = 3a[–(2 + b) + 6(b – 1)] = 3a(–2 – b + 6b – 6) = 3a(–b + 6b – 2 – 6) = 3a(5b – 8) (f) – 4y(x – 2) – 12y(3 – x) = 4y[–(x – 2) – 3(3 – x)] = 4y(–x + 2 – 9 + 3x) = 4y(–x + 3x + 2 – 9) = 4y(2x – 7) = = = = = Review Exercise 4 1. (a) 4a + 5b = 4(–2) + 5(7) = –8 + 35 = 27 (b) 2a2 = 2(–2)2 =8 (c) (2a)2 = [2(–2)]2 = (– 4)2 = 16 (d) a(b – a) = (–2)[7 – (–2)] = (–2)(7 + 2) = (–2)(9) = –18 (e) b – a2 = 7 – (–2)2 =7–4 =3 (f) (b – a)2 = [7 – (–2)]2 = (7 + 2)2 = 92 = 81 3(3) – 5(– 4)2 – 2(3)(– 4)(2) 3 x – 5 y 2 – 2 xyz 2. = 2 (– 4)2 3 x y – – 2 –4 y z 9 – 80 + 48 = 3 16 – – 4 2 5( p – q ) 2q – p 2( p + q ) – – 2 14 7 35( p – q ) 2q – p 4( p + q ) = – – 14 14 14 35( p – q ) – (2 q – p ) – 4( p + q ) = 14 35 p – 35 q – 2 q + p – 4 p – 4 q = 14 35 p + p – 4 p – 35 q – 2 q – 4 q = 14 32 p – 41 q = 14  3( a – 3b ) 4( a + 2 b )  2a + b – (b) – –   2 5  3 8. (a) 2a + b 3( a – 3b ) 4( a + 2 b ) – + 3 2 5 10(2 a + b ) 45( a – 3b ) 24( a + 2 b ) =– – + 30 30 30 –10(2 a + b ) – 45( a – 3b ) + 24( a + 2 b ) = 30 –20 a – 10 b – 45 a + 135 b + 24 a + 48 b = 30 –20 a – 45 a + 24 a – 10 b + 135 b + 48 b = 30 – 41a + 173b = 30 3( f – h ) 7( h + k ) 5( k – f ) (c) – + 4 6 2 9( f – h ) 14( h + k ) 30( k – f ) = – + 12 12 12 9( f – h ) – 14( h + k ) + 30( k – f ) = 12 9 f – 9 h − 14 h – 14 k + 30 k – 30 f = 12 9 f – 30 f – 9 h – 14 h – 14 k + 30 k = 12 –21 f – 23h + 16 k = 12 =– 1 x–y 3( y + 4 z ) 5( x + 3z ) – + 3 4 8 96 8( x – y ) 18( y + z ) 15( x + 3z ) – – + 24 24 24 24 96 – 8( x – y ) – 18( y + z ) + 15( x + 3z ) 24 96 – 8 x + 8 y – 18 y – 18 z + 15 x + 45 z 24 96 – 8 x + 15 x + 8 y – 18 y – 18 z + 45 z 24 96 + 7 x – 10 y + 27 z 24 (d) 4 – = –23 3 32 – – 4 4 –23 35 – 4 22 =2 35 3. (a) 3ab – 5xy + 4ab + 2yx = 3ab + 4ab – 5xy + 2yx = 7ab – 3xy (b) 4(3p – 5q) + 6(2q – 5p) = 12p – 20q + 12q – 30p = 12p – 30p – 20q + 12q = –18p – 8q = 72 (c) 2a + 3[a – (b – a)] + 7(2b – a) = 2a + 3(a – b + a) + 7(2b – a) = 2a + 3(a + a – b) + 7(2b – a) = 2a + 3(2a – b) + 7(2b – a) = 2a + 6a – 3b + 14b – 7a = 2a + 6a – 7a – 3b + 14b = a + 11b (d) –2[3x – (4x – 5y) – 2(3x – 4y)] = –2(3x – 4x + 5y – 6x + 8y) = –2(3x – 4x – 6x + 5y + 8y) = –2(–7x + 13y) = 14x – 26y (e) 4{h – 3[ f – 6( f – h)]} = 4[h – 3( f – 6f + 6h)] = 4[h – 3(–5f + 6h)] = 4(h + 15f – 18h) = 4(15f + h – 18h) = 4(15f – 17h) = 60f – 68h (f) 5(x + 5y) – {2x – [3x – 3(x – 2y) + y]} = 5(x + 5y) – [2x – (3x – 3x + 6y + y)] = 5(x + 5y) – (2x – 7y) = 5x + 25y – 2x + 7y = 5x – 2x + 25y + 7y = 3x + 32y (d) (e) 2x 5–x 8x 3(5 – x ) + = + 3 4 12 12 8 x + 3(5 – x ) = 12 8 x + 15 – 3 x = 12 8 x – 3 x + 15 = 12 5 x + 15 = 12 x–y 3x – 2 y 3( x – y ) 2(3 x – 2 y ) (b) – = – 8 12 24 24 3( x – y ) – 2(3 x – 2 y ) = 24 3x – 3y – 6 x + 4 y = 24 3x – 6 x – 3y + 4 y = 24 −3 x + y = 24 4(2 a – b ) 2(3a + b ) 20(2 a – b ) 6(3a + b ) (c) – = – 3 5 15 15 20(2 a – b ) − 6(3a + b ) = 15 40 a – 20 b – 18 a – 6 b = 15 40 a – 18 a – 20 b – 6 b = 15 22 a – 26 b = 15 4. (a) (f) 5. (a) (b) 6. (a) (b) h+ f f +k 4h – k – + 3 2 5 10( h + f ) 15( f + k ) 6(4 h – k ) = – + 30 30 30 10( h + f ) – 15( f + k ) + 6(4 h – k ) = 30 10 h + 10 f – 15 f – 15 k + 24 h – 6 k = 30 10 f – 15 f + 10 h + 24 h – 15 k – 6 k = 30 –5 f + 34 h – 21k = 30 4 p – 3q q – 4p 3q – – 5 6 90 q 6(4 p – 3q ) 5( q – 4 p ) = – – 30 30 30 90 q – 6(4 p – 3q ) – 5( q – 4 p ) = 30 90 q – 24 p + 18 q – 5 q + 20 p = 30 –24 p + 20 p + 90 q + 18 q – 5 q = 30 – 4 p + 103q = 30 4( x – 5)  5( x – y ) 7 x – y  –  + 6 21  7  4( x – 5) 5( x – y ) 7x – y = – – 7 6 21 24( x – 5) 35( x – y ) 2(7 x – y ) = – – 42 42 42 24( x – 5) – 35( x – y ) – 2(7 x – y ) = 42 24 x – 120 – 35 x + 35 y – 14 x + 2 y = 42 24 x – 35 x – 14 x + 35 y + 2 y – 120 = 42 –25 x + 37 y – 120 = 42 21pq + 14q – 28qr = 7q(3p + 2 – 4r) 4x – 8(y – 2z) = 4[x – 2(y – 2z)] = 4(x – 2y + 4z) Total value of 5-cent coins = 5x cents Total value of 10-cent coins = (3x × 10) cents = 30x cents 3 x 7 3    5 x + 7 x × 10  cents 30    5 x + 7 x  cents 30   35  7 x + 7 x  cents 65 x cents 7 (c) Number of 10-cent coins = Total value of coins = = = = 73 1 x 3 × 60 x = km 180 xy Distance Farhan can cycle in y minutes = km 180 8. (a) Required difference = 3y × 60 – 25y = 180y – 25y = 155y seconds (b) Required sum = 50(3z – 2) × 60 + 4(z + 1) × 3600 = 3000(3z – 2) + 14 400(z + 1) = 9000z – 6000 + 14 400z + 14 400 = 9000z + 14 400z – 6000 + 14 400 = (23 400z + 8400) seconds 9. (i) Total amount Shirley earned = $[(25 – 5) × x + 5 × 1 .5x] = $(20 × x + 5 × 1 .5x) = $(20x + 7 .5x) = $27 .5x Total amount Kate earned = $[(18 – 4) × y + 4 × 1 .5y] = $(14 × y + 4 × 1 .5y) = $(14y + 6y) = $20y Total amount they earned = $(27 .5x + 20y) (ii) Amount Kate was paid per hour = $5 .50 + $0 .50 = $6 Total amount they earned = $[27 .5(5 .5) + 20(6)] = $(151 .25 + 120) = $271 .25 10. (i) Total score obtained by Michael in the first two papers = p – 3q + 13 + 3p + 5q – 4 = p + 3p – 3q + 5q + 13 – 4 = (4p + 2q + 9) marks (ii) Score obtained by Michael in the third paper = 10p + 5q – (4p + 2q + 9) = 10p + 5q – 4p – 2q – 9 = 10p – 4p + 5q – 2q – 9 = (6p + 3q – 9) marks (iii) 6p + 3q – 9 = 3(2p + q – 3) 3. Let the two numbers be xy and xz , where y + z = 10 . xy = 10x + y xz = 10x + z \ xy × xz = (10x + y)(10x + z) = 10x(10x + z) + y(10x + z) = 100x2 + 10xz + 10xy + yz = 100x2 + 10x(y + z) + yz = 100x2 + 10x(10) + yz = 100x2 + 100x + yz = 100x(x + 1) + yz 7. Distance Farhan can cycle in 1 minute = Challenge Yourself 1. Let the number of heads up in the pile of 5 be x . Then the number of tails up in the pile of 5 is 5 – x, the number of heads up in the pile of 7 is 5 – x . After the teacher flips over all the coins in the pile of 5, the number of heads up in that pile is 5 – x . Hence, both piles now have the same number of heads up . (shown) 2. The only possible set of values is {x = 2, y = 3, z = 6} . Proofs 1 1 1 + + < 1 . z y x 1 1 1 If x  3, then y, z > 3 and + + < 1 . z y x If x = 2 and y  4, then z  5 and 1 74 628 6.8 × 60 630 ≈ m/s 7 × 60 a 2 bd 12 (2)(–3) 7. (a) = 3ac – d 3(1)(0) – (–3) –6 = 0+3 –6 = 3 = –2 Revision Exercise A1 6. Average speed = 1. (a) 42 = 2 × 3 × 7 66 = 2 × 3 × 11 78 = 2 × 3 × 13 HCF of 42, 66 and 78 = 2 × 3 =6 (b) 7 = 7 13 = 13 14 = 2 × 7 LCM of 7, 13 and 14 = 2 × 7 × 13 = 182 2. (i) Greatest whole number which is a factor of both 405 and 1960 = HCF of 405 and 1960 =5 (ii) Smallest whole number that is divisible by both 405 and 1960 = LCM of 405 and 1960 = 23 × 34 × 5 × 72 = 158 760 3. (i) 105 = 3 × 5 × 7 126 = 2 × 32 × 7 HCF of 105 and 126 = 3 × 7 = 21 Greatest number of students that the refreshment can cater to = 21 (ii) Number of bags of crisps each student will receive = 105 ÷ 21 =5 (iii) Number of packets of fruit juice each student will receive = 126 ÷ 21 =6 4. (i) Pairs of cards that have a sum of 4 = {–2, 6}, {–1, 5}, {1, 3} (ii) Pairs of cards that have a product of 2 = {–2, –1}, {1, 2} (iii) Groups of three cards that have a sum of 10 = {–1, 5, 6}, {1, 3, 6}, {1, 4, 5}, {2, 3, 5} 5. (a) 101 × (b) (c) 3 bc + d 2 2(0) + (–3)2 = a+b 1+ 2 0+9 = 3 9 = 3 =3 (c) a2 + b2 – c2 + d2 = 12 + 22 – 02 + (–3)2 =1+4–0+9 = 14 3 3 3 3 (d) –a – b + c – d = –13 – 23 + 03 – (–3)3 = –1 – 8 + 0 – (–27) = –9 + 0 – (–27) = –9 – (–27) = –9 + 27 = 18 8. Cost of a pear = (a + b) cents (b)  10 a 12( a + b )  Total cost = $  +  100  100  10 a + 12( a + b )  =$  100   10 a + 12 a + 12 b  = $  100   22 a + 12 b  = $  100  80.7 ≈ 100 × 81 = 100 × 9 = 900  2(11a + 6 b )  =$  100  26 × 502 ÷ 49 ≈ 3 27 × 500 ÷ 50 = 3 × 500 ÷ 50 = 1500 ÷ 50 = 30 65 × 3 = $  11a + 6 b  50   63 ÷ 17 ≈ 64 × 3 64 ÷ 16 = 8 × 4 ÷ 16 = 32 ÷ 16 =2 75 1 6. Number of buttons in Box B after Kate transfers 15 buttons from Box A to Box B = 35 + 15 = 50 Number of buttons in Box A after Kate transfers 15 buttons from Box A to Box B Revision Exercise A2 1. (a) 54 = 2 × 33 126 = 2 × 32 × 7 342 = 2 × 32 × 19 HCF of 54, 126 and 342 = 2 × 32 = 18 (b) 16 = 24 28 = 22 × 7 44 = 22 × 11 68 = 22 × 17 LCM of 16, 18, 44 and 68 = 24 × 7 × 11 × 17 = 20 944 10 2 2. (a) 9216 = 2 × 3 = 50 ÷ = 1 9 ×2 10 10 ≈ (4 × 3) m2 (ii) Cost of carpet ≈ 4 × 3 × $89 .75 ≈ $(4 × 3 × 90) 8. Required answer = –8x + 9 + 15 – 4x – (–7x + 4 + 5x + 7) = –8x + 9 + 15 – 4x – (–7x + 5x + 4 + 7) = –8x + 9 + 15 – 4x – (–2x + 11) = –8x + 9 + 15 – 4x + 2x – 11 = –8x – 4x + 2x + 9 + 15 – 11 = –10x + 13 3 1  4 2 2 1 4  – –3 +3 – +  –  = 3  20  20 5  5 3 61 2 4 = + – 3 5 20 40 183 48 = + – 60 60 60 40 + 183 – 48 = 60 175 = 60 35 = 12 11 =2 12 (b) (i) [– 4 .749 – 6 .558 × (–2 .094)3] ÷ 3 –1.999 = – 44 .030 (to 3 d .p .) 2    5 8   1  (ii)    – 3 – (–0.375)3   × [–p ÷ (–6 .5)] × – 6 33 3       = 0 .313 (to 3 d .p .) 1 7 51 7. (i) Area of carpet = 4 3 3 6 8000 = 2 × 5 = 22 × 5 = 20 2 2 3. 1764 = 2 × 3 × 72 36 = 22 × 32 8820 = 22 × 32 × 5 × 72 \ Value of p = 22 × 32 × 5 = 180 4. (i) Temperature of town at 6 p .m . = – 6 °C + 8° C – 4 °C = –2 °C (ii) Overall increase = –2 °C – (– 6 °C) = –2 °C + 6 °C = 4 °C 5. (a) 50 × = 70 Initial number of buttons in Box A = 70 + 15 = 85 \ – 9216 = – 210 × 32 = –(25 × 3) = –96 (b) 8000 = 26 × 53 \ 10 5 7 76 Chapter 5 Linear Equations and Simple Inequalities TEACHING NOTES Suggested Approach Since many Secondary 1 students are still in the concrete operational stage (according to Piaget), teaching students how to solve linear equations in one variable with the use of algebra discs on a balance can help them to learn the concepts more easily. However, there is still a need to guide students to move from the ‘concrete’ to the ‘abstract’, partly because they cannot use this approach in examinations, and partly because they cannot use this approach to solve linear equations which consist of algebraic terms that have large or fractional coefficients (see Section 5.1). After students learn how to solve linear equations, they will learn how to evaluate an unknown in a formula and formulate linear equations to solve problems in real-world contexts. Since the concept of inequality is harder than that of equation, students only learn how to solve inequalities towards the end of this chapter. Section 5.1: Linear Equations Students have learnt how to complete mathematical sentences such as 7 + = 13 in primary school. Teachers can introduce equations by telling students that when we replace with x, we have 7 + x = 13, which is an equation. Teachers should illustrate the meaning of ‘solving an equation’ using appropriate examples. Students should know the difference between linear expressions and linear equations. Teachers can use the ‘Balance Method’ to show how to solve linear equations which do not involve any brackets before illustrating how to solve those which involve brackets. As this approach cannot be used to solve linear equations which consist of algebraic terms that have large or fractional coefficients, so there is a need to help students consolidate what they have learnt in Worked Examples 1, 2 and 3. The thinking time on page 115 of the textbook reinforces students’ understanding of the concept of equation. For example, since x + 3 = 6, 2x + 3 = 9 and 10x – 4 = 5x + 11 are equivalent equations that can be obtained from x = 3, then the value of x in each of the equations is 3. Section 5.2: Formulae Teachers can use simple formulae such as A = lb, where A, l and b are the area, the length and the breadth of the rectangle respectively, to let students understand that a formula makes use of variables to write instructions for performing a calculation. Teachers may get students to provide examples of formulae which they have encountered in mathematics and the sciences. Section 5.3: Applications of Linear Equations in Real-World Contexts Teachers should illustrate how a word problem is solved using the model method before showing how the same problem can be solved using the algebraic method. Students should observe how the algebraic method is linked to the model method. Also, students should be aware why they need to learn the algebraic method. In this section, students are given ample opportunities to formulate linear equations to solve problems in real-world contexts. Section 5.4: Simple Inequalities In the investigation on page 126 of the textbook, students are required to work with numerical examples before generalising the conclusions for some properties of inequalities. In Secondary 1, students only need to know how to solve linear inequalities of the form ax  b, ax  b, ax < b and ax > b, where a and b are integers and a > 0. Teachers should get students to formulate inequalities based on real-world contexts (see the journal writing on page 128 of the textbook). 77 1 WORKED SOLUTIONS Journal Writing (Page 128) Journal Writing (Page 113) • 1. To solve an equation in x means to find the value of x so that the values on both sides of the equation are equal, i.e. x satisfies the equation. 2. The operations should be applied to both sides of the equation such that the equation is simplified to the form ax = b, where a and b are b constants. Thus x = . a • Teachers may wish to point out common mistakes that students may make in solving a linear equation in order to extract their understanding of the process. Thinking Time (Page 115) Some equivalent equations that have the solution y = –1: • y = –1 • y+1=0 • y – 1= –2 • 3y + 8 = 5 • 2y – 1 = –3 • 2y – 1 = –3 • 10y + 2 = 13y + 5 • 2(2y – 3) = 5(y – 1) Teachers may wish to note that the list is not exhaustive. Practise Now (Page 110) x+3 x+3–3 \x (b) x–7 x–7+7 \x (c) x+3 x+3–3 \x (d) x–2 x–2+2 \x (a) Investigation (Properties of Inequalities) 1. Cases Working Multiplication LHS = 10 × 5 by a positive = 50 number on RHS = 6 × 5 both sides of = 30 the inequality 10 > 6 Division by a positive number on both sides of the inequality 10 > 6 LHS = 10 ÷ 5 =2 Inequality 50 > 30 Is the inequality Conclusion sign reversed? No If x > y and c > 0, then cx > cy. 2x – 5 = 5 2x – 5 + 5 = 5 + 5 2x = 10 \x =5 (b) 3x + 4 = 7 3x + 4 – 4 = 7 – 4 3x = 3 \x =1 (c) –3x + 3 = 9 –3x + 3 – 3 = 9 – 3 –3x = 6 3x = –6 \ x = –2 (d) –5x – 2 = 13 –5x – 2 + 2 = 13 + 2 –5x = 15 5x = –15 \ x = –3 (a) 2 > 1.2 No RHS = 6 ÷ 5 = 1.2 If x > y and c > 0, then x y > . c c 2. Yes, the conclusions drawn from Table 5.3 apply to 10  6. The following conclusions hold for x  y: If x  y and c > 0, then cx  cy and The following conclusions hold for x < y: • If x < y and c > 0, then cx < cy and • If x  y and c > 0, then cx  cy and x y  . c c x y < . c c The following conclusions hold for x  y: 1 =7 =7–3 =4 =6 =6+7 = 13 = –7 = –7 – 3 = –10 = –3 = –3 + 2 = –1 Practise Now (Page 111) Table 5.3 • A bowl of rice contains 5 g of protein. A teenager needs a minimum of 49 g of protein each day. It is given that he only eats rice on a particular day. The inequality which we need to set up to find the least number of bowls of rice he needs to eat in order to meet his minimum protein requirement that day is: 5x  49, where x represents the number of bowls of rice he needs to eat that day. The flag-down fare of a taxi is $5. The taxi charges $0.30 for each 385 m it travels. A person has not more than $50 to spend on his taxi ride. The inequality which we need to set up to find the maximum distance that he can travel on the taxi is: 30x  4500, where x is the number of blocks of 385 m. x y  . c c 78 (d) Practise Now (Page 112) 3x + 4 = x – 10 3x – x + 4 = x – x – 10 2x + 4 = –10 2x + 4 – 4 = –10 – 4 2x = –14 \ x = –7 (b) 4x – 2 = x + 7 4x – x – 2 = x – x + 7 3x – 2 = 7 3x – 2 + 2 = 7 + 2 3x = 9 \x=3 (c) 3x – 2 = –x + 14 3x + x – 2 = –x + x + 14 4x – 2 = 14 4x – 2 + 2 = 14 + 2 4x = 16 \x =4 (d) –2x – 5 = 5x – 12 –2x – 5x – 5 = 5x – 5x – 12 –7x – 5 = –12 –7x – 5 + 5 = –12 + 5 –7x = –7 7x = 7 x=1 (a) –2(3x – 4) –6x + 8 –6x – 8x + 8 –14x + 8 –14x + 8 – 8 –14x 14x = 4(2x + 5) = 8x + 20 = 8x – 8x + 20 = 20 = 20 – 8 = 12 = –12 \x =– 6 7 Practise Now 1 x+9=4 x+9–9=4–9 \ x = –5 (b) 3x – 2 = 4 3x – 2 + 2 = 4 + 2 3x = 6 1. (a) 3x 6 = 3 3 \x =2 (c) 7x + 2 = 2x – 13 7x – 2x + 2 = 2x – 2x – 13 5x + 2 = –13 5x + 2 – 2 = –13 – 2 5x = –15 5x –15 = 5 5 \ x = –3 (d) 3(3y + 4) = 2(2y + 1) 9y + 12 = 4y + 2 9y – 4y + 12 = 4y – 4y + 2 5y + 12 = 2 5y + 12 – 12 = 2 – 12 5y = –10 Practise Now (Page 113) 2(x – 3) = –3x + 4 2x – 6 = –3x + 4 2x + 3x – 6 = –3x + 3x + 4 5x – 6 = 4 5x – 6 + 6 = 4 + 6 5x = 10 \x=2 (b) 2(x + 3) = 5x – 9 2x + 6 = 5x – 9 2x – 5x + 6 = 5x – 5x – 9 –3x + 6 = –9 –3x + 6 – 6 = –9 – 6 –3x = –15 3x = 15 \x=5 (c) –2(x + 2) = 3x – 9 –2x – 4 = 3x – 9 –2x – 3x – 4 = 3x – 3x – 9 –5x – 4 = –9 –5x – 4 + 4 = –9 + 4 –5x = –5 5x = 5 \x=1 (a) 5y –10 = 5 5 \ y = –2 (e) 2(y – 1) + 3(y – 1) = 4 – 2y 2y – 2 + 3y – 3 = 4 – 2y 2y + 3y – 2 – 3 = 4 – 2y 5y – 5 = 4 – 2y 5y + 2y – 5 = 4 – 2y + 2y 7y – 5 = 4 7y – 5 + 5 = 4 + 5 7y = 9 7y 9 = 7 7 2 \y=1 7 79 1 x + 0.7 = 2.7 x + 0.7 – 0.7 = 2.7 – 0.7 \x=2 (b) 2y – 1.3 = 2.8 2y – 1.3 + 1.3 = 2.8 + 1.3 2y = 4.1 2. (a) Practise Now 3 (a) 2y 4.1 = 2 2 \ y = 2.05 Practise Now 2 (a) =5–9 = –4 = 2 × (– 4) = –8 5 1 1 y+2= y+3 7 2 4 5 1 1 1 1 y– y+2= y– y+3 7 2 2 2 4 3 1 y+2=3 4 14 3 1 y+2–2 =3 –2 4 14 3 1 y=1 4 14 14 3 14 1 × ×1 y= 4 3 14 3 5 \y=5 6 3z – 1 z–4 (c) = 2 3 3z – 1 z–4 6× =6× 2 3 3(3z – 1) = 2(z – 4) 9z – 3 = 2z – 8 9z – 2z – 3 = 2z – 2z – 8 7z – 3 = –8 7z – 3 + 3 = –8 + 3 7z = –5 7z 5 =– 7 7 5 \z =– 7 (b) 1 =4 = (2x – 3) × 4 = 4(2x – 3) = 8x – 12 =8 = 8 + 12 = 20 20 8x = 8 8 1 \x =2 2 y–3 3 = (b) y+4 2 y–3 3 2(y + 4) × = 2(y + 4) × y+4 2 2(y – 3) = 3(y + 4) 2y – 6 = 3y + 12 2y – 3y – 6 = 3y – 3y + 12 –y – 6 = 12 –y – 6 + 6 = 12 + 6 –y = 18 \ y = –18 x +9 =5 2 x +9–9 2 x 2 x 2× 2 \x 8 2x – 3 8 (2x – 3) × 2x – 3 8 8 8x – 12 8x – 12 + 12 8x Practise Now 4 F = ma (a) When m = 1000, a = 0.05, F = 1000(0.05) = 50 N Net force acting on body = 50 N (b) When F = 100, a = 0.1, 100 = m(0.1) 100 0.1 = 1000 kg Mass of body = 1000 kg \m= 80 2. Let the number of marks Lixin obtains be x. Then the number of marks Devi obtains is x + 15. x + 15 = 2x x – 2x = –15 –x = –15 \ x = 15 Lixin obtains 15 marks. Practise Now 5 1. x 2 x + y – 3z = 2y y + 3x When x = 1, y = 4, 2 (1) + 4 – 3z 4 + 3 (1) 2 + 4 – 3z 4+3 6 – 3z 7 8(6 – 3z) 48 – 24z –24z –24z 1 2(4) 1 = 8 1 = 8 =7 =7 = 7 – 48 = – 41 = Practise Now 8 Let the number be x. 1 7 =7 x+3 5 10 1 7 x =7–3 5 10 1 3 x =3 5 10 3 \x =5×3 10 1 = 16 2 1 The number is 16 . 2 − 41 –24 17 =1 24 \z = v–u a 2. t = When t = 3, v = 2 1 1 ,u=1 , 2 3 1 1 –1 2 3 3= a 1 1 3= 6 a 2 Practise Now 9 1. (a) 15x > 75 x> 1 6 1 \a=1 ÷3 6 7 = 18 3a = 1 75 15 \x>5 4 5 6 7 –6 –5 –4 –3 8 9 (b) 4x  –16 –16 x 4 x  –4 Practise Now 6 1 (i) A = pr2 2 (ii) When r = 5, 2. 6x > 7 7 6 1 x >1 6 \ The smallest integer value of x is 2. x> 1 (3.142)(5)2 2 = 39.275 cm2 Area of semicircle = 39.275 cm2 A = Practise Now 10 Practise Now 7 Let the number of buses that are needed to ferry 520 people be x. Then 45x  520 1. Let the smaller number be x. Then the larger number is 5x. x + 5x = 24 6x = 24 520 45 5 x  11 9 \ The minimum number of buses that are needed to ferry 520 students is 12. x  24 6 =4 \ The two numbers are 4 and 20. x= 81 1 Exercise 5A 1. (a) (b) (c) (d) (e) (f) (g) 2. (a) (f) x + 8 = 15 x + 8 – 8 = 15 – 8 \x =7 x + 9 = –5 x + 9 – 9 = –5 – 9 \ x = –14 x – 5 = 17 x – 5 + 5 = 17 + 5 \ x = 22 y – 7 = –3 y – 7 + 7 = –3 + 7 \y =4 y + 0.4 = 1.6 y + 0.4 – 0.4 = 1.6 – 0.4 \ y = 1.2 y – 2.4 = 3.6 y – 2.4 + 2.4 = 3.6 + 2.4 y =6 –2.7 + a = – 6.4 –2.7 + 2.7 + a = – 6.4 + 2.7 \ a = –3.7 4x = –28 7 y 21 = 7 7 \y =3 (g) 4y – 1.9 = 6.3 4y – 1.9 + 1.9 = 6.3 + 1.9 4y = 8.2 4y 8.2 = 4 4 \ y = 2.05 (h) –3y – 7.8 = –9.6 –3y – 7.8 + 7.8 = –9.6 + 7.8 –3y = –1.8 3y = 1.8 3y 1.8 = 3 3 \ y = 0.6 3 1 = 4 2 3 3 1 3 7y – 2 + 2 = +2 4 4 2 4 1 7y = 3 4 7y 1 =3 ÷7 7 4 13 \y = 28 1 1 (j) 1 – 2y = 2 4 1 1 1 1 1 – 1 – 2y = –1 2 2 4 2 1 –2y = –1 4 1 2y = 1 4 2y 1 =1 ÷2 2 4 5 \y = 8 3. (a) 3x – 7 = 4 – 8x 3x + 8x – 7 = 4 – 8x + 8x 11x – 7 = 4 11x – 7 + 7 = 4 + 7 11x = 11 (i) 4x –28 = 4 4 \ x = –7 (b) –24x = –144 24x = 144 24 x 144 = 24 24 \x =6 (c) 3x – 4 = 11 3x – 4 + 4 = 11 + 4 3x = 15 3x 15 = 3 3 \x =5 (d) 9x + 4 = 31 9x + 4 – 4 = 31 – 4 9x = 27 9x 27 = 9 9 \x =3 (e) 12 – 7x = 5 12 – 12 – 7x = 5 – 12 –7x = –7 7x = 7 7y – 2 11x 11 = 11 11 \x =1 (b) 4x – 10 = 5x + 7 4x – 5x – 10 = 5x – 5x + 7 –x – 10 = 7 –x – 10 + 10 = 7 + 10 –x = 17 7x 7 = 7 7 1 \x=1 3 – 7y = –18 3 – 3 – 7y = –18 – 3 –7y = –21 7y = 21 82 (c) (d) 4. (a) (b) (c) (d) \ x = –10 2(2x – 2.2) = 4.6 4x – 4.4 = 4.6 4x – 4.4 + 4.4 = 4.6 + 4.4 4x = 9 4x 9 = 4 4 \ x = 2.25 (f) 4(3y + 4.1) = 7.6 12y + 16.4 = 7.6 12y + 16.4 – 16.4 = 7.6 – 16.4 12y = –8.8 \ x = –17 30 + 7y = –2y – 6 30 + 7y + 2y = –2y + 2y – 6 30 + 9y = – 6 30 – 30 + 9y = – 6 – 30 9y = –36 9y –36 = 9 9 \ y = –4 2y – 7 = 7y – 27 2y – 7y – 7 = 7y – 7y – 27 –5y – 7 = –27 –5y – 7 + 7 = –27 + 7 –5y = –20 5y = 20 5y 20 = 5 5 \y =4 2(x + 3) = 8 2x + 6 = 8 2x + 6 – 6 = 8 – 6 2x = 2 2x 2 = 2 2 \x=1 5(x – 7) = –15 5x – 35 = –15 5x – 35 + 35 = –15 + 35 5x = 20 5x 20 = 5 5 \x =4 7(–2x + 4) = – 4x –14x + 28 = – 4x –14x + 4x + 28 = – 4x + 4x –10x + 28 = 0 –10x + 28 – 28 = 0 – 28 –10x = –28 10x = 28 10 x 28 = 10 10 4 \x =2 5 3(2 – 0.4x) = 18 6 – 1.2x = 18 6 – 6 – 1.2x = 18 – 6 –1.2x = 12 1.2x = –12 1.2 x –12 = 1.2 1.2 (e) (g) (h) (i) (j) 12 y –8.8 = 12 12 11 \y =– 15 3(2y + 3) = 4y + 3 6y + 9 = 4y + 3 6y – 4y + 9 = 4y – 4y + 3 2y + 9 = 3 2y + 9 – 9 = 3 – 9 2y = –6 –6 2y = 2 2 \ y = –3 3(y + 1) = 4y – 21 3y + 3 = 4y – 21 3y – 4y + 3 = 4y – 4y – 21 –y + 3 = –21 –y + 3 – 3 = –21 – 3 –y = –24 \ y = 24 3(y + 2) = 2(y + 4) 3y + 6 = 2y + 8 3y – 2y + 6 = 2y – 2y + 8 y+6=8 y+6–6=8–6 \y=2 5(5y – 6) = 4(y – 7) 25y – 30 = 4y – 28 25y – 4y – 30 = 4y – 4y – 28 21y – 30 = –28 21y – 30 + 30 = –28 + 30 21y = 2 21y 2 = 21 21 2 \y = 21 (k) 2(3b – 4) = 5(b + 6) 6b – 8 = 5b + 30 6b – 5b – 8 = 5b – 5b + 30 b – 8 = 30 b – 8 + 8 = 30 + 8 83 1 (l) \ b = 38 3(2c + 5) = 4(c – 3) 6c + 15 = 4c – 12 6c – 4c + 15 = 4c – 4c – 12 2c + 15 = –12 2c + 15 – 15 = –12 – 15 2c = –27 (d) y –8+8 4 y 4 y 4× 4 \y 2c –27 = 2 2 1 \ c = –13 2 (m) 9(2d + 7) = 11(d + 14) 18d + 63 = 11d + 154 18d – 11d + 63 = 11d – 11d + 154 7d + 63 = 154 7d + 63 – 63 = 154 – 63 7d = 91 (c) 6. (a) 1 x =7 3 1 3× x=3×7 3 \ x = 21 3 x = –6 4 4 3 4 × x= × (– 6) 3 4 3 \ x = –8 (b) 1 x+3=4 3 1 x + 3 – 3= 4 – 3 3 1 x=1 3 1 3× x=3×1 3 \x=3 1 =6 =4×6 = 24 =2 =2–3 = –1 =1 =4×1 =4 2 (f) 15 – y = 11 5 2 15 – 15 – y = 11 – 15 5 2 – y = –4 5 2 y=4 5 5 2 5 x y= ×4 2 5 2 \ y = 10 7f –13 = 7 7 6 \ f = –1 7 (b) = –2 + 8 1 (e) 3– y 4 1 3–3– y 4 1 – y 4 1 y 4 1 4× y 4 \y 7d 91 = 7 7 \ d = 13 (n) 5(7f – 3) = 28( f – 1) 35f – 15 = 28f – 28 35f – 28f – 15 = 28f – 28f – 28 7f – 15 = –28 7f – 15 + 15 = –28 + 15 7f = –13 5. (a) y – 8 = –2 4 84 1 x 3 1 1 1 x + x = 12 – x + x 3 3 3 4 x = 12 3 3 4 3 × x= × 12 4 3 4 \x =9 x = 12 – 3 x 5 3 1 x– x 2 5 1 x 10 1 10 × x 10 \x 1 1 x+ 2 2 1 1 1 = x– x+ 2 2 2 1 = 2 1 = 10 × 2 =5 = (c) y 2 5y 6 y 2 y + 3 5y 6 1 – 5 1 5 1 – 5 1 – 5 1 + 5 5y 6 6 5y × 5 6 – –3(2 – x) = 6x – 6 + 3x = 6x – 6 + 3x – 6x = 6x – 6x – 6 –3x = 0 – 6 + 6 – 3x = 0 + 6 –3x = 6 3x = – 6 3x –6 = 3 3 \ x = –2 (b) 5 – 3x = – 6(x + 2) 5 – 3x = – 6x – 12 5 – 3x + 6x = – 6x + 6x – 12 5 + 3x = –12 5 – 5 + 3x = –12 – 5 3x = –17 3x –17 = 3 3 2 \ x = –5 3 (c) –3(9y + 2) = 2(– 4y – 7) –27y – 6 = –8y – 14 –27y + 8y – 6 = –8y + 8y – 14 –19y – 6 = –14 –19y – 6 + 6 = –14 + 6 –19y = –8 19y = 8 19 y 8 = 19 19 8 \y = 19 (d) –3(4y – 5) = –7(–5 – 2y) –12y + 15 = 35 + 14y –12y – 14y + 15 = 35 + 14y – 14y –26y + 15 = 35 –26y + 15 – 15 = 35 – 15 –26y = 20 26y = –20 8. (a) y 3 y y =2– + 3 3 =2– =2 =2+ =2 1 5 1 5 6 1 ×2 5 5 16 \y=2 25 2 3 5 (d) y– = 2y + 3 4 8 2 3 5 y – 2y – = 2y – 2y + 3 4 8 4 3 5 – y– = 3 4 8 4 3 3 5 3 – y– + = + 3 4 4 8 4 4 3 – y =1 3 8 4 3 y = –1 3 8 3 4 3  –1 3  × y= ×   8  4 3 4 1 \ y = –1 32 2 4 7. (a) = x 5 2 4 5x × = 5x × x 5 10 = 4x 4x = 10 = 4x 10 = 4 4 1 \x =2 2 12 2 (b) = y –1 3 12 2 3(y – 1) × = 3(y – 1) × y –1 3 36 = 2(y – 1) 36 = 2y – 2 2y – 2 = 36 2y – 2 + 2 = 36 + 2 2y = 38 2y 38 = 2 2 \ y = 19 26 y –20 = 26 26 10 \y =– 13 (e) 3(5 – h) – 2(h – 2) = –1 15 – 3h – 2h + 4 = –1 15 + 4 – 3h – 2h = –1 19 – 5h = –1 19 – 19 – 5h = –1 – 19 –5h = –20 5h = 20 5h 20 = 5 5 \h =4 85 1 9. (a) 5x + 1 3 5x + 1 3× 3 5x + 1 5x + 1 – 1 5x =7 (e) =3×7 = 21 = 21 – 1 = 20 5x 20 = 5 5 \x =4 (b) 2x – 3 4 2x – 3 12 × 4 3(2x – 3) 6x – 9 6x – 4x – 9 2x – 9 2x – 9 + 9 2x x–3 3 x–3 = 12 × 3 = 4(x – 3) = 4x – 12 = 4x – 4x – 12 = –12 = –12 + 9 = –3 = (f) 2x –3 = 2 2 1 \ x = –1 2 3x – 1 x –1 (c) = 5 3 3x – 1 x –1 15 × = 15 × 5 3 3(3x – 1) = 5(x – 1) 9x – 3 = 5x – 5 9x – 5x – 3 = 5x – 5x – 5 4x – 3 = –5 4x – 3 + 3 = –5 + 3 4x = –2 4x –2 = 4 4 1 \x =– 2 1 1 (d) (5y + 4) = (2y – 1) 4 3 1 1 12 × (5y + 4) = 12 × (2y – 1) 4 3 3(5y + 4) = 4(2y – 1) 15y + 12 = 8y – 4 15y – 8y + 12 = 8y – 8y – 4 7y + 12 = – 4 7y + 12 – 12 = – 4 – 12 7y = –16 9 y 22 = 9 9 4 \y=2 9 2y + 3 y–5 =0 + 4 6 2y + 3 y–5 y–5 5–y – =0– + 4 6 6 6 2y + 3 5–y =– 4 6 2y + 3 y–5 = 4 6 2y + 3 y–5 12 × = 12 × 4 6 3(2y + 3) = 2(5 – y) 6y + 9 = 10 – 2y 6y + 2y + 9 = 10 – 2y + 2y 8y + 9 = 10 8y + 9 – 9 = 10 – 9 8y = 1 8y 1 = 8 8 1 \y= 8 10. (a) 12 x+3 12 (x + 3) × x+3 12 12 2x + 6 2x + 6 – 6 2x =2 = (x + 3) × 2 = 2(x + 3) = 2x + 6 = 12 = 12 – 6 =6 6 2x = 2 2 \x =3 7y –16 = 7 7 2 \ y = –2 7 1 2y – 1 y+3 – =0 5 7 2y – 1 y+3 y+3 y+3 – + =0+ 5 7 7 7 2y – 1 y + 3 = 5 7 2y – 1 y+3 35 × = 35 × 5 7 7(2y – 1) = 5(y + 3) 14y – 7 = 5y + 15 14y – 5y – 7 = 5y – 5y + 15 9y – 7 = 15 9y – 7 + 7 = 15 + 7 9y = 22 86 (b) y+5 5 = y–6 4 y+5 5 = 4(y – 6) × 4(y – 6) × y–6 4 4(y + 5) = 5(y – 6) 4y + 20 = 5y – 30 4y – 5y + 20 = 5y – 5y – 30 –y + 20 = –30 –y + 20 – 20 = –30 – 20 –y = –50 \ y = 50 2y + 1 4 (f) = 3y – 5 7 2y + 1 4 = 7(3y – 5) × 7(3y – 5) × 3y – 5 7 7(2y + 1) = 4(3y – 5) 14y + 7 = 12y – 20 14y – 12y + 7 = 12y – 12y – 20 2y + 7 = –20 2y + 7 – 7 = –20 – 7 2y = –27 11 =4 2x – 1 11 (2x – 1) × = (2x – 1) × 4 2x – 1 11 = 4(2x – 1) 11 = 8x – 4 8x – 4 = 11 8x – 4 + 4 = 11 + 4 8x = 15 (e) 8 x 15 = 8 8 7 \x=1 8 32 1 (c) –3 = 2x – 5 4 32 1 –3+3 = +3 2x – 5 4 32 13 = 2x – 5 4 32 13 4(2x – 5) × = 4(2x – 5) × 2x – 5 4 128 = 13(2x – 5) 128 = 26x – 65 26x – 65 = 128 26x – 65 + 65 = 128 + 65 26x = 193 2y –27 = 2 2 1 \ y = –13 2 2 3 (g) = y–2 y+6 2 3 (y – 2)(y + 6) × = (y – 2)(y + 6) × y–2 y+6 26 x 193 = 26 26 11 \x =7 26 1 1 (d) = –1 2 x+2 1 1 –1 = x+2 2 1 1 +1 –1+1 = x+2 2 1 3 = x+2 2 1 3 2(x + 2) × = 2(x + 2) × x+2 2 2 = 3(x + 2) 2 = 3x + 6 3x + 6 = 2 3x + 6 – 6 = 2 – 6 3x = – 4 2(y + 6) = 3(y – 2) 2y + 12 = 3y – 6 2y – 3y + 12 = 3y – 3y – 6 –y + 12 = – 6 –y + 12 – 12 = – 6 – 12 –y = –18 \ y = 18 2 3 (h) = 7y – 3 9y – 5 2 3 (7y – 3)(9y – 5) × = (7y – 3)(9y – 5) × 7y – 3 9y – 5 2(9y – 5) 18y – 10 18y – 21y – 10 –3y – 10 –3y – 10 + 10 –3y 3y –4 3x = 3 3 1 \ x = –1 3 = 3(7y – 3) = 21y – 9 = 21y – 21y – 9 = –9 = –9 + 10 =1 =1 3y –1 = 3 3 1 \y =– 3 87 1 11. (a) 5x + 4 3 3(10 x ) – (5 x + 4) 3 30 x – 5 x – 4 3 25 x – 4 3 25 x – 4 3× 3 25x – 4 25x – 4 + 4 25x 10x – (d) =7 =7 =7 =7 =3×7 = 21 = 21 + 4 = 25 25 x 25 = 25 25 \x =1 (b) 4x x −1 – 3 2 2(4 x ) – 3( x – 1) 6 8 x – 3x + 3 6 5x + 3 6 5x + 3 12 × 6 2(5x + 3) 10x + 6 10x + 6 – 6 10x 1 4 =1 5 4 5 = 4 5 = 4 = = 12 × 13 y 1 = 13 13 1 \y= 13 6( y – 2) 2( y – 7) (e) – 12 = 7 3 6( y – 2) – 84 2( y – 7) = 7 3 6 y – 12 – 84 2( y – 7) = 7 3 6 y – 96 2( y – 7) = 7 3 6 y – 96 2( y – 7) 21 × = 21 × 7 3 3(6y – 96) = 14(y – 7) 18y – 288 = 14y – 98 18y – 14y – 288 = 14y – 14y – 98 4y – 288 = –98 4y – 288 + 288 = –98 + 288 4y = 190 5 4 = 15 = 15 = 15 – 6 =9 10 x 9 = 10 10 9 \x= 10 x –1 x+3 (c) – = –1 3 4 4( x – 1) – 3( x + 3) = –1 12 4 x – 4 – 3x – 9 = –1 12 4 x – 3x – 4 – 9 = –1 12 x – 13 = –1 12 x – 13 12 × = 12 × (–1) 12 x – 13 = –12 x – 13 + 13 = –12 + 13 \ x=1 1 y+5 3( y – 1) = 3 4 3 – ( y + 5) 3( y – 1) = 3 4 3– y–5 3( y – 1) = 3 4 –y + 3 – 5 3( y – 1) = 3 4 –y – 2 3( y – 1) = 3 4 –y – 2 3( y – 1) 12 × = 12 × 3 4 4(–y – 2) = 9(y – 1) – 4y – 8 = 9y – 9 – 4y – 9y – 8 = 9y – 9y – 9 –13y – 8 = –9 –13y – 8 + 8 = –9 + 8 –13y = –1 13y = 1 1– 4 y 190 = 4 4 1 \ y = 47 2 88 (f) 7 – 2y 2 1 – (2 – y) = 1 2 5 4 5(7 – 2 y ) – 4(2 – y ) 5 = 10 4 5 35 – 10 y – 8 + 4 y = 10 4 5 –10 y + 4 y + 35 – 8 = 10 4 5 –6 y + 27 = 10 4 5 –6 y + 27 20 × = 20 × 10 4 2(–6y + 27) = 25 –12y + 54 = 25 –12y + 54 – 54 = 25 – 54 –12y = –29 12y = 29 14. \ \x= = 4(7x – 4y) × 3 4 = 3(7x – 4y) = 21x – 12y = 21x – 21x – 12y = –12y = –12y + 20y = 8y = –8y x 8 =– y 9 3 x + 26 5 When x = 12, 1. y = 3 (12) + 26 5 1 = 33 5 y 2 – xz 2. a = 5 When x = 2, y = –1, z = –3, y= 1  19  5 + 3  20  6 19 5 + 60 6 =1 3 4 Exercise 5B 19 12. When x = , 20  19  3 LHS = 2   –  20  4 9 3 =1 – 10 4 3 = 1 20 = = 9x –8 y = 9 9 8y x =– 9 1 1  8y ×x = × – y y  9  12 y 29 = 12 12 5 \y=2 12 RHS = 3x – 5 y 7x – 4y 3x – 5 y 4(7x – 4y) × 7x – 4y 4(3x – 5y) 12x – 20y 12x – 21x – 20y –9x – 20y –9x – 20y + 20y –9x 9x (–1)2 – 2(–3) 5 1+ 6 = 5 7 = 5 2 =1 5 3. S = 4pr2 1 (i) When r = 10 , 2 2  22   1  S = 4  7   10 2  a= 3 = LHS 20 19 is the solution of the equation 20 5 3 1 = x+ . 4 3 6 13. 4x + y = 3x + 5y 4x – 3x + y = 3x – 3x + 5y x + y = 5y x + y – y = 5y – y x = 4y 3 3 ×x = × 4y 16 y 16 y 3x 3 \ = 16 y 4 2x – = 1386 89 1 (ii) When S = 616, 7. U = p(r + h)  22  616 = 4   r2  7 88 2 616 = r 7 When U = 16 16 1 2 22 (r + 2) 7 88 2 r = 616 7 7 r2 = × 616 88 2 r = 49 r+2 r+2 \ r = ± 49 \r = ±7 = 7 or –7 (N.A. since r > 0) 1 , h = 2, 2 22 = (r + 2) 7 1 = 16 2 7 1 = × 16 2 22 1 =5 4 1 =5 –2 4 1 4 =3 1 bh 2 (i) When b = 20, h = 45, 4. A = 8. v2 = u2 + 2gs When v = 25, u = 12, g = 10, 252 = 122 + 2(10)s 625 = 144 + 20s 144 + 20s = 625 20s = 625 – 144 20s = 481 1 (20)(45) 2 = 450 cm2 Area of triangle = 450 cm2 (ii) When A = 30, b = 10, A= 1 (10)h 2 30 = 5h 5h = 30 \h =6 Height of triangle = 6 cm 5. (a) P = xyz (b) S = p2 + q3 30 = 481 20 1 = 24 20 a 2c 9. –d= b b When a = 3, b = 4, d = –5, \s = 2c 3 – (–5) = 4 4 3 c 5 = 4 2 c 3 =5 2 4 m+n+ p+q (c) A = 4 (d) T = 60a + b p + 2q 3 When k = 7, q = 9, 6. k = \c =2×5 p + 2(9) 3 p + 18 7= 3 3 × 7 = p + 18 21 = p + 18 p + 18 = 21 \ p = 21 – 18 =3 7= = 11 10. 1 2 1 1 1 1 – = + a b c d 1 1 1 When a = , b = , d = – , 2 4 5 1 1 1 1 – = + 1 1  –1  c  5  4 2 2–4 = –2 –2 + 5 3 3c \c 1 90 3 4 1 –5 c 1 = –5 c 1 = c 1 = c =1 1 = 3 11. N = m x+q 14. When N = 1 4 , m = 9, x = 2, 5 9 4 1 = 2 + q 5 9 9 = 2+q 5 1 1 = 2+q 5 2+q =5 \q =5–2 =3 d–e a 12. c = – f –d b When a = 3, b = 4, c = – 6, d = –5 and e = 2, 6[ n (–2)2 – (–3)] = 5n –5 6(4 n + 3) = 5n –5 6(4n + 3) = –25n 24n + 18 = –25n 24n + 25n = –18 49n = –18 18 49 15. (i) Let the smallest odd number be n. The next odd number will be n + 2. The greatest odd number will be (n + 2) + 2 = n + 4. \ S = n + (n + 2) + (n + 4) =n+n+n+2+4 = 3n + 6 (ii) When the greatest odd number is –101, n + 4 = –101 \ n = –101 – 4 = –105 \ S = 3(–105) + 6 = –309 \n =– –5 – 2 3 – f – (–5) 4 –7 3 – –6 = f +5 4 7 3 –6 = + f +5 4 –6 = 3 4 3 –6 4 27 – 4 –27( f + 5) –27( f + 5) –27f – 135 –27f –27f –6 – m ( nx 2 – y ) = 5n z When m = 6, x = –2, y = –3, z = –5, 7 f +5 7 = f +5 7 = f +5 =4×7 = 28 = 28 = 28 + 135 = 163 = 16. (i) T = c × d + e × ef 100 –145 c (ii) e = 4–c When e = 150, = cd + 163 –27 1 = –6 27 150(4 – c) 600 – 150c –150c + 145c –5c b c–b When a = 3, c = 10, 13. a = 3(10 – b) 30 – 3b –3b – b – 4b \b – 600 –5 = 120 \c = b = 10 – b =b =b = –30 = –30 –30 = –4 =7 –145 c 4–c = –145c = –145c = – 600 = –600 150 = \f = 3 f 100 f +5 50 When d = 3, d= f +5 50 50 × 3 = f + 5 150 = f + 5 f + 5 = 150 \ f = 150 – 5 = 145 3= 1 2 \ T = 120(3) + = 577.50 91 150(145) 100 1 3. Let Priya’s age be x years old. Then Amirah’s age is (x + 4) years, Shirley’s age is (x – 2) years. x + (x + 4) + (x – 2) = 47 x + x + x + 4 – 2 = 47 3x + 2 = 47 3x = 47 – 2 3x = 45 45 x= 3 = 15 \ Priya is 15 years old, Amirah is 15 + 4 = 19 years old and Shirley is 15 – 2 = 13 years old. 4. Let the greater number be x. 2 Then the smaller number is x. 3 17. y = (x – 32) × 5 9 (i) When x = 134, y = (134 – 32) × 5 9 = 56.7 (to 3 s.f.) Required temperature = 56.7 °C (ii) When x = 0, 5 y = (0 – 32) × 9 = –17.8 (to 3 s.f.) Since 0 °F = –17.8 °C, it is less common for the temperature to fall below 0 °F because 0 °F is much lower than 0 °C. (iii) When y = –62.1, – 62.1 = (x – 32) × (x – 32) × 5 9 5 = – 62.1 9 x – 32 = –62.1 × x+ 9 5 x – 32 = –111.78 \ x = –111.78 + 32 = –79.78 = –79.8 (to 3 s.f.) \ The smaller number is Required temperature = –79.8 °F 5. Let the number be x. 3x = x + 28 3x – x = 28 2x = 28 Exercise 5C 1. Let the mass of the empty lorry be x kg. Then the mass of the bricks is 3x kg. x + 3x = 11 600 4x = 11 600 2 (27) = 18. 3 28 2 = 14 The number is 14. 6. Let the number of people going on the holiday be x. 15x = 84 + 12x 15x – 12x = 84 3x = 84 \x = 11 600 4 = 2900 \ The mass of the bricks is 3(2900) = 8700 kg. 2. Let the smallest odd number be n. The next odd number will be n + 2. Then the next odd number will be (n + 2) + 2 = n + 4. The greatest odd number will be (n + 4) + 2 = n + 6. n + (n + 2) + (n + 4) + (n + 6) = 56 n + n + n + n + 2 + 4 + 6 = 56 4n + 12 = 56 4n = 56 – 12 4n = 44 44 n= 4 = 11 \ The greatest of the 4 numbers is 11 + 6 = 17. x= 1 2 x = 45 3 5 x = 45 3 3 x= × 45 5 = 27 84 3 = 28 There are 28 people going on the holiday. 7. Let the number of boys who play badminton be x. Then the number of boys who play soccer is 3x. 3x – 12 = x + 12 3x – x = 12 + 12 2x = 24 \ x = 24 2 = 12 There are 12 boys who play badminton. \x= 92 13. Let Kate’s average speed for the first part of her journey be x km/h. Then her average speed for the second part of her journey is (x – 15) km/h. 8. Let the number be x. 1 9 x + 49 = x 2 4 1 9 x – x = – 49 2 4 7 – x = – 49 4 4 \ x = – × (– 49) 7 = 28 The number is 28. 9. Let the number be x. 68 – 4x = 3(x + 4) 68 – 4x = 3x + 12 – 4x – 3x = 12 – 68 –7x = –56 –56 \x = –7 =8 The number is 8. 10. Let the son’s age be x years. Then the man’s age is 6x years. 6x + 20 = 2(x + 20) 6x + 20 = 2x + 40 6x – 2x = 40 – 20 4x = 20 20 x= 4 =5 \ The man was 6(5) – 5 = 25 years old when his son was born. 11. Let the cost of a mooncake with one egg yolk be $x. Then the cost of a mooncake with two egg yolks is $(x + 2). 6(x + 2) + 5x = 130.8 6x + 12 + 5x = 130.8 6x + 5x = 130.8 – 12 11x = 118.8 Time taken for first part of journey = 350 hours. x 470 – 350 Time taken for second part of journey = x – 15 120 = hours. x – 15 350 120 = x x – 15 350(x – 15) = 120x 350x – 5250 = 120x 350x – 120x = 5250 230x = 5250 5250 x= 230 19 = 22 23 \ Kate’s average speed for the second part of her journey is 19 19 – 15 = 7 km/h. 23 23 14. Let the denominator of the fraction be x. Then the numerator of the fraction is x – 5. 22 \ The fraction is x – 5 +1 x +1 x–4 x +1 3(x – 4) 3x – 12 3x – 2x x x–5 . x 2 3 2 = 3 = 2(x + 1) = 2x + 2 = 2 + 12 = 14 = \ The fraction is 14 – 5 9 = . 14 14 15. Let the number in the tens place be x. Then the number in the ones place is 2.5x. \ The number is 10x + 2.5x = 12.5x. \ The number obtained when the digits are reversed is 10(2.5x) + x = 25x + x = 26x. 26x – 12.5x = 27 13.5x = 27 118.8 11 = 10.8 \ The cost of a mooncake with two egg yolks is $(10.8 + 2) = $12.80. 12. Let the number of 20-cent coins Jun Wei has be x. Then the number of 10-cent coins he has is x + 12. 10(x + 12) + 20x = 540 10x + 120 + 20x = 540 10x + 20x = 540 – 120 30x = 420 x= 27 13.5 =2 The number is 12.5(2) = 25. x= 420 30 = 14 \ Jun Wei has 14 + (14 + 12) = 40 coins. x= 93 1 Exercise 5D 4. 1. (a) If x > y, then 5x > 5y. (b) If x < y, then x y < . 20 20 (c) If x  y, then 3x > 3y. 1 \ The smallest rational value of y is 1 . 7 5. 20x > 33 (d) If x  y, then x y .  10 10 (e) If 15 > 5 and 5 > x, then 15 > x. (f) If x < 50 and 50 < y, then x < y. 2. (a) x x 62 4 2 4 3 14 15 15 1 16 2 –36 3 y < –12 5y > –24 –16 126 12 1 \ x > 10 2 (g) 2y < –5 7 –105 3 x < –35 \ The greatest odd integer value of x is –37. 2y > – 6 7. 5y < 20 and 17 19 18 –14 –15 –13 –12 –11 –6 –5 –4 4 –4 5 –3 –2 1. (a) 1 2 (h) 9y > –20 1 x 2 1 x =1 2 1 x =1 2 \x =2 (b) 2(x – 1) + 3(x + 1) = 4(x + 4) 2x – 2 + 3x + 3 = 4x + 16 5x + 1 = 4x + 16 5x – 4x = 16 – 1 \ x = 15 (c) 2y – [7 – (5y – 4)] = 6 2y – (7 – 5y + 4) = 6 2y – (11 – 5y) = 6 2y – 11 + 5y = 6 7y – 11 = 6 7y = 6 + 11 7y = 17 4 5 6 7 8 10 –10 1 11 2 12 13 –5 –4 –3 –2 1 –2 2 20 9 2 –3 2 –2 –1 0 \ y > –2 –2 9 9 3. Let the number of vans that are needed to ferry 80 people be x. Then 12x  80 y >– 17 7 3 =2 7 \y = 80 12 2 x6 3 \ The minimum number of vans that are needed to ferry 80 people is 7. x 1 x–1 = x– 5 2 y < –2 y– Review Exercise 5 –1 x> y<– 6 2 y  –3 20 5 y<4 \ The possible values are –3, –2, –1, 0, 1, 2 and 3. y> 28 x< 4 \x<7 (f) 12x > 126 6 y< \y< –24 5 4 \ y > –4 5 (e) 4x < 28 5 x< 1 \ x  15 2 (c) 3y < –36 (d) 33 20 13 x>1 20 \ The smallest value of x if x is a prime number is 2. 6. 3x < –105 x> 3x  18 18 3 \x6 (b) 4x  62 8  7y 7y  8 8 y 7 1 y 1 7 94 (d) 3 x – 5 = 0.5x 4 (i) 3 x – 0.5x = 5 4 1 x =5 4 \x =4×5 = 20 (e) 2y + 7 4 2y + 7 2y + 7 2y 2y = 12 = 4 × 12 = 48 = 48 – 7 = 41 41 \y = 2 1 = 20 2 4y – 1 5 (f) = 5y + 1 7 7(4y – 1) = 5(5y + 1) 28y – 7 = 25y + 5 28y – 25y = 5 + 7 3y = 12 12 \y = 3 =4 (g) (h) (j) 2c c –1 c+3 – = 9 6 12 2(2 c ) – 3( c – 1) c + 3 = 18 12 4 c – 3c + 3 c + 3 = 18 12 c+3 c+3 = 18 12 12(c + 3) = 18(c + 3) 2(c + 3) = 3(c + 3) 2c + 6 = 3c + 9 2c – 3c = 9 – 6 –c = 3 \ c = –3 2(3 – 4 d ) 3( d + 7) 1 – = 5d + 3 2 6 4(3 – 4 d ) – 9( d + 7) 6(5 d ) + 1 = 6 6 12 – 16 d – 9 d – 63 30 d + 1 = 6 6 –25 d – 51 30 d + 1 = 6 6 –25d – 51 = 30d + 1 –25d – 30d = 1 + 51 –55d = 52 \d = – a +1 a –1 =4 + 4 3 3( a + 1) + 4( a – 1) =4 12 3a + 3 + 4 a – 4 =4 12 7a – 1 =4 12 7a – 1 = 12 × 4 7a – 1 = 48 7a = 48 + 1 7a = 49 49 \a = 7 =7 52 55 2. (a) 18x < –25 –25 18 7 \ x < –1 18 (b) 10y  –24 x< 24 10 2 \ y > –2 5 3. 3(x – 1) – 5(x – 4) = 8 3x – 3 – 5x + 20 = 8 –2x + 17 = 8 –2x = 8 – 17 –2x = –9 y>– b–4 2b + 1 5b – 1 – = 3 6 2 2(b – 4) – (2 b + 1) 5b – 1 = 6 2 2b – 8 – 2b – 1 5b – 1 = 6 2 –9 5b – 1 = 6 2 –3 5b – 1 = 2 2 –3 = 5b – 1 5b – 1 = –3 5b = –2 2 \b=– 5 –9 –2 1 =4 2 1 1 1 \x–5 =4 –5 2 2 2 = –1 4. 4x  11 x= 11 4 3 x2 4 \ The smallest integer value of x is 3. x 95 1 5. 10. Let the smaller odd number be n. Then the greater number is n + 2. n + 2 + 5n = 92 6n + 2 = 92 6n = 92 – 2 6n = 90 90 n= 6 = 15 \ The two consecutive odd numbers are 15 and 17. 11. Let the mass of Object B be x kg. Then the mass of Object A is (x + 5) kg, the mass of Object C is 2(x + 5) kg. (x + 5) + x + 2(x + 5) = 255 x + 5 + x + 2x + 10 = 255 4x + 15 = 255 4x = 255 – 15 4x = 240 240 x= 4 = 60 \ The mass of Object C is 2(60 + 5) = 130 kg. 12. Let Farhan’s present age be x years. Then Farhan’s cousin’s present age is (38 – x) years. x – 7 = 3(38 – x – 7) x – 7 = 3(31 – x) x – 7 = 93 – 3x x + 3x = 93 + 7 4x = 100 100 \x = 4 = 25 Farhan is 25 years old now. 13. Let Raj’s present age be x years. Then Nora’s present age is 2x years, Ethan’s present age be is 2(2x) years. 2(2x) + 22 = 2(x + 22) 4x + 22 = 2x + 44 4x – 2x = 44 – 22 2x = 22 22 x= 2 = 11 \ Nora is 2(11) = 22 years old now. 14. Let the number of sweets that the man has to give to his son be x. 55 + x = 4(25 – x) 55 + x = 100 – 4x x + 4x = 100 – 55 5x = 45 45 \x = 5 =9 The man has to give 9 sweets to his son. 3y < –24 –24 y< 3 y < –8 \ The greatest integer value of y is –9. 6. 5x < 125 125 5 x < 25 \ The greatest value of x if x is divisible by 12 is 24. 7. 5y  84 x< y 84 5 4 5 \ The smallest value of y if y is a prime number is 17. y  16 4 3 pr 3 (i) When r = 7, 4  22  V =   (7)3 3 7  1 = 1437 3 1 (ii) When V = 113 , 7 4  22  3 1 113 =   r 3 7  7 1 88 3 r 113 = 7 21 88 3 1 r = 113 21 7 21 1 r3 = × 113 88 7 r3 = 27 \ r = 3 27 =3 8. V = 3y – n m When y = 5, m = –3, 9. n – 2y = n – 2(5) = n – 10 –3(n – 10) –3n + 30 –3n + n –2n 3(5) – n –3 15 – n = –3 = 15 – n = 15 – n = 15 – 30 = –15 –15 –2 1 =7 2 \n = 1 96 19. Let the number of tickets Jun Wei can buy be x. Then 12.50x < 250 15. Let the original price of each apple be x cents. 24x = (24 + 6)(x – 5) 24x = 30(x – 5) 24x = 30x – 150 24x – 30x = –150 – 6x = –150 –150 \x = –6 = 25 The original price of each apple is 25 cents. 16. Let the distance between Town A and Town B be x km. 45 minutes = 45 60 x x + 4 6 6x + 4x 24 10 x 24 5x 12 hour = 250 12.5 x < 20 \ The maximum number of tickets Jun Wei can buy with $250 is 20. x< 20. Let the first integer be x. Then the second integer will be (x + 1). x + (x + 1) < 42 2x < 41 3 hour 4 41 2 x < 20.5 \ The largest possible integer x can be is 20. 20 + 1 = 21 212 = 441 \ The square of largest possible integer is 441. x< 3 4 3 = 4 3 = 4 3 = 4 = 5x = 12 × 5x = 9 x= 21. Let Nora’s age be x years. Then Kate’s age is (x – 4) years. x + (x – 4) < 45 2x < 45 + 4 3 4 49 2 x < 24.5 \ Maximum possible age of Nora is 24 years. 24 – 4 = 20 \ The maximum possible age of Kate is 20 years. x< 9 5 =1 4 5 \ The main travels a total distance of 2 × 1 4 3 = 3 km. 5 5 17. Let the denominator of the fraction be x. Then the numerator of the fraction is x – 2. \ The fraction is 22. Let the number of ships needed to carry 400 passengers be . 60x > 400 x–2 . x 400 60 2 x>6 3 \ The minimum number of ships needed to carry 400 passengers is 7. x> x–2–3 x–3 x–5 x–3 4(x – 5) 4x – 20 4x – 3x x 3 = 4 3 = 4 = 3(x – 3) = 3x – 9 = –9 + 20 = 11 11 – 2 9 \ The fraction is = . 11 11 18. Let the number of sets of multimedia equipment that can be bought with $35 000 be x. Then 1900x  35 000 23. Let the number of pencils that can be bought with $27 be x. 2.50x < 27 27 2.5 4 x < 10 5 \ The maximum number of pencils that can be bought with $27 is 10. x< 35 000 1900 8 x  18 19 \ The maximum number of sets of multimedia equipment that can be bought with $35 000 is 18. x 97 1 4. A × B = 8 B × C = 28 C × D = 63 B × D = 36 Challenge Yourself 1. x +2 =0 x = –2 There is no solution since x cannot be a negative number. 2. Since (x + 2)2 and (y – 3)2 cannot be negative. (x + 2)2 = 0 and (y – 3)2 = 0 x+2 =0 and y–3=0 x = –2 and y=3 \ x + y = –2 + 3 =1 3. A + B = 8 — (1) B + C = 11 — (2) B + D = 13 — (3) C + D = 14 — (4) (2) – (3): B + C – B – D = 11 – 13 C – D = –2 — (5) (4) + (5): C + D + C – D = 14 + (–2) 2C = 12 12 \C = 2 (2) ÷ (3): 28 B×C = 63 C×D Since C cannot be equal to 0, then B 4 = , — (5) D 9 (4) × (5): B × D × B 4 = 36 × D 9 Since D cannot be equal to 0, then B2 = 16. \ B = ± 16 = 4 or – 4 (N.A. since B > 0) Substitute B = 4 into (1): A × 4 = 8 8 4 =2 Substitute B = 4 into (2): 4 × C = 28 \A= 28 4 =7 Substitute B = 4 into (4): 4 × D = 36 \C = =6 Substitute C = 6 into (4): 6 + D = 14 \ D = 14 – 6 =8 Substitute C = 6 into (2): B + 6 = 11 \ B = 11 – 6 =5 Substitute B = 5 into (1): A + 5 = 8 \A =8–5 =3 1 — (1) — (2) — (3) — (4) 36 4 =9 \D = 98 Chapter 6 Functions and Linear Graphs TEACHING NOTES Suggested Approach Although the topic on functions and linear graphs is new to most students, they do encounter examples of their applications in their daily lives, e.g. maps show the usage of Cartesian coordinates; escalators and moving walkways illustrate the concept of steepness. Teachers can get students to discuss about in detail these real-life examples. When students are able to appreciate their uses, teachers can proceed to introduce the concept of functions and linear graphs. Section 6.1: Cartesian Coordinates Teachers can build upon prerequisites, namely number lines to introduce the horizontal axis (x-axis) and the vertical axis (y-axis). Teachers can introduce this concept by playing a game (see Class Discussion: Battleship Game (Two Players)) to arouse students’ interest. Teachers should teach students not only on how to draw horizontal and vertical axes and plot the given points, but also to determine the position of points. Teachers can impress upon students that the first number in each ordered pair is with reference to the horizontal scale while the second number is with reference to the vertical scale. As such, students need to take note that the point (3, 4) has a different position compared to the point (4, 3). Section 6.2: Functions Teachers can use the Function Machine (see Investigation: Function Machine) to explore the concept of a function with the students and show that when a function is applied to any input x, it will produce exactly one output y. Once the students have understood the relationship between the input x and the output y, they are then able to represent the function using an equation, a table and a graph. Section 6.3: Graphs of Linear Functions Teachers should illustrate how a graph of a linear function is drawn on a sheet of graph paper. Teachers can impress upon students that when they draw a graph, the graph has to follow the scale stated for both the x-axis and y-axis and the graph is only drawn for the values of x stated in the range. Section 6.4: Applications of Linear Graphs in Real-World Contexts Teachers can give examples of linear graphs used in many daily situations and explain what each of the graphs is used for. Through Worked Example 2, students will learn how functions and linear graphs are applied in realworld contexts and solve similar problems The thinking time on page 151 of the textbook requires students to think further and consider if a negative value is possible or logical in the real world. Teachers should get the students to apply the answer of it to the other problems in real-world contexts. Challenge Yourself To further guide pupils to better understand the concept, teachers may modify the question to giving the x-coordinate of C. 99 1 WORKED SOLUTIONS Class Discussion (Ordered Pairs) Class Discussion (Battleship Game (Two Players)) 1. A single number is not sufficient to describe the exact position of a student in the classroom seating plan. For example, when the number 1 is used to indicate the position of a student in the classroom, the student could be either in row 1 or column 1. From Fig. 6.2, we can see that there are 11 possible positions of the student. Similarly, the location of a seat in a cinema cannot be represented by a single number. An example of a seating plan of a cinema is as shown: The purpose of this Battleship Game is to introduce students to the use of 2D Cartesian coordinates to specify points through an interesting and engaging activity. Teachers may wish to emphasise to students that they should call out a location on the grid by calling the letter before calling the number, e.g. D7 instead of 7D. Teachers may wish to use the grids provided (similar to that in Fig. 6.1) to conduct this activity. A B 1 C 2 D 3 E 4 F 5 G 6 H 7 I 8 J 1 9 B C D E F G H I 3 4 5 6 7 8 9 10 11 12 From the seating plan shown, both the number and the letter are required to represent the location of a seat in the cinema. 2. The order in which two numbers are written are important, i.e. (5, 3) and (3, 5) do not indicate the same position. 10 A 2 J Self Journal Writing (Page 137) 1 1. Guiding Questions: • How do you determine the locations of your house, a bus stop and a shopping mall in your neighbourhood on the map? • How can you obtain the approximate distances between your house, a bus stop and a shopping mall in your neighbourhood? 2. Guiding Questions: • What types of shops can normally be found on the ground floor of a shopping mall? • What is the size of each shop? How many spaces on the map should each shop occupy? • Are there any other considerations, e.g. walkways and washrooms, when designing the map? 3. Guiding Questions: • What types of horizontal and vertical scales are commonly used for the seating plan of a cinema in Singapore? • What are the different types of seats, e.g. wheelchair berths and couple seats, which can be found in a cinema? 2 3 4 5 6 7 8 9 10 A B C D E F G H I J Opponent 1 100 1. y = x + 3 2. (a) Input x = 4 → Output y = 4 + 3 = 7 (b) Input x = –7 → Output y = –7 + 3 = – 4 3. (a) Input x = 9 – 3 = 6 → Output y = 9 (b) Input x = 0 – 3 = –3 → Output y = 0 4. y 9. Investigation (Function Machine) 1 0 –1 –1 x 1 2 3 –2 x –7 –3 2 4 6 y –4 0 5 7 9 –3 –4 –5 Table 6.1 –6 y 5. –7 y = –2x – 1 y=x+3 Fig. 6.8 9 10. Every input x produces exactly one output y . 8 7 6 Thinking Time (Page 143) 5 1. y2 = x is not the equation of a function because • there are two values of y for every positive value of x, e.g. if the input x = 9, then the output y = ±3, • there is no value for the output y if the input x is negative. 2. It is possible for a function to have two input values x with the same output value y. Consider the equation of the function y = x2. If the input x = –3 or 3, then the output y = 9 . 4 3 2 1 –7 – 6 –5 – 4 –3 –2 –1 0 –1 x 1 2 3 4 5 6 –2 Class Discussion (Equation of a Function) –3 1. Since the point A lies on the graph of the function y = 2x, its coordinates satisfy the equation of the function y = 2x . Since the point B do not lie on the graph of the function y = 2x, its coordinates do not satisfy the equation of the function y = 2x . 2. Examples of coordinates of points that satisfy the equation of the function y = 2x include (2, 4), (3, 6), (4, 8), (0.5, 1) and (1.25, 2.5). 3. Amirah is correct to say that ‘the coordinates of every point on the line satisfy the equation of the function y = 2x’. Since a graph is a way to display a function, the coordinates of every point on the graph satisfy the equation of the function. –4 Fig. 6.6 The coordinates of every point on the straight line in Fig. 6.6 satisfy the equation of the function y = x + 3 . 6. Every input x produces exactly one output y . 7. y = –2x – 1 8. x –1 –0 .5 0 2 3 y 1 0 –1 –5 –7 Table 6.2 Thinking Time (Page 147) (i) The y-coordinate of each point that lies on the line y = 2x is twice its x-coordinate, i.e. the coordinates are given by (x, 2x) 101 1 (ii) (ii) When x = 35, y = 100 – 35 × 3 = 100 – 105 = –5 This means that 35 days after Nora receives her monthly allowance, she has –$5. Logically speaking, she should not have a negative amount of money. However, in the real world, it is possible for her to have –$5 as she may have borrowed $5 from her friends. y y=x+3 9 8 7 6 5 4 Practise Now (Page 138) 3 y 2 1 –7 – 6 –5 – 4 –3 –2 –1 0 –1 B(–2, 3) x 1 2 3 4 5 6 3 A(2, 2) 2 –2 1 –3 –2 –1 0 –1 –4 C(–1, –2) y –2 x 1 2 3 D(3, –1) Practise Now (Page 143) 1 0 –1 –1 1. (i) When x = 4, y = 2(4) – 3 =8–3 =5 (ii) When y = –5, –5 = 2x – 3 –5 + 3 = 2x –2 = 2x \ x = –1 2. (i) When x = 0, x 1 2 3 –2 –3 –4 –5 –6 –7 y = –2x – 1 Yes, I agree with Lixin. As shown above, the graphs of the functions y = x + 3 and y = –2x – 1 are straight lines. Hence, they are linear. 1 2 (0) – 3 5 2 =0– 5 2 =– 5 2 (ii) When y = – , 3 2 1 2 – =– x– 3 3 5 2 2 1 – + =– x 3 5 3 4 1 – =– x 15 3 4 \x = 5 y=– Thinking Time (Page 151) (i) When x = –2, y = 100 – (–2) × 3 = 100 + 6 = 106 This means that 2 days before Nora receives her monthly allowance, she has $106. However, in the real world, it is not possible for her to have more money before she receives her monthly allowance than when she receives her monthly allowance. 1 102 (ii) Amount of money the passenger has to pay if taxi travels 6 km = $3 + 6 × $0.50 = $3 + $3 = $6 (iii) Amount of money the passenger has to pay if taxi travels 10 km = $3 + 10 × $0.50 = $3 + $5 = $8 Practise Now 1 x 0 2 4 y = 2x + 1 1 5 9 1. (i) y y = 2x + 1 9 8 (b) 7 (ii) 6 5 x 3 6 10 y 4 .50 6 8 y (c) 4 3 8 2 1 7 x 0 1 2 3 4 6 (ii) From the graph in (i), when y = 6, q = x = 2 .5 2. 5 x –2 0 2 y = 3x –6 0 6 4 3 x –2 0 2 y = 2 – 2x 6 2 –2 2 y 1 y = 3x y = 2 – 2x 6 0 5 x 1 2 3 4 5 6 7 8 9 10 4 3 Exercise 6A 2 1. A(– 4, –3), B(–2, 4), C(3, 4), D(4, 2), E(1, 1), F(3, –3) y 2. 1 –2 –1 0 –1 x 1 2 A(2, 5) 5 –2 4 –3 F(–1, 2) –4 –5 3 2 B(1, 2) 1 –6 –3 –2 –1 0 –1 C(–2, –1) –2 Practise Now 2 x 1 2 3 E(3, –2) 4 5 6 D(6, –2) 3. (i) When x = 3, y = 4(3) + 5 = 12 + 5 = 17 (a) (i) Amount of money the passenger has to pay if taxi travels 3 km = $3 + 3 × $0.50 = $3 + $1.50 = $4.50 103 1 (ii) When x = –2, y = 4(–2) + 5 = –8 + 5 = –3 4. (i) When y = 34, 34 = 25 – 3x 3x = 25 – 34 3x = –9 \ x = –3 (ii) When y = –5, –5 = 25 – 3x 3x = 25 + 5 3x = 30 \ x = 10 y (c) 8 (0, 8) 7 6 5 (5, 4) 4 3 y 5. (a) (– 6, 4) (6, 4) 4 2 3 1 2 1 – 6 –5 – 4 –3 –2 –1 0 –1 1 2 3 4 5 x 0 (0, 0) x 6 1 2 4 y (d) –3 (–1, 4) –4 4 (6, – 4) 2 1 y 5 (0, 5) x 1 –2 (–5, –2) 3 The figure shows a quadrilateral. 2 1 – 6 –5 – 4 –3 –2 –1 0 –1 (1, 0) –5 – 4 –3 –2 –1 0 –1 4 (– 6, 0) (0, 3) 3 The figure shows a rectangle. (b) 5 The figure shows an isosceles triangle. –2 (– 6, – 4) 3 (6, 0) 1 2 3 4 5 y (e) (–1, 3) 3 x 6 (5, 2) 2 –2 1 –3 –1 0 –1 –4 –5 (0, –5) –2 The figure shows a rhombus. x 1 2 3 4 5 (5, –2) –3 (–1, –3) The figure shows a trapezium. 1 104 (b) (i) When y = 1, y 6. 8 C(1, 8) 2 1 x+ 3 3 2 = x 3 2 = x 3 =1 1 = 1 3 2 3 \x 7 1– 6 5 4 3 (ii) When y = – 2 1 A(1, 0) 0 1 B(7, 0) 2 3 4 5 6 1 6 1 1 – – 6 3 1 – 2 – x 7 Area of ABC = 1 ×6×8 2 = 24 units2 \x y 7. (–3, 7) 1 , 6 2 1 = x+ 3 3 2 = x 3 2 = x 3 3 =– 4 Exercise 6B 7 x 0 2 4 5 y = 2x + 8 8 12 16 4 y = 2x + 2 2 6 10 3 y = 2x – 3 –3 1 5 2 y = 2x – 6 –6 –2 2 1. (a) 6 (–2, 5) (–1, 3) 1 –3 –2 –1 0 –1 (0, 1) y x 1 y = 2x + 8 2 3 (1, –1) 16 14 –2 –3 (2, –3) 12 –5 y = 2x + 2 10 –4 8 (3, –5) y = 2x – 3 6 The points lie on a straight line. 8. (a) (i) When x = –3, 4 y = 2x – 6 2 2 1 (–3) + 3 3 1 = –2 + 3 2 = –1 3 1 (ii) When x = 1 , 2 2  1 1 y = 1  + 3 2 3 1 =1+ 3 1 =1 3 y= 0 –2 x 1 2 3 4 –4 –6 (b) They are parallel lines. 105 1 x –4 0 4 y = 3x + 7 –5 7 y = 3x + 5 –7 5 y = 3x – 3 –15 y = 3x – 6 –18 2. (a) x –4 0 4 19 y = –2x + 5 13 5 –3 17 y = –2x + 3 11 3 –5 –3 9 y = –2x – 4 4 –4 –12 –6 6 y = –2x – 7 1 –7 –15 3. (a) y y y = –2x + 5 y = 3x + 7 20 18 12 y = 3x + 5 16 y = –2x + 3 10 8 14 6 12 y = –2x – 4 y = 3x – 3 10 8 y = –2x – 7 4 2 y = 3x – 6 6 4 0 – 4 –3 –2 –1 –2 2 –4 0 – 4 –3 –2 –1 –2 –6 x 1 2 3 4 –8 –4 –10 –6 –12 –8 –14 –10 –16 –12 (b) They are parallel lines. –14 –16 –18 (b) They are parallel lines. 1 14 106 x 1 2 3 4 4. (a) x –4 0 4 y = – 4x + 8 24 8 –8 y = – 4x + 2 18 2 –14 y = – 4x – 3 13 –3 –19 y = – 4x – 6 10 –6 –22 x –3 0 3 y = 6 – 3x 15 6 –3 5. (i) y y = 6 – 3x 15 14 y 13 y = – 4x + 8 12 24 y = – 4x + 2 22 11 20 10 18 9 16 8 14 y = – 4x – 3 y = – 4x – 6 7 (ii) 12 6 10 5 8 4 6 3 4 2 1 2 0 – 4 –3 –2 –1 –2 (ii) x 1 2 3 –3 4 –2 –1 0 –1 –4 –2 –6 –3 –8 x 1 2 3 (ii) From the graph in (i), when y = 0, a=x=2 From the graph in (i), when x = –2, b = y = 12 From the graph in (i), when y = 1.5, c = x = 1 .5 –10 –12 –14 –16 –18 –20 –22 (b) y = mx + c1, y = mx + c2, y = mx + c3, y = mx + c4, where m, c1, c2, c3 and c4 are constants 107 1 x –2 0 y = 2x + 4 0 4 8 y = 2 – 3x 8 2 –4 6. 2. (a) (i) Distance car can travel if it has 3 l of petrol = 27 km (ii) Distance car can travel if it has 5.2 l of petrol = 47 km (b) Amount of petrol required to travel 36 km = 4 l \ Cost of petrol required to travel 36 km = 4 × $1.40 = $5.60 2 y y = 2 – 3x y = 2x + 4 3. (i) 8 7 5 4 3 30 50 70 C 100 200 300 400 Review Exercise 6 2 y 1. (a) 1 –1 10 (ii) There is a fixed overhead of $50. (iii) Amount of money Devi has to pay for 68 T-shirts = $390 (iv) Number of T-shirts Devi can order with $410 = 72 6 –2 N (–2, 6) x 0 –1 1 2 5 –2 4 –3 3 –4 (–2, 2) Exercise 6C x 3 6 10 y 105 90 70 2 3 5 4 (–2, 2) 3 (6, 2) 2 1 –2 –1 0 –1 –2 x 1 2 3 (2, –2) The figure shows a square. 100 90 80 70 60 50 40 30 20 10 x 1 2 3 4 5 6 4 (2, 6) 6 110 1 x 1 y (b) 120 0 (4, 2) 1 The figure shows a rectangle. y (c) 2 –2 –1 0 1. (a) (i) Amount of money left after 3 days = $120 – 3 × $5 = $120 – $15 = $105 (ii) Amount of money left after 6 days = $120 – 6 × $5 = $120 – $30 = $90 (iii) Amount of money left after 10 days = $120 – 10 × $5 = $120 – $50 = $70 (b) (4, 6) 6 7 8 9 10 108 4 5 6 y (c) (iii) When x = – (6, 8) 8  1 1 y = 4 –  – 1  2 2 1 =–2–1 2 1 = –3 2 4. (i) When y = 150, 150 = 250 – 20x 150 – 250 = –20x –100 = –20x \x =5 (ii) When y = 450, 450 = 250 – 20x 450 – 250 = –20x 200 = –20x \ x = –10 (iii) When y = –1150, –1150 = 250 – 20x –1150 – 250 = –20x –1400 = –20x \ x = 70 7 6 5 (–2, 4) (8, 4) 4 3 2 1 –2 –1 0 –1 x 1 2 3 4 5 6 7 8 –2 –3 –4 (2, – 4) The figure shows a trapezium. y (d) 7 (0, 7) (2, 7) 6 5 (2, 5) 4 1 , 2 x –3 0 3 1 y=2 x+3 2 – 4 .5 3 10 .5 5. (i) 3 2 (– 4, 1) y 1 – 4 –3 –2 –1 0 x 1 1 y=2 x+3 2 11 2 10 The figure shows a kite. 2. (a) A(–5, 0), B(– 4, 3), C(–3, 4), D(0, 5), E(3, 4), F(4, 3), G(4, –3), H(3, – 4), I(–3, – 4), J(– 4, –3) (b) (i) H (ii) G 3. (i) When x = 12, 9 8 7 6 5 1 y = 4(12) – 1 2 1 = 48 – 1 2 1 = 46 2 1 (ii) When x = 2 , 2 1  1 y = 4 2  – 1  2 2 1 = 10 – 1 2 1 =8 2 4 3 2 1 –3 –2 (ii) –1 0 –1 x 1 2 3 –2 –3 –4 –5 109 1 (ii) From the graph in (i), when x = –2, a = y = –2 From the graph in (i), when y = 3, b=x=0 Challenge Yourself y 6 B 4 2 A C2 –6 –4 –2 0 C1 2 4 6 x 8 Teachers can guide students to first draw the line y = 1 on a piece of graph paper. The base of the triangle will be AC and its height will be the perpendicular height from B to AC i.e 4 units. Hence, the base has to be 6 units. Counting 6 units to the right of A will give the point C1(7, 1), and counting 6 units to the left of A will give the point C2(–5, 1). 1 110 Chapter 7 Number Patterns TEACHING NOTES Suggested Approach Students have done word problems involving number sequences and patterns in primary school. These word problems required the students to recognise simple patterns from various number sequences and determine either the next few terms or a specific term. However, they were not taught to use algebra to solve problems involving number patterns. Teachers can arouse students’ interest in this topic by bringing in real-life applications (see chapter opener on page 157 and Investigation: Fibonacci Sequence). Section 7.1: Number Sequences In primary school, students were only asked how to find the next few terms and a specific term of number sequences but they have not been taught how to state the rule. Teachers can build upon this by getting students to work in pairs to state the rules of number sequences and then write down the next few terms (see Class Discussion: Number Sequences). Students should learn that they can add, subtract, multiply or divide or use a combination of arithmetic operations to get the next term of a number sequence. Section 7.2: General Term of a Number Sequence Teachers can build upon what students have learnt in Chapter 4 (Basic Algebra and Manipulation) and teach students how to observe a number sequence and look for a pattern so that they can use algebra and find a formula for the general term, Tn = nth term. Teachers can get students to work in pairs to find a formula for the general term and hence find a specific term for different number sequences (see Class Discussion: Generalising Simple Sequences). After the students have learnt how to generalise simple sequences, they should know that the aim is not to simply solve the problem but to represent it so that it becomes a general expression which can be used to find specific terms. Section 7.3: Number Patterns In primary school, students have attempted questions involving number patterns. In this section, teachers can ask the students to apply what they have learnt for number sequences on number patterns. Teachers can get students to work in pairs to find a formula for the general term and hence find a specific term for different number patterns (see Class Discussion: The Triangular Number Sequence). Through this class discussion, students should learn that they need not use a large number of coins to find the total number of coins needed to form a triangle with a base that has 100 coins. They need only to find the formula for the general term and they are able to find the total number of coins by substituting n = 100 into the formula. They should also learn that with the formula, they can find Tn easily for any n. Section 7.4: Number Patterns in Real-World Contexts Teachers may get students to discover number patterns in real-world contexts (e.g. shells, pine cones, rocks, wallpaper, floor tiles) and ask them to represent that number pattern into a general expression. Through Worked Example 5, students will learn that in the real world, which in this case in Chemistry, the general term of a number sequence is important and advantageous in finding specific terms. In this worked example, finding the general term of the number of hydrogen atoms allowed one to find the member number, number of carbon atom(s) and number of hydrogen atoms easily without going through tedious workings, especially if the value of the specific term is large. For other figures, students should consider drawing the next figure in the sequence so as to identify the pattern. 111 1 Challenge Yourself Some of the questions (e.g. Questions 3, 4 and 5) are not easy for average students while others (Questions 1 and 2) should be manageable if teachers guide them as follows: Questions 1 and 2: Teachers can get the students to draw a table and write down the first 6 terms. The students have to observe carefully how each term in the sequence can be obtained and find a formula for the general term to get the final answer to the question. Questions 3, 4 and 5: Teachers have to get the students to think beyond just the four operations. The students have to consider more ways and observe carefully how each term in the sequence is obtained. Once they have figured this out, they are able to search on the Internet to find out the names for the sequences. 1 112 WORKED SOLUTIONS Figure Number, n Number of Coins at the Base of the Triangle, n 1 1 1 =1 = 1× 2 2 2 2 1+2 =3 = 2×3 2 3 3 1+2+3 =6 = 3× 4 2 4 4 1+2+3+4 = 10 = 4×5 2 5 5 1+2+3+4+5 = 15 = 5×6 2 Start with 1, then multiply each term by 2 to get the next term . 6 6 1 + 2 + 3 + 4 + 5 + 6 = 21 = 6×7 2 : : 1, 3, 9, 27, 81, 243, 729, … Start with 1, then multiply each term ×3 ×3 ×3 ×3 ×3 ×3 by 3 to get the next term . n n 2. Class Discussion (Number Sequences) 1. Sequence Positive even numbers 2, 4, 6, 8, 10, 12, 14, … Positive odd numbers 1, 3, 5, 7, 9, 11, 13, …      +2 +2 +2 +2 +2 +2      +2 +2 +2 +2 +2 +2 Multiples of 3 3, 6, 9, 12, 15, 18, 21, … Powers of 2 1, 2, 4, 8, 16, 32, 64, …      +3 +3 +3 +3 +3 +3      ×2 ×2 ×2 ×2 ×2 ×2 Start with 2, then add 2 to each term to get the next term . Start with 1, then add 2 to each term to get the next term . Start with 3, then add 3 to each term to get the next term .     Powers of 3 Rule : 1 + 2 + 3 + 4 + ··· + n = 1 n(n + 1) 2 Table 7.6 3. When n = 100, Table 7.1 1 1 n(n + 1) = × 100 × (100 + 1) 2 2 1 = × 100 × 101 2 = 5050 Total number of coins needed to form a triangle with a base that has 100 coins = 5050 2. The sequence of positive odd numbers can be obtained by subtracting 1 from each term of the sequence 2, 4, 6, 8, 10, … . Teachers may wish to note that there are other possible answers to this question. 3. (a) Rule: Find the square of the position of each term . The next two terms are 36 and 49 . (b) Rule: Find the cube of the position of each term . The next two terms are 216 and 343 . Investigation (Fibonacci Sequence) 1. 2. 3. 4. Class Discussion (Generalising Simple Sequences) (a) Hence, Tn = 3n . 100th term, T100 = 3 × 100 = 300 2 (b) Hence, Tn = n . 100th term, T100 = 1002 = 10 000 (c) Hence, Tn = n3 . 100th term, T100 = 1003 = 1 000 000 1; 5; 13; 21 3, 5, 8, 13, 21, 34 Michaelmas Daisy has 55 petals . 4, 6; 7, 10 Journal Writing (Page 169) Pascal’s Triangle was developed by the French Mathematician Blaise Pascal . It is formed by starting with the number 1 . Each number in the subsequent rows is obtained by finding the sum of the number which is diagonally above it to the left and that which is diagonally above it to the right . 0 is used as a substitute in the absence of a number in either of the two positions . Class Discussion (The Triangular Number Sequence) 1 1 1. 1 1 Figure 5 Total Number of Coins, Tn Figure 6 1 1 1 1 113 5 7 2 21 10 5 8 13 21 1 4 10 20 35 3 1 3 6 15 2 1 3 4 6 1 1 5 15 35 1 6 21 1 7 1 1 (d) Since the common difference is 3, Tn = 3n + ? . The term before T1 is c = T0 =1–3 = –2 . \ General term of sequence, Tn = 3n – 2 2. (i) 23, 27 (ii) Since the common difference is 4, Tn = 4n + ? . The term before T1 is c = T0 =3–4 = –1 . \ General term of sequence, Tn = 4n – 1 (iii) T50 = 4(50) – 1 = 200 – 1 = 199 The Fibonacci sequence is a set of numbers that begins with 1 and 1, and each subsequent term is the sum of the previous two terms, i .e . 1, 1, 2, 3, 5, 8, 13, 21, … The sums of the numbers on the diagonals of Pascal’s Triangle form the Fibonacci sequence, as illustrated . Teachers may wish to get students to describe the symmetry in Pascal’s Triangle and to identify other patterns that can be observed from the triangle. Practise Now 1 1. (a) Rule: Add 5 to each term to get the next term . The next two terms are 28 and 33 . (b) Rule: Subtract 6 from each term to get the next term . The next two terms are –50 and –56 . (c) Rule: Multiply each term by 3 to get the next term . The next two terms are 1215 and 3645 . (d) Rule: Divide each term by –3 to get the next term . The next two terms are –18 and 6 . 2. (a) 22, 29 (b) 15, 11 Practise Now 4 1. (i) Figure 5 Figure 6 Practise Now 2 (ii) (i) T4 = 4(4) + 7 = 16 + 7 = 23 (ii) T7 = 4(7) + 7 = 28 + 7 = 35 Sum of 4th term and 7th term of sequence = T4 + T7 = 23 + 35 = 58 Practise Now 3 Number of Dots 1 2+1×4=6 2 2 + 2 × 4 = 10 3 2 + 3 × 4 = 14 4 2 + 4 × 4 = 18 5 2 + 5 × 4 = 22 6 2 + 6 × 4 = 26 : : n 2 + n × 4 = 4n + 2 (iii) When n = 2013, 4n + 2 = 4(2013) + 2 = 8054 Number of dots in 2013th figure = 8054 2. (i) 8th line: 72 = 8 × 9 (ii) Since 110 = 10 × 11 = 10(10 + 1), k = 10 . 1. (a) Since the common difference is 4, Tn = 4n + ? . The term before T1 is c = T0 =5–4 = 1 . \ General term of sequence, Tn = 4n + 1 (b) Since the common difference is 5, Tn = 5n + ? . The term before T1 is c = T0 =7–5 = 2 . \ General term of sequence, Tn = 5n + 2 (c) Since the common difference is 6, Tn = 6n + ? . The term before T1 is c = T0 =2–6 = – 4 . \ General term of sequence, Tn = 6n – 4 1 Figure Number Practise Now 5 (i) 114 Member Number Number of carbon atoms Number of hydrogen atoms 1 2 4 2 3 6 3 4 8 4 5 10 5 6 12 6 7 14 : : : n n+1 2n + 2 (ii) Let h + 1 = 55 . h = 55 – 1 = 54 When n = h = 54, 2n + 2 = 2(54) + 2 = 110 Number of hydrogen atoms the member has = 110 (iii) Let 2k + 2 = 120 . 2k = 120 – 2 = 118 k = 59 When n = k = 59, n + 1 = 59 + 1 = 60 Number of carbon atoms the member has = 60 Exercise 7B 1. (a) Since the common difference is 6, Tn = 6n + ? . The term before T1 is c = T0 =7–6 = 1 . \ General term of sequence, Tn = 6n + 1 (b) Since the common difference is 3, Tn = 3n + ? . The term before T1 is c = T0 = –4 – 3 = –7 . \ General term of sequence, Tn = 3n – 7 (c) Since the common difference is 7, Tn = 7n + ? . The term before T1 is c = T0 = 60 – 7 = 53 . \ General term of sequence, Tn = 7n + 53 (d) Since the common difference is –3, Tn = –3n + ? . The term before T1 is c = T0 = 14 + 3 = 17 . \ General term of sequence, Tn = –3n + 17 2. (i) T5 = 2(5) + 5 = 10 + 5 = 15 (ii) T8 = 2(8) + 5 = 16 + 5 = 21 (iii) 15 = 3 × 5 21 = 3 × 7 LCM of 5th term and 8th term of sequence = 3 × 5 × 7 = 105 3. (i) 18, 21 (ii) Since the common difference is 3, Tn = 3n + ? . The term before T1 is c = T0 =3–3 = 0 . \ General term of sequence, Tn = 3n (iii) T105 = 3(105) = 315 4. (i) 30, 34 (ii) Since the common difference is 4, Tn = 4n + ? . The term before T1 is c = T0 = 10 – 4 = 6 . \ General term of sequence, Tn = 4n + 6 (iii) T200 = 4(200) + 6 = 800 + 6 = 806 Exercise 7A 1. (a) Rule: Add 5 to each term to get the next term . The next two terms are 39 and 44 . (b) Rule: Subtract 8 from each term to get the next term . The next two terms are 40 and 32 . (c) Rule: Multiply each term by 2 to get the next term . The next two terms are 384 and 768 . (d) Rule: Divide each term by 2 to get the next term . The next two terms are 50 and 25 . (e) Rule: Divide each term by – 4 to get the next term . The next two terms are 16 and – 4 . (f) Rule: Multiply each term by –2 to get the next term . The next two terms are –288 and 576 . (g) Rule: Subtract 7 from each term to get the next term . The next two terms are –87 and –94 . (h) Rule: Add 10 to each term to get the next term . The next two terms are –50 and – 40 . (i) Rule: Add 10 to each term to get the next term . The next two terms are 50 and 60 . (j) Rule: Add 7 to each term to get the next term . The next two terms are 80 and 87 . (k) Rule: Multiply each term by 3 to get the next term . The next two terms are 324 and 972 . 2. (a) 9, 15 (b) 12, 8 (c) –33, –32 (d) 88, 85 (e) 21, 28 3. (a) –67, –131 (b) 8, 13 (c) 144, 196 (d) –216, 343 (e) 81, 243 115 1 (b) (i) General term of sequence, Tn = 2n2 + 1 – 2 = 2n2 – 1 2 (ii) T388 = 2(388) – 1 = 301 088 – 1 = 301 087 8. (i) 5. (i) Number of points 1 2 3 4 5 6 Number of segments 1+1 =2 2+1 =3 3+1 =4 4+1 =5 5+1 =6 6+1 =7 (ii) Let the number of points be n . Number of segments = n + 1 . When n = 49, number of segments = 49 + 1 = 50 (iii) 101 = n + 1 \ n = 101 – 1 = 100 Figure 5 Figure 6 6. (i) (ii) Figure 5 (ii) Figure Number Number of Intersection(s) between the Circles 1 0 2 1 3 2 4 3 5 4 6 5 : : n n–1 Number of Small Triangles 1 4 2 9 3 16 4 25 5 36 6 49 : : n (n + 1)2 (iii) When n = 20, (n + 1)2 = (20 + 1)2 = 212 = 441 Number of triangles in 20th figure = 441 (iv) Let (n + 1)2 = 121 . n + 1 = 11 or n + 1 = –11 n = 11 – 1 or n = –11 – 1 = 10 or = –12 (N .A . since n > 0) 9. (i) 6th line: 54 = 6 × 9 (ii) Since 208 = 13 × 16 = 13(13 + 3), k = 13 . 10. (i) 5th line: 1 + 3 + 5 + 7 + 9 + 11 = 36 = 62 = (5 + 1)2 (iii) Let n – 1 = 28 . n = 28 + 1 = 29 7. (a) When n = 1, 2n2 + 1 = 2(1)2 + 1 =2+1 =3 When n = 2, 2n2 + 1 = 2(2)2 + 1 =8+1 =9 When n = 3, 2n2 + 1 = 2(3)2 + 1 = 18 + 1 = 19 When n = 4, 2n2 + 1 = 2(4)2 + 1 = 32 + 1 = 33 The first four terms of the sequence are 3, 9, 19 and 33. 1 Figure Number Figure 6 (ii) c = 169 = 13 d + 1 = 13 d = 13 – 1 = 12 a = 13 + 12 = 25 116 11 . (a) (i) (ii) Number of people 4 6 8 10 12 14 Number of tables 4–2 =1 2 6–2 =2 2 8–2 =3 2 10 – 2 =4 2 12 – 2 =5 2 14 – 2 =6 2 Number of tables 1 2 3 4 5 6 Number of people 2(1) + 2 = 4 2(2) + 2 = 6 2(3) + 2 = 8 2(4) + 2 = 10 2(5) + 2 = 12 2(6) + 2 = 14 (b) (i) From (a)(i): When n = 20, n – 2 20 – 2 = 2 2 =9 \ 9 tables will be needed to seat 20 people . (ii) When n = 30, n–2 30 – 2 = 2 2 = 14 \ 14 tables will be needed to seat 30 people . (c) (i) From (a)(ii): When n = 22, 2(22) + 2 = 46 \ 46 people can be seated at 22 tables . (ii) When n = 36, 2(36) + 2 = 74 \ 74 people can be seated at 36 tables . 12 . (i) Number of points on the line segments AB (including the points A and B) Number of possible line segments 2 3 4 5 6 7 2 × (2 – 1) 2 3 × (3 – 1) 2 4 × (4 – 1) 2 5 × (5 – 1) 2 6 × (6 – 1) 2 7 × (7 – 1) 2 = 10 = 15 =1 =3 =6 = 21 (ii) Number of points including AB = 18 + 2 = 20 20 × (20 – 1) 2 = 190 Number of possible line segments = 117 1 13. (i) 1 (ii) 5 Row 10 10 5 1 Sum 1 1 = 1 = 20 2 1 + 1 = 2 = 21 3 1 + 2 + 1 = 4 = 22 4 1 + 3 + 3 + 1 = 8 = 23 5 1 + 4 + 6 + 4 + 1 = 16 = 24 6 1 + 5 + 10 + 10 + 5 + 1 = 32 = 25 : n 14. (a) : 1 + (n – 1) + ··· + (n – 1) + 1 = 2n – 1 Figure 1 Number of black 1 squares (b) Number of white 1×2+1=3 squares (w) Area of whole 4 figure (b + w) Perimeter of 2(1 + 4) = 10 whole figure (cm) 2 3 4 5 6 2 3 4 5 6 2×2+1=5 3×2+1=7 4×2+1=9 7 10 13 16 19 2(2 + 4) = 12 2(3 + 4) = 14 2(4 + 4) = 16 2(5 + 4) = 18 2(6 + 4) = 20 (b) (i) Number of white squares in Figure 9 = 9 × 2 + 1 = 19 (ii) Perimeter of Figure 9 = 2(9 + 4) = 26 cm (iii) Number of white squares in Figure n = n (2 + 1) = 2n + 1 (iv) Perimeter of Figure n = 2(n + 4) = (2n + 8) cm 15. (i) 8th line: 16. (a) (i) 11, 13 (ii) 24, 28 (iii) 84, 112 (iv) 85, 113 (b) 6th line: 132 + 842 = 852 7th line: 152 + 1122 = 1132 17. (i) 1 2 1 2 = – + 8 9 10 8 × 9 × 10 (ii) Based on the pattern, nth line: 2 1 2 1 – + = n ( n + 1)( n + 2) n n +1 n+2 1 2 1 2 \ – + = 10 11 12 10 × 11 × 12 2 = 1320 1 = 660 2 1 2 1 (iii) = – + p +1 p+2 7980 p 2 2 1 1 – + = p ( p + 1)( p + 2) p +1 p+2 p Member Number Number of carbon atoms Number of hydrogen atoms 1 3 4 2 4 6 3 5 8 4 6 10 5 7 12 6 8 14 : : : n n+2 2n + 2 (ii) Let h + 2 = 25 . h = 25 – 2 = 23 When n = h = 23, 2n + 2 = 2(23) + 2 = 48 Number of hydrogen atoms the member has = 48 \ p(p + 1)(p + 2) = 7980 (p2 + p)(p + 2) = 7980 p3 + 2p2 + p2 + 2p – 7980 = 0 p3 + 3p2 + 2p – 7980 = 0 \ p = 19 or p = –11 + 17 .292 (5 s .f .) (reject, p is a whole number) or p = –11 – 17 .292 (5 s .f .) (reject, p > 1) 1 5 × 2 + 1 = 11 6 × 2 + 1 = 13 118 (iii) Let 2k + 2 = 64 . 2k = 64 – 2 = 62 k = 31 When n = k = 31, n + 2 = 31 + 2 = 33 Number of carbon atoms the member has = 33 18. (i) 3. (i) Figure 5 (ii) M 1st Generation Ancestor F M 2nd Generation Ancestors F F M M F F M F 4rd Generation Ancestors Number of 4th generation ancestors a male bee has = 5 (ii) The number of nth generation ancestors forms a sequence: 1, 2, 3, 5, ... The first two numbers of the sequence are 1 and 2, and each subsequent term is the sum of the previous two terms . (iii) Number of 5th generation ancestors a male bee has = 3 + 5 =8 M F 1 5×1+1=6 2 5 × 2 + 1 = 11 3 5 × 3 + 1 = 16 4 5 × 4 + 1 = 21 5 5 × 5 + 1 = 26 : : n 5 × n + 1 = 5n + 1 1st Generation Ancestor M F F Number of Buttons (iii) When n = 56, 5n + 1 = 5(56) + 1 = 281 Number of buttons in 56th figure = 281 (iv) Let 5n + 1 = 583 . 5n = 583 – 1 = 582 2 n = 116 5 2 Since n = 116 ∉ Z+, it is not possible for a figure in the 5 sequence to be made up of 583 buttons . 4. (i) 3rd Generation Ancestors F Figure Number M 2nd Generation Ancestors F 3rd Generation Ancestors Figure 5 F M F M F M F M F F F M (ii) 4rd Generation Ancestors F 5th Generation Ancestors (iv) The sequence for the number of nth generation ancestors is 1, 2, 3, 5, 8, 13, 21, 34, 55, . . . Number of 10th generation ancestors a male bee has = 34 + 55 = 89 Review Exercise 7 1. (a) 53, 44 (b) 28, 40 1 1 , 27 81 (d) 121, 169 2. (i) 64, 81 (ii) General term of sequence, Tn = (n + 2)2 (iii) T25 = (25 + 2)2 = 272 = 729 (c) Figure Number Number of Triangles 1 1= 1× 2 2 2 3= 2×3 2 3 6= 3× 4 2 4 10 = 4×5 2 5 15 = 5×6 2 : : n 1 n(n + 1) 2 (iii) When n = 77, 1 1 n(n + 1) = × 77 × (77 + 1) 2 2 1 = × 77 × 78 2 = 3003 Number of triangles in 77th figure = 3003 119 1 1 n(n + 1) = 66 . 2 n(n + 1) = 132 Since 132 = 11 × 12 = 11(11 + 1), n = 11 . 5. (i) 7th line: 13 + 23 + 33 + 43 + 53 + 63 + 73 = 784 = (1 + 2 + 3 + 4 + 5 + 6 + 7)2 3 3 (ii) 1 + 2 + 33 + 43 + ··· + 153 = (1 + 2 + 3 + 4 + ··· + 15)2 = 1202 = 14 400 3 3 3 3 (iii) Since 1 + 2 + 3 + 4 + ··· + k3 = 1296 = 362 = (1 + 2 + 3 + 4 + ··· + 8)2, k = 8 . (iv) Let 12 + 35 2 6. (i) a = = 2. Number of Handshakes 2 1= 2 ×1 2 3 3= 3× 2 2 4 6= 4×3 2 5 10 = 5×4 2 6 15 = 6×5 2 : : n 1 n(n – 1) 2 2 1369 Number of handshakes that will take place = = 37 (ii) Since the common difference is 2, TnA = 2n + ? . The term before T1A is c = T0A =4–2 = 2 . \ General term of sequence A, TnA = 2n + 2 (iii) General term of sequence C, TnC = = (2 n + 2)2 + ( n 2 + 2 n )2  2 (18 ) + 2  + 18 2 + 2 (18 )  = 38 2 + 360 2 = 131 044 2 = 362 Challenge Yourself Value 31 32 33 34 35 36 ··· Last Digit 3 9 7 1 3 9 ··· 2015 ÷ 4 = 503 R 3 \ Last digit of 32015 = 7 1 1 n(n – 1) 2 3. (i) 4, 9 (ii) The general term, Tn, of the sequence is obtained by continuously finding the sum of the digits of n2 until a single-digit number is left, e .g . to obtain T7, 72 = 49 → 4 + 9 = 13 → 1 + 3 = 4, \ T7 = 4 . 4. (i) 11, 18 (ii) For n  3, Tn = Tn – 1 + Tn – 2 . (iii) Lucas Numbers (which is different from Lucas Sequence) 5. (i) 10, 12 (ii) For n  4, Tn = Tn – 2 + Tn – 3 . (iii) Perrin Numbers (or Perrin Sequence) Tn2A + Tn2B 2 T18C = 1. Number of People 120 5. Let the time the motorist spends on the expressway be x hours . Then the time he spends on the stretch of road is 2x hours . 95x + 65 × 2x = 375 95x + 130x = 375 225x = 375 Revision Exercise B1 1. (a) 0 .15x + 2 .35(x – 2) = 1 .3 0 .15x + 2 .35x – 4 .7 = 1 .3 2 .5x – 4 .7 = 1 .3 2 .5x = 1 .3 + 4 .7 2 .5x = 6 \ x = 2 .4 7 5 (b) – =4 2 – 2y 1– y 5 7 =4 – 1– y 2(1 – y ) 10 – 7 =4 2(1 – y ) 3 =4 2(1 – y ) 3 = 8(1 – y) 3 = 8 – 8y 3 – 8 = –8y –5 = –8y \y = 2 3 \ Total time taken = x + 2x = 3x  2 = 3 1   3 = 5 hours x=1 6. x –4 0 6 y= 1 x+3 2 1 3 6 y = –x + 6 10 6 0 y y = –x + 6 10 5 8 9 2. (a) 12x > 60 \x > 5 (b) 15y < –24 8 24 y<– 15 3 \ y < –1 5 x – 4y 3 3. = 5x + y 5 5(x – 4y) = 3(5x + y) 5x – 20y = 15x + 3y 5x – 15x = 3y + 20y –10x = 23y 5 1 y= x+3 2 7 6 4 3 2 1 – 4 –3 –2 –1 0 x 1 2 3 4 5 6 7. (i) Since the common difference is 9, Tn = 9n + ? . The term before T1 is c = T0 =6–9 = –3 . \ General term of sequence, Tn = 9n – 3 (ii) Let 9k – 3 = 159 . 9k = 159 + 3 = 162 \ k = 18 8. (i) 8th line: 1 + 3 + 5 + 7 + 9 + 11 + 13 + 15 = 82 (ii) Since 144 = 122, k = 12 . x 23 =– y 10 x 23 \ =– 3y 30 4. Let the number of 20-cent coins in the box be x . Then the number of 50-cent coins in the box is 54 – x . 0 .2x + 0 .5(54 – x) = 20 .7 0 .2x + 27 – 0 .5x = 20 .7 – 0 .3x + 27 = 20 .7 – 0 .3x = 20 .7 – 27 – 0 .3x = – 6 .3 x = 21 There are twenty-one 20-cent coins in the box . 121 1 y 5. (i) Revision Exercise B2 C(2, 2) 1 (x – 3) – x + 5 = 3(x – 1) 3 1 x – 1 – x + 5 = 3x – 3 3 2 – x + 4 = 3x – 3 3 2 – x – 3x = –3 – 4 3 11 – x = –7 3 10 \x=1 11 2 3 (b) – +1=3 y y 1 – +1 =3 y 1 – =3–1 y 1 – =2 y 1. (a) 2 D(–2, 0) 1 –2 –1 0 –1 2 3 B(4, –2) –3 –4 A(0, – 4) (ii) Coordinates of D = (–2, 0) 6. (a) x –5 0 5 y=x+2 y=x–3 –3 2 7 –8 –3 2 y y=x+2 7 6 5 4 1 2 3 2. (a) 14x > –110 2 6 7 1 \ x > –7 – 5 – 4 –3 –2 –1 0 –1 (b) –18 < 3y 18 <y 3 \ y > –6 3. Let the smallest even number be x . Then the next 6 even numbers are x + 2, x + 4, x + 6, x + 8, x + 10 and x + 12 . x + x + 2 + x + 4 + x + 6 + x + 8 + x + 10 + x + 12 = 336 7x + 42 = 336 7x = 336 – 42 7x = 294 \ x = 42 The smallest of the 7 numbers is 42 . 4. 8 .5x + 3 .6(2x + 5) = 206 .4 8 .5x + 7 .2x + 18 = 206 .4 15 .7x + 18 = 206 .4 15 .7x = 206 .4 – 18 15 .7x = 188 .4 \ x = 12 – y=2 x 1 2 3 4 5 –2 –3 y = –3 –4 –5 –6 –7 y=x–3 –8 1 1 ×5×5+ ×5×5 2 2 1 1 = 12 + 12 2 2 = 25 units2 Since the common difference is –3, Tn = –3n + ? . The term before T1 is c = T0 = 44 + 3 = 47 . \ General term of sequence, Tn = –3n + 47 Let –3k + 47 = –13 . –3k = –13 – 47 = –60 \ k = 20 6th line: 62 – 2 × 6 = 24 Since 92 – 2 × 9 = 63, k = 9 . (b) Area enclosed by the four lines = 7. (i) (ii) 8. (i) (ii) 1 4 –2 1 –y = 2 \y =– x 1 122 Chapter 8 Percentage TEACHING NOTES Suggested Approach Although students have learnt percentage in primary school (i.e. how to express a part of a whole as a percentage, write fractions and decimals as percentages, and vice versa, find a percentage part of a whole and solve up to 2-step word problems involving percentage), many may still struggle with percentage. Teachers can introduce percentage as fractions by going right back to the fundamentals. Teachers can give students practical applications of percentages and show the changes in fractions and proportions through the examples to give them a better understanding of the concept. Section 8.1: Introduction to Percentage Teachers can get students to work in pairs to find an advertisement/article in which percentage(s) can be found and discuss about it together (see Class Discussion: Percentages in Real Life). After the discussion, students should understand the meaning of percentage(s) better and interpret information more accurately. Students need to be able to comment critically on the usefulness of percentages before they can have a confident grasp of the topic. Teachers can then build upon what students have learnt about percentage in primary school. Students may be able to learn how to accurately calculate a percentage but they might struggle to explain the meaning behind it. Teachers should emphasise on the basics of fractions and proportions before getting the students to calculate and interpret percentages. In Worked Example 5, students should learn that it is easy to see that more people passed the entrance test in 2011 but it is not easy to see which year had a higher proportion of people passing the entrance test. Teachers can highlight to the students that two quantities can be easily compared using percentages because the proportions are converted to the same base i.e. 100. Section 8.2: Percentage Change and Reverse Percentage Teachers should guide students on how to use algebra in percentage change and reverse percentage. Students may draw models, wherever applicable, to help them understand the problem. Through the worked examples in this section, students should be able to tackle percentage change and reverse percentage problems involving algebra. They should also learn how to identify whether the problem is a reverse percentage or a percentage change problem. Teachers can highlight to the students that percentage change is when they are given both the new value and the original value while a reverse percentage is when they need to find the original value given a quantity after a percentage increase or decrease. 123 1 The term ‘up to’ in the phrase ‘Discount up to 80% on All Items’ suggests that the greatest percentage discount given on the items in the shop is 80%. This means that some items in the shop may be sold at a discount that is less than 80%. Hence, the shopkeeper is not dishonest. Most of the time, such advertisements come with terms and conditions that may state the items which are not subjected to a discount, such as ‘New Arrivals’. These terms and conditions are normally shown in fine print. As some items may be sold at a discount of less than 80% and some items may not be subjected to a discount, the prices of the items in the shop may not be low. In addition, the original prices of some items may be very high, such that their prices are still high even after a discount. WORKED SOLUTIONS Class Discussion (Percentage in Real Life) 1. Guiding Questions: • What is the advertisement/article about? • Are the percentages found in the advertisement/article expressed using the percentage symbol or in words? • What do the percentages mean in the context of the advertisement/article? Teachers may use this to assess students’ prior knowledge of percentage, e.g. whether students are able to relate percentages to fractions and to perform relevant calculations using the given percentages to illustrate the meaning of the percentages in the context of the advertisement/article. Teachers may also use this as a trigger to show students the need to learn percentage, and link back to the different scenarios in the advertisement/article. Alternatively, teachers may wish to use the article titled ‘A smaller and cheaper iPad’ (Today, 5 July 2012) and/or the apparel advertisement (Page 3, Today, 5 July 2012) for this question. Teachers may wish to ask students to list other instances where such phrases are used. They may also want to take this opportunity to highlight to students the importance of being informed consumers. Students should not take information at face value. Instead, they should learn how to interpret information accurately. 3. Guiding Questions: The following shows examples of statements with percentages more than 100%: • In Singapore, the number of employers hiring ex-offenders has increased by more than 100% from 2004 to 2011. • The total number of registrants for a school during Phase 2B of the Primary 1 registration is 120% of the number of vacancies available. • The number of mobile subscriptions in Singapore in 2011 is about 150% of her population. • The population of Singapore in 2010 is about 240% of that in 1970. • The total fertility rate in Singapore in 1970 is about 270% of that in 2010. The phrase ‘this year’s sales is 200% of last year’s sales’ means that the sales this year is 2 times of that of last year. The article ‘A smaller and cheaper iPad?’ is about Apple’s apparent intention to launch an iPad which is smaller and less expensive. This is in order to counter its rivals’ new products so as to maintain its stronghold in the tablet market. The percentage found in the article is 61 per cent, which is expressed in words and in the context of the article, it means that Apple has 61% of the tablet market share. A conclusion that can be drawn is that the iPad is the most popular tablet in the market as the total tablet market share is the base, i.e. 100%. The other brands have a total of 100% – 61% = 39% of the tablet market share. The advertisement shows that a particular brand of apparel is holding a storewide end-of-season sale. The percentages found in the advertisement are 70% and 10%, which are expressed using the percentage symbol. In the context of the advertisement, it means that shoppers can enjoy a discount of up to 70% on the items and those with UOB cards are entitled to an additional 10% off if they purchase a minimum of 3 items. Teachers may ask students to refer to page 197 of the textbook for an example of how this phrase may be used. Teachers should also highlight to students that the use of percentages can be misleading, e.g. a salesman who sold a car in January and two cars in February can say that his sales in February is 200% of that in January. Teachers may wish to ask students how the additional 10% off for UOB card members is calculated, i.e. whether a shopper with a UOB card gets to enjoy a maximum discount of 70% + 10% = 80% on selected items. Teachers may also wish to get students to use any amount, e.g. $100, to illustrate the meaning of the percentages, i.e. 70% and 10%, in the context of the advertisement. 2. Guiding Questions: • What is the meaning of the term ‘up to’? • An advertisement with the phrase ‘Discount up to 80% on All Items’ is displayed at the entrance of a shop. If an item in the shop is sold at a discount of 10%, does this mean that the shopkeeper is dishonest? • Is it true that all the items in the shop are sold at a discount? Could there be any exceptions? • Does it mean that the prices of all the items in the shop are very low? Class Discussion (Expressing Two Quantities in Equivalent Forms) 40 × 100% 50 = 80% There are 80% as many male teachers as female teachers. The number of male teachers is 80% of the number of female teachers. 4 The number of male teachers is of the number of 5 female teachers. 1. (a) (i) Required percentage = • • • 124 2. Mr Lee’s monthly salary in 2011 = 110% × $x 110 = × $x 100 = $1.1x Mr Lee’s monthly salary in 2012 = 90% × $1.1x 50 × 100% 40 = 125% There are 125% as many female teachers as male teachers. The number of female teachers is 125% of the number of male teachers. (ii) Required percentage = • • • (b) = 90 × $1.1x 100 = $0.99x Hence, it is not correct to say that Mr Lee’s monthly salary in 2012 was $x. 5 The number of female teachers is of the number of 4 male teachers. In words A is 80% of B. B is 125% of A. Percentage A = 80% × B B = 125% × A Fraction 4 A = (fraction) × B 5 5 B = (fraction) × A 4 Decimal A = 0.8 × B B = 1.25 × A Practise Now 1 (a) (i) 45% = 45 100 9 = 20 305 (ii) 305% = 100 61 = 20 1 = 3 20 5.5 (iii) 5.5% = 100 55 = 1000 11 = 200 5 61 (iv) 8 % = % 7 7 61 = ÷ 100 7 61 1 = × 7 100 61 = 700 17 17 (b) (i) = × 100% 20 20 1700 = 20 = 85% Table 8.1 2. (i) In words P is 20% of Q. Percentage P = 20% × Q 1 (fraction) × Q 5 Fraction P= Decimal P = 0.2 × Q R is 50% of S. T is 125% of U. R = 50% × S T = 125% × U R= 1 5 (fraction) × S T = (fraction) × U 2 4 R = 0.5 × S T = 1.25 × Q Table 8.2 (ii) The relationship between P and Q can be illustrated as follows: P Q The relationship between R and S can be illustrated as follows: R S The relationship between T and U can be illustrated as follows: T U (ii) 23 Thinking Time (Page 198) 20% + 80% , i.e. 50% of the total 2 number of students in the two groups had done the survey. This is because there may be a different number of students in each of the two groups, e.g. if Ethan conducted the survey on 20% of a group of 100 students and on 80% of another group of 1. No, it is not correct to say that 1 116 = 5 5 116 = × 100% 5 11 600 = % 5 = 2320% 20% × 100 + 80% × 200 × 100% = 60% of the 100 + 200 total number of students in the two groups had done the survey. 200 students, then 125 1 Practise Now 2 Practise Now (Page 190) 1. (a) 20% of $13.25 = 20% × $13.25 (a) (i) 12% = 12 100 = 0.12 20 = × $13.25 100 = $2.65 (ii) 413% = 413 100 = 4.13 (b) 15 3 3 % of $640 = 15 % × $640 4 4 3 15 4 = × $640 100 = $100.80 2. 2500% of $4.60 = 2500% × $4.60 (iii) 23.6% = 23.6 100 = 0.236 (iv) 6 1 25 %= % 4 4 25 = ÷ 100 4 25 1 = × 4 100 25 = 400 = 0.0625 Alternatively, 2500 = × $4.60 100 = $115 Practise Now 4 1. Method 1: Number of students who were late for school = 3% × 1500 = 3 × 1500 100 = 45 Number of students who were punctual for school = 1500 – 45 = 1455 Method 2: Percentage of students who were punctual for school = 100% – 3% = 97% Number of students who were punctual for school = 97% × 1500 6 1 % = 6.25% 4 6.25 = 100 = 0.0625 (b) (i) 0.76 = 0.76 × 100% = 76% (ii) 2.789 = 2.789 × 100% = 278.9% = 97 × 1500 100 = 1455 Practise Now 3 1. (i) Total number of teachers in the school = 45 + 75 = 120 2. Percentage of children who attended the dinner = 100% – 35.5% – 40% = 24.5% Number of children who attended the dinner = 24.5% × 1800 Percentage of male teachers in the school = 45 × 100% 120 = 37.5% = 24.5 × 1800 100 = 441 (ii) Method 1: Percentage of female teachers in the school = 75 × 100% 120 = 62.5% Practise Now 5 Method 2: Percentage of female teachers in the school = 100% – 37.5% = 62.5% 1400 ml 2. Required percentage = × 100% 2.1 l 1400 ml = × 100% 2100 ml Percentage of people who attended the New Year party in Village A 4000 = × 100% 30 000 = 13 1 % 3 Percentage of people who attended the New Year party in Village B 2800 = × 100% 25 000 = 2 × 100% 3 2 = 66 % 3 1 = 11.2% \ Village A had a higher percentage of people who attended its New Year party. 126 Practise Now 6 Practise Now 8 1. (a) Value of award for a Secondary 1 student in 2009 = 140% × $250 70% of the books = 35 35 1% of the books = 70 35 100% of the books = × 100 70 = 50 There are 50 books on the bookshelf. 140 = × $250 100 = $350 (b) (i) Percentage increase in value of award from 2008 to 2009 for a Primary 1 student $250 – $150 = × 100% $150 $100 = × 100% $150 Practise Now 9 1. Method 1: 109% of original cost = $654 2 = 66 % 3 (ii) Percentage increase in value of award from 2008 to 2009 for a Primary 6 student $300 – $200 = × 100% $200 $100 = × 100% $200 = 50% 2. (a) Required result = 75% × 32 1% of original cost = $654 109 $654 100% of original cost = × 100 109 = $600 The original cost of the article is $600. Method 2: Let the original cost of the article be $x. 0% 109% x 75 = × 32 100 = 24 (b) Percentage decrease in value of car $127 000 – $119 380 = × 100% $127 000 $7620 × 100% = $127 000 654 From the model, we form the equation: 109% × x = 654 1.09x = 654 x = 600 The original cost of the article is $600. 2. 120% of value in 2011 = $180 000 = 6% Practise Now 7 $180 000 120 $180 000 100% of value in 2011 = × 100 120 = $150 000 The value of the vase was $150 000 in 2011. 120% of value in 2010 = $150 000 $150 000 1% of value in 2010 = 120 $150 000 100% of value in 2010 = × 100 120 = $125 000 The value of the vase was $125 000 in 2010. 1% of value in 2011 = Original Cost Percentage Change New Cost Rental $2400 –5% 95 × $2400 = $2280 100 Wages $1800 –6% 94 × $1800 = $1692 100 Utilities $480 +7% 107 × $480 = $513.60 100 Business $4680 $4485.60 Percentage decrease in monthly cost of running business $4680 – $4485.60 × 100% = $4680 $194.40 = × 100% $4680 =4 100% 2 % 13 127 1 (d) 6 3 33 % = % 5 5 33 = ÷ 100 5 33 1 = × 5 100 33 = 500 4 2. (a) 4% = 100 = 0.04 Practise Now 10 1. Method 1: 97% of original monthly salary = $3346.50 1% of original monthly salary = $3346.50 97 $3346.50 100% of original monthly salary = × 100 97 = $3450 Devi’s original monthly salary is $3450. Method 2: Let Devi’s original monthly salary be $x. 0% 97% (b) 633% = 633 100 = 6.33 100% x (c) 0.02% = 0.02 100 = 0.0002 0.97x 3346.50 (d) 33 From the model, we form the equation: 97% × x = 3346.50 0.97x = 3346.50 x = 3450 Devi’s original monthly salary is $3450. 2. 85% of value in 2011 = $86 700 $86 700 85 $86 700 100% of value in 2011 = × 100 85 = $102 000 The value of the car was $102 000 in 2011. 85% of value in 2010 = $102 000 1% of value in 2011 = 3. (a) (b) $102 000 85 $102 000 100% of value in 2010 = × 100 85 = $120 000 The value of the car was $120 000 in 2010. 1% of value in 2010 = (c) (d) (e) Exercise 8A 1. (a) 28% = 28 100 7 = 25 158 (b) 158% = 100 79 = 50 29 = 1 50 12.4 (c) 12.4% = 100 124 = 1000 31 = 250 1 3 3 = × 100% 5 5 = 60% 9 9 = × 100% 10 10 = 90% 6 6 = × 100% 125 125 = 4.8% 6 6 = × 100% 5 5 = 120% 12 12 = × 100% 25 25 = 48% 6 31 = × 100% 25 25 = 124% 0.78 = 0.78 × 100% = 78% 0.25 = 0.25 × 100% = 25% 0.07 = 0.07 × 100% = 7% 0.095 = 0.095 × 100% = 9.5% 1.35 = 1.35 × 100% = 135% (f) 1 4. (a) (b) (c) (d) (e) 128 2 101 %= % 3 3 101 = ÷ 100 3 101 1 = × 3 100 101 = 300 = 0.337 (to 3 s.f.) (f) 2.6 = 2.6 × 100% = 260% 5. (a) 50% of $70 = 50% × $70 45 minutes × 100% 1 hour 45 minutes = × 100% 60 minutes (b) Required percentage = 50 = × $70 100 = $35 (b) 80% of 4.5 m = 80% × 4.5 m 3 × 100% 4 = 75% 1 year (c) Required percentage = × 100% 4 months 12 months = × 100% 4 months = 80 = × 4.5 m 100 = 3.6 m 6. (i) Total number of students in the class = 20 + 18 = 38 = 3 × 100% = 300% 15 mm (d) Required percentage = × 100% 1m 15 mm = × 100% 1000 mm 3 × 100% = 200 = 1.5% 335 cm (e) Required percentage = × 100% 5m 335 cm = × 100% 500 cm Percentage of boys in the class = 20 × 100% 38 12 = 52 % 19 12 (ii) Percentage of girls in the class = 100% – 52 % 19 7 =47 % 19 7. Percentage of cars which are not blue = 100% – 30% = 70% Number of cars which are not blue = 70% × 120 = 70 × 120 100 = 84 8. Percentage of annual income Jun Wei donated to charitable organisations $1200 = × 100% 12 × $1600 $1200 = × 100% $19 200 67 × 100% 100 = 67% 1 kg (f) Required percentage = × 100% 800 g 1000 g = × 100% 800 g = 5 × 100% 4 = 125% = = 6.25% Percentage of annual income Lixin donated to charitable organisations $4500 = × 100% 12 × $6800 $4500 = × 100% $81 600 60° × 100% 360° 1 × 100% = 6 2 = 16 % 3 63 cents (h) Required percentage = × 100% $2.10 63 cents = × 100% 210 cents (g) Required percentage = 35 = 5 % 68 \ Jun Wei donated a higher percentage of his annual income to charitable organisations. 25 seconds 9. (a) Required percentage = × 100% 3.5 minutes 25 seconds = × 100% 210 seconds = 3 × 100% 10 = 30% 10. (a) 6 5 × 100% 42 19 = 11 % 21 = 129 1 1 % of 1.35 ml = 6 % × 1.35 ml 5 5 1 6 5 = × 1.35 ml 100 837 = ml 10 000 1 14. Number of remaining pages after Friday = 600 – 150 = 450 Number of pages that remains to be read = (100% – 40%) × 450 = 60% × 450 (b) 56 7 7 % of 810 m = 56 % × 810 m 8 8 7 56 8 × 810 m = 100 11 = 460 m 16 (c) 0.56% of 15 000 l = 0.56% × 15 000 l = 60 × 450 100 = 270 Required percentage = 270 × 100% 600 = 45% = 0.56 × 15 000 l 100 = 84 l (d) 2000% of 5¢ = 2000% × 5¢ Exercise 8B 2000 = × 5¢ 100 = 100¢ = $1 1. (a) Required value = 135% × 60 135 = × 60 100 = 81 (b) Required value = 225% × 28 11. Percentage of marks Kate obtains = \ Kate gets a bronze award. 40 × 100% 60 2 = 66 % 3 225 = × 28 100 = 63 (c) Required value = 55% × 120 Percentage of marks Priya obtains = \ Priya gets a silver award. 46 × 100% 60 2 = 76 % 3 55 = × 120 100 = 66 (d) Required value = 62 Percentage of marks Nora obtains = 49 × 100% 60 2 = 81 % 3 \ Nora gets a gold award. 12. Percentage of employees who were unaffected by the financial crisis = 100% – 2.5% – 50.75% = 46.75% Number of employees who were unaffected by the financial crisis = 46.75% × 12 000 2. (a) 1% of number = 17 20 17 100% of number = × 100 20 = 85 The number is 85. (b) 175% of number = 49 46.75 = × 12 000 100 = 5610 13. Amount Ethan spent on room rental = 20.5% × $1850 1% of number = 49 175 49 100% of number = × 100 175 = 28 The number is 28. (c) 115% of number = 161 = 20.5 × $1850 100 = $379.25 Amount Ethan overspent = $379.25 + $690 + $940 – $1850 = $159.25 $159.25 Required percentage = × 100% $1850 = 8.61% (to 2 d.p.) 1 1 % × 216 2 1 62 = 2 × 216 100 = 135 20% of number = 17 1% of number = 161 115 161 100% of number = × 100 115 = 140 The number is 140. 130 (d) 80% of number = 192 9. $58.50 90 $58.50 100% of original bill = × 100 90 = $65 The original bill is $65. 10. Value obtained after initial increase = 130% × 2400 3. Percentage increase in length of elastic band = 90 – 72 × 100% 72 18 = × 100% 72 = 25% 4. (i) Value of award for a Secondary 1 student in the top 5% in 2009 = 130% × $500 = Final number = 80% × 3120 = 108 ×x 100 = 1.08x Number of train passengers in 2012 = 108% × 1.08x = 108 × 1.08x 100 = 1.1664x Percentage increase in number of train passengers from 2010 to 2012 1.1664 x – x = × 100% x 0.1664 x = × 100% x = 0.1664 × 100% = 16.64% = 80 × $120 000 100 = $96 000 Value of car at the end of 2011 = 90% × $96 000 Original Cost Percentage Change New Cost Raw materials $100 +11% 111 × $100 = $111 100 Overheads $80 +20% 120 × $80 = $96 100 Wages $120 –15% 85 × $120 = $102 100 Printer $300 12. = 45% of the students = 135 130 × 2400 100 = 3120 80 = × 3120 100 = 2496 11. Let the number of train passengers in 2010 be x. Number of train passengers in 2011 = 108% × x 130 = × $500 100 = $650 (ii) Percentage increase in value of award from 2008 to 2009 for a Primary 4 student in the next 5% $350 – $250 = × 100% $250 $100 = × 100% $250 = 40% 5. Percentage decrease in price of desktop computer $1360 – $1020 = × 100% $1360 $340 = × 100% $1360 = 25% 6. Value of car at the end of 2010 = 80% × $120 000 7. 90% of original bill = $58.50 1% of original bill = 192 1% of number = 80 192 100% of number = × 100 80 = 240 The number is 240. 90 × $96 000 100 = $86 400 1% of the students = 135 45 135 100% of the students = × 100 45 = 300 There are 300 students who take part in the competition. 8. 136% of original cost = $333 200 $309 Percentage increase in production cost of printer $309 – $300 = × 100% $300 $9 = × 100% $300 = 3% $333 200 136 $333 200 100% of original cost = × 100 136 = $245 000 The cost of the house when it was built is $245 000. 1% of original cost = 131 1 16. Let Amirah’s height be x m. 108% of Huixian’s height = x m 13. 115% of value in 2011 = $899 300 $899 300 1% of value in 2011 = 115 $899 300 100% of value in 2011 = × 100 115 = $782 000 The value of the condominium was $782 000 in 2011. 115% of value in 2010 = $782 000 1% of Huixian’s height = x m 108 x 100% of Huixian’s height = × 100 108 25 = xm 27 25 Huixian’s height is x m. 27 25 Priya’s height = 90% × x 27 90 25 = × x 100 27 5 =x m 6 x Required percentage = × 100% 5 x 6 1 = × 100% 5 6 $782 000 115 $782 000 100% of value in 2010 = × 100 115 = $680 000 The value of the condominium was $680 000 in 2010. 14. 75% of value in 2011 = $11 250 1% of value in 2010 = $11 250 75 $11 250 100% of value in 2011 = × 100 75 = $15 000 The value of the surveying machine was $15 000 in 2011. 75% of value in 2010 = $15 000 1% of value in 2011 = = 120% $15 000 75 $15 000 100% of value in 2010 = × 100 75 = $20 000 The value of the surveying machine was $20 000 in 2010. 15. 105% of value at the end of 2010 = $61 824 1% of value in 2010 = Review Exercise 8 1m × 100% 56 mm 1000 mm = × 100% 56 mm 1. Required percentage = = 125 × 100% 7 5 = 1785 % 7 2. (i) Pocket money Michael receives in a year = 52 × $28 = $1456 $61 824 105 $61 824 100% of value at the end of 2010 = × 100 105 = $58 880 The value of the investment portfolio was $58 880 at the end of 2010. 92% of original value = $58 880 1% of value at the end of 2010 = Savings in a year = 20 × $1456 100 = $291.20 (ii) Spending in a year = $1456 – $291.20 = $1164.80 30 ×b a 3. = 100 4b 4b 30 100 = 4 3 = 40 $58 880 92 $58 880 100% of original value = × 100 92 = $64 000 The original value of the investment portfolio was $64 000. 1% of original value = 1 132 68 × 100% 80 = 85% 4. Huixian’s percentage score = Challenge Yourself 1. Let the number of red jellybeans Amirah moves from Bottle A to Bottle B be x, the number of yellow jellybeans Amirah moves from Bottle A to Bottle B be y. Priya’s percentage score = 86 × 100% 120 2 = 71 % 3 120 Rui Feng’s percentage score = × 100% 150 = 80% \ Huixian performs the best in her Science test. 120 × 120 100 = 144 Red 300 150 Yellow 100 150 Red 300 – x 150 + x Yellow 100 – y 150 + y After 300 – 100 – 300 – 100 – x 80 = y 20 x 4 = y 1 300 – x = 4(100 – y) 300 – x = 400 – 4y 4y – x = 100 — (1) 150 + x 60 \ = 150 + y 40 150 + x 3 = 150 + y 2 2(150 + x) = 3(150 + y) 300 + 2x = 450 + 3y 2x – 3y = 150 — (2) 2 × (1): 8y – 2x = 200 — (3) (2) + (3): 5y = 350 y = 70 Substitute y = 70 into (1): 4(70) – x = 100 280 – x = 100 x = 180 Number of jellybeans Amirah moves from Bottle A to Bottle B =x+y = 180 + 70 = 250 2. Percentage of water which is poured from Cup B into Cup A \ 120 1% of number of pears = 60 120 100% of number of pears = × 100 60 = 200 Number of pears the vendor has = 200 Total number of fruits the vendor has = 120 + 144 + 200 = 464 6. 120% of number of pages Kate reads on the second day = 60 1% of number of pages Kate reads on the second day = Bottle B Before 5. Number of apples the vendor has = 60% of number of pears = 120 Bottle A 60 120 100% of number of pages Kate reads on the second day 60 = × 100 120 = 50 Number of pages Kate reads on the second day = 50 Number of pages in the book = 6 × 50 = 300 7. Percentage of goats left = 94 × 86% 100 = 80.84% 80.84% of original number of goats = 8084 1% of original number of goats = 8084 80.84 8084 100% of original number of goats = × 100 80.84 = 10 000 The original number of goats in the village is 10 000. 8. Let Mr Neo’s original salary be $x. 60 = × 70% 100 = 42% Percentage of water in Cup A before 60% of solution in Cup A is poured into Cup B = 40% + 42% = 82% Percentage of water in Cup A after 60% of solution in Cup A is poured into Cup B Mr Neo’s reduced salary = 85 × $x 100 = $0.85x $ x – $0.85 x Required percentage = × 100% $0.85 x $0.15 x = × 100% $0.85 x = 40 × 82% 100 = 32.8% = 0.15 × 100% 0.85 11 = 17 % 17 133 1 Chapter 9 Ratio, Rate, Time and Speed TEACHING NOTES Suggested Approach Students have learnt how to solve problems involving ratios and speed in primary school. Teachers can bring in real-life examples for ratio, rate, time and speed to arouse students’ interest in this topic. Students will also learn how to solve problems involving ratio, rate, time and speed through worked examples that involve situations in real-world contexts. Section 9.1: Ratio Teachers can build upon what students have learnt about ratio in primary school and introduce equivalent ratios through a recap of equivalent fractions. Teachers should emphasise that ratio does not indicate the actual size of quantities involved. Practical examples can be given to the students to let them recognise what equivalent ratios are (e.g. using 2 different kinds of fruits). Teachers should highlight some common errors in ratio (i.e. the ratio of a part of a whole with the ratio of two parts, incorrect order of numbers expressed when writing ratio and incorrect numerator expressed when writing ratio as a fraction). To make learning interesting, students can explore more about the Golden Ratio (see chapter opener and Investigation: Golden Ratio). Teachers can also get the students to find out what other man-made structures or natural occurrences have in common with the Golden Ratio (see Performance Task at page 210 of the textbook). Section 9.2: Rate Teachers should explain that rate is a relationship between two quantities with different units of measure (which is different from ratio). Teachers can give real life examples (e.g. rate of flow, consumption) for students to understand the concept of rate. Teachers can also get students to interpret using tables which show different kinds of rates (e.g. interest rate, postage rate, parking rate etc.). Students can get more practice by learning to calculate rates they are familiar with (see Investigation: Average Pulse Rate). Teachers should impress upon them to distinguish between constant and average rates. Section 9.3: 1 Time Teachers should emphasise that the addition and subtraction of times are not simply the same as adding and subtracting the numbers. For example, teachers can ask students why 30 + 40 = 70 = 110 (where 110 refers to 1 h 10 min). To prevent students from making careless mistakes, teachers should help students understand that: 6 hours 45 minutes is not the same as 6.45 hours, 1 hour is not the same as 100 minutes, 1 minute is not the same as 100 seconds. Another important learning point would be dealing with time during the period before midnight and early morning. Teachers may also compare the time displayed on a digital clock with that on an analogue clock, and show students how the time is read. 134 Section 9.4: Speed Teachers should inform students that speed is a special type of rate, i.e. speed is the distance covered per unit time. Teachers can get students to match appropriate speed to examples given (e.g. speed of a moving bicycle, lorry, car and aeroplane) to bring across the notion of speed. Teachers can build upon what students have learnt about distance, time and speed in primary school. Students need to know that average speed is defined as the total distance travelled by the object per unit time and not the average of the speeds of the object. Teachers should also impress upon students that there are differences between average speed and constant speed. Teachers should teach students the conversion of units and highlight to them to use appropriate units when solving problems. Challenge Yourself Questions 1 and 2: Teachers can guide the students by getting them to use appropriate algebraic variables to represent the rates involved in the question. Students have to read the question carefully and form the linear equations which then can be solved to get the answers. 135 1 WORKED SOLUTIONS Journal Writing (Page 208) Class Discussion (Making Sense of the Relationship between Ratios and Fractions) 1. Aspect ratio is used to describe the relationship between the width and height of an image . It does not represent the actual length and height, but instead represents the proportion of its width and height . This is usually represented by two numbers separated by a colon, for example, 4 : 3 and 16 : 9 . The standard size of televisions has an aspect ratio of 4 : 3 which means the image is 4 units wide for every 3 units of height . Meanwhile, the latest size of televisions for the aspect ratio is 16 : 9 which is 16 units of width for every 9 units of height . The following are some examples of aspect ratio used in our daily lives: • 16 : 10 is used mainly in widescreen computer monitors. • 16 : 9 is the aspect ratio used in cinema halls as well as High Definition TV. • 14 : 9 is a compromise aspect ratio used to create an image that is viewable to both 4 : 3 and 16 : 9 televisions . • 5 : 4 is a computer monitor resolution and also in mobile phones. • 4 : 3 is used in the older TVs (mainly non-widescreen) and computer monitors . • 1 : 1 is an uncommon aspect ratio that is used mainly in photography . 2. Example 1: Scale drawings of maps and buildings are often represented by ratios . This is because it is impossible for a map to be exactly of the same size as the area it represents . Therefore, the measurements are scaled down in a fixed proportion so that the map can be used easily. Similarly, a scale drawing of a building will have the same shape as the actual building except that is scaled down . Example 2: In Chemistry and Biology, ratios are used for simple dilution of chemicals. A fixed unit volume of a chemical is added to an appropriate volume of solvent in order to dilute the chemical . For example, a 1 : 5 dilution (verbalize as “1 to 5” dilution) entails combining 1 unit volume of solute (the material to be diluted) + 4 unit volumes (approximately) of the solvent to give 5 units of the total volume . There are 40 green balls and 60 red balls in a bag . Let A and B represent the number of green balls and red balls respectively . 1. Find the ratio of A to B . A : B = 40 : 60 = 2:3 We can conclude that: The ratio of A to B is 2 : 3 . The following statement is equivalent to the above statement . 2 2 A (fraction) of B, i .e . = (fraction) . 3 3 B 2. Find the ratio of B to A . B : A = 60 : 40 = 3:2 We can conclude that: The ratio of B to A is 3 : 2 . The following statement is equivalent to the above statement . A is 3. B is 3 B 3 (fraction) of A, i .e . = (fraction) . 2 A 2 A 20 20 B 20 20 20 4. Example: There are 30 girls and 10 boys in a class . Let G and B represent the number of girls and boys respectively . G : B = 30 : 10 = 3:1 We can conclude that: The ratio of G to B is 3 : 1 . The following statement is equivalent to the above statement . G is 3 G 3 (fraction) of B, i .e . = (fraction) . 1 B 1 OR B : G = 10 : 30 = 1:3 We can conclude that: The ratio of B to G is 1 : 3 . The following statement is equivalent to the above statement . B is 1 B 1 (fraction) of G, i .e . = (fraction) . 3 G 3 G 10 B 10 10 Investigation (Golden Ratio) 1. 10 2. 3. 1 136 AB = 1 .7 cm BC = 1.05 cm AC AB AB BC XY YZ 2.75 = 1 .62 (to 2 s .f .) 1.7 1.7 = = 1 .62 (to 2 s .f .) 1.05 = 2.75 cm = 1 .7 cm = XY 2.75 = = 1 .62 (to 2 s .f .) YZ 1.7 1+ 5 = 1 .62 (to 2 s .f .) 2 4. All the values in the previous questions are all equal . 5. – 6. (a) (b) Thinking Time (Page 224) Do a recap on the definition of average speed which is defined as the total distance travelled by the total time taken . Average speed is different from the general meaning of ‘average’ in statistics . The word ‘average’ here does not refer to the sum of all individual speeds divided by the total number of individual speeds . 3+ 5 2 3+ 5 ϕ+1= 2 Both answers are the same . ϕ2 = 1 = –1 + 5 2 ϕ 1 = ϕ – ____ ϕ 1+ 5 –1 + 5 ϕ– 1 = – 2 2 ϕ =1 It is equal to 1 . Performance Task (Page 225) 1. (a) Teachers may wish to assign this activity as a pair work for the students to do. Students can help to record each other’s walking speed. Average walking speed of a human = 5 km/h (b) Minimum average speed 2.4 = 11 × 60 + 30     3600 2.4 =  690   3600  Performance Task (Page 210) Teachers may wish to give some examples of • man-made structures such as the a) Acropolis of Athens (468–430 BC), including the Parthenon; b) Great Mosque of Kairouan (built by Uqba ibn Nafi c. 670 A.D); c) Cathedral of Chartres (begun in the 12th century), Notre-Dame of Laon (1157–1205), and Notre Dame de Paris (1160); d) Mexico City Metropolitan Cathedral (1667–1813) . • natural occurrences a) spiral growth of sea shells; b) spiral of a pinecone; c) petals of sunflower; d) horns of antelopes, goats and rams; e) tusks of elephants; f) body dimensions of penguins . = 12.5 km/h (to 3.s.f.) (c) Average speed of a bicycle = 22.5 km/h Average speed of a sports car = 280 km/h (d) Average speed of an MRT train = 45 km/h (e) Average speed of an aeroplane = 805 km/h (f) Average speed of the spaceship = 28 000 km/h Investigation (Average Pulse Rate) Average speed (km/h) (a) Walking 5 km/h (b) Running 12.5 km/h (c) Bicycle 22.5 km/h Thinking Time (Page 216) (d) Sports car 280 km/h 1. The parking charges per minute are $0 .40 is a constant rate as the rate of charges per minute is the same throughout . The rate of petrol consumption is 13.5 km per litre is an average rate as the rate of consumption is not the same per minute . 2. The following are 3 examples of average rate that can be found in daily life: • Average speed • Downloading rate of a file • Average daily population growth The following are 3 example of constant rate that can be found in daily life: • Simple interest rate • Income Tax rate • Currency exchange rate (e) MRT train 45 km/h (f) Aeroplane 805 km/h (g) Spaceship 28 000 km/h First reading Second reading Third reading Pulse rate (per minute) 2. Priya cycles to school while Devi walks to school . 3. Number of times a spaceship is as fast as an aeroplane = 28 000 805 = 34 .8 4. Other examples of speeds which can be encountered in real life: • Speed of a bus • Speed of a cheetah • Speed of the Singapore Flyer capsule 137 1 5. Teachers may wish to ask the students to present their findings to the class . 2. Let the amount of money Kate had initially be $3x . Then the amount of money Nora had initially is $5x . Kate Nora Before $3x $5x After $(3x + 150) $(5x – 150) Practise Now 1 (i) Ratio of the number of lemons to the number of pears = 33 : 20 (ii) Ratio of the number of pears to the number of fruits in the basket = 20 : (33 + 20) = 20 : 53 3 x + 150 7 = 5 x – 150 9 9(3x + 150) = 7(5x – 150) 27x + 1350 = 35x – 1050 27x – 35x = –1050 – 1350 –8x = –2400 x = 300 \ Amount of money Kate had initially = $[3(300)] = $900 \ Practise Now 2 (a) 240 g : 1 .8 kg = 240 g : 1800 g = 2 : 15 Alternatively, 2.40 g 240 g = 1.8 kg 1800 g 2 15 \ 240 g : 1.8 kg = 2 : 15 = 3 8 3 8 : = × 45 : 5 9 5 9 = 27 : 40 (c) 0 .36 : 1 .2 = 0 .36 × 100 = 36 = 3 (b) Practise Now 5 x:y = 5:6 ↓ ×2 = 10 : 12 (i) x : y : z = 10 : 12 : 27 (ii) x : z = 10 : 27 × 45 : 1 .2 × 100 : 120 : 10 y:z = 4:9 ↓ ×3 = 12 : 27 Practise Now 6 Practise Now 3 Let the amount of money Khairul had initially be $6x . Then the amount of money Michael and Ethan had initially is $4x and $5x respectively . 3a : 7 = 8 : 5 3a 8 = 7 5 15a = 56 a =3 11 15 Practise Now 4 x Non-fiction From the model, we form the equation: 5x + 2x = 1421 7x = 1421 x = 203 There are 3 × 203 = 609 more fiction than non-fiction books in the library . 1 Michael Ethan Before $6x $4x $5x After $(6x – 45) $(4x + 30) $(5x + 15) 7 6 x – 45 = 4 x + 30 6 6(6x – 45) = 7(4x + 30) 36x – 270 = 28x + 210 36x – 28x = 210 + 270 8x = 480 x = 60 \ Amount of money Khairul had initially = $[6(60)] = $360 \ 1. Let the number of fiction books = 5x . Then the number of non-fiction book = 2x . Fiction Khairul 138 Practise Now 7 Number of words per minute that Amirah can type Practise Now 10 10 45 11 00 720 16 = 45 Number of words per minute that Lixin can type 23 00 23 11 = 15 min Practise Now 11 1. (i) 25 minutes = 25 hours 60 16.8 Speed of the train = = 40.32 km/h  25   60  798 19 = 42 Thus, Lixin is the fastest typist . = (ii) 16 .8 km = 16 800 m 25 minutes = 25 × 60 = 1500 seconds Practise Now 8 1. (a) Amount each child have to pay 16 800 = 11.2 m/s 1500 55 km (55 × 1000) m 5 2. 55 km/h = = = 15 m/s 1h 3600s 18 12 minutes 30 seconds = (12 × 60) + 30 = 750 seconds Speed of the train = $2.70 × 32.5 = 36 = $2 .44 (b) (i) Distance travelled on 1 litre of petrol Distance travelled = 15 5 1 × 750 = 11 458 m 18 3 3. Let the speed of the bus be x km/h. 265 25 265 = 25 = 10 .6 km Distance travelled on 58 litres of petrol = 10 .6 × 58 = 614 .8 km (ii) Amount of petrol required to travel a distance of 1007 km = 3 hours 13 20 hours 16 20 hours Distance the car travelled in 3 hours = 90 × 3 = 270 km 270 + (3 × x) = 510 3x = 510 – 270 3x = 240 x = 80 The speed of the bus is 80 km/h. 1007 10.6 = 95 litres Amount that the car owner has to pay = 95 × $1.95 = $185.25 2. In 1 minute, 5 people can finish = Practise Now 12 1. (i) Speed of the train = 48.6 km/h 48.6 km = 1h 48 600 m = (convert 48 .6 km to m and 1 h into s) 3600 s 20 60 = 6 buns In 5 minutes, 5 people can finish =6×5 = 30 buns In 5 minutes, 10 people can finish = 30 × 2 = 60 buns = 13.5 m/s (ii) Speed of the train = 48.6 km/h 48.6 km = 1h 4 860 000 cm = (convert 48 .6 km to cm and 1 h into min) 60 min = 81 000 cm/min Practice Now 9 7 1 h = 7 h 15 min 4 22 45 +7h 11 min 15 min + 11 min = 26 min \ The bus journey was 12 h 26 min long . 828 = 18 = 46 Number of words per minute that Shirley can type = 20 ÷ 3 12 h + 15 min 29 45 06 00 (05 45) \ The ship arrived at Port Y at 06 00 or 6 a .m . on Saturday . 139 1 2. Speed of the fastest human sprinter 100 m = 9.58 s (100 ÷ 1000) km = (9.58 ÷ 3600) h Practise Now 14 Let the distance for the car to travel from Town A to Town B to meet the truck = x km . Then the time taken for the car to travel from Town A to Town B to meet the truck at an average speed of 72 km/h 277 km/h 479 No . of times a cheetah is as fast as the fastest human sprinter 110 = 277 37 479 x hour, and 72 the time taken for the truck to travel from Town B to Town A to meet the car at an average speed of 38 km/h = 37 = 550 – x hour 38 x 550 – x \ = 72 38 38x = 39 600 – 72x 38x + 72x = 39 600 110x = 39 600 x = 360 km Hence, the time taken for the two vehicles to meet = = 2 .93 Practise Now 13 Time taken for Farhan to swim a distance of 1.5 km 1.5 h 2.5 3 = h 5 Total time taken = 360 72 = 5 hours = 3 1 1 +1 +1 5 2 9 3 3 10 = + + 5 2 9 54 135 100 = + + 90 90 90 289 = hours 90 Distance that Farhan runs = Practise Now 15 Radius of the wheel of the car 0.75 2 = 0.375 m Circumference of the wheel of the car = 2 × p × 0.375 = 2 × 3 .142 × 0.375 = 2.3565 m Distance travelled by a car in 1 minute = 14 × 60 = 840 m Number of revolutions made by the wheel per minute = 1 9 10 =9× 9 = 10 km Total distance travelled = 1.5 + 40 + 10 = 51.5 km Average speed for the entire competition =9×1 = 840 2.3565 = 356 (to the nearest whole number) = Total distance travelled Total time taken Exercise 9A 51.5  289   90  11 km/h = 16 289 = 1. (a) 1.5 kg : 350 g = 1500 g : 350 g = 30 : 7 Alternatively, 1.5 kg 1500 g = 350 g 350 g 30 7 \ 1.5 kg : 350 g = 30 : 7 = (b) 1 140 15 9 15 9 : = × 168 : × 168 24 7 24 7 = 105 : 216 = 35 : 72 (c) 0.45 : 0.85 = 0.45 × 100 : 0.85 × 100 = 45 : 85 = 9 : 17 (d) 580 ml : 1 .12 l : 104 ml =580 : 1120 : 104 = 145 : 280 : 26 From the model, we form the equation: 9x – 5x = 44 4x = 44 x = 11 Total amount of money that is shared between the two boys = (5 + 9) × 11 = $154 2 3 5 2 3 5 : : = × 24 : × 24 : × 24 3 2 8 3 2 8 =16 : 36 : 15 (f) 0 .33 : 0 .63 : 1 .8 = 0 .33 × 100 : 0 .63 × 100 : 1 .8 × 100 = 33 : 63 : 180 = 11 : 21 : 60 2. (a) a : 400 = 6 : 25 a 6 = 400 25 25a = 2400 a = 96 (b) 5b : 8 = 2 : 5 (e) 8. (i) Number of toys Huixian makes = 1530 × 16 12 + 16 + 17 1530 = × 16 45 = 544 1530 × 17 12 + 16 + 17 1530 = × 17 45 = 578 Amount of money Priya earns = 578 × $1.65 = $953.70 (ii) Number of toys Priya makes = 5b 2 = 8 5 25b = 16 16 b= 25 3. 4. 5. 6. 7. 9. (a) 4 (b) 0.75 : 3 2x 3y = 5 8 16x = 15y x 15 = y 16 x : y = 15 : 16 a : b : c = 75 : 120 : 132 (i) a : b : c = 25 : 40 : 44 (ii) b : a = 40 : 25 = 8 : 5 (iii) b : c = 40 : 44 = 10 : 11 (i) Ratio of the number of boys to the number of girls = 14 : 25 (ii) Ratio of the number of girls to the total number of players in the team = 25 : 39 (i) Ratio of the number of athletes to the number of volunteers = 3600 : 20 000 = 9 : 50 (ii) Ratio of the number of media representatives to the number of athletes to the number of spectators = 1200 : 3600 : 370 000 =3 : 9 : 925 Let the amount of money that Rui Feng gets = 5x . Then the amount of money that Vishal gets = 9x . Rui Feng 1 kg : 630 g = 4200 g : 630 g 5 = 20 : 3 (c) 0 .6 kg : (d) 75 5 : 3 100 16 3 53 = : 4 16 3 53 = × 16 : × 16 4 16 =12 : 53 3 kg : 400 g = 600 g : 750 g : 400 g 4 = 12 : 15 : 8 1 3 1 15 : 2.5 : 3 = × 12 : 2.5 × 12 : × 12 3 4 3 4 = 4 : 30 : 45 (e) 1 .2 : 3 10. (a) 5 = 16 2 1 4 9 4 3 33 : 5.5 = 1.2 × 10 : × 10 : 5.5 × 10 10 10 = 12 : 33 : 55 : 6 = m : 1 : 6 = m : 1 5 6 5 9 6 × 20 : 6 × 20 = m × 20 : × 20 4 5 45 : 120 = 20m : 24 9 : 24 = 20m : 24 9 = 20m 20m = 9 9 m = 20 x $44 Vishal 141 1 (b) x : 3 : 9 2 = 15 : 4 4 Amount of profit Farhan received 1 : y 2 11 × $494 500 7 + 11 + 5 = $236 500 Amount of profit Michael received 5 = × $494 500 7 + 11 + 5 = 9 9 15 x×4:3×4: × 4= ×4: ×4:y×4 4 2 2 4x : 12 : 18 = 15 : 18 : 4y 18 4x 15 12 = = 4y 12 18 18 9 x 5 2 = = 2y 3 6 3 6x = 15 4y = 27 = $107 500 14. Let the number that must be added be x . 3+ x 2 = 8+x 3 3(3 + x) = 2(8 + x) 9 + 3x = 16 + 2x 3x – 2x = 16 – 9 x =7 The number is 7 . 15. Let the amount of money Ethan had initially be $5x . Then the amount of money Jun Wei and Raj had initially is $6x and $9x respectively . 15 27 y = 6 4 3 5 x = y =6 4 2 1 x =2 2 3 1 1 11. p : q = :2 p:r = : 4 3 2 ↓ ×4 ↓ ×6 = 3 :8 = 2 : 3 ↓ ↓ = 6 : 16 = 6 : 9 (i) p : q : r = 6 : 16 : 9 (ii) q : r = 16 : 9 12. (i) Let the initial number of teachers in the school be x . Then the number of students in the school is 15x . 15x = 1200 x = 80 The initial number of teachers in the school is 80 . (ii) Let the number of teachers who join the school be y . x = Before Jun Wei Raj $5x $6x $9x $6x $9x $(5x – 50) After 5 x – 50 3 = 6x 4 4(5x – 50) = 3(6x) 20x – 200 = 18x 20x – 18x = 200 2x = 200 x = 100 \ Amount of money Ethan has after giving $50 to his mother = $[5(100) – 50] = $450 x y 5 3 16. = = y z 8 4 4x = 3y 8y = 5z \ 80 + y 3 = 1200 40 40(80 + y) = 3(1200) 3200 + 40y = 3600 40y = 3600 – 3200 40y = 400 y = 10 The number of teachers who join the school is 10 . 13. Ratio of Ethan, Farhan and Michael’s property investment = $427 000 : $671 000 : $305 000 = 427 : 671 : 305 = 7 : 11 : 5 Total amount of profit earned = $1 897 500 – ($427 000 + $671 00 + $305 000) = $494 500 Amount of profit Ethan received 3 y 4 2y = 3x – y + 2 z x = = = 7 × $494 500 7 + 11 + 5 = $150 000 = = = 1 Ethan 142 z= 2y 3  8  3 y – y + 2  y 4  5  2y 9 16 y–y+ y 4 5 2y 45 64 20 y– y+ y 20 20 20 2y 89 y 20 40 89 8 y 5 (ii) Amount of petrol required to travel a distance of 2013 .2 km Exercise 9B 2013.2 11.8 36 = 170 litres 59 Amount that the car owner has to pay = 1. (a) Number of words that she can type per minute 1800 60 (1 hour = 60 minutes) = 30 (b) Cost of one unit of electricity 120.99 =$ 654 = $0 .19 (c) His monthly rental rate 4800 =$ 3 = $1600 (d) Its mass per metre 15 = 3.25 8 =4 kg/m 13 2. Time taken for Ethan to blow 1 balloon = 36 × $1 .99 59 = $339.51 6. (i) Amount of fertiliser needed for a plot of land that has an area of 1 m2 = 170 200 8 = 25 g Amount of fertiliser needed for a plot of land that has an area of 14 m2 = 25 × 14 = 350 g (ii) Area of land that can be fertilised by 450 g of fertiliser = 450 25 = 18 m2 7. (i) Temperature of the metal after 9 minutes = 428 °C – [(23 °C × 3) + (15 °C × 6)] = 269 °C (ii) Temperature of the metal after 18 minutes = 428 °C – [(23 °C × 3) + (15 °C × 15)] = 134 °C Amount of temperature needed for the metal to fall so that it will reach a temperature of 25 °C = 134 °C – 25 °C = 109 °C Time needed for the metal to reach a temperature of 25 °C = 20 15 = 1.3 minutes Time taken for Jun Wei to blow 1 balloon = = 25 18 = 1.38 minutes Time taken for Vishal to blow 1 balloon 21 16 = 1.3125 minutes Thus, Vishal can blow balloons at the fastest rate. 3. 3 hours = 180 minutes Number of ornaments made in 3 hours = 109 8 5 = 13 minutes 8 8. 4 weeks ⇒ fifteen 2-litre bottles of cooking oil = 180 ×4 15 = 48 Amount earned by the worker = 48 × $1.15 = $55.20 4. (i) Amount he is charged for each minute of outgoing calls = 15 × 2 = 7.5 litres of cooking oil 4 10 weeks ⇒ 10 × 7.5 = 75 litres of cooking oil Number of 5-litre tins of cooking oil needed for a 10-week period 1 week ⇒ 39 650 = $0 .06 (ii) Amount he has to pay = $0 .06 × 460 = $27 .60 5. (i) Distance travelled on 1 litre of petrol =$ 75 5 = 15 9. (i) Total amount to be paid to the man = 224 × $7.50 = $1680 = 259.6 22 = 11 .8 km Distance travelled on 63 litres of petrol = 11 .8 × 63 = 743 .4 km = 143 1 (ii) Number of normal working hours from 9 a .m . to 6 p .m . excluding lunch time = 8 hours Let the number of overtime hours needed to complete the project in 4 days by each worker be x . 4[4(8 + x)] = 224 16(8 + x) = 224 128 + 16x = 224 16x = 224 – 128 16x = 96 x =6 Overtime hourly rate = 1.5 × $7.5 = $11.25 Total amount to be paid to the 4 men if the project is to be completed in 4 days = 4{4[(8 × $7.5) + (6 × $11.25)]} = $2040 10. 10 chefs can prepare a meal for 536 people in 8 hours and so 1 chef can prepare a meal for 536 people in 8 × 10 = 80 hours . Hence, 22 chefs can prepare a meal for 536 × 22 = 11 792 people in 536 × 22 80 hours and so 22 chefs can prepare a meal for people 80 80 in = 1 hour . 80 536 × 22 Thus 22 chefs can prepare a meal for × 5 people in 80 1 × 5 = 5 hours. 3. Journey Time Arrival Time (a) 02 40 55 minutes 03 35 (b) 22 35 8 hours 06 35 (next day) (c) 15 45 2 (d) 09 48 (e) 20 35 (Tuesday) (f) 22 35 1 h or 2 h 15 min 4 17 50 12 7 h or 12 h 28 min 15 22 16 10 2 h or 10 h 40 min 3 07 15 (Wednesday) 1 1 h 4 23 50 4. 8.35 a.m. ⇒ 08 35 3 .12 p .m . ⇒ 15 12 08 35 15 00 15 12 09 00 25 min 7h 12 min 25 min + 12 min = 37 min \ The journey took 6 h 37 min . 5. 21 55 +9h 6. +4h + 5 min + 13 min 30 55 07 00 07 13 (06 55) \ The train arrived at its destination at 07 13 or 7 .13 a .m . on Tuesday . 536 × 22 ×5 80 = 737 Exercise 9C 1. (a) (b) (c) (d) 2. (a) (b) (c) (d) Departure Time 08 00 21 42 00 00 02 42 3 .30 a .m . 11 .12 p .m . 7.15 p.m. 12 .00 a .m . ? 10 51 –4h ? 14 51 + 15 min – 9 min 15 00 – 6 min 15 06 15 06 Working backwards, the car started the journey at 10 51. 7. (i) 22 55 23 00 00 00 5 min 06 00 06 05 5 min + 5 min = 10 min \The journey took 7 h 10 min . (ii) 35 min = 30 min + 5 min 5 min before 06 05 is 06 00 30 min before 06 00 is 05 30 \ The coach reached its destination at 05 30. 8. Assume time taken includes breaks in between stations . (a) Depart from A: 21 30; Arrive at C: 02 25 21 30 22 00 30 min 00 00 4h 30 min + 25 min = 55 min \ The time taken from A to C was 4 h 55 min. 1 5 min 7h 144 02 00 02 25 25 min (b) Depart from B: 22 30; Arrive at E: 07 50 22 30 23 00 00 00 30 min 3. (a) 8.4 km/min 8.4 km = 1 min 8.4 km =  1  60  h = 504 km/h (b) 315 m/s 315 m = 1s 07 50 07 00 50 min 8h 30 min + 50 min = 80 min = 1 h 20 min \ The time taken from B to E was 9 h 20 min . (c) Depart from C: 02 30; Arrive at F: 09 20 02 30 03 00 09 00 09 20 30 min 6h 20 min 30 min + 20 min = 50 min \ The time taken from C to F was 6 h 50 min. (d) Depart from D: 04 20; Arrive at G: 10 45 04 20 05 00 10 00 5h 40 min 22 00 10 00 30 min  1   3600  h = 1134 km/h (c) 242 m/min 242 m = 1 min 10 45 45 min 40 min + 45 min = 85 min = 1 h 25 min \ The time taken from D to G was 6 h 25 min. (e) Depart from A: 21 30; Arrive at G: 10 45 21 30 = 12 h  242   1000  km =  1  60  h 13 km/h 25 (d) 125 cm/s 125 cm = 1s = 14 10 45 45 min 30 min + 45 min = 75 min = 1 h 15 min \ The time taken from A to G was 13 h 15 min.  125   100 000  km =  1   3600  h = 4.5 km/h 4. (a) 65 cm/s 65 cm = 1s Exercise 9D 30 1 = hour 60 2 Speed of the particle 24.6 km =  1  2  hour 1. (i) 30 minutes =  65   100  km = 1s 13 m/s = 20 (b) 367 km/h 367 km = 1h 367 × 1000 m = 3600 s 367 000 m = 3600 s = 49.2 km/h (ii) 24 .6 km = 24 .6 × 1000 = 24 600 m 30 minutes = 30 × 60 = 1800 s Speed of the particle 24 600 m = 1800 s = 13 2 m/s 3 2. 12 24 hours 1 hour 48 minutes 14 12 hours = 107 48 4 1 hour 48 minutes = 1 = 1 hours 60 5 Distance between the two stations = 200 × 1  315  km  1000  17 m/s 18 4 5 = 360 km = 360 × 1000 m = 360 000 m 145 1 (c) 1000 cm/min 1000 cm = 1 min 7.  1000   100  m = 60 s Time taken = 12 s X 60 15 =4s (ii) Average speed of the object for its entire journey from X to Y Total distance travelled = Total time taken = 1 m/s 6 (d) 86 km/min 86 km = 1 min 86 × 1000 m = 60 s 120 12 + 4 120 = 16 1 = 7 m/s 2 = 1 m/s 3 5. Speed of the fastest Singaporean sprinter 100 m = 10.37 s = 1433 8. L  100   1000  km =  10.37   3600  h 742 km/h 1037 Number of times the bullet train is as fast as the fastest Singaporean sprinter 365 742    34 1037  19 57 1 = h 3 Time taken to travel the remaining part of the journey = 160 6+4 160 = 10 = 16 m/s 9. Time taken to travel the first 50 km of its journey 50 × 1000 m = 25 m/s = 55 110 1 = h 2 Average speed of the car for its entire journey Total distance travelled = Total time taken = = 2000 s 2000 h 3600 5 = h 9 Time taken to travel the next 120 km of its journey = 19 + 55 1 1 + 3 2 74 5 6 120 80 1 =1 h 2 = 4 km/h 5 1 N 100 25 =4s Average speed of the object for its entire journey from L to N Total distance travelled = Total time taken 3701 7200 6. Time taken to travel the first part of the journey = 88 Speed = 25 m/s M 160 m = = 10 = Speed = 10 m/s Time taken = 6 s Distance travelled from L to M = 10 × 6 = 60 m Thus, distance travelled from M to N = 160 – 60 = 100 m Time taken to travel from M to N = 34 = Y (i) Time taken to travel from M to Y = = Speed = 15 m/s M 120 m 146 Then the time taken for Nora to meet Lixin will be (x + 6) min . Distance between Town A and Town B = 100 × (x + 6) + 75 × (x + 6) = 100x + 600 + 75x + 450 = (175x + 1050) m Thus, 180x = 175x + 1050 10x – 175x = 1050 5x = 1050 x = 210 Distance between Town A and Town B = 180 × 210 = 37 800 m Distance travelled for the last part of its journey 35 = 90 × 60 = 52.5 km Average speed of the object for its entire journey Total distance travelled = Total time taken 50 + 120 + 52.5 1 35 5 +1 + 9 2 60 6 = 84 km/h 19 10. Radius of the wheel of the car = 60 2 = 30 cm = 0 .3 m Circumference of the wheel of the car = 2 × p × 0 .3 = 2 × 3 .142 × 0 .3 = 1.8852 m Distance travelled by the car in 1 hour = 13 .2 × 60 × 60 = 47 520 m Number of revolutions made by the wheel per minute = Review Exercise 9 1 1 : b:c=3:4 2 3 ↓ ×2 ↓ ×6 = 3: 2 =6:8 ↓ ×3 = 9:6 \ a : c = 9 : 8 . 2. (i) Let the mass of type A coffee beans in the mixture be 3x kg . Then the mass of type B and C coffee beans in the mixture be 5x kg and 7x kg respectively . \ 3x + 5x + 7x =35 15x =45 x =3 Mass of type A coffee beans in the mixture = 3 × 3 = 9 kg Mass of type B coffee beans in the mixture = 5 × 3 = 15 kg Mass of type C coffee beans in the mixture = 7 × 3 = 21 kg (ii) Cost of the mixture per kg 1. a : b = 47 520 1.8852 = 25 207 (to the nearest whole number) 11. Length of the goods train = 8  8  =  72 × +  54 × 3600  3600    7 km 25 7 = × 1000 25 = 280 m = 12. (9 × $7) + (15 × $10) + (21 × $13) 45 = $10 .80 3. (i) Let the number of books in the box be 4x . Then the initial number of toys in the box be 5x . \ 4x = 36 x =9 So the initial number of toys in the box is 5 × 9 = 45. (ii) Let the number of toys that are given away be y . 36 12 \ = 45 – y 11 11(36) = 12(45 – y) 396 = 540 – 12y 12y =540 – 396 12y = 144 y = 12 The number of toys that are given away is 12 . = Nora: 100 m/min Town A Town B Kate: 80 m/min Let the time taken for Nora to meet Kate be x min . So distance between Town A and Town B = 100 × x + 80 × x = 180x m Nora: 100 m/min Town A Town B Kate: 80 m/min 6 min Lixin: 75 m/min 147 1 4. (i) Total cost of placing an advertisement containing 22 words = 350 + (22 × 25) = 900 cents = $9 (ii) Let the number of words he can use be x . Then 350 + 25x  1500 25x  1500 – 350 25x  1150 x  46 The greatest number of words he can use is 46 . +2h + 15 min 5. (i) ? ? 12 06 09 51 11 51 12 00 12 06 –2h – 9 min Time taken for the athlete to run 5 × 1000 3 2 = 1666 s 3 2 = 1666 ÷ 3600 3 25 = h 54 His average speed for the entire competition Total distance travelled = Total time taken 750 + 20 + 5 = 1000 15 30 25 + + 60 60 54 30 = 21 km/h 131 2 8. Let the distance from A to C be x m . 5 3 Then the distance from C to B be x m . 5 = – 6 min Working backwards, the time the journey started was 09 51. (ii) Total distance = 198 km 1 9 Total time = 2 h 15 min = 2 h = h 4 4 198 Average speed = 9 4 = 88 km/h 195 6. (i) Time taken = 52 3 =3 h 4 = 3 h 45 min +3h + 15 min Time Taken = 30 s A 2xm 5 + 30 min 5 min 3h 3 x ÷ 30 5 3 1 = x× 5 30 1 = xs 50 (ii) Average speed of the object for its entire journey from A to B xm = 1    30 + 50 x  s 18 15 15 min 5 min + 15 min = 20 min \ The time taken was 3 h 20 min . 1 3 h 20 min = 3 h 3 10 = h 3 195 Average speed = 10 3 = 58.5 km/h 7. Distance that the athlete cycles 50 x m/s 1500 + x 9. Let the first part of the journey be x km . Then the remaining part of the journey be (150 – x) km . Time taken for the entire journey = 4.5 h = x 150 – x + 35 5 x 150 – x 35 × + 35 × 35 5 x + 7(150 – x) x + 1050 – 7x x – 7x – 6x x \ Distance = 148.75 km \ 30 = 40 × 60 = 20 km 1 B 3xm 5 = (ii) 18 00 C (i) Time taken for the object to travel from C to B 3 x = 5 30 08 45 11 45 12 00 12 30 \ The time at which the lorry arrives at its destination is 12 30 . 14 55 15 00 Average speed = 30 m/s 148 = 4.5 = 35 × 4.5 = 157.5 =157.5 =157.5 – 1050 =–892.5 =148.75 For the second race, Let the time for the first person to pass the end point be t s . Time taken for Vishal to finish the 100 m race 10. Radius of the wheel of the car 48 = 2 = 24 cm Circumference of the wheel of the car = 2 × p × 24 = 2 × 3 .142 × 24 = 150.816 cm Distance travelled by a car in 1 minute = 3.5 ÷ 60 110 x 110 100 y = ) (Substitute x = 90  100 y   90  = 99 s y Time taken for Jun Wei to finish the 100 m race 100 = s y At time t s, distance that Vishal covered 99 t= y = 7 km 120 7 = × 100 000 cm 120 1 = 5833 cm 3 Number of revolutions made by the wheel per minute 1 5833 3 = 150.816 = 39 (to the nearest whole number) = ty = 99 m At time t s, distance that Jun Wei covered 100 t= y ty = 100 m \ Vishal win the race by 100 – 99 = 1 m. Challenge Yourself 1. Let the rate of the moving escalator be x steps per second . When she is walking down at a rate of 2 steps per second, then the total steps (including the steps covered by the moving escalator) covered in 1 second is (x + 2) . Since she use 18 steps to reach the bottom from the top, therefore, the time taken is (18 ÷ 2) = 9 seconds . When she is exhausted, then the total steps (including the steps covered by the moving escalator) covered in 1 second is (x + 1) . Since she use 12 steps to reach the bottom from the top, therefore, the time taken is (12 ÷ 1) = 12 seconds . Hence, 9(x + 2) = 12(x + 1) 9x + 18 = 12x + 12 9x – 12x = 12 – 18 –3x = –6 x =2 \ Total steps covered by the moving escalator = 9(2 + 2) = 36 . Hence the time taken for her to reach the bottom from the top if she stands on the escalator 36 2 = 18 s 2. Let Vishal’s speed be x m/s and Jun Wei’s speed be y m/s. Then in the first race, when Vishal ran pass the end point 100 m, Jun Wei is only at 90 m of the race. Hence, at the same time, 90 100 = y x 100y = 90x 100 y x= 90 = 149 1 Chapter 10 Basic Geometry TEACHING NOTES Suggested Approach Students have learnt angle measurement in primary school. They have learnt the properties, namely, angles on a straight line, angles at a point and vertically opposite angles. However, students are unfamiliar with the types of angles and using algebraic terms in basic geometry. There is a need to guide students to apply basic algebra and linear equations in this topic. Students will learn how to do this through the worked examples in this topic. Teachers can introduce basic geometry by showing real-life applications (see chapter opener on page 231). Section 10.1: Points, Lines and Planes Teachers should illustrate what a point, a line, intersecting lines and planes look like. Teachers can impress upon the students that there is a difference between a line and a ray. A ray has a direction while a line has no direction. Teachers can highlight to the students that for a ray, the arrowhead indicates the direction in which the ray extends while for a line, its arrowhead is to indicate that the line continues indefinitely. The thinking time on page 234 of the textbook requires students to think and determine whether each of the statements is true or false. Teachers should make use of this opportunity to highlight and clear some common misconceptions about points, lines and planes. Section 10.2: Angles Teachers can build upon prerequisites, namely angle measurement, to introduce the types of angles by classifying angle measurements according to their sizes. To make practice more interesting, teachers can get the students to work in groups to measure and classify the various types of angles of different objects (i.e. scissors, set square, compass and the hands of a clock). Teachers should recap with students on what they have learnt in primary school, i.e. angles on a straight line, angles at a point and vertically opposite angles. After going through Worked Examples 1 to 4, students should be able to identify the properties of angles and use algebraic terms to form and solve a linear equation to find the value of the unknowns. Students are expected to state reasons in their working. Section 10.3: Angles Formed by Two Parallel Lines and a Transversal Teachers can get students to discuss examples where they encounter parallel lines in their daily lives and ask them what happens when a line or multiple lines cut the parallel lines. To make learning more interactive, students are given the opportunity to explore the three angle properties observed when a pair of parallel lines is cut by a transversal (see Investigation: Corresponding Angles, Alternate Angles and Interior Angles). Through this investigation, students should be able to observe the properties of angles associated with parallel lines. The investigation also helps students to learn how to solve problems involving angles formed by two parallel lines and a transversal. Students are expected to use appropriate algebraic variables to form and solve linear equations to find the value of the unknowns. Teachers should emphasise the importance of stating the properties when the students are solving questions on basic geometry. Challenge Yourself Question 1: Teachers can guide the students by hinting to them that this question is similar to a problem involving number patterns. Students have to draw a table and write down the first few numbers of rays between OA and OB, and their respective number of different angles. The students will then have to observe carefully and find an expression that represents rays between OA and OB. Question 2: Teachers can guide the students by telling them to find the different angles that both the hour hand and minute hand makes from one specific position to another. Question 3: Teachers can guide the students by telling them to find the number of times the bell will sound between certain times of the day. 1 150 2. 2c° + 100° + 3c° 2c° + 3c° 5c° c° \c WORKED SOLUTIONS Thinking Time (Page 234) (a) False. There are an infinite number of points lying on a line segment. (b) False. There is exactly one line that passes through any three distinct points which are collinear; there is no line that passes through any three distinct points which are non-collinear. (c) False. There is exactly one line that passes through any two distinct points. (d) False. Two distinct lines intersect at one point; two coincident lines intersect at an infinite number of points; two parallel lines do not intersect at any point. (e) True. Practise Now 2 1. 58° + 148° + 7a° = 360° (/s at a point) 7a° = 360° – 58° – 148° = 154° a° = 22° \ a = 22 2. b° + 90° + b° + 4b° = 360° (/s at a point) b° + b° + 4b° = 360° – 90° 6b° = 270° b° = 45° \ b = 45 Investigation (Corresponding Angles, Alternate Angles and Interior Angles) /a = /b /a = /c /a + /d = 180° /a = /b (corr. /s) /a = /c (alt. /s) /a + /d = 180° (int. /s) /b = /a (corr. /s) /c = /b (vert. opp. /s) = /a \ /a = /c (proven) 6. Method 1: /b = /a (corr. /s) /b + /d = 180° (adj. /s on a str. line) \ /a + /d = 180° (proven) Method 2: /c = /a (alt. /s) /c + /d = 180° (adj. /s on a str. line) \ /a + /d = 180° (proven) 1. 2. 3. (a) (b) (c) 5. Practise Now 3 (i) AOC + 90° + 53° = 180° (adj. /s on a str. line) AOC = 180° – 90° – 53° = 37° (ii) BOD = AOC = 37° (vert. opp. /s) Practise Now 4 3a° + 40° = a° + 60° (vert. opp. /s) 3a° – a° = 60° – 40° 2a° = 20° a° = 10° \ a = 10 a° + 60° + 4b° + 10° = 180° (adj. /s on a str. line) 10° + 60° + 4b° + 10° = 180° 4b° = 180° – 10° – 60° – 10° = 100° b° = 25° \ b = 25 Practise Now (Page 236) (a) (b) (c) (d) (e) (f) = 180° (adj. /s on a str. line) = 180° – 100° = 80° = 16° = 16 Acute Reflex Obtuse Obtuse Reflex Acute Practise Now (Page 246) (a) (i) /a and /m, /b and /n, /c and /o, /d and /p, /e and /i, /f and /j, /g and /k, /h and /l (ii) /c and /m, /d and /n, /g and /i, /h and /j (iii) /c and /n, /d and /m, /g and /j, /h and /i (b) No, /c ≠ /g as PQ is not parallel to RS . Practise Now 1 1. (a) 122° + a° = 180° (adj. /s on a str. line) a° = 180° – 122° = 58° \ a = 58 (b) 95° + 65° + b° = 180° (adj. /s on a str. line) b° = 180° – 95° – 65° = 20° \ b = 20 151 1 DCQ = CDR = 35° (alt. /s, PQ // RS) b° + ACQ + DCQ = 360° (/s at a point) b° + 48° + 35° = 360° b° = 360° – 35° – 48° = 277° \ b = 277 Practise Now 5 a° = 54° (corr. /s, AB // CD) \ a = 54 c° + 106° = 180° (int. /s, AB // CD) c° = 180° – 106° = 74° \ c = 74 b° = c° (vert. opp. /s) = 74° \ b = 74 d° = c° (corr. /s, AB // CD) = 74° \ d = 74 2. 2e° + 30° = 69° (corr. /s, AB // CD) 2e° = 69° – 30° = 39° e° = 19 .5° \ e = 19 .5 f ° = 2e° (corr. /s, AB // CD) = 39° \ f = 39 1. Practise Now 7 C 19° X A C 2a° 20° P D Q E XCQ = 44° (alt. /s, AB // PQ) ECQ = 20° (alt. /s, PQ // DE) 2a° = XCQ + ECQ = 44° + 20° = 64° a° = 32° \ a = 32 2. 228° X b° C E 32° Since BWQ = DYQ (= 122°), then AB // CD (converse of corr. /s) . \ BXS = CZR = 65° (alt. /s, AB // CD) Q D R 293° S Exercise 10A 1. (a) (b) (c) (d) 2. (a) (b) (c) (d) (e) (f) F XAC = 228° – 180° = 48° ACQ = XAC (alt. /s, XB // PQ) = 48° EDR = 32° (alt. /s, RS // EF) EDR + CDR + 293° = 360° (/s at a point) 32° + CDR + 293° = 360° CDR = 360° – 32° – 293° = 35° 1 E 245° Practise Now 8 B A P D P B 44° 2b° B DEF + 245° = 360° (/s at a point) DEF = 360° – 245° = 115° 5a° = 115° (corr. /s, BC // EF) a° = 23° \ a = 23 CDQ = 19° (alt. /s, BC // PQ) EDQ + DEF = 180° (int. /s, PQ // EF) EDQ + 115° = 180° EDQ = 180° – 115° = 65° 2b° = CDQ + EDQ = 19° + 65° = 84° b° = 42° \ b = 42 Practise Now 6 1. Q 5a° A F 152 a = 79, b = 106, c = 98 d = 50, e = 228 f = 117, g = 45 h = 243, i = 94, j = 56 Obtuse Reflex Acute Reflex Acute Obtuse 3. (a) Complementary angle of 18° = 90° – 18° = 72° (b) Complementary angle of 46° = 90° – 46° = 44° (c) Complementary angle of 53° = 90° – 53° = 37° (d) Complementary angle of 64° = 90° – 64° = 26° 4. (a) Supplementary angle of 36° = 180° – 36° = 144° (b) Supplementary angle of 12° = 180° – 12° = 168° (c) Supplementary angle of 102° = 180° – 102° = 78° (d) Supplementary angle of 171° = 180° – 171° = 9° 5. (a) a° + 33° = 180° (adj. /s on a str. line) a° = 180° – 33° = 147° \ a = 147 (b) b° + 42° + 73° = 180° (adj. /s on a str. line) b° = 180° – 42° – 73° = 65° \ b = 65 (c) 4c° + 80° + c° = 180° (adj. /s on a str. line) 4c° + c° = 180° – 80° 5c° = 100° c° = 20° \ c = 20 (d) 4d° + 16° + 2d° + 14° = 180° (adj. /s on a str. line) 4d° + 2d° = 180° – 16° – 14° 6d° = 150° d° = 25° \ d = 25 6. (a) x° + y° + z° = 180° (adj. /s on a str. line) When y° = 45°, z° = 86°, x° + 45° + 86° = 180° x° = 180° – 45° – 86° = 49° \ x = 49 (b) x° + y° + z° = 180° (adj. /s on a str. line) When x° = 2y°, z° = 3y°, 2y° + y° + 3y° = 180° 6y° = 180° y° = 30° \ y = 30 7. (a) a° + 67° + 52° + 135° = 360° (/s at a point) a° = 360° – 67° – 52° – 135° = 106° \ a = 106 (b) 5b° + 4b° + 3b° = 360° (/s at a point) 12b° = 360° b° = 30° \ b = 30 8. 9. 10. 11. 153 (c) 16c° + 4c° + 90° + 4c° = 360° (/s at a point) 16c° + 4c° + 4c° = 360° – 90° 24c° = 270° c° = 11 .25° \ c = 11 .25 (d) (7d + 23)° + 6d° + 139° + 5d° = 360° (/s at a point) 7d° + 23° + 6d° + 139° + 5d° = 360° 7d° + 6d° + 5d° = 360° – 23° – 139° 18d° = 198° d° = 11° \ d = 11 (i) AOC = 48° (vert. opp. /s) (ii) 90° + DOE + 48° = 180° (adj. /s on a str. line) DOE = 180° – 90° – 48° = 42° (a) 40° + 30° + a° = 117° (vert. opp. /s) a° = 117° – 40° – 30° = 47° \ a = 47 (b) 7b° + 3b° = 180° (adj. /s on a str. line) 10b° = 180° b° = 18° \ b = 18 c° = 7b° (vert. opp. /s) = 7(18°) = 126° \ c = 126 (a) x° + y° + z° = 180° (adj. /s on a str. line) y° + x° + z° = 180° When y° = x° + z°, y° + y° = 180° 2y° = 180° y° = 90° \ y = 90 (b) x° + y° + z° = 180° (adj. /s on a str. line) When x° = y° = z°, z° + z° + z° = 180° 3z° = 180° z° = 60° \ z = 60 AOB + DOA = 180° (adj. /s on a str. line) AOB + 5AOB = 180° 6AOB = 180° \ AOB = 30° BOC = 2AOB = 2 × 30° = 60° COD = 4AOB = 4 × 30° = 120° DOA = 5AOB = 5 × 30° = 150° 1 = 180° (adj. /s on a str. line) = 180° = 180° = 180° – 132° = 48° y° = 16° \ y = 16 (ii) Obtuse AOD = (186 – 4x)° + 34° = [186 – 4(22)]° + 34° = 98° + 34° = 132° Reflex COE = 180° + (186 – 4x)° = 180° + 98° = 278° 12. (a) 7a° + 103° = 180° (adj. /s on a str. line) 7a° = 180° – 103° = 77° a° = 11° \ a = –11 2b° + 13° = 103° (vert. opp. /s) 2b° = 103° – 13° = 90° b° = 45° \ b = 45 (b) 62° + 49° + 3c° = 180° (adj. /s on a str. line) 3c° = 180° – 62° – 49° = 69° c° = 23° \ c = 23 d° = 3c° (vert. opp. /s) = 69° \ d = 69 e° = 62° + 49° (vert. opp. /s) = 111° \ e = 111 (c) 7f ° + 5° = 2f ° + 35° (vert. opp. /s) 7f ° – 2f ° = 35° – 5° 5f ° = 30° f ° = 6° \f =6 2f ° + 35° + 5g° + 18° = 180° (adj. /s on a str. line) 2(6°) + 35° + 5g° + 18° = 180° 12° + 35° + 5g° + 18° = 180° 5g° = 180° – 12° – 35° – 18° = 115° g° = 23° \ g = 23 (d) 24° + 90° + h° = 104° + 32° (vert. opp. /s) h° = 104° + 32° – 24° – 90° = 22° \ h = 22 24° + 90° + h° + 2i° = 180° (adj. /s on a str. line) 24° + 90° + 22° + 2i° = 180° 2i° = 180° – 24° – 90° – 22° = 44° i° = 22° \ i = 22 j° = 2i° (vert. opp. /s) = 44° \ j = 44 13. (i) (186 – 4x)° + 34° = 6x° (vert. opp. /s) 186° – 4x° + 34° = 6x° 6x° + 4x° = 186° + 34° 10x° = 220° x° = 22° \ x = 22 1 6x° + 3y° 6(22°) + 3y° 132° + 3y° 3y° Exercise 10B 1. (a) (i) BXR and DZR, AXR and CZR, AXS and CZS, BXS and DZS, BWP and DYP, AWP and CYP, AWQ and CYQ, BWQ and DYQ (ii) AXS and DZR, BXS and CZR, AWQ and DYP, BWQ and DYP (iii) AXS and CZR, BXS and DZR, AWQ and CYP, BWQ and DYP (b) No, BWQ ≠ AXR as PQ is not parallel to RS . (c) No, the sum of DYP and CZR is not equal to 180° as PQ is not parallel to RS . 2. (a) a° = 117° (vert. opp. /s) \ a = 117 b° = 117° (corr. /s, AB // CD) \ b = 117 c° + a° = 180° (int. /s, AB // CD) c° + 117° = 180° c° = 180° – 117° = 63° \ c = 63 d° = 78° (corr. /s, AB // CD) \ d = 78 (b) e° = 31° (alt. /s, AB // CD) \ e = 31 f ° = 35° + 31° (alt. /s, AB // CD) = 66° \ f = 66 (c) g° = 83° (alt. /s, AB // CD) \ g = 83 h° = 69° (corr. /s, AB // CD) \ h = 69 (d) i° + 75° + 60° = 180° (int. /s, AB // CD) i° = 180° – 75° – 60° = 45° \ i = 45 j° = 60° (alt. /s, AB // CD) \ j = 60 154 a° = 38° (corr. /s, AB // CD) \ a = 38 a° + 30° = 2b° (corr. /s, AB // CD) 38° + 30° = 2b° 2b° = 68° b° = 34° \ b = 34 (b) 7c° = 140° (corr. /s, AB // CD) c° = 20° \ c = 20 2d° = 7c° (vert. opp. /s) = 140° d° = 70° \ d = 70 (c) 7e° + 3e° = 180° (int. /s, AB // CD) 10e° = 180° e° = 18° \ e = 18 (d) (2f + 6)° = (3f – 23)° (alt. /s, AB // CD) 2f ° + 6° = 3f ° – 23° 3f ° – 2f ° = 6° + 23° f ° = 29° \ f = 29 4. (a) A 5. (a) 3. (a) D C CEQ + 128° = 180° (int. /s, PQ // CD) CEQ = 180° – 128° = 52° AEQ = 92° – CEQ = 92° – 52° = 40° a° + AEQ = 180° (int. /s, AB // PQ) a° + 40° = 180° a° = 180° – 40° = 140° \ a = 140 P (b) X A B (4b – 10)° B Y D (2b – 2)° Q Q 114° CYP = (2b – 2)° (vert. opp. /s) CYP + (4b – 10)° = 180° (int. /s, AB // CD) (2b – 2)° + (4b – 10)° = 180° 2b° – 2° + 4b° – 10° = 180° 2b° + 4b° = 180° + 2° + 10° 6b° = 192° b° = 32 \ b = 32 D AEQ + 142° = 180° (int. /s, AB // PQ) AEQ = 180° – 142° = 38° CEQ + 114° = 180° (int. /s, PQ // CD) CEQ = 180° – 114° = 66° a° = AEQ + CEQ = 38° + 66° = 104° \ a = 104 (c) R C 69° E 37° A P B A C Q C C b° 92° 128° a° P E P E (b) B a° 142° P A B 28° E Q 94° F c° S 19° D AEP = 28° (alt. /s, AB // PQ) FEP = 94° – AEP = 94° – 28° = 66° EFS = FEP = 66° (alt. /s, PQ // RS) DFS = 19° (alt. /s, RS // CD) c° + EFS + DFS = 360° (/s at a point) c° + 66° + 19° = 360° c° = 360° – 66° – 19° = 275° \ c = 275 Q D AEP = 69° (alt. /s, AB // PQ) CEP = 37° (alt. /s, PQ // CD) b° = AEP + CEP = 69° + 37° = 106° \ b = 106 155 1 BAC + DCA = 180° (int. /s, AB // CE) BAC + 122° = 180° BAC = 180° – 122° = 58° 7y° + BAC = 360° 7y° + 58° = 360° 7y° = 360° – 58° = 302° y° = 43.1° (to 1 d.p.) \ y = 43 .1 6. (i) CDF = 86° (alt. /s, CE // FG) (ii) HDE = 86° (vert. opp. /s) EDA = HDE – 47° = 86° – 47° = 39° BAD + EDA = 180° (int. /s, AB // CE) BAD + 39° = 180° BAD = 180° – 39° = 141° 7. (i) AEB = 68° (alt. /s, BF // AD) (ii) EAB = 58° (alt. /s, AB // CD) FBA + EAB = 180° (int. /s, BF // AD) FBA + 58° = 180° FBA = 180° – 58° = 122° ABE = FBA – 68° = 122° – 68° = 54° 8. B C A P 46° 72° D E 52° F 10. A Q x° B D P 5y° 147° E x° = 147° (corr. /s, BC // EF) \ x = 147 CDQ = 32° (alt. /s, BC // PQ) QDE + 147° = 180° (int. /s, PQ // EF) QDE = 180° – 147° = 33° 5y° + CDQ + QDE = 360° (/s at a point) 5y° + 32° + 33° = 360° 5y° = 360° – 32° – 33° = 295° y° = 59° \ y = 59 11. Since AXS + CZR = 104° + 76° = 180°, then AB // CD (converse of int. /s) . \ BWP = DYP = 46° (corr. /s, AB // CD) A 12. Q G (i) CGF = 52° (corr. /s, FD // GC) (ii) BCG + CGF = 180° (int. /s, AC // EG) BCG + 52° = 180° BCG = 180° – 52° = 128° BCF = BCG – 72° = 128° – 72° = 56° (iii) BDQ = 46° (alt. /s, AC // PQ) FDQ = 52° (alt. /s, PQ // EG) Reflex BDF + BDQ + FDQ = 360° (/s at a point) Reflex BDF + 46° + 52° = 360° Reflex BDF = 360° – 46° – 52° = 262° 9. FDC + 58° = 180° (adj. /s on a str. line) FDC = 180° – 58° = 122° DCA = FDC (alt. /s, DF // AC) = 122° DCA + 4x° = 360° (/s at a point) 122° + 4x° = 360° 4x° = 360° – 122° = 238° x° = 59 .5° \ x = 59 .5 1 C 32° B w° C P R x° y° Q D S z° E F G QCA + w° = 180° (int. /s, AB // PQ) QCA = 180° – w° QCD = x° – QCA = x° – (180° – w°) = x° – 180° + w° CDR = QCD (alt. /s, PQ // RS) = x° – 180° + w° FDR = y° – CDR = y° – (x° – 180° + w°) = y° – x° + 180° – w° FDR + z° = 180° (int. /s, RS // EG) y° – x° + 180° – w° + z° = 180° w° + x° = y° + z° \w+x=y+z 156 F 4. (a) A Review Exercise 10 B a° 1. (a) 32° + 4a° + 84° = 180° (adj. /s on a str. line) 4a° = 180° – 32° – 84° = 64° a° = 16° \ a = 16 84° + 2b° = 180° (adj. /s on a str. line) 2b° = 180° – 84° = 96° b° = 48° \ b = 48 (b) c° + 68° = 180° (adj. /s on a str. line) c° = 180° – 68° = 112° \ c = 112 68° + 3d° – 5° + 30° = 180° (adj. /s on a str. line) 3d° = 180° – 68° + 5° – 30° = 87° d° = 29° \ d = 29 2. (a) 4a° + 2a° + a° + a° + 2a° = 360° (/s at a point) 10a° = 360° a° = 36° \ a = 36 (b) (3b – 14)° + (4b – 21)° + (2b + 1)° + (b + 34)° = 360° (/s at a point) 3b° – 14° + 4b° – 21° + 2b° + 1° + b° + 34° = 360° 3b° + 4b° + 2b° + b° = 360° + 14° + 21° – 1° – 34° 10b° = 360° b° = 36° \ b = 36 3. (a) COF = 4a° – 17° (vert. opp. /s) 2a° + COF + 3a° – 10° = 180° (adj. /s on a str. line) 2a° + 4a° – 17° + 3a° – 10° = 180° 2a° + 4a° + 3a° = 180° + 17° + 10° 9a° = 207° a° = 23° \ a = 23 (b) COF = 2b° + 15° (vert. opp. /s) 2b° + 2b° + 15° + b° = 180° (adj. /s on a str. line) 2b° + 2b° + b° = 180° – 15° 5b° = 165° b° = 33° \ b = 33 b° + 3c° + 2b° + 15° = 180° (adj. /s on a str. line) 33° + 3c° + 2(33°) + 15° = 180° 33 + 3c° + 66° + 15° = 180° 3c° = 180° – 33° – 66° – 15° = 66° c° = 22° \ c = 22 E P 250° Q 126° C D DEP + 126° = 180° (int. /s, PQ // CD) DEP = 180° – 126° = 54° BEP + DEP + 250° = 360° (/s at a point) BEP + 54° + 250° = 360° BEP = 360° – 54° – 250° = 56° a° + BEP = 180° (int. /s, AB // PQ) a° + 56° = 180° a° = 180° – 56° = 124° \ a = 124 (b) (6b – 21)° + (5b – 52)° = 180° (int. /s, AB // CD) 6b° – 21° + 5b° – 52° = 180° 6b° + 5b° = 180° + 21° + 52° 11b° = 253° b° = 23° \ b = 23 3c° = (6b – 21)° = [6(23) – 21]° = 117° c° = 39° \ c = 39 (c) A B 276° P (5d – 13)° E Q (4d + 28)° C D QEA + (5d – 13)° = 180° (int. /s, AB // PQ) QEA + 5d° – 13° = 180° QEA = 180° – 5d° + 13° = 193° – 5d° QEC + (4d + 28)° = 180° (int. /s, PQ // CD) QEC + 4d° + 28° = 180° QEC = 180° – 4d° – 28° = 152° – 4d° 276° + QEA + QEC = 360° (/s at a point) 276° + 193° – 5d° + 152° – 4d° = 360° 5d° + 4d° = 276° + 193° + 152° – 360° 9d° = 261° d° = 29° \ d = 29 157 1 A (d) F R 285° E P Q e° S 18° C 5. (i) CDF = 148° (alt. /s, GC // DF) CDE + 84° + CDF = 360° (/s at a point) CDE + 84° + 148° = 360° CDE = 360° – 84° – 148° = 128° (ii) ABC = CDE (alt. /s, AB // DE) = 128° ABH = ABC – 74° = 128° – 74° = 54° 6. (i) DEH + 26° = 180° (adj. /s on a str. line) DEH = 180° – 26° = 154° (ii) BEH = 62° (alt. /s, EC // GH) DEB + BEH + DEH = 360° (/s at a point) DEB + 62° + 154° = 360° DEB = 360° – 62° – 154° = 144° ABC = DEB (corr. /s, AB // DF) = 144° B 7. A C B 37° D AEP = 37° (alt. /s, AB // PQ) AEP + 285° + FEP = 360° (/s at a point) 37° + 285° + FEP = 360° FEP = 360° – 37° – 285° = 38° EFS = 38° (alt. /s, PQ // RS) DFS = 18° (alt. /s, RS // CD) e° = EFS + DFS = 38° + 18° = 56° \ e = 56 (e) C B A 123° X D 122° f° 316° E XEC = 122° (alt. /s, CD // XE) XEB + 123° = 180° (int. /s, AB // XE) XEB = 180° – 123° = 57° f ° = XEC – XEB = 122° – 57° = 65° \ f = 65 (f) Q X D C 238° 5g° E A P X BEQ = 37° (alt. /s, AB // PQ) ECX = 238° – 180° = 58° CEQ = ECX (alt. /s, PQ // XD) = 58° 5g° = BEQ + CEQ = 37° + 58° = 95° g° = 19° \ g = 19 1 58° Y F G (i) DEF + reflex DEF = 360° (/s at a point) DEF + 316° = 360° DEF = 360° – 316° = 44° BDE = DEF (alt. /s, DB // FE) = 44° (ii) DYF = 58° (corr. /s, YB // FE) ABD = DYF (alt. /s, AC // XG) = 58° 8. Since BWQ + DYP = 123° + 57° = 180°, then AB // CD (converse of int. /s) . \ DZR = AXS = 118° (alt. /s, AB // CD) B 37° E D 158 9. A P D 1 × 360° 12 = 30° Angle hour hand moves from 12 noon to 7 p.m. = 7 × 30° = 210° Q x° R 2. Angle hour hand moves in 1 hour = B w° C y° S z° F Angle hour hand moves from 7 p.m. to 7.20 p.m. = 20 × 30° 60 = 10° Angle hour hand moves from 12 noon to 7.20 p.m. = 210° + 10° = 220° E BCQ = w° (alt. /s, AB // PQ) DCQ = x° – BCQ = x° – w° CDS + DCQ = 180° (int. /s, PQ // RS) CDS + x° – w° = 180° CDS = 180° – x° + w° EDS = y° – (180° – x° + w°) = y° – 180° + x° – w° EDS = z° (alt. /s, RS // FE) y° – 180° + x° – w° = z° w° + z° + 180° = x° + y° \ w + z + 180 = x + y Angle minute hand moves from 7 p.m. to 7.20 p.m. = 20 × 360° 60 = 120° Smaller angle between minute hand and hour hand at 7.20 p.m. = 220° – 120° = 100° Larger angle between minute hand and hour hand at 7.20 p.m. = 360° – 100° (/s at a point) = 260° 3. From 9 a.m. to before 9 p.m. on any particular day, the bell will sound twice every hour, except for the hour from 1 p.m. to before 2 p.m. and the hour from 2 p.m. to before 3 p.m., when it only sounds once during each hour. Likewise, from 9 p.m. on any particular day to before 9 a.m. the next day, the bell will sound twice every hour, except for the hour from 1 a.m. to before 2 a.m. and the hour from 2 a.m. to before 3 a.m, when it only sounds once during each hour. \ Number of times bell will sound from 9 a.m. on a particular day to before 9 p.m. the next day = 2 × 30 + 1 × 6 = 60 + 6 = 66 Since the bell will sound at 9 p.m. the next day, Number of times bell will sound from 9 a.m. on a particular day to 9 p.m. the next day = 66 + 1 = 67 Challenge Yourself 1. Number of Rays between OA and OB Number of Different Angles 0 1= 1 ×1×2 2 1 3= 1 ×2×3 2 2 6= 1 ×3×4 2 3 10 = 1 ×4×5 2 4 15 = 1 ×5×6 2 : n : 1 (n + 1)(n + 2) 2 Number of different angles in the figure = 1 (n + 1)(n + 2) 2 159 1 Chapter 11 Triangles, Quadrilaterals and Polygons TEACHING NOTES Suggested Approach Students have learnt about triangles, and quadrilaterals such as parallelograms, rhombuses and trapeziums in primary school. They would have learnt the properties and finding unknown angles involving these figures. In this chapter, students begin from 3-sided triangles, to 4-sided quadrilaterals and finally n-sided polygons. The incremental approach is to ensure that students have a good understanding before they move on to a higher level. Teachers may want to dedicate more time and attention to the section on polygons in the last section of this chapter. Section 11.1: Triangles Students have learnt about isosceles triangles, equilateral triangles and right-angled triangles in primary school. In this chapter, students should be aware that triangles can be classified by the number of equal sides or the types of angles. Teachers may want to check students’ understanding on the classification of triangles (see Thinking Time on page 260). Teachers should highlight to the students that equilateral triangles are a special type of isosceles triangles while scalene triangles are triangles that are not isosceles, and are definitely not equilateral triangles. Students should explore and discover that the longest side of a triangle is opposite the largest angle, and the sum of two sides is always larger than the third side (see Investigation: Basic Properties of a Triangle). Teachers should ensure students are clear what exterior angles are before stating the relation between exterior angles and its interior opposite angles. Some may think that the exterior angle of a triangle is the same as the reflex angle at a vertex of a triangle. Section 11.2: Quadrilaterals Teachers may want to first recap students’ knowledge of parallelograms, rhombuses and trapeziums based on what they have learnt in primary school. Teachers can use what students have learnt in Chapter 10, reintroduce and build up their understanding of the different types of quadrilaterals and their properties (see Investigation: Properties of Special Quadrilaterals and Investigation: Symmetric Properties of Special Quadrilaterals). For further understanding, teachers may wish to show the taxonomy of quadrilaterals to demonstrate their relations. Before proceeding onto the next section, teachers may want to go through with the students the angle properties of triangles and quadrilaterals. This reinforces the students’ knowledge as well as prepares them for the section on polygons. Section 11.3: Polygons Teachers should emphasise to the students that triangles and quadrilaterals are polygons so that they are aware that all the concepts which they have learnt so far remains applicable in this topic. Students should learn the different terms with regards to polygons. In this section, most polygons studied will be simple, convex polygons. Students need to know the names of polygons with 10 sides or less and the general naming convention of polygons (see Class Discussion: Naming of Polygons). Through the class discussion, students should be able to develop a good understanding on polygons and be able to name them. They should also know and appreciate the properties of regular polygons (see Investigation: Properties of a Regular Polygon and Investigation: Symmetric Properties of Regular Polygons). Teachers can ask students to recall the properties of triangles and quadrilaterals during the investigation of the sum of interior angles and sum of exterior angles of a polygon. Students should see a pattern in how the sum of interior angles differs as the number of sides increases and understand its formula, (see Investigation: Sum of Interior Angles of a Polygon) as well as discover that the sum of exterior angles is always equal to 360° regardless of the number of sides of the polygon (see Investigation: Sum of Exterior Angles of a Pentagon). 1 160 Challenge Yourself Some of the questions (e.g. Questions 1 and 2) may be challenging for most students while the rest of the questions can be done with guidance from teachers. Question 1: Two new points need to be added. The first point (say, E) is the midpoint of BC and the second point (say, F) lies on the line AE such that nBCF is equilateral. Draw the lines AE, CF and DF. Begin by finding ABC and continue from there. Question 2: Draw DG such that BC // DG, and mark E at the point where DG cuts CD. Join E and F. Begin by finding ACB and continue from there. 161 1 6. (a) Square : All sides are equal in length . Parallelogram : Opposite sides are equal in length . Rhombus : All sides are equal in length . Trapezium : All sides are not equal in length . Kite : There are two pairs of equal adjacent sides . (b) Square : All four interior angles are right angles . Parallelogram : Opposite interior angles are equal . Rhombus : Opposite interior angles are equal . Trapezium : All four interior angles are not equal . Kite : One pair of opposite interior angles is equal . (c) Square : The two diagonals are equal in length . Parallelogram : The two diagonals are not equal in length . Rhombus : The two diagonals are not equal in length . Trapezium : The two diagonals are not equal in length . Kite : The two diagonals are not equal in length . (d) Square : The diagonals bisect each other . Parallelogram : The diagonals bisect each other . Rhombus : The diagonals bisect each other . Trapezium : The diagonals do not bisect each other . Kite : The diagonals do not bisect each other . (e) Square : The diagonals are perpendicular to each other . Parallelogram : The diagonals are not perpendicular to each other . Rhombus : The diagonals are perpendicular to each other . Trapezium : The diagonals are not perpendicular to each other . Kite : The diagonals are perpendicular to each other . (f) Square : The diagonals bisect the interior angles . Parallelogram : The diagonals do not bisect the interior angles . Rhombus : The diagonals bisect the interior angles . Trapezium : The diagonals do not bisect the interior angles . Kite : One diagonal bisects the interior angles . WORKED SOLUTIONS Thinking Time (Page 260) A represents isosceles triangles . B represents scalene triangles . C represents acute-angled triangles . D represents right-angled triangles . Investigation (Basic Properties of a Triangle) 1. The side opposite /B is b and the side opposite /C is c . 2. The largest angle is /C and the smallest angle is /B . The side opposite the largest angle, /C is the longest side and the side opposite the smallest angle, /B is the shortest side . 3. The bigger the angle, the longer the side opposite it . The angle opposite the side shortest in length will be the smallest angle. This applies to the longest side as well i.e. the longest side is always opposite the largest angle . 4. The sum of the lengths of the two shorter sides of a triangle is always longer than the length of the longest side . 5. Yes, since the sum of the angles facing the two shorter sides are greater than the largest angle facing the longest side, hence, the sum of the lengths of the two shorter sides of a triangle is always longer than the length of the longest side . 6. No, it is not possible to form a triangle . 7. a + b = c . It is still not possible to form a triangle . 8. The sum of the lengths of any two line segments has to be greater than the length of the third line segment From the investigation, two basic properties of a triangle are: • The largest angle of a triangle is opposite the longest side, and the smallest angle is opposite the shortest side . • The sum of the lengths of any two sides of a triangle must be greater than the length of the third side . Thinking Time (Page 271) Investigation (Properties of Special Quadrilaterals) (a) Yes (b) Yes (d) Yes (e) Yes A represents kites . B represents parallelograms . C represents rhombus . D represents squares . 1. AB = 2 .8 cm, BC = 1 .8 cm, DC = 2 .8 cm, AD = 1 .8 cm AB = DC and BC = AD (Opposite sides are equal in length .) 2. BAD = 90°, ABC = 90°, BCD = 90°, ADC = 90° BAD = ABC = BCD = ADC = 90° (All four interior angles are right angles .) 3. AE = 1 .7 cm, BE = 1 .7 cm, CE = 1 .7 cm, DE = 1 .7 cm AE = BE = CE = DE = 1 .7 cm (Diagonals bisect each other .) 4. AE + CE = 1 .7 + 1 .7 = 3 .4 cm, BE + DE = 1 .7 + 1 .7 = 3 .4 cm Both of the sums are equal . (The two diagonals are equal in length .) 5. The following properties hold: • Opposite sides are equal in length. • All four interior angles are right angles. • Diagonals bisect each other. • The two diagonals are equal in length. 1 162 (c) Yes Class Discussion (Naming of Polygons) Journal Writing (Page 278) Triangle (3-sided) Quadrilateral (4-sided) Pentagon (5-sided) Hexagon (6-sided) Heptagon (7-sided) Octagon (8-sided) Nonagon (9-sided) Decagon (10-sided) Since a regular polygon is a polygon with all sides equal and all angles equal, the statement made by Devi is correct as she stated one of the two properties of a regular polygon . On the other hand, the statement made by Michael is wrong as he stated an incomplete definition of a regular polygon, i .e . the conditions of a regular polygon . A polygon with all sides equal may not be regular, e .g . a square is a regular polygon (see Fig . (a)) but a rhombus is not a regular polygon (see Fig . (b)) . This is because even though a rhombus is a polygon with all sides equal, not all its angles are equal . The hexagon shown in Fig . (d) is a regular polygon but the hexagon shown in Fig . (e) is not a regular polygon because even though all its sides are equal, not all its angles are equal . Hence, it does not mean that a polygon with all sides equal is regular . Thinking Time (Page 277) Fig. (a) Fig. (b) Fig. (c) Fig. (d) Fig. (e) Fig. (f) The name of a regular triangle is an equilateral triangle and the name of a regular quadrilateral is a square . Investigation (Properties of a Regular Polygon) 1. Yes . (a) Rhombus (b) In addition, a polygon with all angles equal may not be regular . For example, a rectangle is a polygon (see Fig . (c)) but it is not regular because not all its sides are equal although all its angles are equal . Another example is the hexagon as shown in Fig . (f) . It is not a regular polygon because even though all its angles are equal, not all its sides are equal . Hence, it does not mean that a polygon with all angles equal is regular . 2. Yes . (a) Square and Rectangle (b) In conclusion, a regular polygon is a polygon with all sides equal and all angles equal . 163 1 Investigation (Sum of Interior Angles of a Polygon) Investigation (Sum of Exterior Angles of a Pentagon) 1. 1. – 2. The sum of exterior angles of a pentagon is 360° as all the exterior angles will meet at a vertex . From the investigation, we observe that the sum of exterior angles of a pentagon is 360° . A proof of the above result is given as follows: Consider the pentagon in Fig . 11 .24 . We have /a + /p = 180°, /b + /q = 180°, /c + /r = 180°, /d + /s = 180° and /e + /t = 180° . \ /a + /p + /b + /q + /c + /r + /d + /s + /e + /t = 5 × 180° (/a + /b + /c + /d + /e) + (/p + /q + /r + /s + /t) = 900° Since the sum of interior angles of a pentagon = /a + /b + /c + /d + /e = (5 – 2) × 180° = 540°, 540° + (/p + /q + /r + /s + /t) = 900° . \ /p + /q + /r + /s + /t = 900° – 540° = 360° By using this method, we can show that the sum of exterior angles of a hexagon, of a heptagon and of an octagon is also 360° . Polygon Number of sides Number of Triangle(s) formed Sum of Interior Angles 3 1 1 × 180° = (3 – 2) × 180° 4 2 2 × 180° = (4 – 2) × 180° 5 3 3 × 180° = (5 – 2) × 180° Triangle Quadrilateral Pentagon Thinking Time (Page 285) 6 4 4 × 180° = (6 – 2) × 180° 7 5 5 × 180° = (7 – 2) × 180° 8 6 6 × 180° = (7 – 2) × 180° n (n – 2) (n – 2) × 180° 1. (i) No . Since 70° is not an exact divisor of 360°, hence a regular polygon to have an exterior angle of 70° is not possible . (ii) Since 360° = 3 × 120°, 360° = 4 × 90°, 360° = 6 × 60°, 360° = 8 × 45°, 360° = 9 × 40°, 360° = 10 × 36°, 360° = 12 × 30°, 360° = 15 × 24°, 360° = 18 × 20°, 360° = 20 × 18°, 360° = 25 × 15°, 360° = 30 × 12°, 360° = 40 × 9°, 360° = 45 × 8°, 360° = 60 × 6°, 360° = 90 × 4°, 360° = 120 × 3°, 360° = 180 × 2°, All the possible values of the angle are 2°, 3°, 4°, 6°, 8°, 9°, 12°, 15°, 18°, 20°, 24°, 30°, 36°, 40°, 45°, 60°, 90° and 120° . 2. No, it is not possible as a concave polygon has one or more interior angles that are greater than 180° while as a regular polygons has all interior angles that are less than 180° . Hexagon Heptagon Octagon n-gon 2. If a polygon has n sides, then it will form (n – 2) triangles . Investigation (Tesellation) 1. The only regular polygons that tessellate on their own are equilateral triangles, squares and regular hexagons . Combinations of other regular polygons such as a square and a regular octagon can produce tessellations . 2. See Fig . 11 .17 in the textbook for an example . 3. The sum of the corner angles will add up to 360° . 1 164 54° 2 = 27° \ CDE = 27° Practise Now 1 x° = 1. 90° + 65° + a° = 180° (/ sum of n) a° = 180° – 90° – 65° = 25° \ a = 25 2. Since AC = BC, \ CAB = CBA = b° b° + 52° + b° = 180° (/ sum of n) 2b° = 180° – 52° = 128° Practise Now 4 1. (i) ABC = 108° (opp . /s of // gram) 9x° = 108° 108° 9 = 12° \ x = 12 (ii) (DCE + 38°) + 108° = 180° (int . /s, AD // BC) DCE = 180° – 38° – 108° = 34° 2. (5x + 6)° + (2x + 13)° = 180° (int . /s, AB // DC) 7x° + 19° = 180° 7x° = 180° – 19° = 161° x= 128° 2 = 64° \ b = 64 b° = Practise Now 2 a° = 53° + 48° (ext . / of n) = 101° \ a = 101 (b) FDE = 93° (vert . opp . /s) b° + 33° + 93° = 180° (/ sum of n) b° = 180° – 33° – 93° = 54° \ b = 54 c° = 41° + 93° (ext . / of nABD) = 134° \ c = 134 (a) 161° 7 = 23° \ x = 23 [5(23) + 6]° + (y + 17°) = 180° (int . /s, AB // DC) y° = 180° – 121° – 17° = 42° \ y = 42 x° = Practise Now 3 Practise Now 5 1. (i) DAE = 90° (right angle) 51° + 90° + AED = 180° (/ sum of nAED) AED = 180° – 51° – 90° = 39° (ii) CDE + 51° = 90° (/ADC is a right angle) CDE = 90° – 51° = 39° 68° + 39° + CED = 180° (/ sum of nCDE) CED = 180° – 68° – 39° = 73° 2. (i) Since EB = EC (diagonals bisect each other), \ EBC = 63° 63° + BEC + 63° = 180° (/ sum of nBEC) BEC = 180° – 63° – 63° = 54° (ii) DEC + 54° = 180° (adj . /s on a str . line) DEC = 180° – 54° = 126° Since ED = EC (diagonals bisect each other), \ CDE = DCE = x° . x° + 126° + x° = 180° (/ sum of nCDE) 2x° = 180° – 126° = 54° 1. (i) CAB = 32° (alt . /s, AB // DC) Since BA = BC, \ ACB = CAB = 32° 32° + ABC + 32° = 180° (/sum of nABC) ABC = 180° – 32° – 32° = 116° (ii) Since AC = CE, \ CEA = CAE = 32° 32° + (32° + BCE) + 32° = 180° (/ sum of nABC) BCE = 180° – 32° – 32° – 32° = 84° 2. BDC = (3x + 13)° (diagonals bisect interior angles of a rhombus) DAC = (x + 45)° (diagonals bisect interior angles of a rhombus) 2(3x + 13)° + 2(x + 45)° = 180° (int . / s, AB // DC) 6x° + 26° + 2x° + 90° = 180° 8x° = 180° – 26° – 90° 8x° = 64° 64° 8 = 8° \x =8 x° = 165 1 2. The sum of exterior angles of the regular decagon is 360° . \ Size of each exterior angle of the regular decagon Practise Now 6 1. Sum of interior angles of a pentagon = (n – 2) × 180° = (5 – 2) × 180° = 540° a° + 121° + a° + a° + 107° = 540° 3a° = 540° – 121° – 107° 3a° = 312° 360° 10 = 36° \ Size of each interior angle of the regular decagon = 180° – 36° = 144° 3. The sum of exterior angles of an n-sided polygon is 360° . 25° + 26° + 3(180° – 161°) + (n – 5)(180° – 159°) = 360° 25° + 26° + 3(19°) + (n – 5)(21°) = 360° 25° + 26° + 57° + n(21°) – 105° = 360° n(21°) = 360° – 25° – 26° – 57° + 105° = 357° = 312° 3 = 104° \ a = 104 2. Sum of interior angles of a hexagon = (n – 2) × 180° = (6 – 2) × 180° = 720° 3b° + 4b° + 104° + 114° + 128° + 122° = 720° 7b° = 720° – 104° – 114° – 128° – 122° 7b° = 252° a° = 357° 21° = 17 n= Practise Now 9 E 252° 7 = 36° \ b = 36 D b° = F Practise Now 7 C A (i) Sum of interior angles of a regular polygon with 24 sides = (n – 2) × 180° = (24 – 2) × 180° = 3960° (ii) Size of each interior angle of a regular polygon with 24 sides B G Size of each exterior angle of the hexagon 360° 6 = 60° CBG = BCG = 60° BGC + 60° + 60° = 180° (/ sum of nBCG) BGC = 180° – 60° – 60° = 60° = 3960° 24 = 165° = Practise Now 8 Practise Now 10 1. (a) The sum of exterior angles of the regular polygon is 360° . \ Number of sides of the polygon (i) Sum of interior angles of a pentagon = (n – 2) × 180° = (5 – 2) × 180° = 540° Since PBC is an interior angle of a pentagon, 360° 40° =9 (b) Size of each exterior angle of a regular polygon = 180° – 178° = 2° The sum of exterior angles of the regular polygon is 360° . \ Number of sides of the polygon = 540° = 108° . 5 (ii) Since CRQ is an interior angle of a pentagon, \ CRQ = 108° . Let QCR = CQR = x° (base /s of isos . nCQR) x° + x° + 108° = 180° (/ sum of nCQR) 2x° = 180° – 108° 2x° = 72° \ PBC = 360° 2° = 180 = 72° 2 = 36° \ QCR = 36° x° = 1 166 A 18° y° = 2 = 9° \ BDC = 9° (v) Let the exterior angle of the n-sided polygon be a° . a° + 162° = 180° (adj /s on a str . line) a° = 180° – 162° = 18° Since the sum of the exterior angles of the n-sided polygon is 360°, 2. (a) (b) \n = (c) Exercise 11A (d) 360° 18° = 20 C 1. (a) 3. (a) 60° 20° A B /C = 180° – 20° – 60° (/ sum of n) = 100° It is a scalene triangle and an obtuse-angled triangle . (b) C (d) (iii) BCD + 108° + 90° = 360° (/s at a point) BCD = 360° – 108° – 90° = 162° (iv) Let BDC = BCD = y° (base /s of isos . nBCD) y° + y° + 162° = 180° (/ sum of nBCD) 2y° = 180° – 162° 2y° = 18° (b) C 42° 48° B /C = 180° – 42° – 48° (/ sum of n) = 90° It is a scalene triangle and right-angled triangle . Third angle of the triangle = 180° – 40° – 40° (/ sum of n) = 100° Third angle of the triangle = 180° – 87° – 87° (/ sum of n) = 6° Third angle of the triangle = 180° – 15° – 15° (/ sum of n) = 150° Third angle of the triangle = 180° – 79° – 79° (/ sum of n) = 22° 39° + 90° + a° = 180° (/ sum of n) a° = 180° – 39° – 90° = 51° \ a = 51 68° + 2b° + 64° = 180° (/ sum of n) 2b° = 180° – 68° – 64° = 48° 48° 2 = 24° \ b = 24 (c) 4c° + 3c° + 40° = 180° (/ sum of n) 4c° + 3c° = 180° – 40° 7c° = 140° b° = A 70° 40° B /C = 180° – 70° – 40° (/ sum of n) = 70° It is an isosceles triangle and acute-angled triangle . (c) 140° 7 = 20° \ c = 20 (d) 3d° + 4d° + d° = 180° (/ sum of n) 8d° = 180° c= C 180° 8 = 22 .5° \ d = 22 .5 (e) Since BA = BC, \ BCA = BAC = 62° 62° + e° + 62° = 180° (/ sum of n) e° = 180° – 62° – 62° = 56° \ e = 56 d° = A 60° 60° B /C = 180° – 60° – 60° (/ sum of n) = 60° It is an equilateral triangle and acute-angled triangle . 167 1 (f) Since AC = BC = AB, \ CAB = CBA = ACB = f ° f ° + f ° + f ° = 180° (/ sum of n) 3f ° = 180° 7. (a) a° + 90° = 115° (ext . / of nBCE) a° = 115° – 90° = 25° \ a = 25 b° = 90° + 32° (ext . / of nEFG) = 122° \ b = 122 (b) ABE = ABD = 89° + 27° (ext . / of nBCD) = 116° c° = 116° + 22° (ext . / of nABE) = 138° \ c = 138 8. (a) 82° + 40° + a° = 180° (/ sum of n) a° = 180° – 82° – 40° = 58° \ a = 58 ADB = 82° (vert . opp . /s) b° = 45° + 82° (ext . / of nABD) = 127° \ b = 127 (b) EDF + 44° + 57° = 180° (/ sum of n) EDF = 180° – 44° – 57° = 79° ADB = 79° (vert . opp . /s) c° = 51° + 79° (ext . / of nABD) = 130° \ c = 130 9. (a) BAC + ACD = 180° (int . /s, AB // CD) 108° + (a° + 37°) = 180° a° = 180° – 108° – 37° = 35° \ a = 35 b° = 71° + 37° (ext . / of nABD) = 108° \ b = 108 (b) AHF = 45° (vert . opp . /s) AHI + CIH = 180° (int . /s, AB // CD) (45° + 64°) + (32° + c°) = 180° c° = 180° – 45° – 64° – 32° = 39° \ c = 39 d° + 39° + 64° = 180° (/ sum of n) d° = 180° – 39° – 64° = 77° \ d = 77 180° 3 = 60° \ f = 60 4. (a) a° = 47° + 55° (ext . / of n) = 102° \ a = 102 (b) 90° + b° + 50° = 180° (/ sum of n) b° = 180° – 90° – 50° = 40° \ b = 40 90° + c° + 35° = 180° (/ sum of n) c° = 180° – 90° – 35° = 55° \ c = 55 (c) d° + 110° = 180° (adj . /s on a str . line) d° = 180° – 110° = 70° \ d = 70 2e° + 3e° = 110° (ext . / of n) 5e° = 110° f° = 110° 5 = 22° \ e = 22 5. 3x° + 4x° + 5x° = 180° (/ sum of n) 12x° = 180° e° = 180° 12 = 15° \ x = 15 Smallest angle of the triangle = 3(15°) = 45° 6. (i) Let ADB = BDC = x° 90° + 20° + 2x° = 180° (/ sum of n) 2x° = 180° – 90° – 20° = 70° x° = 70° 2 = 35° \ BDC = 35° (ii) CBD + 20° + 35° = 180° (/ sum of n) CBD = 180° – 20° – 35° = 125° x° = 1 168 (c) Since EB = EC, \ ECB = EBC = 2e° f ° = 2e° + 2e° (ext . / of nBCE) = 4e° e° + f ° = 120° (ext . / of nBEF) e° + 4e° = 120° 5e° = 120° 12. (i) DCE + 61° + 41° = 180° (/ sum of n) DCE = 180° – 61° – 41° = 78° ACB = 78° (vert . opp . /s) (ii) ABC + 78° + 50° = 180° (/ sum of n) ABC = 180° – 78° – 50° = 52° 13. (i) DEC = BCE = 47° (alt . /s, AC // ED) 32° + 47° + DFE = 180° (/ sum of nDEF) DFE = 180° – 32° – 47° = 101° (ii) CBD = BDE = 32° (alt /s, AC // ED) 106° + EBD + 32° = 180° (adj . /s on a str . line) EBD = 180° – 106° – 32° = 42° BDC = EBD = 42° (alt . /s, BE // CD) 14. Let CBO be x° . 120° 5 = 24° \ e = 24 f ° = 4(24°) = 96° \ f = 96 ABE = DEB (alt /s, AB // CD) g° + 2(24°) = 96° g° = 96° – 48° = 48° \ g = 48 (d) AFE = CGF = 68° (corr . /s, AB // CD) 68° + h° = 180° (adj . /s on a str . line) h° = 180° – 68° = 112° \ h = 112 FJI = KJB = 65° (vert . opp . /s) i° = 65° (corr . /s, AB // CD) \ i = 65 IGH = CGF = 68° (vert . opp . /s) 68° + j° + 65° = 180° (/ sum of nGHI) j° = 180° – 68° – 65° = 47° \ j = 47 ° 1 10. (x – 35)° + (x – 25)° +  x – 10  = 180° (/ sum of n)  2 5 x° – 70° = 180° 2 5 x° = 180° + 70° 2 5 x° = 250° 2 250° x° =  5  2  = 100° \ x = 100 11. (i) ABC + 50° + 26° = 180° (/ sum of n) ABC = 180° – 50° – 26° = 104° (ii) CBD = 50° + 26° (ext . / of n) = 76° e° = 1 1 x° and BAO = 1 x° . 2 2 1 Since OA = OC, \ ACO = CAO = x° . 2 Since OB = OC, \ CBO = BCO = x° . Then CAO = Since OA = OB, \ BAO = ABO = 1 1 x° . 2 Hence, CAB + ABC + BCA = 180° (/ sum of nABC) 1    1 1  1  2 x ° + 1 2 x ° +  1 2 x ° + x ° +  2 x ° + x ° = 180° 6x° = 180 180° 6 = 30° x° = 1 (30°) = 15° . 2 15. Since AB = AC, then let ABC = ACB = x° . DBE = 180° – x° (adj . /s on a str . line) Since BD = BE, then \ CAO = 180° – (180° – x ° ) x° = . 2 2 Since AF = DF, \ FAD = FDA BDE = BED = FAD = FDA = BDE = x° . 2 x° + x + x° = 180° (/ sum of nABC) 2 1 2 x° = 180° 2 180° x= 1 2 2 = 72° \ ABC = 72° 169 1 4. (a) Since ABCD is a square, \ DAC = BAC = 45° and hence DAE = 45° . (Diagonals bisect the interior angles) AED + 82° = 180° (adj . /s on a str . line) AED = 180° – 82° = 98° 45° + 98° + a° = 180° (/ sum of nADE) a° = 180° – 45° – 98° = 37° \ a = 37 Since ABCD is a square, \ BAC = DAC = 45° and hence EAF = 45° . (Diagonals bisect the interior angles) AEF = 82° (vert . opp . /) b° = 45° + 82° (ext . / of nAEF) = 127° \ b = 127 (b) Since ABCD is a square, \ BCA = DCA = 45° and hence ECF = 45° . (Diagonals bisect the interior angles) c° + 45° + c° = 180° (/ sum of nCEF) 2c° = 180° – 45° = 135° Exercise 11B 1. (a) a° + 54° = 90° (BCD is a right angle) a° = 90° – 54° = 36° \ a = 36 b° = 36° (alt . /s, AB // DC) \ b = 36 (b) EBC = 90° (right angle) 90° + 39° + c° = 180° (/ sum of nBCE) c° = 180° – 90° – 39° = 51° \ c = 51 DCE + 39° = 90° (BCD is a right angle) DCE = 90° – 39° = 51° 51° + d° + 78° = 180° (/ sum of nCDE) d° = 180° – 51° – 78° = 51° \ d = 51 2. (a) a° = 106° (opp . /s of // gram) \ a = 106 b° = 48° (alt . /s, AD // BC) \ b = 48 (b) 4c° + 5c° = 180° (int . /s, AB // DC) 9c° = 180° 135° 2 = 67 .5° \ c = 67 .5 Since ABCD is a square, \ CED = 90° . (Diagonals bisect each other at right angles) Hence, d° + 67 .5° = 90° d° = 90° – 67 .5° = 22 .5° \ d = 22 .5 5. (a) Since ABCD is a rhombus, \ ACB = ADC = 114° (Opposite angles are equal) and hence a = 114 . Since ABCD is a rhombus, \ AB = CB and hence ACB = CAB = b° . b° + 114° + b° = 180° (/ sum of nABC) 2b° = 180° – 114° = 66° c° = 180° 9 = 20° \ c = 20 2d° = 4(20°) (opp . / s of // gram) c° = 80° 2 = 40° \ d = 40 3. (a) Since ABCD is a kite, \ AD = CD and so ACD = CAD = a° a° + 100° + a° = 180° (/ sum of nACD) 2a° = 180° – 100° = 80° d° = 80° 2 = 40° \ a = 40 Since ABCD is a kite, \ AB = CB and so CAB = ACB = 61° . 61° + b° + 61° = 180° (/sum of nABC) b° = 180° – 61° – 61° = 58° \ b = 58 (b) Since ABCD is a kite, \ DAC = BAC = 40° . (One diagonal bisects the interior angles) 40° + 26° + c° = 180° (/ sum of nACD) c° = 180° – 40° – 26° = 114° \ c = 114 a° = 1 66° 2 = 33° \ b = 33 (b) CBD = BDA = 38° (alt . /s, AD // BC) c° = 38° \ c = 38 Since ABCD is a rhombus, \ AB = AD and hence BDA = DBA = 38° . 38° + d° + 38° = 180° (/ sum of nABD) d° = 180° – 38° – 38° = 104° \ d = 104 b° = 170 9. ADB = (3x + 7)° (diagonals bisect interior angles of a rhombus) DAC = (2x + 53)° (diagonals bisect interior angles of a rhombus) 2(3x + 7)° + 2(2x + 53)° = 180° (int . /s, AB // DC) 6x° + 14° + 4x° + 106° = 180° 10x° = 180° – 14° – 106° 10x° = 60° 60° x° = 10 = 6° \x =6 10. 5x° + x° = 180° (int . /s, AB // DC) 6x° = 180° 180° x° = 6 = 30° \ x = 30 2 .2(30°) + y° = 180° (int . /s, AB // DC) y° = 180° – 66° = 114° \ y = 114 11. (i) Since ABCD is a kite, \ BAC = DAC = 25° (One diagonal bisects the interior angles) and since AB = AD, \ BDA = DBA = x° x° + 2(25°) + x° = 180° (/ sum of nABD) 2x° = 180° – 50° = 130° 130° x° = 2 = 65° \ ABD = 65° (ii) Since ABCD is a kite, \ BCA = DCA = 44° One diagonal bisects the interior angles) and since CB = CD, \ BDC = DBC = y° y° + 2(44°) + y° = 180° (/ sum of nBCD) 2y° = 180° – 88° = 92° 92° y° = 2 = 46° \ CBD = 46° 12. D C (c) DCA = CAB = 42° (alt, /s, AB // DC) e° = 42° \ e = 42 Since ABCD is a rhombus, \ ADB = CDB = f ° . (Diagonals bisect the interior angles) Also, AD = CD and hence CAD = ACD = 42° 42° + 2f ° + 42° = 180° (/ sum of nACD) 2f ° = 180° – 42° – 42° = 96° 96° 2 = 48° \ f = 48 6. (i) AED = 52° (vert . opp . /s) Since AE = DE, \ ADE = DAE = x° . x° + 52° + x° = 180° (/ sum of nADE) 2x° = 180° – 52° = 128° f° = 128° 2 = 64° \ ADB = ADE = 64° ADC = 90° (right angle of a rectangle) 64° + 90° + ACD = 180° (/ sum of nACD) ACD = 180° – 64° – 90° = 26° ADE + 65° = 180° (int . /s, AB // DC) ADE = 180° – 65° = 115° BCD = 65° (opp . /s of // gram) CBE + 65° = 125° (ext . / of nBCE) CBE = 125° – 65° = 60° ABD = 46° (alt . /s, AB // DC) Since ABCD is a rhombus, \ AB = AD and hence BDA = DBA = 46° . 46° + BAD + 46° = 180° (/ sum of nABD) BAD = 180° – 46° – 46° = 88° DBC = 46° (alt . /s, AD // BC) Since BC = BE, \ BCE = BEC = x° . x° + x° = 46° (ext . / of nBCE) 2x° = 46° x° = (ii) 7. (i) (ii) 8. (i) (ii) 118° A 46° x° = 2 = 23° \ BCE = 23° E B (i) Since E is the midpoint of AB, \ CE = DE and hence CDE = DCE = x° . x° + 118° + x° = 180° (/ sum of nCDE) 2x° = 180° – 118° = 62° 62° x = 2 = 31° ADE + 31° = 90° (ADC is a right angle) = 59° (ii) From (i), DCE = x° = 31° . 171 1 13. S Since AB = AD, \ ABD = ADB = x° x° + 118° + x° = 180° (/ sum of nABD) 2x° = 180° – 118° = 62° R 70° 62° 2 = 31° \ ABD = 31° (ii) ABC + 52° = 180° (int . /s, AB // DC) ABC = 180° – 52° = 128° From (i), ABD = 31° . 36° + CBD = 128° CBD = 128° – 31° = 97° x° = P 42° Q (i) PQR + 70° = 180° (int . /s, PQ // SR) PQR = 180° – 70° = 110° (ii) 42° + 110° + PRQ = 180° (/ sum of nPQR) PRQ = 180° – 42° – 110° = 28° 14. Z S 16. Y 64° P 108° W R 42° X (i) Since WXYZ is a rhombus, WZY = WXY = 108° (opp . /s of a // gram) and XZY = XZW = x° (Diagonal bisect the interior angles), hence WZY = 2x° 2x° = 108° Q (i) Since PS = RS, \ RPS = PRS = x° . x° + 64° + x° = 180° (/ sum of nPRS) 2x° = 180° – 64° = 116° 108° 2 = 54° \ XZY = 54° (ii) XYZ + 108° = 180° (int . /s, WX // ZY) XYZ = 180° – 108° = 72° (iii) Since WXYZ is a rhombus, XWZ = XYZ = 72° (opp . /s of a // gram) and XWY = ZWY = y° (Diagonals bisect the interior angles), hence XWZ = 2y° 2y° = 72° x° = 116° 2 = 58° \ PRS = 58° (ii) Since PQ = QR, \ QPR = QRP = 42° . 42° + PQR + 42° = 180° (/ sum of nPQR) PQR = 180° – 42° – 42° = 96° x° = Exercise 11C 72° 2 = 36° \ XWY = 36° y° = 15. D 1. (a) Sum of interior angles of a 11-gon = (n – 2) × 180° = (11 – 2) × 180° = 1620° (b) Sum of interior angles of a 12-gon = (n – 2) × 180° = (12 – 2) × 180° = 1800° (c) Sum of interior angles of a 15-gon = (n – 2) × 180° = (15 – 2) × 180° = 2340° C 52° 62° A B (i) BAD + 62° = 180° (int . /s, AB // DC) BAD = 180° – 62° = 118° 1 172 (b) (i) Sum of interior angles of a regular polygon with 18 sides = (n – 2) × 180° = (18 – 2) × 180° = 2880° (ii) Hence, size of each interior angle of a regular polygon with 18 sides (d) Sum of interior angles of a 20-gon = (n – 2) × 180° = (20 – 2) × 180° = 3240° 2. (a) Sum of interior angles of a quadrilateral = (n – 2) × 180° = (4 – 2) × 180° = 360° 78° + 62° + a° + 110° = 360° a° = 360° – 78° – 62° – 110° = 110° \ a = 110 (b) Sum of interior angles of a quadrilateral = (n – 2) × 180° = (4 – 2) × 180° = 360° b° + 78° + 2b° + 84° = 360° 3b° = 360° – 78° – 84° = 198° 2880° 18 = 160° 4. (a) The sum of exterior angles of the regular polygon is 360° . \ Size of each exterior angle of the regular polygon = 360° 24 = 15° \ Size of each interior angle of a regular polygon with 24 sides = 180° – 15° = 165° (b) The sum of exterior angles of the regular polygon is 360° . \ Size of each exterior angle of the regular polygon = 360° 36 = 10° \ Size of each interior angle of a regular polygon with 36 sides = 180° – 10° = 170° 5. (a) The sum of exterior angles of the regular polygon is 360° . \ Number of sides of the polygon 198° 3 = 66° \ b = 66 (c) Sum of interior angles of a pentagon = (n – 2) × 180° = (5 – 2) × 180° = 540° c° + 152° + 38° + 2c° + 101° = 540° 3c° = 540° – 152° – 38° – 101° 3c° = 249° = b° = 360° 90° =4 (b) The sum of exterior angles of the regular polygon is 360° . \ Number of sides of the polygon = 249° 3 = 83° \ c = 83 (d) Sum of interior angles of a hexagon = (n – 2) × 180° = (6 – 2) × 180° = 720° 102° + 5d° + 4d° + 4d° + 108° + 4d° = 720° 17d° = 720° – 102° – 108° = 510° c° = 360° 45° =8 (c) The sum of exterior angles of the regular polygon is 360° . \ Number of sides of the polygon = 360° 12° = 30 (d) The sum of exterior angles of the regular polygon is 360° . \ Number of sides of the polygon = 510° 17 = 30° \ d = 30 3. (a) (i) Sum of interior angles of a hexagon = (n – 2) × 180° = (6 – 2) × 180° = 720° (ii) Hence, size of each interior angle of a hexagon 360° 4° = 90 6. (a) Size of each interior angle of a regular polygon = 180° – 140° = 40° The sum of exterior angles of the regular polygon is 360° . \ Number of sides of the polygon d° = = 360° 40° =9 720° 6 = 120° = = 173 1 (b) Size of each interior angle of a regular polygon = 180° – 162° = 18° The sum of exterior angles of the regular polygon is 360° \ Number of sides of the polygon 9. The sum of exterior angles of an n-sided polygon is 360° . 15° + 25° + 70° + (n – 3) × 50° = 360° 15° + 25° + 70° + n(50°) – 150° = 360° n(50°) = 360° – 15° – 25° – 70° + 150° n(50°) = 400° 360° 18° = 20 (c) Size of each interior angle of a regular polygon = 180° – 172° = 8° The sum of exterior angles of the regular polygon is 360° . \ Number of sides of the polygon 400° 50° =8 10. The sum of exterior angles of a n-sided polygon is 360° . 3(50°) + (180° – 127°) + (180° – 135°) + (n – 5)(180° – 173°) = 360° 150° + 53° + 45° + (n – 5)(7°) = 360° 150° + 53° + 45° + n(7°) – 35° = 360° n(7°) = 360° – 150° – 53° – 45° + 35° = 147° n= = 360° 8° = 45 (d) Size of each interior angle of a regular polygon = 180° – 175° = 5° The sum of exterior angles of the regular polygon is 360° \ Number of sides of the polygon = 147° 7° = 21 n = 11. E D F 360° 5° = 72 7. Sum of interior angles of a pentagon = (n – 2) × 180° = (5 – 2) × 180° = 540° 2x° + 3x° + 4x° + 5x° + 6x° = 540° 20x° = 540° = C G H B A Size of each exterior angle of the heptagon 360° 7 = 51 .43° BHC + 51 .43° + 51 .43° = 180° (/ sum of nBCH) BHC = 180° – 51 .43° – 51 .43° = 77 .1° (to 1 d .p .) = 540° 20 = 27° Hence, the largest interior angle of the pentagon = 6(27°) = 162° 8. (i) The sum of exterior angles of the triangle is 360° . 3y° + 4y° + 5y° = 360° 12y° = 360° x° = D B A 360° 12 = 30° \ y = 30 (ii) Smallest interior angle of the triangle = 180° – 5(30°) = 180° – 150° = 30° (i) Sum of interior angles of a regular polygon with 20 sides = (n – 2) × 180° = (20 – 2) × 180° = 3240° Hence, size of each interior angles of a regular polygon with 20 sides y° = 1 C 12. 3240° 20 = 162° \ ABC = 162° = 174 (ii) Since size of each interior angle of a regular polygon with 20 sides = 162°, \ BCD = 162° Let CBD = CDB = x° (base /s of isos . nBCD) x° + x° + 162° = 180° (/ sum of nBCD) 2x° = 180° – 162° 2x° = 18° Hence, ACD = BCD – BCA = 108° – 36° = 72° (vi) Since size of each interior angle of a hexagon = 120°, \ BAS = 120° Since size of each interior angle of a pentagon = 108°, \ BAE = 108° 120° + 108° + SAE = 360° (/s at a point) SAE = 360° – 120° – 108° = 132° Let ASE = AES = x° (base / of isos . nAES) x° + x° + 132° = 180° (/ sum of nAES) 2x° = 180° – 132° 2x° = 48° 18° 2 = 9° \x =9 x° = Hence, ABD = ABC – CBD = 162° – 9° = 153° 13. (i) Sum of interior angles of a hexagon = (n – 2) × 180° = (6 – 2) × 180° = 720° \ Size of each interior angle of a hexagon 48° 2 = 24° \ ASE = 24° Let the interior angle be 5x° and the exterior angle be x° . 5x° + x° = 180° (adj . /s on a str . line) 6x° = 180° x° = 14. (i) 720° 6 = 120° Since ABP is an interior angle of a hexagon, \ ABP = 120° . (ii) Since PQR is an interior angle of a hexagon, \ PQR = 120° . = 180° 6 = 30° Since sum of exterior angles of a n-sided polygon is 360°, x° = 360° = 12 30° (ii) ABC = 5(30°) = 150° (int . / of a 12-sided polygon) Let BAC = BCA = x° (base /s of isos . nABC) x° + x° + 150° = 180° (/ sum of nABC) 2x° = 180° – 150° 2x° = 30° 30° x° = 2 = 15° Hence, ACD = BCD – BCA = 150° – 15° = 135° (iii) ABC = BCD = 150° (int . / of a 12-sided polygon) BAD = ADC = y° (base /s of isos . quadrilateral, BA = CD) y° + y° + 150° + 150° = 360° (/ sum of quadrilateral) 2y° = 360° – 150° – 150° 2y° = 60° \n= 120° PQX = (QA is a line of symmetry) 2 = 60° 360° (/s at a point) 6 = 60° (iv) Sum of interior angles of a pentagon = (n – 2) × 180° = (5 – 2) × 180° = 540° \ Size of each interior angle of a pentagon (iii) AXB = 540° 5 = 108° Since ABC is an interior angle of a pentagon, \ ABC = 108° . (v) Since size of each interior angle of a pentagon = 108°, \ BCD = 108° Let BAC = BCA = x° (base /s of isos . nABC) x° + x° + 108° = 180° (/ sum of nABC) 2x° = 180° – 108° 2x° = 72° = 60° 2 = 30° \ ADC = 30° CDE = 150° (int . / of a 12-sided polygon) Hence, ADE = CDE – ADC = 150° – 30° = 120° y° = 72° 2 = 36° \ x = 36 x° = 175 1 15. (i) Since sum of exterior angles of a n-sided polygon is 360°, 17. Sum of interior angles of a pentagon = 540° Let the exterior angle of the pentagon be x° . 5(180° – x°) = 540° 900° – 5x° = 540° –5x° = 540° – 900° –5x° = –360° 360° \n= = 10 36° (ii) Size of an interior angle of the n-sided polygon = 180° – 36° (adj . /s on a str . line) = 144° Let CBD = CDB = x° (base /s of isos . nBCD) x° + x° + 144° = 180° (/ sum of nBCD) 2x° = 180° – 144° 2x° = 36° 360° 5 = 72° /a + 72° + 72° = 180° /a = 180° – 72° – 72° = 36° Hence, /a + /b + /c + /d + /e = 5 × 36° = 180° 18. a1 + x1 = 180° (adj . /s on a str . line) a2 + x2 = 180° (adj . /s on a str . line) a3 + x3 = 180° (adj . /s on a str . line) a4 + x4 = 180° (adj . /s on a str . line) an + xn = 180° (adj . /s on a str . line) Hence, a1 + x1 + a2 + x2 + a3 + x3 + a4 + a4 + ··· + an + xn = n × 180° x° = 36° 2 = 18° \ CDB = 18° CDE = 144° (int . / of a 10-sided polygon) Hence, BDE = CDE – CDB = 144° – 18° = 126° (iii) Let XCD = XDC = 18° (base /s of isos . nCDX, CX = DX) 18° + 18° + CXD = 180° (/ sum of nCDX) CXD = 180° – 18° – 18° = 144° 16. ACE = /a + /b (ext . / of nABC) JCE + /a + /b = 180° (adj . /s on a str . line) JCE = 180° – /a – /b DEC = /c + /d (ext . / of nDEF) GEC + /c + /d = 180° (adj . /s on a str . line) GEC = 180° – /c – /d HGJ = /e + /f (ext . / of nGHI) EGJ + /e + /f = 180° (adj . /s on a str . line) EGJ = 180° – /e – /f CJG = /g + /h (ext . / of nJKL) CGJ + /g + /h = 180° (adj . /s on a str . line) CJG = 180° – /g – /h Sum of interior angles of quadrilateral = (4 – 2) × 180° = 360° \ JCE + GEC + EGJ + CJG = 360° (180° – /a – /b) + (180° – /c – /d) + (180° – /e – /f ) + (180° – /g – /h) = 360° –/a – /b – /c – /d – /e – /f – /g – /h = 360° – 180° – 180° – 180° – 180° –/a – /b – /c – /d – /e – /f – /g – /h = –360° Hence, /a + /b + /c + /d + /e + /f + /g + /h = 360° x° = 1 a1 + a2 + a3 + a4 + ··· + an + x1 + x2 + x3 + x4 + ··· + xn = n × 180° (n – 2) × 180° + x1 + x2 + x3 + x4 + ··· + xn = n × 180° x1 + x2 + x3 + x4 + ··· + xn = n × 180° – (n – 2) + 360° x1 + x2 + x3 + x4 + ··· + xn = 180°n – 180°n + 360° \ x1 + x2 + x3 + x4 + ··· + xn = 360° 19. (i) Two regular polygons are equilateral triangles and squares . (ii) The interior angles of the polygons meeting at a vertex must add to 360° . (iii) Shape Interior Angle in degrees Triangle 60 Square 90 Pentagon 108 Hexagon 120 More than six sides More than 120 degrees Since the interior angles of the polygon meeting at a vertex must add to 360°, hence the interior angle must be an exact divisor of 360° . This will work only for triangles, squares and hexagons as the interior angle are all divisor of 360° . (iv) The reason is that the hexagon has the smallest perimeter for a given area as compared to the square and the triangle . This will allow the bees to make more honey using less wax and less work . 176 Since DA = DC, \ DCA = DAC = 39° . 39° + ABC + 39° = 180° (/ sum of nACD) ADC = 180° – 39° – 39° = 102° 39° + d° = 102° d° = 102° – 39° = 63° \ d = 63 (c) e° + 62° + 52° = 180° (/ sum of nBCD) e° = 180° – 62° – 52° = 66° \ e = 66 48° + f ° + 66° = 180° (adj . /s on a str . line) f ° = 180° – 48° – 66° = 66° \ f = 66 (d) 110° + DBC = 180° (adj . /s on a str . line) DBC = 180° – 110° = 70° Since DB = DC, \ DCB = DBC = 70° . Hence, g° = 70° \ g = 70 70° + h° = 110° (ext . / of nBCD) h° = 110° – 70° = 40° \ h = 40 (e) Since DB = DC, \ DBC = DCB = 3i° . (5i + 4)° + 3i° = 180° (adj . /s on a str . line) 8i° = 180° – 4° = 176° Review Exercise 11 1. (a) Since AB = AC, \ ACB = ABC = 3a° . 3a° + 2a° + 3a° = 180° (/ sum of nABC) 8a° = 180° 180° 8 = 22 .5° \ a = 22 .5 (b) Since DA = DB, \ DBA = DAB = 32° . 32° + ADB + 32° = 180° (/ sum of nABD) ADB = 180° – 32° – 32° = 116° 116° + b° = 360° (/s at a point) b° = 360° – 116° = 244° \ b = 244 Since CA = CB, \ CAB = CBA = x° . x° + 64° + x° = 180° (/ sum of nABC) 2x° = 180° – 64° = 116° a° = 116° 2 = 58° c° + 32° = 58° c° = 58° – 32° = 26° \ c = 26 2. (a) Since BA = BD, \ BDA = BAD = a° . a° + 40° + a° = 180° (/ sum of nABD) 2a° = 180° – 40° = 140° x° = 176° 8 = 22° \ i = 22 3(22°) + 2j° = [5(22) + 4]° (ext . / of nBCD) 2j° = 114° – 66° = 48° i° = 140° 2 = 70° \ a = 70 CBD + 40° = 180° (adj . / s on a str . line) CBD = 180° – 40° = 140° Since BC = BD, \ BCD = BDC = b° . b° + 140° + b° = 180° (/ sum of nBCD) 2b° = 180° – 140° = 40° a° = 48° 2 = 24° \ j = 24 (f) k° + 78° = 3k° (ext . / of nABD) 3k° – k° = 78° 2k° = 78° j° = 40° 2 = 20° \ b = 20 (b) Since BA = BD, \ BDA = BAD = c° . c° + c° = 78° (ext . / of nABD) 2c° = 78° b° = 78° 2 = 39° \ k = 39 39° + l° + 78° = 180° (/ sum of nABD) l° = 180° – 39° – 78° = 63° \ l = 63 k° = 78° 2 = 39° \ c = 39 c° = 177 1 4. (a) 112° + ABC = 180° (adj . /s on a str . line) ABC = 180° – 112° = 68° 62° + HED = 180° (adj . /s on a str . line) HED =180° – 62° = 118° a° + BCD = 180° (adj . /s on a str . line) BCD = 180° – a° Sum of the interior angles of a pentagon = (5 – 2) × 180° = 540° . \ 114° + 68° + 180° – a° + 95° + 118° = 540° –a° = 540° – 114° – 68° – 180° – 95° – 118° = –35° \ a = 35 (b) Sum of exterior angles of a hexagon = 360° \ 2b° + 4b° + 3b° + b° + b° + b° = 360° 12b° = 360° 3. (a) Since AB = AC, ACB = ABC = a° . a° + BAC + a° = 180° (/ sum of nABC) BAC = 180° – 2a° DCA = 180° – 2a° (alt . /s, AB // DC) Since AC = AD = CD, DCA = CDA = CAD = 60° 180° – 2a° = 60° –2a° = 60° – 180° = –120° –120° –2 = 60° \ a = 60 (b) b° + b° + 76° = 180° (int . /s, AB // DC) 2b° = 180° – 76° = 104° a° = 104° 2 = 52° \ b = 52 c° + c° + 118° = 180° (int . /s, AB // DC) 2c° = 180° – 118° = 62° b° = 62° 2 = 31° \ c = 31 52° + 31° + d° = 180° (/ sum of nABE) d° = 180° – 52°– 31° = 97° \ d = 97 (c) Since EA = EB, EAB = EBA = 58° . 58° + e° = 180° (int . /s, AB // DC) e° = 180° – 58° = 122° \ e = 122 f ° = 58° (corr . /s, AB // DC) \ f = 58 Since ED = EC, EDC = ECD = 58° . 58° + g° + 58° = 180° (/ sum of nCDE) . g° = 180° – 58° – 58° = 64° \ g = 64 c° = 1 360° 12 = 30° \ b = 30 c° + 3(30°) = 180° (adj . /s on a str . line) c° = 180° – 90° = 90° \ c = 90 ACD = 40° (alt . /s, AB // DC) CAD + 108° + 40° = 180° (int . /s, AD// BC) CAD = 180° – 108° – 40° = 32° Since AB = AD, \ ADB = ABD = 62° . 62° + BAD + 62° = 180° (/ sum of nABD) BAD = 180° – 62° – 62° = 56° Since CB = CD, \ BDC = DBC = x° . x° + 118° + x° = 180° (/ sum of nBCD) 2x° = 180° – 118° = 62° b° = 5. (i) (ii) 6. (i) (ii) 62° 2 = 31° \ BDC = 31° x° = 178 7. Since nABE is an equilateral triangle, AB = AE = BE and EAB = EBA = AEB = 60° . DAE + 60° = 90° (right angle of a square) DAE = 90° – 60° = 30° Since AD = AB, \ AE = AD and AED = ADE = x° . x° + 30° + x° = 180° (/ sum of nADE) 2x° = 180° – 30° = 150° 10. Sum of interior angles of a pentagon = (5 – 2) × 180° = 3 × 180° = 540° Let the 5 interior angles be 3x°, 4x°, 5x°, 5x° and 7x° . 3x° + 4x° + 5x° + 5x° + 7x° = 540° 24x° = 540° 540° 24 = 22 .5° (i) Largest interior angle = 7 × 22 .5° = 157 .5° (ii) Largest exterior angle = 180° – 3 × 22 .5° = 112 .5° 11. Sum of exterior angles of a n-sided polygon = 360° 35° + 72° + (n – 2) × 23° = 360° 23°n = 360° – 35° – 72° + 46° = 299° x° = 150° 2 = 75° CBE + 60° = 90° (right angle of a square) CBE = 90° – 60° = 30° Since BC = AB, \ BE = BC and BEC = BCE = y° . y° + 30° + y° = 180° (/ sum of nBEC) 2y° = 180° – 30° = 150° x° = 299° 23° = 13 12. Let the interior angle be 13x° and the exterior angle be 2x° . 13x° + 2x° = 180° (adj . /s on a str . line) 15x° = 180° n= 150° 2 = 75° 75° + 60° + 75° + CED = 360° (/s at a point) CED = 360° – 75° – 60° – 75° = 150° 8. Sum of interior angles of a (2n – 3)-sided polygon = [(2n – 3) – 2] × 180° Hence, [(2n – 3) – 2] × 180° = 62 × 90° (2n – 5) × 180° = 5580° 360°n – 900° = 5580° 360°n = 5580° + 900° 360°n = 6480° y° = 180° 15 = 12° Sum of exterior angles of a n-sided polygon = 360° Hence, 360° n= 2(12° ) x° = = 15 13. Sum of the interior angles of a n-sided polygon = (n – 2) × 180° Sum of the exterior angles of a n-sided polygon = 360° (n – 2) × 180° = 4 × 360° 180°n = 1440° + 360° = 1800° 6480° 360° = 18 9. Sum of interior angles of a n-sided polygon = (n – 2) × 180° 126° + (n – 1) × 162° = (n – 2) × 180° 126° + 162°n – 162° = 180°n – 360° 180°n – 162°n = 360° + 126° –162° 18°n = 324° n = 1800° 180° = 10 n= 324° 18° = 18 n = 179 1 180° – 20° (base /s of isos . nABC) 2 160° = 2 = 80° \ DCF = DCA = ACB – 60° = 80° – 60° = 20° BFC + FCB + 50° = 180° (/ sum of nBCF) BFC + ACB + 50° = 180° BFC + 80° + 50° = 180° BFC = 180° – 80° – 50° = 50° Since CBF = BFC = 50°, i .e . CB = CF, then nBCF is an isosceles triangle . Challenge Yourself 1. 2. ACB = 180° – 20° (base /s of isos . nABC) 2 160° = 2 = 80° ABC = A 20° D G F A B C E 20° Draw E on BC such that AE ^ BC . Draw F on AE such that nBCF is an equilateral triangle . Then ABF = 80° – 60° = 20° and BF = BC = AD . Consider the quadrilateral ABFD . A F E 20° B D 50° 60° C Draw G on AG such that DG // BC . Draw BG to cut CD at E . Draw EF . By symmetry, BE = CE, so nBCE is an isosceles triangle . Since the base angle of nBCE is 60°, then nBCE is an equilateral triangle, i .e . BEC = 60° and EBF = 60° – 50° = 10° . \ CE = CB (sides of equilateral nBCE) = CF (sides of isosceles nBCF) Since CE = CF, then nCEF is an isosceles triangle . ˆ 180° – ECF CFE = (base /s of isos . nCEF) 2 ˆ 180° – DCF = 2 180° – 20° = 2 160° = 2 = 80° \ BFE = CFE – BFC = 80° – 50° = 30° G F 20° B Since ABF = BAD = BAC = 20° and BF = AD, then by symmetry, AB // DF and ABFD is an isosceles trapezium . In the isosceles trapezium ABFD, by symmetry, AG = BG, so nABG is an isosceles triangle . 20° = 10° (AE bisects BAC), 2 then ABG = BAG = 10° (base /s of isos . nABG) . \ ADB + ABD + BAD = 180° (/ sum of nABD) ADB + ABG + 20° = 180° ADB + 10° + 20° = 180° ADB = 180° – 10° – 20° = 150° Teachers may wish to note the usefulness of the symmetric properties of an isosceles trapezium. Otherwise, formal proofs using congruent triangles are beyond the scope of Secondary 1 syllabus. Since BAG = BAE = 1 G D 180 4. (i) An exterior angle of a concave polygon has a negative measure and is inside the polygon as shown in the diagram below . E .g . FEG = EBF + BFE (ext . / of nBEF) = 10° + 30° = 40° DEG = BEC (vert . opp . /s) = 60° DGE = CBE (alt . /s, DG // BC) = 60° Since the base angle of nDEG is 60°, then nDEG is an equilateral triangle, i .e . EDG = 60° and DE = DG . AGD = ACB (corr . /s, DG // BC) = 80° \ FGE + DGE + AGD = 180° (adj . /s on a str . line) FGE + 60° + 80° = 180° FGE = 180° – 60° – 80° = 40° Since FEG = FGE = 40°, then nEFG is an isosceles triangle, i .e . FE = FG . Consider the quadrilateral DEFG . e4° i5° i4° e5° i1° e3° e1° i2° e2° (ii) Yes . Exterior angle of the vertex which is “pushed in” will flip over into the inside of the polygon and becomes negative. Adding all the exterior angles as before, they will still add to 360° . E .g . i1° + (–e1°) + i2° + e2° + i3° + e3° + i4° + e4° + i5° + e5° + = 5 × 180° [i1° + i2° + i3° + i4° + i5°] + (–e1°) + e2° + e3° + e4° + e5° = 900° (–e1°) + e2° + e3° + e4° + e5° = 900 – [i1° + i2° + i3° + i4° + i5°] (–e1°) + e2° + e3° + e4° + e5° = 900° – (5 – 2) × 180° (–e1°) + e2° + e3° + e4° + e5° = 900° – 540° (–e1°) + e2° + e3° + e4° + e5° = 360° The above proof holds for any n-sided polygon . G D F E 5. In a n-sided polygon, each diagonal connects one vertex to another vertex which is not its next-door neighbour . Since there are n vertices in an n-sided polygon, therefore there are n starting points for the diagonals . For each diagonal, it (e .g . V1) can join to other (n – 3) vertices since it cannot join itself (V1) or either of the two neighbouring vertices (V2 and Vn) . So the total number of diagonals formed is n × (n – 3) . However, in this way, each diagonal would be formed twice (to and from each vertex), so the product n(n – 3) must be divided by 2 . Hence the formula is n ( n – 3) . 2 E .g . Since DE = DG and FE = FG, then DEFG is a kite . In the kite DEFG, the longer diagonal DF bisects EDG . \ CDF = EDF 60° 2 = 30° 3. Yes . For any n-sided concave polygon, it can still form (n – 2) triangles in the polygon . Hence the sum of the interior angles is still the same . E .g . = V1 Vn V2 Vn – 1 V3 V4 181 1 Chapter 12 Geometrical Constructions TEACHING NOTES Suggested Approach Students have learnt how to draw triangles and quadrilaterals using rulers, protractors and set squares in primary school. Teachers need to reintroduce these construction tools and demonstrate the use of these if students are still unfamiliar with them. When students are comfortable with the use of these construction tools and the compasses, teachers can proceed to the sections on construction of triangles and quadrilaterals. Section 12.1: Introduction to Geometrical Constructions Teachers may wish to recap with students how rulers, protractors and set squares are used. More emphasis should be placed on the use of protractors, such as the type of scale (inner or outer) to use, depending on the type of angle (acute or obtuse). Teachers need to impress upon students to avoid parallax errors when reading the length using a ruler, or an angle using a protractor. Teachers should show and lead students on the use of compasses. Students are to know and be familiar with the useful tips in using the construction tools. Section 12.2: Perpendicular Bisectors and Angle Bisectors Teachers should state and define perpendicular bisectors and angle bisectors. Stating what perpendicular and bisect means individually will help students to remember their meanings. For the worked examples in this section, teachers are encouraged to go through the construction steps one by one with the students. Students should follow and construct the same figures as shown in the worked examples. Teachers should allow students to use suitable geometry software to explore and discover the properties of perpendicular bisectors and angle bisectors (see Investigation: Property of a Perpendicular Bisector and Investigation: Property of an Angle Bisector), that is, their equidistance from end-points and sides of angles respectively. Section 12.3: Construction of Triangles Students should be able to construct the following types of triangles at the end of this section: • Given 2 sides and an included angle • Given 3 sides • Given 1 side and 2 angles As a rule of thumb, students should draw the longest line as a horizontal line. Teachers are to remind their students to mark all angles, vertices, lengths and other markings (same angles, same sides, right angles etc.) clearly. Students should not erase any arcs that they draw in the midst of construction and check their figure at the end. Section 12.4: Construction of Quadrilaterals Students should be able to construct parallelograms, rhombuses, trapeziums and other quadrilaterals at the end of this section. As a rule of thumb, students should draw the longest line as a horizontal line. Teachers are to remind their students to mark all angles, vertices, lengths and other markings (same angles, same sides, right angles etc.) clearly. Students should not erase any arcs they draw in the midst of construction and check their figure at the end. 1 182 WORKED SOLUTIONS Investigation (Property of a Perpendicular Bisector) 4. The length of AC is equal to the length of BC . 5. Any point on the perpendicular bisector of AB is equidistant from A and B . 6. Any point which is not on the perpendicular bisector of AB is not equidistant from A and B . Investigation (Property of an Angle Bisector) 5. The length of PR is equal to the length of QR . 6. Any point on the angle bisector of BAC is equidistant from AB and AC . 7. Any point which is not on the angle bisector of BAC is not equidistant from AB and AC . Practise Now 1 A B 8 cm Practise Now 2 C 78° A B Practise Now 3 C 4 .8 cm S 130° A 7 .6 cm B (i) Length of AC = 11 .3 cm (ii) Length of BS = 4 .0 cm 183 1 Practise Now 4 R T 9 .8 cm 7 .2 cm P Q 8 .4 cm (i) Required angle, QPR = 77° (ii) Length of QT = 5 .3 cm Practise Now 5 Z U X 56° 48° 8 cm Y (iii) The point U is equidistant from the points Y and Z, and equidistant from the lines XY and XZ . 1 184 Practise Now 6 D 1. C 5 .5 cm 120° A 8 .5 cm B Length of AC = 12 .2 cm 2. D C 6 .5 cm A B 10 .5 cm Length of AC = 12 .3 cm 185 1 Practicse Now 7 S 1. 6 cm 9 cm 9 cm P R 4 .5 cm 6 cm Q QRS = 71° 2. S P R 12 cm 7 .5 cm Q QRS = 74° 1 186 Practise Now 8 R 9 .2 cm 95° S 6 .2 cm 80° P Q 5 .6 cm (i) Length of PS = 7 .0 cm (ii) PSR = 54° Exercise 12A 1. A B 9 .5 cm 2. C A 56° B 187 1 C 3. P 5. 6 .5 cm 10 cm 10 cm 80° A B 8 cm Length of AC = 9 .4 cm 4. C Q 9 cm QPR = 53° 6. 9 cm A 5 cm B Length of AC = 7 .5 cm 9 .5 cm 1 188 R 7. Z 60° 45° X Y 10 .2 cm Length of XZ = 9 .1 cm C 8. S A 6 .5 cm 88° 9 .8 cm B (i) Length of AC = 11 .6 cm (ii) Length of BS = 5 .9 cm 189 1 C 9. 8 .8 cm S 60° A B 9 .4 cm (i) Required angle, BAC = 52° (ii) Length of CS = 3 .9 cm 10. P T 9 .5 cm Q 8 .5 cm R 9 .8 cm (i) Required angle, PQR = 52° (ii) Length of QT = 8 .0 cm 1 190 11. P 9 .2 cm T 8 .8 cm Q R 10 .4 cm (i) Required angle, QPR = 71° (ii) PT = 4 .2 cm 12. Z 13. Z U 55° 64° X U 8 cm Y 74° X (i) Length of XZ = 7 .5 cm (ii) Length of UY = 7 .2 cm 49° 8 cm Y (iii) The point U is equidistant from the points X and Y, and equidistant from the lines XY and YZ . 191 1 14. 5 cm C 11 cm S 62° T A B 10 .2 cm (i) Length of BC = 10 .9 cm (iii) Length of ST = 4 .7 cm C 15. 4 .6 cm 54° A S 1 8 .5 cm B 192 R 16. 7 .9 cm 9 .2 cm T P 8 .3 cm 17. Q P O Q (ii) Diameter 193 1 Exercise 12B A 1. D 10 cm 80° B C 12 cm Length of diagonal BD = 16 .9 cm 2. 96 mm = 9 .6 cm 84 mm = 8 .4 cm 84 mm 96 mm Length of each of the two diagonals = 12 .8 cm 1 194 3. D C 115° A B 6 cm Length of each of the two diagonals = 10 .1 cm, 6 .5 cm 4. 9 cm S P 9 cm Q 7 .5 cm R 12 cm QPS = 133° 5. 60 mm = 6 cm 9 mm = 0 .9 cm R S 9 mm P 60 mm Q QPS = 171° 195 1 6. Q 75° 5 .3 cm 6 .3 cm P 60° R S 6 .7 cm (i) Length of PR = 7 .1 cm (ii) RPS = 70° 7. 6 cm W Z 4 .5 cm 60° X Y 8 cm Length of YZ = 3 .9 cm Length of WY = 6 .9 cm 8. 56 mm = 5 .6 cm 112 mm = 11 .2 cm Z W 56 mm 80° X 70° Y 112 mm Length of WY = 11 .6 cm Length of XZ = 10 .7 cm 1 196 9. T D C 6 cm (i) Length of diagonal BD = 8 .4 cm (ii) Length of AT = 7 .1 cm 115° A B 9 cm R 10. 4 .8 cm S U 4 .8 cm P 4 cm (i) Length of QS = 5 .4 cm (ii) Length of SU = 4 .5 cm Q 11. S P R 10 cm (iii) Length of PQ = 7 .0 cm Q 197 1 R 12. S 6 cm 3 .5 cm P 60° 45° 10 cm T Q (i) QRS = 119° (ii) Length of PT = 5 .4 cm 13. U S R 7 cm 3 .1 cm 50° P Q 11 cm (i) QRS = 109° (ii) Length of RU = 4 .1 cm 1 198 14. 5 cm X S W 120° U 6 cm Y 9 .8 cm Z T (i) Length of WY = 8 .6 cm (ii) Length of ST = 6 .5 cm (iii) WUX = 105° 15. S T R 5 .8 cm 4 .6 cm 120° 105° P 6 .5 cm Q Review Exercise 12 C 1. 6 cm S 60° A 4 .5 cm B (i) Length of AC = 5 .4 cm (ii) Length of CS = 3 .3 cm 199 1 2. R 8 .8 cm 10 .2 cm P T Q 12 cm 3. X 60° Y 60° U Z 8 cm (iii) Rhombus V 1 200 (i) Required angle, QPR = 46° (ii) RT = 7 .9 cm 4. T D C 5 .5 cm 120° A S 8 cm B (i) Length of BD = 7 .1 cm (ii) Length of ST = 6 .5 cm 5. U S 6 cm R 2 cm 60° P 8 cm Q (i) Required angle, QRS = 123° (ii) Length of QU = 6 .5 cm 201 1 6. B S A C 10 cm 45° D (iii) AB = BC = AD = CD = 7 .1 cm ABCD is a square . (iv) Length of DS = 9 .3 cm Challenge Yourself 1. 2. R 5 cm P 6 cm 6 cm 7 cm P 8 cm Q Q Incircle Circumcircle 1 202 8 cm R (ii) ACD = 56° (alt. /s, AB // DC) ACB = 70° – ACD = 70° – 56° = 14° 7. (i) (2x + 17)° + (3x – 25)° + (2x + 49)° + (x + 40)° + (4x – 17)° + (3x – 4)° = (6 – 2) × 180° 2x° + 17° + 3x° – 25° + 2x° + 49° + x° + 40° + 4x° – 17° + 3x° – 4° = 4 × 180° 2x° + 3x° + 2x° + x° + 4x° + 3x° = 720° – 17° + 25° – 49° – 40° + 17° + 4° 15x° = 660° x° = 44° \ x = 44 (ii) Smallest interior angle = (x + 40)° = (44 + 40)° = 84° (iii) Smallest exterior angle = 180° – largest interior angle (adj. /s on a str. line) = 180° – (4x – 17)° = 180° – [4(44) – 17]° = 180° – (176 – 17)° = 180° – 159° = 21° C 8. (i) Revision Exercise C1 1. 35% of students = 140 140 1% of students = 35 140 100% of students = × 100 35 = 400 \ The total number of students who take part in the competition is 400 . 2. Value first obtained = (100 – 15)% of 5600 = 85% × 5600 85 = × 5600 100 = 4760 1100 110% of 4760 = × 4760 100 = 5236 \ The final number is 5236. 3. Height of hall 6 = 7 28 6 Height of hall = × 28 7 = 24 m Ratio of breadth of hall to height of hall = 21 : 24 = 7:8 4. 1035 hours 53 minutes 1128 hours 53 × 575 25 = 1219 Number of words in the report = 6 hours 25 minutes X 6.5 cm 5. (i) 0845 hours 1510 hours The train takes 6 hours 25 minutes to travel from Town A to Town B . 7 cm (ii) Distance between Town A and Town B = 108 × 6 25 60 5 = 108 × 6 12 = 693 km 6. D A 6 cm B (ii) Length of BX = 5.6 cm C 70° A 56° B (i) ABC + 70° = 180° (int. /s, AB // DC) ABC = 180° – 70° = 110° 203 1 195 20 3 60 195 = 1 3 3 1 = 58 km/h 2 Average speed of lorry on the return journey = Revision Exercise C2 3780 – 3500 × 100% 3500 280 = × 100% 3500 = 8% 1. Percentage increase in salary = 6. 26° + x° = 180° (adj. /s on a str. line) x° = 180° – 26° = 154° \ x = 154 BAD = 62° (alt. /s, AB // CD) BAQ = BAD – 26° = 62° – 26° = 36° RSA = SAQ (alt. /s, PQ // RS) = 36° RSA + y° + y° = 180° (adj. /s on a str. line) 36° + y° + y° = 180° y° + y° = 180° – 36° 2y° = 144° y° = 72° \ y = 72 7. 95° + (n – 1) × 169° = (n – 2) × 180° 95 + 169n – 169 = 180n – 360 169n – 180n = –360 – 95 + 169 –11n = –286 \ n = 26 2. Percentage of students who did not have to stay back = 100% – 25% = 75% 75 75% of 40 = × 40 100 = 30 \ 30 students did not have to stay back for detention. 3. X : Y = 8 : 15 = 56 : 105 Y : Z = 21 : 32 = 105 : 160 \ X : Z = 56 : 160 = 7 : 20 10 × 31 4 1 = 77 2 1230 hours 4. Number of times light can circle the world = 5. 0845 hours 3 hours 45 minutes 45 60 3 = 52 × 3 4 = 195 km Distance between Town A and Town B = 52 × 3 1455 hours 3 hours 20 minutes 1815 hours 8. Length of BC = 7 .7 cm Length of CD = 11 .7 cm C D 82° 5 .3 cm 105° 110° A 1 7.6 cm 204 B Chapter 13 Perimeter and Area of Plane Figures TEACHING NOTES Suggested Approach In the previous chapter, students have learnt the construction of plane figures such as triangles and quadrilaterals. Here, they will learn how to convert units of area, as well as find the perimeter and area of triangles and quadrilaterals. Students will revise what they have learnt in primary school as well as learn the perimeter and area of parallelograms and trapeziums. Teachers should place more focus on the second half of the chapter and ensure students are able to solve problems involving the perimeter and area of parallelograms and trapeziums. Section 13.1: Conversion of Units Teachers may wish to recap with the students the conversion of unit lengths from one unit of measurement to another (i.e. mm, cm, m and km) before moving onto the conversion of units for areas. Teachers may ask students to remember simple calculations such as 1 cm2 = 1 cm × 1 cm to help them in their calculations when they solve problems involving the conversion of units. Section 13.2: Perimeter and Area of Basic Plane Figures This section is a recap of what students have learnt in primary school. Students are reminded to be clear of the difference in the units used for perimeter and area (e.g. cm and cm2). Teachers can impress upon the students that the value of p in calculators is used when its value is not stated in the question. Unless specified, all answers that are not exact should be rounded off to 3 significant figures. Section 13.3: Perimeter and Area of Parallelograms Teachers should illustrate the dimensions of a parallelogram to the students so that they are able to identify the base and height of parallelograms. It is important to emphasise to the students that the height of a parallelogram is with reference to the base and it must be perpendicular to the base chosen. Also, the height may lie within, or outside of the parallelogram. Teachers can highlight to the students that identifying the height of a parallelogram is similar to identifying the height of a triangle. Teachers should guide students in finding the formula for the area of a parallelogram (see Investigation: Formula for Area of a Parallelogram). Both possible methods should be shown to students (The second method involves drawing the diagonal of the parallelogram and finding the area of the two triangles). Section 13.4: Perimeter and Area of Trapeziums Teachers should recap with students the properties of a trapezium. Unlike the parallelogram, the base of the trapezium is not required and the height must be with reference to the two parallel sides of the trapezium. Thus, the height lies either inside the trapezium, or it is one of its sides (this occurs in a right trapezium, where two adjacent angles are right angles). Teachers should guide students in finding the formula for the area of a trapezium (see Investigation: Formula for Area of a Trapezium). Both possible methods should be shown to students (Again, the second method involves drawing the diagonal of the trapezium and finding the area of the two triangles). Teachers can enhance the students’ understanding and appreciation of the areas of parallelograms and trapeziums by showing them the link between the area of a trapezium, a parallelogram and a triangle (see Thinking Time on page 329). 205 1 Area of parallelogram ABCD = area of nABC + area of nADC WORKED SOLUTIONS 1 1 × AB × CF + × DC × DE 2 2 1 1 = bh + bh 2 2 = bh = Class Discussion (International System of Units) 1. The seven basic physical quantities and their base units are shown in the following table: Basic Physical Quantity Base Unit Length metre (m) Mass kilogram (kg) Time second (s) Electric current ampere (A) Thermodynamic Temperature kelvin (K) Amount of substance mole (mol) Luminous intensity candela (cd) Thinking Time (Page 325) From the geometry software template ‘Area of Parallelogram’, we can conclude that the formula for the area of parallelogram is also applicable to oblique parallelograms . Investigation (Formula for Area of a Trapezium) 1. The new quadrilateral AFGD is a parallelogram . 2. Length of AF = length of AB + length of EF =b+a =a+b Scientists developed the International System of Units (SI units) so that there is a common system of measures which can be used worldwide . 2. Measurements of Lengths: 1 foot (ft) = 0 .3048 m 1 inch (in) = 0 .0254 m 1 yard (yd) = 0 .9144 m 1 mile = 1609 .344 m Measurement of Areas: 1 acre = 4046 .8564 m2 1 × area of parallelogram AFGD 2 1 = × AF × h 2 1 = (a + b)h 2 3. Area of trapezium ABCD = 4. Method 1: Divide the trapezium ABCD into two triangles ABD and DCB by drawing the diagonal BD as shown below: a C D F Investigation (Formula for Area of a Parallelogram) 1. The new quadrilateral CDEF is a rectangle . 2. Length of CF = length of DE = h Length of EF = length of EB + length of BF = length of EB + length of AE =b 3. Area of parallelogram ABCD = area of rectangle CDEF = EF × CF = bh 4. Divide the parallelogram ABCD into two triangles ABC and ADC by drawing the diagonal AC as shown below: D h A E B b Length of FB = length of DE = h Area of trapezium ABCD = area of nABD + area of nDCB C 1 2 1 = 2 1 = 2 1 = 2 = h A E b B F Length of CF = length of DE = h 1 206 × AB × DE + ×b×h+ (b + a)h (a + b)h 1 × DC × FB 2 1 ×a×h 2 Method 2: Divide the trapezium ABCD into a parallelogram AFCD and a triangle FBC by drawing a line FC // AD as shown below: a Practise Now 1 (a) 16 m2 = 16 × 10 000 cm2 = 160 000 cm2 (b) 357 cm2 = 357 × 0 .0001 m2 = 0 .0357 m2 C D Practise Now (Page 318) (a) h h b A E F b G B (b) Length of CG = length of DE = h Length of AF = length of DC = a \ Length of FB = length of AB – length of AF =b–a Area of trapezium ABCD = area of parallelogram AFCD + area of nFBC = AF × DE + =a×h+ h b 1 × FB × CG 2 1 × (b – a) × h 2 (c) 1 (2a + b – a)h 2 1 = (a + b)h 2 = h b Teachers may wish to get higher-ability students to come up with more methods to find a formula for the area of a trapezium. (d) b Thinking Time (Page 329) 1. (i) The new figure is a parallelogram. h 1 (ii) Area of trapezium = (a + b)h 2 When a = b, 1 1 (a + b)h = (b + b)h 2 2 1 = (2b)h 2 = bh = area of parallelogram 2. (i) The new figure is a triangle. (ii) Area of trapezium = (e) b h 1 (a + b)h 2 When a = 0, 1 1 (a + b)h = (0 + b)h 2 2 1 = bh 2 = area of triangle 207 1 (f) Practise Now (Page 324) b (a) h b h (b) Practise Now 2 b h 1. Length of each side of square field = 64 4 = 16 m Area of field = 162 = 256 m2 Area of path = (16 + 3 .5 + 3 .5)2 – 256 = 232 – 256 = 529 – 256 = 273 m2 2. Area of shaded region = area of rectangle ABCD – area of nARQ – area of nBRS – area of nCPS – area of nDPQ (c) h b (d) b h 1 1 = 25 × 17 – × (25 – 14) × 5 – × 14 × 3 2 2 1 1 – × (25 – 8) × (17 – 3) – × (17 – 5) × 8 2 2 1 1 1 = 425 – × 11 × 5 – 21 – × 17 × 14 – × 12 × 8 2 2 2 1 = 425 – 27 – 21 – 119 – 48 2 1 2 = 209 m 2 (e) h b Practise Now 3 (f) 3 × 2p(14) + 2(14) 4 = 21p + 28 = 94 .0 cm (to 3 s .f .) (i) Perimeter of unshaded region = h 3 × p(14)2 4 = 147p = 462 cm2 (to 3 s .f .) (iii) Area of shaded region = area of square – area of unshaded region = (2 × 14)2 – 147p = 282 – 147p = 784 – 147p = 322 cm2 (to 3 s .f .) b (ii) Area of unshaded region = 1 Practise Now 4 (i) Area of parallelogram = 24 × 7 = 168 m2 (ii) Perimeter of parallelogram = 2(30 + 7) = 2(37) = 74 m 208 Practise Now 7 Practise Now 5 Area of parallelogram = PQ × ST = 480 m2 20 × ST = 480 ST = 24 Length of ST = 24 m 1 × (5 + 13 .2) × 4 2 1 = × 18 .2 × 4 2 = 36 .4 m2 (ii) Perimeter of trapezium = 5 + 6 + 13 .2 + 5 .5 = 29 .7 m (i) Area of trapezium = Practise Now 6 1. Total area of shaded regions = area of parallelogram ABJK + area of parallelogram CDIJ + area of parallelogram DEGH = 4 × 12 + (2 × 4) × 12 + 4 × 12 = 48 + 8 × 12 + 48 = 48 + 96 + 48 = 192 m2 2. Area of nCDF = Practise Now 8 (i) Area of trapezium = 1 × DC × CF = 60 cm2 2 1 × DC × 3CG = 60 2 3 × DC × CG = 60 2 DC × CG = 40 Area of parallelogram ABCD = DC × CG = 40 cm2 1 × (PQ + RS) × PS 2 1 × (14 + 10) × PS 2 1 × 24 × PS 2 12 × PS PS = 72 m2 = 72 = 72 = 72 =6 Length of PS = 6 m (ii) Perimeter of trapezium = PQ + QR + RS + PS 14 + QR + 10 + 6 30 + QR QR Length of QR = 7 .2 m = 37 .2 m = 37 .2 = 37 .2 = 7 .2 Practise Now (Page 328) Practise Now 9 (a) Area of figure = area of trapezium + area of semicircle 1 1 1  × (48 + 16) × 20 + p  1424  2 2 2  1 1 = × 64 × 20 + p × 356 2 2 = 640 + 178p = 1200 m2 (to 3 s .f .) 2 = h (b) Exercise 13A h (c) 1. (a) 40 m2 = 40 × 10 000 cm2 = 400 000 cm2 2 (b) 16 cm = 16 × 0 .0001 m2 = 0 .0016 m2 (c) 0 .03 m2 = 0 .03 × 10 000 cm2 = 300 cm2 (d) 28 000 cm2 = 28 000 × 0 .0001 m2 = 2 .8 m2 h 259 18.5 = 14 cm (ii) Perimeter of rectangle = 2(18 .5 + 14) = 2(32 .5) = 65 cm 2. (i) Breadth of rectangle = 209 1 3. Area of figure = area of square – area of triangle 5. Let the diameter of the semicircle be x cm . 1 = 92 – × 3 × 2 .5 2 = 81 – 3 .75 = 77 .25 m2 4. (a) Diameter of circle = 2 × 10 = 20 cm Circumference of circle = 2p(10) = 20p = 62 .8 cm (to 3 s .f .) Area of circle = p(10)2 = 100p = 314 cm2 (to 3 s .f .) 1 × p × x + x = 144 2 1 22 × × x + x = 144 2 7 11 x + x = 144 7 18 x = 144 7 x = 56 \ Diameter of semicircle = 56 cm = 0 .56 m  21  6. (a) (i) Perimeter of figure = 2p   + 2(36 – 21)  2 = 2p(10 .5) + 2(15) = 21p + 30 = 96 .0 cm (to 3 s .f .) (ii) Area of figure = area of two semicircles + area of rectangle = p(10 .5)2 + 15 × 21 = 110 .25p + 315 = 661 cm2 (to 3 s .f .) 3.6 2 = 1 .8 m Circumference of circle = 2p(1 .8) = 3 .6p = 11 .3 m (to 3 s .f .) Area of circle = p(1 .8)2 = 3 .24p = 10 .2 m2 (to 3 s .f .) (b) Radius of circle = (b) (i) Perimeter of figure = 1 × 2p(5) + 2(5) + 200 2 = 5p + 10 + 200 = 39 .9 cm (to 3 s .f .) (ii) Area of figure = area of semicircle + area of triangle 176 2p 88 = p = 28 .0 mm (to 3 s .f .) (c) Radius of circle = 1 1 × p(5)2 + × 10 × 10 2 2 25 = p + 50 2 = 89 .3 cm2 (to 3 s .f .)  18   18  1 (c) (i) Perimeter of figure = × 2p   + 2p    4  2 2 = 88 p Diameter of circe = 2 × 176 p = 56 .0 mm (to 3 s .f .) =  88  Area of circle = p    π 2 1 × 2p(9) + 2p(4 .5) 2 = 9p + 9p = 18p = 56 .5 cm (to 3 s .f .) =  7744  = p 2   π  7744 = p = 2460 mm2 (to 3 s .f .) (ii) Area of figure = area of big semicircle + area of two small semicircles 616 p = 14 .0 cm (to 3 s .f .) (d) Radius of circle = 1 × p(9)2 + p(4 .5)2 2 81 p + 20 .25p = 2 = 60 .75p = 191 cm2 (to 3 s .f .) 7. (i) Perimeter of figure = 2p(2) + 2(9 – 2 × 2) + 2(3) = 4p + 2(5) + 6 = 4p + 10 + 6 = 4p + 16 = 28 .6 m (to 3 s .f .) = 616 p = 28 .0 cm (to 3 s .f .) Diameter of circle = 2 ×  616  Circumference of circle = 2p    π  = 88 .0 cm (to 3 s .f .) 1 210 (ii) Area of figure = area of rectangle – area of four quadrants = 9 × [2(2) + 3] – p(2)2 =9×7–4p = 63 – 4p = 50 .4 m2 (to 3 s .f .) 8. Let the breadth of the rectangular field be x m . Then the length of the field is (x + 15) m . 2[(x + 15) + x] = 70 2(2x + 15) = 70 2x + 15 = 35 2x = 20 x = 10 \ Breadth of field = 10 m Length of field = 10 + 15 = 25 m Area of field = 25 × 10 = 250 m2 Area of path = (25 + 2 .5 + 2 .5) × (10 + 5 + 5) – 250 = 30 × 20 – 250 = 600 – 250 = 350 m2 9. Area of shaded region = area of quadrilateral PQRS Cost incurred = 28p × $55 = $4838 .05 (to the nearest cent) 7 1 × 2p   + 2(5 .7) 2  2 1 = × 2p(3 .5) + 11 .4 2 = 3 .5p + 11 .4 = 22 .4 cm (to 3 s .f .) (ii) Area of figure = area of semicircle BCD + area of nABD 13. (i) Perimeter of figure = 1 1 × p(3 .5)2 + × 7 × (8 – 3 .5) 2 2 1 = 6 .125p + × 7 × 4 .5 2 = 6 .125p + 15 .75 = 35 .0 cm2 (to 3 s .f .) = 14. (i) Perimeter of shaded region  10  3 1 1 × 2p(10) + × 2p   + × 2p(10 – 3) + 3 + (10 – 3)  2 4 2 4 1 1 = 15p + × 2p(5) + × 2p(7) + 3 + 7 2 4 7 = 15p + 5p + p + 10 2 47 = p + 10 2 = 83 .8 cm (to 3 s .f .) (ii) Area of shaded region = area of big semicircle + area of small semicircle + area of region ABCE = 1 1 × AR × RP + × RB × RP 2 2 1 = × RP × (AR + RB) 2 1 = × AD × AB 2 1 = × 23 × (7 + 13 .5) 2 1 = × 20 .5 × 23 2 = 235 .75 m2 10. Area of shaded region = area of nABC – area of nADE = 1 1 × p(10)2 + × p(5)2 2 2 1 + × p(102 – 72) 4 25 1 = 50p + p+ × p(100 – 49) 2 4 25 1 = 50p + × p(51) p+ 2 4 25 51 = 50p + p+ p 2 4 301 = p 4 = 236 cm2 (to 3 s .f .)  200  1 15. (i) Perimeter of shaded region = × 2p   + 2(10) 2  2  = 1 1 × 20 × 21 – × 10 × 10 .5 2 2 = 210 – 52 .5 = 157 .5 m2 = 1 1 × AC × BD = × CD × AE 2 2 1 1 × 20 × BD = × 22 × 16 2 2 10 × BD = 176 BD = 17 .6 Length of BD = 17 .6 cm 11. Area of nACD = 12 12. (i) Area of surface of circular pond = p    2 200 p + 20 2 = 42 .2 m (to 3 s .f .) = 2 (ii) Area of shaded region = area of semicircle BCD – area of nBCD = p(6) = 36p = 113 m2 (to 3 s .f .) 2 2   1 1 × p  200  – × 10 × 10 2 2  2  = 25p – 50 = 28 .5 m2 (to 3 s .f .) = (ii) Area of path = p(6 + 2)2 – 36p = p(8)2 – 36p = 64p – 36p = 28p m2 211 1 1 × (35 .5 + 20) × 15 2 1 = × 55 .5 × 15 2 = 416 .25 cm2 (ii) Perimeter of trapezium = 35 .5 + 18 + 20 + 16 = 89 .5 cm 5. (i) Area of trapezium = 0.785 cm p 0.785 1 Area of shaded region = ×2 × p 2 16. Radius of each circle = 0.785 p 0.785 p = 0 .250 cm2 (to 3 s .f .) 17. Area of grass within the goat’s reach = p(1 .5)2 = 2 .25p m2 Time the goat needs = 2 .25p × 14 = 99 .0 minutes (to 3 s .f .) = 6. (i) Area of trapezium = Exercise 13B 1. (a) Area of parallelogram = 12 × 7 = 84 cm2 = 150 m2 = 150 = 150 = 30 = 18 Length of RS = 18 m (ii) Perimeter of trapezium = PQ + QR + RS + PS = 54 .7 m 12 + QR + 18 + 13 = 54 .7 43 + QR = 54 .7 QR = 11 .7 Length of QR = 11 .7 m 7. Area of shaded regions = area of trapezium ABCD – area of nBCE 42 6 =7m (b) Base of parallelogram = 42.9 7.8 = 5 .5 mm (c) Height of parallelogram = 1 1 × (10 + 14) × 12 – × 14 × 12 2 2 1 = × 24 × 12 – 84 2 = 144 – 84 = 60 cm2 = 1 2. (a) Area of trapezium = × (7 + 11) × 6 2 1 = × 18 × 6 2 = 54 cm2 126 (b) Height of trapezium = 1 × (8 + 10) 2 126 = 1 × 18 2 126 = 9 = 14 m 702 2 = 351 m2 8. Area of parallelogram ABFG = 351 27 = 13 m Height of parallelogram ABFG with reference to base FG = 1 × (2 × 27) × 13 2 1 = × 54 × 13 2 = 351 m2 9. (a) Total area of shaded regions = area of rectangle – area of parallelogram – area of circle – area of triangle = (12 + 14) × (15 + 10) 1 – (12 + 14 – 5 – 2) × 10 – p(4)2 – × 12 × 15 2 = 26 × 25 – 19 × 10 – 16p – 90 = 650 – 190 – 16p – 90 = 370 – 16p = 320 cm2 (to 3 s .f .) (b) Area of shaded region = area of trapezium – area of circle Area of shaded region = 72 –5 1 ×8 2 72 = –5 4 = 18 – 5 = 13 mm (c) Length of parallel side 2 of trapezium = 3. (i) Area of parallelogram = 6 × 9 = 54 cm2 (ii) Perimeter of parallelogram = 2(10 + 6) = 2(16) = 32 cm 4. Area of parallelogram = PQ × ST = QR × SU PQ × 8 = 10 × 11 .2 PQ × 8 = 112 PQ = 14 Length of PQ = 14 m 1 1 × (PQ + RS) × PT 2 1 × (12 + RS) × 10 2 5 × (12 + RS) 12 + RS RS 1 × (35 + 18) × 18 – p(6)2 2 1 = × 53 × 18 – 36p 2 = 477 – 36p = 364 cm2 (to 3 s .f .) = 212 10. Area of figure = area of trapezium ABCE – area of parallelogram GHDE – area of semicircle 2  15  1 1 = × (12 + 13 + 15) × 24 – 13 × 16 – × p    2 2 2 (b) Total area of shaded regions = area of circle – area of triangle – area of rectangle 1 × (2 × 13 .6) × 13 .6 – 16 × 11 2 1 = 184 .96p – × 27 .2 × 13 .6 – 176 2 = 184 .96p – 184 .96 – 176 = 184 .96p – 360 .96 = 220 cm2 (to 3 s .f .) (c) Total area of shaded regions = p(13 .6)2 – 1 1 × 40 × 24 – 208 – × p(7 .5)2 2 2 = 480 – 208 – 28 .125p = 272 – 28 .125p = 184 cm2 (to 3 s .f .) = 11. Area of nAED = 1 × AE × ED = 25 cm2 2 AE × ED = 50 1 1 × (48 + 16) × 20 + × (30 + 20) × 16 2 2 1 1 = × 64 × 20 + × 50 × 16 2 2 = 640 + 400 = 1040 cm2 (d) Area of shaded region = area of trapezium – area of triangle = 1 × (EB + DC) × ED 2 1 = × (3AE + 4AE) × ED 2 1 = × 7AE × ED 2 7 = × AE × ED 2 7 = × 50 2 = 175 cm2 12. (i) Let the height of the parallelogram ABCD with reference to the base BC be h cm . Area of parallelogram ABCD = BC × h = 80 cm2 Area of trapezium BCDE = 1 1 × (17 + 9) × (2 × 6) – × 17 × 6 2 2 1 = × 26 × 12 – 51 2 = 156 – 51 = 105 cm2  28   28  1 × 2p   + 2p   2. (i) Perimeter of shaded region =  4  2 2 = 1 × 2p(14) + 2p(7) 2 = 14p + 14p = 28p = 88 .0 cm (to 3 s .f .) = 1 × BE × h Area of nABE = 2 1 = × 2BC × h 2 = BC × h = 80 cm2 (ii) Let the height of the parallelogram ABCD with reference to the base DC be h′ cm . Area of parallelogram ABCD = DC × h′ = 80 cm2 (ii) Area of shaded region = area of big semicircle – area of two small semicircles 1 × p(14)2 – p(7)2 2 = 98p – 49p = 49p = 154 cm2 (to 3 s .f .) 3. Area of shaded region = area of one square of sides (2 × 12) cm = (2 × 12)2 = 242 = 576 cm2 4. (i) Area of parallelogram = 9 × 25 = 225 m2 (ii) Perimeter of parallelogram = 2(9 + 30 .8) = 2(39 .8) = 79 .6 m 5. Let AB = BC = CD = DE = EF = AF = x cm . (x + x) × x = 24 2x × x = 24 2x2 = 24 x2 = 12 Since x > 0, x = 12 Area of parallelogram BCEF = 12 × 12 = 12 cm2 = 1 × DF × h′ 2 1 1 = × DC × h′ 2 2 1 = × DC × h′ 4 1 = × 80 4 = 20 cm2 Area of nADF = Review Exercise 13 1. (a) Area of shaded region = 11 × 13 + 7 × (14 + 13) + 8 × (35 – 20) + 9 × 35 – 12 × 9 = 143 + 7 × 27 + 8 × 15 + 315 – 108 = 143 + 189 + 120 + 315 – 108 = 659 cm2 213 1 1 × (8 + 8 ÷ 2) × (6 ÷ 2) 2 1 = × (8 + 4) × 3 2 1 = × 12 × 3 2 = 18 cm2 6. Area of trapezium ABPQ = Challenge Yourself 1. Let the length of AB be x cm . Then the length of BC = the length of AC = 2x cm . Area of nABC = area of nABD + area of nBCD + area of nACD 1 1 1 ×x×9+ × 2x × 7 + × 2x × 7 2 2 2 = 4 .5x + 7x + 7x = 18 .5x cm2 Case 1: The base of nABC is taken to be AB . 7. Area of figure = area of rectangle ABCF + area of trapezium FCDE = 20 × 15 + = 1 × (20 + 3 .5) × 7 2 1 × 23 .5 × 7 2 = 300 + 82 .25 = 382 .25 m2 = 382 .25 × 0 .0001 ha = 0 .038 225 ha 36 8. (i) x + y = 1 ×6 2 36 = 3 = 12 (ii) Since x = 2y, 2y + y = 12 3y = 12 y =4 \x=2×4 =8 9. Length of each side of square = 1 =1m Perimeter of square = 4 × 1 =4m 1 Radius of circle = m p  1 Circumference of circle = 2p   m  π 1 × x × h1 = 18 .5x 2 \ h1 = 37 cm Case 2: The base of nABC is taken to be BC or AC . = 300 + 1 × 2x × h2 = 18 .5x 2 \ h2 = 18 .5 cm 2. (i) Perimeter of figure = pr1 + pr2 + pr3 + pr4 + pr5 + AB = p(r1 + r2 + r3 + r4 + r5) + AB AB + AB 2 70 =p× + 70 2 = 35p + 70 = 180 cm (to 3 s .f .) (ii) Perimeter of figure = pr + AB =p× AB + AB 2 70 =p× + 70 2 = 35p + 70 = 180 cm (to 3 s .f .) (iii) Given a line segment AB of fixed length, regardless of the number of semicircles drawn on the line segment, the perimeter of the figure will be the same. =p×  1 Required difference = 4 – 2p    π = 0 .455 m (to 3 s .f .)  21  10. Circumference of drum = 2p    2 = 21p cm Number of complete turns of handle required 9.89 × 100 21π 989 = 21p = 15 (rounded up to the nearest whole number) = 1 214 3. C D A E B BD 5 ED 1 = ⇔ = BE 4 BE 4 Area of n AED ED 1 = = Area of n ABE BE 4 Area of nAED 1 = 4 20 \ Area of nAED = 5 cm2 Since nACD shares the same base AD and the same height as nABD, area of nACD = area of nABD . Since nAED is a common part of nACD and nABD, area of nDCE = area of nABE = 20 cm2 . Area of n BCE BE 4 = = Area of n DCE ED 1 Area of nBCE 4 = 20 1 \ Area of nBCE = 80 cm2 Area of trapezium = area of nABE + area nAED + area of nDCE + area of nBCE = 20 + 5 + 20 + 80 = 125 cm2 215 1 Chapter 14 Volume and Surface Area of Prisms and Cylinders TEACHING NOTES Suggested Approach Students have learnt the conversion of unit area and perimeter and area of plane figures in the last chapter. This chapter will be dealing with the conversion of unit volumes and the volume and surface area of solids, which is a natural transition from the last chapter, from two-dimensional to three-dimensional. To assist in the students’ understanding, teachers should continually remind students to be aware of the linkages between both topics, as well as introducing real-life applications that can reinforce learning. Section 14.1: Conversion of Units Teachers should recap the unit conversion of lengths and areas, proceed to introduce of volume by stating actual applications (see Class Discussion: Measurement in Daily Lives), and then stating the different units associated with volume (e.g. m, cm3 and m3). Students should recognise how the number of dimensions and the unit representation for lengths, areas and volumes are related (e.g. cm, cm2 and cm3). Students should recall calculations such as 1 cm3 = 1 cm × 1 cm × 1 cm and solve problems involving conversion of unit volumes. Section 14.2: Nets Teachers should first define and explain that nets are basically flattened figures that can be folded to its threedimensional solids. Teachers should show the nets of the various solids. Students are encouraged to make their own nets and form the different three-dimensional solids. They should also be able to visualise the solids from different viewpoints. Section 14.3: Volume and Surface Area of Cubes and Cuboids Teachers can state that the volume of an object refers to the space it occupies, so the greater the volume, the more space the object occupies. Students should be informed and know that the volume of cubes and cuboids is the product of its three sides (base × height = (length × breadth) × height). The formulas for the total surface area of cubes and cuboids can be explored and discovered by students (see Class Discussion: Surface Area of Cubes and Cuboids). It is important for the students to observe that the total surface area is the total area of all its faces. Section 14.4: Volume and Surface Area of Prisms Teachers can introduce prisms to the students by stacking a few cubes to form a prism and show them how a prism looks like. Students should know terms like lateral faces and cross-sections, and learn that prisms are solids with uniform polygonal cross-sections. Teachers can ask the students to name some real-life examples of prisms and use this opportunity to get them to explain why certain objects are not prisms so that they can get a better understanding about prisms. Observant students should realise that cuboids are prisms. Teachers can highlight to the students that prisms do not necessarily have square bases and challenge students to think of bases of other possible shapes (see Fig. 14.2 on page 348). Teachers should illustrate and derive the formulas for the volume and total surface area. Students need to understand the definitions of volume and total surface area rather than memorise the formulas. 1 216 Section 14.5: Volume and Surface Area of Cylinders Similar to the last section, teachers can introduce cylinders by stacking coins or showing students real-life examples of cylindrical objects. Only right circular cylinders are covered in this syllabus. Some students may think that cylinders are also prisms since both have uniform cross-sections. Teachers need to impress upon students that this is not the case even though cylinders and prisms share similarities (see Investigation: Comparison between a Cylinder and a Prism). Teachers should also cover the formulas for the volume and total surface area of cylinders. Again, students need to understand the definitions of volume and total surface area rather than memorise formulas. Section 14.6: Volume and Surface Area of Composite Solids Teachers should go through Worked Example 10 closely with students. Other than assessing their understanding, teachers can inform students to be aware of any sides that should be omitted in finding total surface areas. Challenge Yourself Teachers should challenge students to think how the cross-section of the cuboid looks like in finding the volume and surface area. 217 1 Part III: WORKED SOLUTIONS Name Class Discussion (Measurements in Daily Lives) Figure Net Other 4% 1. (i) Wash Basin 10% Triangular Prism Showers 29% Laundry 19% Kitchen sink 22% Flushing 16% Cylinder Source: http://www.pub.gov.sg/conserve/Households/Pages/Watersavinghabits.aspx Class Discussion (Surface Area of Cubes and Cuboids) The activity which requires the greatest amount of water is shower . (ii) – Some measures: • Take shorter showers. • Turn off the shower tap while soaping. • Use a tumbler when brushing your teeth. • Do not thaw food under running water. Let it defrost overnight inside the refrigerator instead . • Wash vegetables and dishes in a sink or container filled with water . • Install thimbles or water saving devices at taps with high flow rate. • Turn off taps tightly to ensure they do not drip. • Do not leave the tap running when not in use. 2. (i) The volume of one teaspoon of liquid is 5 ml . (ii) This corresponds to 2 litres of water. 1. A cube has 6 surfaces. Each surface is in the shape of a square . The area of each face is equal . \ The total surface area of a cube is 6l 2 . A cuboid has 6 surfaces. Each surface is in the shape of a rectangle . \ The total surface area of a cube is 2(b × l + b × h + l × h) . 2. The total surface of the object is equal to the total area of all the faces of the net . Thinking Time (Page 349) 1. (i) The shape of all the lateral faces of a right prism is a rectangle. (ii) The shape of all the lateral faces of an oblique prism is a parallelogram. 2. Investigation (Cubes, Cuboids Prisms and Cylinders) Part II: Name Figure Net Cube 3. Examples of building structures and items are European-style houses and chocolates. They are shaped as prisms as they have a uniform cross-section. Cuboid Thinking Time (Page 354) Examples are can drinks, toilet rolls and iron rods. They are shaped as cylinders as they have a uniform circular cross-section. 1 218 (ii) Investigation (Comparison between a Cylinder and a Prism) 1 cm3 = 1 ml 165 000 cm3 = 165 000 ml 165 000 l 1000 = 165 l 1. The polygon will become a circle. 2. The prism will become like a cylinder. = Thinking Time (Page 358) Practise Now 2 1. (i) Volume of the cuboid = l × 18 × 38 = 35 568 35 568 l= 18 × 38 = 52 (ii) Volume of each small cube = 2 × 2 × 2 = 8 cm3 Number of cubes to be obtained r Circumference = 2pr h r 35 568 8 = 4446 2. Volume of the open rectangular tank = 55 × 35 × 36 = 69 300 cm3 Volume of water in the open rectangular tank initally = r Total outer surface of a closed cylinder = pr2 + 2prh + pr2 = 2pr2 + 2prh 1 × 69 300 2 = 34 650 cm3 Total volume of water in the open rectangular tank after 7700 cm3 of water are added to it = 34 650 + 7700 = 42 350 cm3 Let the depth of water in the tank be d cm . 55 × 35 × d = 42 350 1925d = 42 350 d = 22 Depth of water = 22 cm = Class Discussion (Total Surface Area of Other Types of Cylinders) (a) an open cylinder Total outer surface of an open cylinder = 2prh + pr2 (b) a pipe of negligible thinkness Practise Now 3 External volume = (180 + 30 + 30) × (80 + 30 + 30) × (120 + 30) = 240 × 140 × 150 = 5 040 000 cm3 Internal volume = 180 × 80 × 120 = 1 728 000 cm3 Volume of concrete used = 5 040 000 – 1 728 000 = 3 312 00 cm3 Total outer surface of a pipe of negligible thickness = 2prh Practise Now 1 (a) (i) 1 m3 = 1 000 000 cm3 10 m3 = 10 × 1 000 000 cm3 = 10 000 000 cm3 (ii) 1 cm3 = 1 ml 10 000 000 cm3 = 10 000 000 ml 10 m3 = 10 000 000 ml (b) (i) 1 000 000 cm3 = 1 m3 165 000 165 000 cm3 = m3 1 000 000 Practise Now 4 1. (i) Volume of cuboid = 8 × 5 × 10 = 400 cm3 (ii) Surface area of the cuboid = 2(8 × 5 + 8 × 10 + 5 × 10) = 340 cm2 = 0.165 m3 219 1 2. (i) Volume of water in the tank = 16 × 9 × 8 = 1152 cm3 = 1152 ml Practise Now 7 1. Base radius = 18 ÷ 2 = 9 cm Height of the cylinder = 2.5 × 9 = 22.5 cm Volume of the cylinder = pr2h = p(9)2(22.5) = 5730 cm3 (to 3 s.f.) 2. Base radius = 12 ÷ 2 = 6 cm Volume of the cylinder = p(6)2h = 1000 1000 h= p (6)2 h = 8.84 cm (to 3 s.f.) 1152 l 1000 = 1.152 l (ii) Surface area of the tank that is in contact with the water = (16 × 9) + 2(16 × 8 + 9 × 8) = 544 cm2 3. Let the length of the cube be l cm . l × l × l = 27 cm3 l 3 = 27 = Practise Now 8 l = 3 27 l =3 Total area of the faces that will be coated with paint = 6(3 × 3) = 54 cm2 1. Since petrol is discharged through the pipe at a rate of 2.45 m/s, i.e. 245 cm/s, in 1 second, the volume of petrol discharged is the volume of petrol that fills the pipe to a length of 245 cm. In 1 second, volume of petrol discharged = volume of pipe of length 245 cm = pr2h = p(0.6)2(245) = 88 .2p cm3 In 3 minutes, volume of petrol discharged = 88 .2p × 3 × 60 = 49 900 cm3 = 49 .9 l (to 3 s.f.) 2. Base radius = 0 .036 ÷ 2 = 0 .018 m Since water is discharged through the pipe at a rate of 1.6 m/s, i.e. in 1 second, the volume of water discharged is the volume of water that fills the pipe to a length of 1.6 m. In 1 second, volume of water discharged = volume of pipe of length 1.6 m = pr2h = p(0.018)2(1.6) = 0.000 518 4p cm3 Volume of the cylindrical tank = pr2h = p(3.4)2(1.4) = 16 .184p cm3 Time required to fill the tank 16.184 p = 0.000 518 4 p Practise Now 5 1. Base area = area of square =4×4 = 16 m2 Volume of the prism = base area × height = 16 × 10 = 160 m3 2. Base area = area of triangle 1 × 5.6 × x 2 = 2 .8x cm2 Volume of the prism = base area × height = 2 .8x × 12 = 151.2 33 .6x = 151.2 x = 4.5 = Practise Now 6 (i) Volume of the prism = base area × height  1   =   × 3 × 4  + (6 × 5)  × 4.5 2     = 36 × 4.5 = 162 cm3 (ii) Total surface area of the prism = perimeter of the base × height + 2 × base area = (3 + 4 + 6 + 5 + 6) × 4.5 + 2 × 36 = 180 cm2 11 s 81 = 520 min (to the nearest minute) = 3 1219 Practise Now 9 1. (i) Total surface area of the can = 2pr2 + 2prh = 2p(3.5)2 + 2p(3.5)(10) = 24.5p + 70p = 94.5p = 297 cm2 (to 3 s.f.) 1 220 (ii) Area of the can that is painted = pr2 + 2prh (An open cylinder has only one = p(3.5)2 + 2p(3.5)(10) base and a curved surface) = 12.25p + 70p = 82.25p Ratio of the area of the can that is painted, to the total surface area found in (i) . = 94.5p : 82.25p = 94.5 : 82.25 =54 : 47 2. (i) Area of the cross section of the pipe = p(2.5)2 – p(2.1)2 = 6.25p – 4.41p = 1 .84p cm2 (ii) Internal curved surface area of the pipe = 2p(2.1)(12) = 50.4p = 158 cm2 (to 3 s.f.) (iii) Total surface area of the pipe = 2(1.84p) + 50.4p + 2p(2.5)(12) = 3 .68p + 50.4p + 60p = 114 .08p = 358 cm2 (to 3 s.f.) Exercise 14A 1. (a) (i) 1 m3 = 1 000 000 cm3 4 m3 = 4 × 1 000 000 cm3 = 4 000 000 cm3 (ii) 1 m3 = 1 000 000 cm3 0.5 m3 = 0.5 × 1 000 000 cm3 = 500 000 cm3 (b) (i) 1 000 000 cm3 = 1 m3 250 000 m3 250 000 cm3 = 1 000 000 = 0.25 m3 3 (ii) 1 000 000 cm = 1 m3 67 800 67 800 cm3 = m3 1 000 000 = 0.0678 m3 2. (a) (i) 1 m = 1 000 000 cm3 0 .84 m3 = 0 .84 × 1 000 000 cm3 = 840 000 cm3 (ii) 1 cm3 = 1 ml 840 000 cm3 = 840 000 ml (b) (i) 1 000 000 cm3 = 1 m3 2560 2560 cm3 = m3 1 000 000 3 = 0.002 56 m3 (ii) 1 cm = 1 ml 2560 cm3 = 2560 ml Practise Now 10 3 1. (i) Volume of the container = 20 × 9 × 14 + 1 × p(14)2(20) 4 = 2520 + 980p = 5600 cm3 (to 3 s.f.) (ii) Total surface area of the container 1 2 = 2  4 × π (14)  + 2(9 × 14) + 2(20 × 9) + 2(14 × 20)   1 + × 2p(14)(20) 4 = 98p + 252 + 360 + 560 + 140p = 238p + 1172 = 1920 cm2 (to 3 s.f.) 2. (i) Volume of the solid = 6 × 12 × 8 – 2560 l 1000 = 2.56 l Volume of the cuboid = 6 × 8 × 10 = 480 cm3 Surface area of the cuboid = 2(6 × 8 + 8 × 10 + 6 × 10) = 376 cm2 Volume of the cuboid = 7 × 12 × 5 = 420 cm3 Surface area of the cuboid = 2(7 × 12 + 5 × 7 + 5 × 12) = 358 cm2 Volume of the cuboid = 120 × 10 × 96 = 115 200 mm3 Surface area of the cuboid = 2(120 × 10 + 96 × 10 + 120 × 96) = 27 360 mm2 = 3. (a) (i) (ii) (b) (i) (ii) (c) (i) 1 × p(3)2(12) 2 (ii) = 576 – 54p = 406 cm3 (to 3 s.f.) (ii) Total surface area of the solid (d) (i) Volume of the cuboid = 1 1 1 × × 10 2 2 1 = 7 cm3 2 (ii) Surface area of the cuboid 1   1 1 1 = 2  1 × + × 10 + 1 × 10  2   2 2 2 1 1 = 2  8 × 6 – × π (3)2  + 2(8 × 12) + × 2p(3)(12) + 6 × 12 2 2   = 96 – 9p + 192 + 36p + 72 = 360 + 27p = 445 cm2 (to 3 s.f.) = 41 221 1 cm2 2 1 6. Volume of the rectangular block of metal = 0 .24 × 0 .19 × 0.15 = 0 .00684 m3 Let the length of the cube be l cm . Volume of each small cube = l × l × l = 0 .00684 m3 l3 = 0 .00684 (e) (i) Volume of the cuboid = 1 2 3 5 × × 5 8 8 21 3 = cm 64 (ii) Surface area of the cuboid 2 5  2 3 3 5 = 2 1 × + × + 1 ×  5 8  5 8 8 8 l = 3 0.00684 = 0.190 (to 3 s.f.) \ Length of each side = 0 .190 m 7. Volume of the open rectangular tank = 4 × 2 × 4 .8 = 38 .4 m3 Volume of water in the open rectangular tank initally 43 cm2 160 (f) (i) Volume of the cuboid = 3.9 × 0.7 × 1.5 = 4.095 cm3 (ii) Surface area of the cuboid = 2(3.9 × 0.7 + 0.7 × 1.5 + 3.9 × 1.5) = 19 .26 cm2 =3 Length Breadth Height Volume Total surface area (a) 24 mm 18 mm 2160 mm3 1284 mm2 (b) 5 cm 5 mm 4. 3 3 cm 8 cm 120 cm 52.5 cm3 (c) 2.5 cm 6 cm 3.5 cm (d) 12 m 8m 6m 576 m3 3 × 38 .4 4 = 28 .8 m3 4000 l = 4000 × 1000 ml = 4 000 000 ml = 4 000 000 cm3 4 000 000 3 = m 1 000 000 = 158 cm2 89.5 cm2 432 m2 = 4 m3 Total volume of water in the open rectangular tank after 4000 litres of water are added to it = 28 .8 + 4 = 32 .8 m3 Let the depth of water in the tank be d m. 4 × 2 × d = 32 .8 8d = 32 .8 d = 4 .1 \ Depth = 4.1 m 8. External volume = (3.2 + 0.2 + 0.2) × (2.2 + 0.2 + 0.2) × (1.5 + 0.2) = 3 .6 × 2 .6 × 1.7 = 15.912 m3 Internal volume = 3.2 × 2 .2 × 1.5 = 10.56 m3 Volume of wood used = 15.912 – 10.56 = 5.352 m3 9. External volume = 15 × 10 × 45 = 6750 cm3 Internal volume = 3 × 2 × 45 = 270 cm3 Volume of the hollow glass structure = 6750 – 270 = 6480 cm3 10. (i) Volume of water in the tank = 0 .2 × 0.15 × 0 .16 = 0 .0048 m3 = 0 .0048 × 1 000 000 cm3 = 4800 cm3 = 4800 ml (a) Volume = 24 × 18 × 5 = 2160 mm3 Surface area = 2(24 × 18 + 24 × 5 + 18 × 5) = 1284 mm2 (b) Let the height of the cuboid be h cm . Volume = 5 × 3 × h = 120 cm3 120 = 8 cm 5×3 Surface area = 2(5 × 3 × 5 × 8 + 3 × 8) = 158 cm2 (c) Let the length of the cuboid be l cm . Volume = l × 6 × 3.5 = 52.5 cm3 \h = 52.5 = 2.5 cm 6 × 3.5 Surface area = 2(2.5 × 6 + 6 × 3.5 + 2.5 × 3.5) = 89.5 cm2 (d) Let the breadth of the cuboid be b m . Volume = 12 × b × 6 = 576 m3 \l = 576 =8m 12 × 6 Surface area = 2(12 × 8 + 6 × 8 + 12 × 6) = 432 m2 (i) Volume of the cuboid = 28 × b × 15 = 6720 cm3 \b = 5. 6720 28 × 15 = 16 \ Breadth = 16 cm (ii) Volume of each small cube = 4 × 4 × 4 = 64 cm3 Number of cubes to be obtained \b = 6720 64 = 105 4800 l 1000 = 4 .8 l = 1 = 222 16. In one minute, the water will flow through 22 × 60 = 1320 cm along the drain . Amount of water that will flow through in one minute = 30 × 3.5 × 1320 = 138 600 cm3 = 138 600 ml (ii) Surface area of the tank that is in contact with the water = (0.2 × 0.15) + 2(0.2 × 0.16 + 0.15 × 0 .16) = 0 .142 m2 11. (i) Volume of water in the tank = 80 × 40 × 35 = 112 000 cm3 = 112 000 ml 138 600 l 1000 = 138 .6 l 17. (i) Let the height of the cuboid be h cm . Surface area of the cuboid = 2(12 × 9 + 12 × h + 9 × h) = 426 cm2 2(108 + 12h + 9h) = 426 2(108 + 21h) = 426 108 + 21h = 213 21h = 213 – 108 21h = 105 h =5 \ Height of cuboid = 5 cm (ii) Volume of the cuboid = 12 × 9 × 5 = 540 cm3 18. (i) Floor area of Room A = 26 × 1 = 26 m2 Volume of Room A = 26 × 1 × 3 = 78 m3 Floor area of Room B = 5 × 5 = 25 m2 Volume of Room B = 5 × 5 × 3 = 75 m3 Floor area of Room C = 6 × 6 = 36 m2 Volume of Room C = 6 × 6 × 1 .8 = 64 .8 m3 (ii) No. If both rooms, A and B, have the same height, then we will use the floor area as the gauge. If the rooms do not have the same height, then we will use the volume to decide. = 112 000 = l 1000 = 112 l (ii) Surface area of the tank that is in contact with the water = (80 × 40) + 2(80 × 35 + 40 × 35) = 11 600 cm2 11 600 2 m = 10 000 = 1 .16 m2 12. Let the length of the cube be l cm . l × l × l = 64 cm3 l 3 = 64 l = 3 64 =4 Total area of the faces that will be coated with paint = 6(4 × 4) = 96 cm2 13. Let the length of the cube be l cm . 6(l × l) = 433.5 6l 2 =433.5 l 2 =72.25 l = 72.25 = 8.5 Volume of the cube = 8.5 × 8.5 × 8.5 = 614.125 cm3 14. (i) Number of trips required to fill the entire quarry 2.85 × 1 000 000 6.25 = 456 000 (ii) Cost to fill the quarry = 456 000 × $55 = $25 080 000 (iii) 3 hectares = 30 000 m2 Cost to fill 1 m2 of the land 25 080 000 =$ 30 000 = Exercise 14B 1. (a) Volume of the prism = base area × height 1  =  × (75 + 59) × 46  × 120 2  = 3028 × 120 = 369 840 cm3 (b) Volume of the prism = base area × height 1  =  × (16 + 28) × (18 – 7) + 7 × 28  × 38  2 = 438 × 38 = 16 644 cm3 = $836 15. Volume of wood used to make this trough = (185 × 45 × 28) – [(185 – 2.5 – 2.5) × (45 – 2.5 – 2.5) × (28 – 2.5)] = (185 × 45 × 28) – (180 × 40 × 25.5) = 233 100 – 183 600 = 49 500 cm3 49 500 = m3 1 000 000 = 0.0495 m3 223 1 (c) Volume of the prism = base area × height = [9 × 5 + 9 × 3 + (16 – 8) × (9 – 6)] × 10 = 96 × 10 = 960 cm3 (d) Volume of the prism = base area × height 1  =  × (14 + 18) × 6  × 12 2   = 96 × 12 = 1152 cm3 (e) Volume of the prism = base area × height 1  =  × 6 × 8 + 13 × 10  × 5  2 = 154 × 5 = 770 cm3 (f) Volume of the prism = base area × height (d) Area of nABC 1 × 7.8 × 24 .6 2 = 95.94 cm2 Volume of prism = 95.94 × CD = 38 376 CD = 400 cm 3. Air space in the hall = Volume of the prism = base area × height = =  1 × 42 × (38 – 23) + 42 × 23 × 80   2 = 1281 × 80 = 102 480 m3 4. (a) (i) Volume of the prism = base area × height 1 =  × 6 × 4  × 15  2 = 12 × 15 = 180 cm3 (ii) Total surface area of the prism = perimeter of the base × height + 2 × base area = (5 + 5 + 6) × 15 + 2 × 12 = 264 cm2 (b) (i) Volume of the prism = base area × height = [2 × 7 + (5 – 2) × (7 – 6)] × 9 = 17 × 9 = 153 cm3 (ii) Total surface area of the prism = perimeter of the base × height + 2 × base area = (7 + 2 + 6 + 3 + 1 + 5) × 9 + 2 × 17 = 250 cm2 5. (i) Volume of water in the pool when it is full = Volume of the prism = base area × height 1 =  × 18 × (12 – 3) + 3 × 18  × 35 2  = 135 × 35 = 4725 cm3 AB BC BC Area of nABC Volume of prism (a) 3 cm 4 cm 7 cm 6 cm2 42 cm3 (b) 9 cm 14 cm 11 cm 63 cm2 32 cm 15 cm 2. (c) (d) 24 .6 cm 7.8 cm 2 300 cm 240 cm 400 cm 95.94 cm2 693 cm3 72 000 cm3 38 376 cm3 (a) Area of nABC 1 ×4×3 2 = 6 cm2 Volume of prism =6×7 = 42 cm3 (b) Area of nABC = 1 =  × (1.2 + 2) × 50  × 25 2  = 80 × 25 = 2000 m3 (ii) Area of the pool which is in contact with the water = [(1.2 + 50 + 2 + 50.01) × 25 + 2 × 80] – (25 × 50) = 1490.25 cm2 1 × BC × 9 = 63 2 4.5BC = 63 BC = 14 cm Volume of prism = 63 × 11 = 693 cm3 (c) Volume of prism = Area of nABC × 300 = 72 000 Area of nABC = 240 cm2 Area of nABC = = Exercise 14C 1. (a) (i) Volume of the closed cylinder = pr2h = p(7)2(12) = 1850 cm3 (to 3 s.f.) (ii) Total surface area of the closed cylinder = 2pr2 + 2prh = 2p(7)2 + 2p(7)(12) = 98p + 168p = 266p = 836 cm2 (to 3 s.f.) 1 × 15 × AB = 240 2 7.5AB = 240 AB = 32 cm 1 224 (c) r = 4 ÷ 2 = 2 cm Volume = 528 cm3 p(2)2h = 528 (b) Base radius = 1 .2 ÷ 2 = 0 .6 m (i) Volume of the closed cylinder = pr2h = p(0.6)2(4) = 4.52 m3 (to 3 s.f.) (ii) Total surface area of the closed cylinder = 2pr2 + 2prh = 2p(0.6)2 + 2p(0.6)(4) = 0.72p + 4 .8p = 5.52p = 17.3 m2 (to 3 s.f.) (c) (i) Volume of the closed cylinder = pr2h = p(15)2(63) = 44 500 mm3 (to 3 s.f.) (ii) Total surface area of the closed cylinder = 2pr2 + 2prh = 2p(15)2 + 2p(15)(63) = 450p + 1890p = 2340p = 7350 mm2 (to 3 s.f.) 528 4p = 42.0 cm (to 3 s.f.) Total surface area = 2pr2 + 2prh = 2p(2)2 + 2p(2)(42.02) = 553 cm2 (to 3 s.f.) (d) d = 4 × 2 = 8 m Volume = 1056 m3 p(4)2h = 1056 h= 1056 16 p = 21.0 m (to 3 s.f.) Total surface area = 2pr2 + 2prh = 2p(4)2 + 2p(4)(21.01) = 629 m2 (to 3 s.f.) 3. Base radius = 0 .4 ÷ 2 = 0 .2 m h= Diameter Radius Height Volume Total surface area (a) 8 .00 cm 4 .00 cm 14 cm 704 cm3 453 cm2 (b) 28 .0 cm 14 .0 cm 20 cm 12 320 cm3 2990 cm2 (c) 4 cm 2 cm 42 .0 cm 528 cm3 553 cm2 (d) 8m 4m 21 .0 m 2. 1056 m3 3 × 0.2 = 0.15 m 4 2 Volume of the cylinder = pr h = p(0.2)2(0.15) = 0 .006p m3 = 6000p cm3 Height of the cylinder = 6000 p l 1000 = 18 .8 l 4. Let the depth of water in the drum be d cm . Base radius = 48 ÷ 2 = 24 cm 150 l = 150 000 ml = 150 000 cm3 Volume of water in the drum = pr2d = 150 000 cm3 p(24)2d = 150 000 150 000 d= p (24)2 = 82 .9 \ Depth of water = 82.9 cm 5. Base radius = 15 ÷ 2 = 7.5 cm Capacity of the drink trough = 629 m2 (a) Volume = 704 cm3 pr2(14) = 704 r2 = r= 704 14 p 704 14 p = 4.00 cm (to 3 s.f.) \ d = 2 × 4.00 = 8.00 cm (to 3 s.f.) Total surface area = 2pr2 + 2prh = 2p(4.001)2 + 2p(4.001)(14) = 453 cm2 (b) Volume = 12 320 cm3 pr2(20) = 12 320 r2 = 1 × p × (7.5)2 × 84 2 = 7420 cm3 (to 3 s.f.) = 7.42 l 6. 35 mm = 35 ÷ 10 = 3.5 cm Base radius = 3.5 ÷ 2 = 1.75 cm Total surface area that need to be painted for 1 wooden closed cylinder = 2pr2 + 2prh = 2p(1.75)2 + 2p(1.75)(7) = 6.125p + 24.5p = 30.625p = 12 320 20 p 12 320 20 p = 14.0 cm (to 3 s.f.) \ d = 2 × 14.0 = 28.0 cm (to 3 s.f.) Total surface area = 2pr2 + 2prh = 2p(14.00)2 + 2p(14.00)(20) = 2990 cm2 (to 3 s.f.) r = 225 1 Total surface area that need to be painted for 200 wooden closed cylinders = 200 × 30.625p = 19 200 cm2 (to 3 s.f.) 7. Base radius = 2 .4 ÷ 2 = 1 .2 m Volume of the tank = pr2h = p(1.2)2(6.4) = 9 .216p m3 = 9 216 000p cm3 Volume of the cylinder container = pr2h = p(8.2)2(28) = 1882.72p cm3 Number of completed cylindrical container which can be filled by the oil in the tank 11. Base radius of the pipe = 64 ÷ 2 = 32 mm Since water is discharged through the pipe at a rate of 2.05 mm/s, i.e. in 1 second, the volume of water discharged is the volume of water that fills the pipe to a length of 2.05 mm. In 1 second, volume of water discharged = volume of pipe of length 2.05 mm = pr2h = p(32)2(2.05) = 2099 .2p mm3 Base radius of the cylindrical tank = 7.6 ÷ 2 = 3.8 cm = 38 mm 2 .3 m = 230 cm = 2300 mm Volume of the cylindrical tank = pr2h = p(38)2(2300) = 3 321 200p cm3 Time required to fill the tank 9 216 000 p 1882.72 p = 4895 (to the nearest whole number) 8. External base radius = 28 ÷ 2 = 14 mm = 1.4 cm Internal base radius = 20 ÷ 2 = 10 mm = 1 cm Volume of the metal used in making the pipe = p(1.4)2(35) – p(1)2(35) = 68 .6p – 35p = 33 .6p = 106 cm3 (to 3 s.f.) 9. Base radius of the copper cylindrical rod = 14 ÷ 2 = 7 cm Volume of the copper cylindrical rod = pr2h = p(7)2(47) = 2303p Let the length of the wire be l . Base radius of the wire = 8 ÷ 2 = 4 mm = 0 .4 cm Volume of the wire = p(0.4)2 l = 2303p (0.4)2 l = 2303 2303 l= 0.16 = 14 400 cm (to 3 s.f.) = 144 m 10. Base radius = 2 .4 ÷ 2 = 1 .2 cm Since water is discharged through the pipe at a rate of 2.8 m/s, i.e. 280 cm/s, in 1 second, the volume of water discharged is the volume of water that fills the pipe to a length of 280 cm. In 1 second, volume of water discharged = volume of pipe of length 280 cm = pr2h = p(1.2)2(280) = 403 .2p cm3 Half an hour = 30 minutes In 30 minutes, volume of water discharged = 403 .2p × 30 × 60 = 725 760p cm3 = 2 280 000 cm3 (to 3 s.f.) = 2280 l = 1 3 321 200 p 2099.2 p 83 = 1582 s 656 = 26 min (to the nearest minute) 12. (i) Volume of water in the tank = 18 × 16 × 13 = 3744 cm3 (ii) Let the height of water in the cylindrical container be h . Base radius of the cylindrical container = 17 ÷ 2 = 8.5 cm Volume of water in the cylindrical container = p(8.5)2h = 3744 3744 h= p (8.5)2 = 16.5 \ Height of water = 16.5 cm (iii) Surface area of the cylindrical container that is in contact with the water = pr2 + 2prh = p(8.5)2 + 2p(8.5)(16.49) = 72.25p + 280 .33p = 352.58p = 1110 cm2 (to 3 s.f.) 13. (i) Base radius = 186 ÷ 2 = 93 mm = 9 .3 cm = 1 × 93 = 31 mm = 3 .1 cm 3 Total surface area of the container = 2pr2 + 2prh = 2p(9.3)2 + 2p(9.3)(3.1) = 172.98p + 57.66p = 230 .64p = 725 cm2 (to 3 s.f.) Height = 226 (ii) Area of the container that is painted = pr2 + 2prh (An open cylinder has only one = p(9.3)2 + 2pr(9.3)(3.1) base and a curved surface) = 86 .49p + 57.66p = 144.15p Fraction (ii) Total surface area of the solid = 2(5 × 12) + 2(5 × 8) + 2(12 × 8) + 2(2 × 7) + 2(3 × 2) = 120 + 80 + 192 + 28 + 12 = 432 cm2 2. (i) Volume of the solid = p(2.5)2(8) + 7 × 11 × 3 = 50p + 231 = 388 cm3 (to 3 s.f.) (ii) Total surface area of the solid = 7 × 11 + 2p(2.5)(8) + 2(3 × 7) + 2(3 × 11) + (7 × 11) = 77 + 40p + 42 + 66 + 77 = 262 + 40p = 388 cm2 (to 3 s.f.) 3. (i) Volume of the solid = p(5)2(3) + p(12.5)2(2) = 75p + 312.5p = 387.5p = 1220 cm3 (to 3 s.f.) (ii) Total surface area of the solid = p(12.5)2 + 2p(5)(3) + 2p(12.5)(2) + p(12.5)2 = 156.25p + 30p + 50p + 156.25p = 392.5p = 1230 cm2 (to 3 s.f.) 4. (i) Volume of the glass block 144.15 p 230.64 p 5 = 8 14. (i) Base radius = 23 ÷ 2 = 11.5 mm = 1.15 cm Height = 4 mm = 0 .4 cm Volume of water and metal discs in the tank = (32 × 28 × 19) + 2580[p × (1.15)2 × 0.4] = (17 024 + 1364.82p) cm3 Let the new height in the tank be h . Volume in the tank = 32 × 28 × h = 17 024 + 1364.82p 896h = 17 024 + 1364.82p = 17 024 + 1364.82 π 896 = 23.8 (to 3 s.f.) \ New height of water in the tank = 23 .8 cm (ii) Surface area of the tank that is in contact with the water after the discs have been added = 2(32 × 23.79 + 28 × 23.79) + 32 × 28 = 3750 cm2 (to 3 s.f.) 15. Total surface area of the pipe = 2[p(3.8 + 0.8)2 – p(3.8)2] + 2p(3.8 + 0.8)(15) + 2p(3.8)(15) = 13 .44p + 138p + 114p = 265.44p = 834 cm2 (to 3 s.f.) 16. 124 mm = 12 .4 cm = 0 .124 m 28 km2 = 28 000 000 m2 Volume of the rain = 28 000 000 × 0 .124 = 3 472 000 m3 Volume of each channel = 18 × 26 .4 = 475.2 m3 Time required for the channels to drain off the rain h= 1 × p(24)2(56) + 24 × 56 × 40 4 = 8064p + 53 760 = 79 100 cm3 (to 3 s.f.) (ii) Total surface area of the glass block 1 2 = 2  × π (24)  + 2(40 × 24) + 2(40 × 56) + 2(24 × 56)  4 1 + × 2p(24)(56) 4 = 288p + 1920 + 4480 + 2688 + 672p = 960p + 9088 = 12 100 cm2 (to 3 s.f.) 5. (i) Volume of the solid = = 10 × 12 × 7 – 1 × p(2)2(12) 2 = 840 – 24p = 765 cm3 (to 3 s.f.) (ii) Total surface area of the solid 1   = 2  7 × 10 – × π (2)2  + 2(7 × 12) + 2(3 × 12) 2   1 + × 2p(2)(12) + 10 × 12 2 = 140 – 4p + 168 + 72 + 24p + 120 = 500 + 20p = 563 cm2 (to 3 s.f.) 6. (i) Volume of the remaining solid = p(12)2(32) – p(5)2(14) = 4608p – 350p = 4258p = 13 400 cm3 (to 3 s.f.) 3 472 000 475.2 × 2 59 = 3653 s 297 = 61 minutes (to the nearest minute) = Exercise 14D 1. (i) Volume of the solid = 7 × 3 × 2 + 12 × 8 × 5 = 42 + 480 = 522 cm3 227 1 (ii) Area that will covered in paint = 2p(12)(32) + 2p(5)(14) + 2[p(12)2] = 768p + 140p + 288p = 1196p = 3760 cm2 (to 3 s.f.) 7. (i) Volume of the solid 1  =  × (40 + 88) × 70  × 25 – p(15)2(25) 2  (c) (i) Volume of the solid = 4 × 5 × 1 – 2(1 × 3) = 20 – 6 = 14 cm3 (ii) Total surface area of the solid = 2(1 × 4) + 8(1 × 1) + 2(1 × 3) + 2[4 × 5 – 2(1 × 3)] = 8 + 8 + 6 + 40 – 12 = 50 cm2 (d) (i) Volume of the solid =1×1×5+2×4×1+1×1×3 = 5+8+3 = 16 cm3 (ii) Total surface area of the solid = 2(1 × 5) + 2(1 × 1) + 2(1 × 3) + 2(1 × 4) + 2[1 × 5 + 2 × 3 + 1 × 5] = 10 + 2 + 6 + 8 + 32 = 58 cm2 2. 4.5 m = 450 cm, 3.6 m = 360 cm Number of bricks required 450 18 360 = × × 18 9 6 = 3000 3. Volume of the rectangular block of metal = 256 × 152 × 81 = 3 151 872 mm3 Let the length of the cube be l mm . l 3 = 3 151 872 = 112 000 – 5625p = 94 300 cm3 (to 3 s.f.) (ii) Total surface area of the solid 1 2 = (74 + 40 + 74 + 88) × 25 + 2  × (40 + 88) × 70 – π (15)  2   + 2p(15)(25) = 6900 + 8960 – 450p + 750p = 15 860 + 300p = 16 800 cm2 (to 3 s.f.) 8. (i) Volume of the solid 1 × [p(6 + 1.5)2 – p(6)2] × 8 2 = 4(56.25p – 36p) = 4(20.25p) = 81p = 254 cm3 (to 3 s.f.) (ii) Total surface area of the solid = =2× 1 1 × [p(7.5)2 – p(6)2] + × 2p(7.5) × 8 2 2 1 × 2p(6) × 8 + 2(1.5 × 8) 2 = 20.25p + 60p + 48p + 24 = 24 + 128.25 p = 427 cm2 (to 3 s.f.) + l = 3 3151 872 = 147 (to 3 s.f.) \ Length of each side = 147 mm 4. Let the length of the cube be l cm . l 3 = 343 l = 3 343 =7 Total surface area of a cube = 6l 2 = 6(7)2 = 294 cm2 5. (i) Its volume Review Exercise 14 1. (a) (i) Volume of the solid = 6 × 3 × 2 + 12 × 2 × 3 = 36 + 72 = 108 cm3 (ii) Total surface area of the solid = 2(2 × 12) + 2(3 × 2) + 2(3 × 3) + 2(2 × 6) + 2(3 × 2) + 3 × 6 + 3 × 12 = 48 + 12 + 18 + 24 + 12 + 18 + 36 = 168 cm2 (b) (i) Volume of the solid =6×8×2–2×2×2 = 96 – 8 = 88 cm3 (ii) Total surface area of the solid = 2(2 × 6) + 2 × 8 + 2(3 × 2) + 3(2 × 2) + 2(6 × 8 – 2 × 2) = 24 + 16 + 12 + 12 + 88 = 152 cm2 1 1 × (20 + 22.5) × 17 × 45.5 2 = 16 436.875 cm3 (ii) Volume of a gold bar with a mass of 200 g 16 436.875 = × 200 250 000 = = 13.1495 cm3 = 13.1495 × 1000 mm3 = 13 149.5 mm3 228 (iii) Volume of the gold bar weighing 200 g = 13 149.5 mm3 9. (i) Volume of the solid = p(6)2(14) + 22 × 18 × 8 = 504p + 3168 = 4750 cm3 (to 3 s.f.) (ii) Total surface area of the solid = 2(22 × 18) + 2p(6)(14) + 2(8 × 22) + 2(8 × 18) = 792 + 168p + 352 + 288 = 1432 + 168p = 1960 cm2 (to 3 s.f.) 10. (i) Volume of the remaining solid = 15 × 24 × 16 – p(4)2(7) = 5760 – 112p = 5410 cm3 (to 3 s.f.) (ii) Area that will be covered in paint = 2p(4)(7) + 2(15 × 24) + 2(16 × 24) + 2(16 × 15) = 56p + 720 + 768 + 480 = 56p + 1968 = 2140 cm2 (to 3 s.f.) 1 × (20 + x) × 15 × 50 = 13 149.5 2 375(20 + x) = 13 149.5 13149.5 375 13149.5 x= – 20 375 49 \ x = 15 750 6. Base radius of the cylindrical barrel = 70 ÷ 2 = 35 cm Volume of water in the cylindrical barrel that is drained away = p(35)2(6) = 7350p cm3 0 .2 l = 200 ml = 200 cm3 Time taken for the water level in the barrel to drop by 6 cm 7350 p = 200 20 + x = = 115 minutes (to 3 s.f.) 7. (i) Volume of water in the pail = p(32)2(25) = 25 600p = 80 400 cm3 (to 3 s.f.) (ii) Volume of water in the pail after 2000 metal cubes are added to it = 25 600p + 2000(2 × 2 × 2) = 25 600p + 16 000 Let the new height of water in the pail be h cm . p(32)2h = 25 600p + 16 000 25600 π + 16000 h= π (32)2 = 30.0 (to 3 s.f.) \ New height = 30 .0 cm 8. (i) Internal radius = 4.2 ÷ 2 = 2.1 cm External radius = 5 ÷ 2 = 2.5 cm Volume of metal used in making the pipe = [p(2.5)2 – p(2.1)2] × 8 .9 = 16.376p = 51.4 cm3 (to 3 s.f.) (ii) 51.45 cm3 = 0.00 005 145 m3 Cost of the pipe = 0.00 005 145 × 2700 × $8 = $1 .11 Challenge Yourself (i) Volume of the solid = 50 × 70 × 30 – 10 × 10 × 70 – 2(10 × 10 × 10) – 2(10 × 10 × 20) = 105 000 – 7000 – 4000 – 2000 = 92 000 cm3 (ii) Total surface area of the solid = 2(30 × 70 – 10 × 10) + 2(50 × 30 – 10 × 10) + 2(50 × 70 – 10 × 10) + 4(10 × 60) + 8(10 × 10) + 8(10 × 20) = 4000 + 2800 + 6800 + 2400 + 800 + 1600 = 18 400 cm2 229 1 Chapter 15 Statistical Data Handling TEACHING NOTES Suggested Approach In primary school, students have learnt statistical diagrams such as pictograms, bar graphs, pie charts and line graphs. Here, students revisit what they have learnt and they are expected to know and appreciate the advantages and disadvantages of each diagram. With such knowledge, students can choose the most appropriate diagram given a certain situation. Teachers may want to give more examples when introducing the various stages of a statistical study and engage with students in evaluating and discussing the issues involved in each stage. Knowledge from past chapters may be required (i.e. percentage). Section 15.1: Introduction to Statistics Teachers should define statistics as the collection, organisation, display and interpretation of data. Teachers may want to briefly cover each stage of a statistical study and give real-life examples for discussion with students, in the later sections. Students are expected to solve problems involving various statistical diagrams. Section 15.2: Pictograms and Bar Graphs Using the example in the textbook, teachers can show how each stage is involved in a statistical study, where the data is displayed in the form of a pictogram and bar graph. Students should appreciate what happens in each stage, cumulating in the conclusion through the interpretation of the data. Through the example, students should also learn to read, interpret and solve problems using information presented in these statistical diagrams. Students should know the characteristics of pictograms and bar graphs and take note of the merits and limitations of pictograms and bar graphs (see Attention on page 370 and Thinking Time on page 371). Section 15.3: Pie Charts Some students may still be unfamiliar with calculating the size of the angle of each sector in a pie chart. As such, teachers may wish to illustrate how this is done. Students need to recall the characteristics of a pie chart (see Attention on page 376). Other than the examples given in the textbooks, teachers may give more examples where a data set is represented by a pie chart, such as students’ views on recent current affairs. Section 15.4: Line Graphs Teachers may want to recap how line graphs are drawn. Students need to know the advantage, disadvantage and the cases line graphs are best used in. (see Attention on page 378). Teachers can discuss some situations where pictograms, bar graphs, pie charts or line graphs are most suitable and assess students’ understanding of statistical diagrams (see Class Discussion: Comparison of Various Statistical Diagrams). Section 15.5: Statistics in Real-World Contexts Teachers can use the examples given in the textbooks and further illustrate in detail how each stage in a statistical study is carried out using real-life examples. Teachers can get the students to discuss and think of more ways to collect data besides conducting questionnaires. Other ways can include telephone interviews, emails, online surveys etc. Teachers may want to assign small-scale projects for students where they conduct their own statistical studies. Such projects allow students to apply what they have learnt about statistical data handling in real-world contexts. Section 15.6: Evaluation of Statistics Teachers should go through the various examples in the textbook and discuss with students the potential issues that can arise at each stage of a statistical study. The importance of not engaging in any unethical behaviors, ensuring objectivity and providing the complete picture without omitting any forms of misrepresentation need to be inculcated into students. 1 230 (b) A line graph should be used to display the data as we need to display the trend of the change in the population of Singapore from the year 2004 to the year 2013 . (c) A pie chart cannot be used to display the data as we will not be able to directly determine the exact number of Secondary 1 students who travel to school by each of the 4 modes of transport . A line graph is inappropriate as it is used to display trends over time . Hence, a pictogram or a bar graph should be used to display the data . Since there are only 4 categories, we may wish to use a pictogram instead of a bar graph as it is more visually appealing and is easier to read . (d) A pie chart should be used to display the data as it is easier to compare the relative proportions of Secondary 1 students who prefer the different drinks . WORKED SOLUTIONS Thinking Time (Page 371) 1. Michael is correct . In a pictogram, each icon represents the same number . Hence, since there are 3 buses and 4 cars, more students travel to school by car than by bus . 2. To avoid a misinterpretation of the data, we can replace each bus and each car in the pictogram with a standard icon . Alternatively, we can draw the buses and the cars to be of the same size . Class Discussion (Comparison of Various Statistical Diagrams) 1. Statistical Diagram Advantages • It is more colourful and appealing . • It is easy to read. • It is difficult to use icons to represent exact values . • If the sizes of the icons are inconsistent, the data may easily be misinterpreted . • If the data has many categories, it is not desirable to use a pictogram to display it as it is quite tedious to draw so many icons . • The data sets with the lowest and the highest frequencies can be easily identified. • It can be used to compare data across many categories . • Two or more sets of data with many categories can be easily compared . • If the frequency axis does not start from 0, the displayed data may be misleading . • The categories can be rearranged to highlight certain results . • The relative size of each data set in proportion to the entire set of data can be easily observed . • It can be used to display data with many categories . • It is visually appealing. • The exact numerical value of each data set cannot be determined directly . • The sum of the angles of all the sectors may not be 360° due to rounding errors in the calculation of the individual angles . • It is not easy to compare across the categories of two or more sets of data . • Intermediate values can be easily obtained . • It can better display trends over time as compared to most of the other graphs . • The trends of two or more sets of data can be easily compared . • Intermediate values may not be meaningful . • If the frequency axis does not start from 0, the displayed data may be misleading . • It is less visually appealing as compared to most of the other graphs . Pictogram Bar graph Pie chart Line graph Disadvantages Performance Task (Page 381) 1. Collection of Data Guiding Questions: • What are the types of food that are sold in your current school canteen? • What other types of food would students like to be sold in the school canteen? How many choices would you like to include in the questionnaire? • What should be the sample size? How do you ensure that the sample chosen is representative of the entire school? • How many choices would you like each student surveyed to select? 2. Organisation of Data Guiding Questions: • How can you consolidate the data collected and present it in a table? • How should you organise the data such that it is easy to understand? 3. Display of Data Guiding Question: • Which statistical diagram, i.e. pictogram, bar graph, pie chart or line graph, is the most suitable to display the data obtained? 4. Interpretation of Data Guiding Questions: • How many more food stalls can your school canteen accommodate? • What is the conclusion of your survey, i.e. based on the statistical diagram drawn, which types of food stalls should your school engage for the school canteen? Teachers may wish to refer students to pages 380 and 381 of the textbook for an example on how they can present their report. 2. (a) A bar graph should be used to display the data as we need to compare data across 12 categories . The categories with the lowest and the highest frequencies can also be easily identified. 231 1 Part IV: Interpretation of Data 1. The conclusion was obtained based on a simple majority, i .e . since more than 50% of the employees were satisfied with working in the company, the survey concluded that the employees were satisfied with the company and that the company was a good place to work in . Class Discussion (Evaluation of Statistics) Part I: Collection of Data 1. Teachers to conduct poll to find out the number of students who know Zidane, Beckenbauer and Cruyff. It is most likely that some students will know who Zidane is, but most (if not all) students will not know who Beckenbauer and Cruyff are. 2. It is stated in the article that the poll was conducted on the UEFA website . As such, the voters who took part in the poll were most likely to belong to the younger generation who are more computer-savvy and hence, the voters were unlikely to be representative of all football fans . 3. As shown in the article, the number of votes for the three footballers were close, with 123 582 votes for Zidane, 122 569 votes for Beckenbauer and 119 332 votes for Cruyff . This is despite the fact that most of the younger generation, who were most likely to have voted in the poll, may not know who Beckenbauer and Cruyff are as they were at the peak of their careers in the 1970s . Hence, if older football fans were to participate in the poll, Zidane would probably not have come in first place. 4. The choice of a sample is important as if the sample chosen for collection of data is not representative of the whole population, the figures that are obtained may be misleading. Hence, a representative sample should be chosen whenever possible . 2. 40% × 300 = 40 × 300 100 = 120 employees It is stated in the article that 40% of the employees, i .e . 120 employees were not satisfied with working in the company. As such, even though a simple majority of the employees was satisfied with working in the company, it cannot be concluded that most of the employees were satisfied. This shows that we should not use simple majorities to arrive at conclusions or make decisions . 3. The amendment of the constitution of a country is a very serious matter where the agreement of a simple majority is insufficient, therefore there is a need for a greater percentage of elected Members of Parliament (MPs) to agree before the constitution can be amended . As a result, the Singapore government requires the agreement of at least a two-third majority before the constitution can be amended . Teachers may wish to take this opportunity to get students to search on the Internet for some laws that have been passed in the Singapore Parliament that resulted in a constitutional amendment. 4. It is important to have a basis or contention in order to decide on an issue, and that in some occasions, it is insufficient to make decisions based on a simple majority . Part II: Organisation of Data 1. Banks and insurance firms, timeshare companies and motor vehicle companies received the most number of complaints . 2. The article states that banks and insurance firms, which were grouped together, received the most number of complaints . If banks and insurance firms were not grouped together, it is possible that timeshare companies received the most number of companies . For example, if the 1416 complaints were split equally between banks and insurance firms, they would have received 708 complaints each, then the number of complaints received by timeshare companies, i .e . 1238 complaints, would have been the greatest . 3. This shows that when organising data, it is important to consider whether to group separate entities as doing so might mislead consumers and result in inaccurate conclusions . Teachers may wish to ask students whether a simple majority, i.e. more than 50% of the votes, is necessary to decide on an issue. For example, in the 2011 Singapore Presidential Elections, Dr Tony Tan was elected President of the Republic of Singapore with 35.2% of the total valid votes cast. Part V: Ethical Issues It is unethical to use statistics to mislead others as it is essentially a form of misrepresentation and people may arrive at the wrong conclusions or make the wrong decisions . The rationale for teaching students to be aware of how statistics can be used to mislead others is so that the students will be more discerning when they encounter statistics and will not be misled by others. Teachers should also impress upon students that they should not use statistics to mislead others because it is unethical to do so. Part III: Display of Data 1. Although the height of the bar for Company E appears to be twice that of the bar for Company C, Company E’s claim is not valid as the bars do not start from 0 . By reading off the bar graph, Company E sold 160 light bulbs in a week, which is not twice as many as the 130 light bulbs sold by Company C in a week . 2. For bar graphs, if the vertical axis does not start from 0, the height of each bar will not be proportional to its corresponding frequency, i .e . number of light bulbs sold by each company in a week . Such display of statistical data may mislead consumers . 1 Practise Now (Page 371) 1. (a) (i) Profit earned by the company in 2010 = 5.5 × $1 000 000 = $5 500 000 (ii) Profit earned by the company in 2012 = 7 × $1 000 000 = $7 000 000 (b) The company earned the least profit in 2009. The profit decreased by 1 .5 × $1 000 000 = $1 500 000 in 2009 as compared to 2008 . 232 2. (a) Practise Now (Page 376) Number of television sets Sales of Television Sets in 5 Shops 120 100 Farhan’s total expenditure on the holiday = $1000 + $1200 + $400 + $1200 + $200 = $4000 November December 80 60 Item Angle of sector 40 20 0 Shop 1 Shop 2 Shop 3 Shop 4 Shop 5 Shop 6 Shop 7 Food $1000 × 360° = 90° $4000 Shopping $1200 × 360° = 108° $4000 Hotel $400 × 360° = 36° $4000 Shop (b) (i) Total number of television sets sold in the seven shops in November = 60 + 30 + 50 + 70 + 40 + 64 + 70 = 384 (ii) Total number of television sets sold in the seven shops in December = 90 + 48 + 80 + 112 + 80 + 88 + 96 = 594 384 (c) Required percentage = × 100% 384 + 594 384 = × 100% 978 43 = 39 % 163 70 + 96 (d) (i) Required percentage = × 100% 978 166 = × 100% 978 476 = 16 % 489 (ii) No, I do not agree with the manager . Since Shop 2 sold the least number of televsion sets in November and December, it should be closed down . (e) The company performed better in terms of sales in December . This could be due to the fact that Christmas is in December when people buy television sets as gifts for others . Air Ticket $1200 × 360° = 108° $4000 $200 × 360° = 18° $4000 Others Others 18° Food Air Ticket 108° 108° 36° Shopping Hotel Practise Now 1 1. (i) 4x° + 2x° + 237 .6° = 360° (/s at a point) 4x° + 2x° = 360° – 237 .6° 6x° = 122 .4° x° = 20 .4° \ x = 20 .4 4(20.4° ) (ii) Required percentage = × 100% 360° 81.6° × 100% 360° 2 = 22 % 3 360° (iii) Amount of fruit punch in the jar = × 759 ml 237.6° = 1150 ml = 233 1 2. (i) The least popular colour is black . (ii) Total number of cars sold = 2000 + 3500 + 5000 + 6000 + 1500 = 18 000 Angle of sector that represents number of blue cars sold 2000 = × 360° 18 000 = 40° Angle of sector that represents number of grey cars sold 3500 = × 360° 18 000 = 70° Angle of sector that represents number of white cars sold 5000 = × 360° 18 000 Exercise 15A 1. (i) The greatest number of buses registered was in 2012 . Number of buses registered in 2012 ≈ 6 .5 × 40 000 = 260 000 (ii) Total number of buses registered from 2008 to 2012 ≈ 24 × 40 000 = 960 000 (iii) Total amount the Registry of Vehicles collected in 2010 ≈ 4 .5 × 40 000 × $1000 = $180 000 000 (iv) Percentage increase in number of buses registered from 2011 to 2012 1 × 100% 5.5 2 = 18 % 11 = = 100° Angle of sector that represents number of red cars sold 6000 = × 360° 18 000 = 120° Angle of sector that represents number of black cars sold 1500 = × 360° 18 000 = 30° (iii) No, I do not agree with her . This is because the number of cars indicated on the y-axis is in thousands, thus 3500 grey cars and 1500 black cars are sold . 2. (i) Students who Play Volleyball, Basketball or Tennis Volleyball Basketball Tennis Each circle represents 10 students . (ii) Required ratio = 4 : 5 5 × 100% 6 1 = 83 % 3 (iii) Required percentage = Practise Now 2 3. (i) The number of fatal road casualties was the highest in 2008 . Year 2005 2006 2007 2008 2009 Number of fatal road casualties 173 190 214 221 183 300 Number of copies (in thousands) (ii) (iii) Percentage decrease in number of fatal road casualties from 2008 to 2009 221 – 183 = × 100% 221 = Newspaper Distribution to Households 350 38 × 100% 221 43 % 221 (iv) There are traffic cameras installed along more roads. = 17 250 200 150 100 50 0 1 234 2008 2009 2010 Year 2011 2012 (iv) The percentage of successful candidates increases over the six years as they practise past-year papers and learn from their mistakes . 6. (i) Total number of workers employed in the housing estate =4×1+6×2+5×3+3×4+2×5 = 4 + 12 + 15 + 12 + 10 = 53 (ii) Total number of shops in the housing estate =4+6+5+3+2 = 20 Number of shops hiring 3 or more workers = 5 + 3 + 2 = 10 10 \ Required percentage = × 100% 20 = 50% (iii) Some shops have more customers as they are located at places with higher human traffic, thus they need to employ more workers . 4. (a) Class Class 1A Class 1B Class 1C Class 1D Class 1E Number of students who score a distinction in Mathematics 9 11 16 12 20 Number of students who score a distinction in Science 8 13 12 16 15 Number of students Students who Score a Distinction in Mathematics or Science 25 Mathematics Science 20 15 10 5 0 1A 1B 1C Class 1D 1E Exercise 15B (b) (i) Total number of students in the 5 classes who score a distinction in Mathematics = 9 + 11 + 16 + 12 + 20 = 68 (ii) Total number of students in the 5 classes who score a distinction in Science = 8 + 13 + 12 + 16 + 15 = 64 1. Total number of students surveyed = 768 + 256 + 64 + 192 = 1280 12 × 100% 68 11 = 17 % 17 (d) Percentage of students in Class 1D who score a distinction in Science (c) Required percentage = 16 × 100% 40 = 40% (e) No, Jun Wei is not correct to say that there are 35 students in Class 1E . There may be students in the class who do not score distinctions in both Mathematics and Science . There may also be students in the class who score distinctions in both Mathematics and Science . 5. (i) Number of candidates who sat for the examination in 2009 = 950 (ii) Number of candidates who failed the examination in 2012 = 500 (iii) Total number of candidates who failed the examination in the six years = 400 + 350 + 350 + 400 + 450 + 500 = 2450 Mode of transport Angle of sector Bus 768 × 360° = 216° 1280 Car 256 × 360° = 72° 1280 Bicycle 64 × 360° = 18° 1280 Foot 192 × 360° = 54° 1280 = Foot Bicycle 18° 54° 72° Car 216° Bus 2. (i) Angle of sector that represents number of students who prefer yam = 90° (ii) Angle of sector that represents number of students who prefer vanilla = 360° – 120° – 90° – 50° (/s at a point) = 100° 500 × 100% 2450 20 = 20 % 49 \ Required percentage = 235 1 (ii) Percentage increase in mass of the baby from the 4th to 6th month 5 – 4.2 = × 100% 4.2 0.8 = × 100% 4.2 1 = 19 % 21 6. (a) Total angle of sectors that represent number of female students and teachers in the school = 360° – 240° (/s at a point) = 120° Angle of sector that represents number of teachers in the school 1 = × 120° 6 = 20° (b) (i) Number of female students in the school = 5 × 45 = 225 100° × 100% 360° 7 = 27 % 9 (iii) Required percentage = 360° ×5 50° = 36 (iv) Total number of students in the class = 180° × 100% 360° = 50% 72° (ii) Required percentage = × 100% 360° = 20% 1 17 2 × 360° (iii) x° = 100 = 63° \ x = 63 4. (i) Total number of cars in the survey = 20 + 25 + 20 + 30 + 25 = 120 (ii) Total number of people in all the cars = 20 × 1 + 25 × 2 + 20 × 3 + 30 × 4 + 25 × 5 = 20 + 50 + 60 + 120 + 125 = 375 (iii) Number of cars with 4 or more people = 30 + 25 = 55 3. (i) Required percentage = 240° × 45 20° = 540 (ii) Number of male students in the school = (c) Total school population = 45 + 225 + 540 = 810 2 × 45 3 = 30 Number of females in the school = 225 + 30 = 255 255 \ Required percentage = × 100% 810 13 = 31 % 27 5 7. × 360° = 120° 1+ x + 5 5 120° = 6+x 360° 5 1 = 6+x 3 15 = 6 + x \x =9 8. (i) The town had the greatest increase in the number of people from 2011 to 2012 . (ii) Number of female teachers in the school = 55 × 100% 120 5 = 45 % 6 (iv) Angle of sector that represents number of cars with 1 people 20 = × 360° 120 = 60° Angle of sector that represents number of cars with 2 people 25 = × 360° 120 = 75° Angle of sector that represents number of cars with 3 people 20 = × 360° 120 = 60° Angle of sector that represents number of cars with 4 people 30 = × 360° 120 = 90° Angle of sector that represents number of cars with 5 people 25 = × 360° 120 = 75° \ Required percentage = 5. (i) Month Mass (kg) 0 1 2 3 4 5 6 3 .2 3 .4 3 .8 4 4 .2 4 .4 5 Year 2002 2003 2004 2005 2006 2007 2008 2009 2010 2011 2012 Number of people (in thousands) 6 9 9 .5 12 14 15 16 18 19 25 (iii) Percentage increase in number of people in the town from 2009 to 2012 25 000 – 16 000 = × 100% 16 000 9000 = × 100% 16 000 1 % 4 (iv) There are more new immigrants in the town . = 56 1 8 236 (ii) Number of students who read more than 4 books = 5 + 1 =6 Total number of students in the class = 2 + 5 + 9 + 8 + 6 + 5 + 1 = 36 6 \ Required percentage = × 100% 36 2 = 16 % 3 (iii) Number of students who read fewer than 3 books = 2 + 5 + 9 = 16 Angle of sector that represents number of students who read fewer than 3 books 16 = × 360° 36 = 160° 3. Percentage of students who are enrolled in the Arts course = 100% – 25% – 30% – 15% = 30% 9. (i) Temperature (°C) Temperature of Patient 40 39 38 37 36 35 1500 1800 2100 0000 0300 0600 0900 Time (hours) 10. 11. 12. 13. (ii) Temperature of the patient at 1700 hours ≈ 39 °C Temperature of the patient at 0100 hours ≈ 38 °C The majority of the respondents in Kate’s survey are most likely females while those in Khairul’s survey are most likely males . Kate and Khairul may have conducted each of their surveys at a different location, e .g . Kate may have conducted her survey at Orchard Road while Khairul may have conducted his survey at a housing estate . No, I do not agree with Nora . The temperatures in both countries range from 24 °C to 35 °C . The temperatures in Country X seem to change more drastically than those in Country Y because the vertical axis of the line graph which shows the temperatures of Country X starts from 23 °C instead of 0 °C . (i) Based on the 3-dimensional pie chart, Raj spends the most on luxury goods . (ii) Based on the 2-dimensional pie chart, Raj spends the most on rent and luxury goods . (iii) In a 3-dimensional pie chart, the sizes of the sectors will look distorted . The sectors towards the back of the pie chart will appear smaller than those towards the front . No, I do not agree with Amirah . As there are more cars than motorcycles in Singapore, it is not surprising that there are more accidents involving cars than motorcycles . Moreover, there may be a higher chance of accidents involving motorcycles occurring due to the nature of the vehicle . Type of course Angle of sector Science 25 × 360° = 90° 100 Engineering 30 × 360° = 108° 100 Business 15 × 360° = 54° 100 Arts 30 × 360° = 108° 100 Science Arts 108° 54° Review Exercise 15 Business 108° Engineering 1. (i) Required ratio = 6 : 3 =2:1 7 × 100% 4 = 175% 2. (i) Total number of books read by the students in the class in a month =2×0+5×1+9×2+8×3+6×4+5×5+1×6 = 0 + 5 + 18 + 24 + 24 + 25 + 6 = 102 (ii) Required percentage = 4. (i) Total angle of sectors that represent amount Devi spends on clothes and food = 360° – 36° – 90° – 90° (/s at a point) = 144° Angle of sector that represents amount Devi spends on food 1 × 144° 4 = 36° = 36° × 100% 90° = 40% \ Required percentage = 237 1 (ii) Devi’s monthly income = 360° × $400 36° = $4000 Devi’s annual income = 12 × $4000 = $48 000 5. (i) Year Number of laptops 2008 2009 2010 2011 2012 70 30 44 90 26 (ii) Percentage decrease in number of laptops purchased by the company from 2008 to 2009 70 – 30 × 100% 70 40 = × 100% 70 1 = 57 % 7 (iii) The company might have had a tighter budget in 2009 . = Challenge Yourself The better way to display the data using a bar graph is as follows: Breathing rate 150 50 40 100 30 20 50 1 Swimming Jogging Running Walking 10 Breathing rate (per minute) Pulse rate (per minute) Pulse rate 238 (ii) Volume of wood used = 72 × 54 × 48 – 158 355 = 186 624 – 158 355 = 28 269 cm3 (iii) Mass of box = 0 .9 × 28 269 = 25 442 .1 g Revision Exercise D1 1. Let the radius of the quadrant be x cm . 1 × 2px + 2x = 71 .4 4 1 px + 2x = 71 .4 2 150° × 100% 360° 2 = 41 % 3 72° (ii) Required percentage = × 100% 360° = 20% 15 (b) x° = × 360° 100 = 54° \ x = 54 6. (a) (i) Required percentage =  1 x  π + 2  = 71 .4  2 71.4 \x= 1 π+2 2 = 20 .00 (to 4 s .f .) 1 × p(20 .00)2 4 = 314 cm2 (to 3 s .f .) 1  12  Perimeter of shaded region = × 2p(12) + 2p   2  2 = 12p + 2p(6) = 12p + 12p = 24p = 75 .4 cm (to 3 s .f .) Area of shaded region = area of big semicircle 1 = × p(12)2 2 = 72p = 226 cm2 (to 3 s .f .) Area of trapezium ABEF = area of rectangle ACEF – area of nBCE 1 = area of rectangle ACEF – × area of nBDE 2 1 = 12 × 8 – × 24 2 = 96 – 12 = 84 cm2 Volume of solid = base area × height = [12 × 12 + (18 – 12) × 3] × 6 = (144 + 6 × 3) × 6 = (144 + 18) × 6 = 162 × 6 = 972 cm3 Total surface area of solid = perimeter of base × height + 2 × base area = [12 + 12 + 18 + 3 + 6 + (12 – 3)] × 6 + 2 × 162 = (12 + 12 + 18 + 3 + 6 + 9) × 6 + 324 = 60 × 6 + 324 = 360 + 324 = 684 cm2 (i) Capacity of box = (72 – 1 .5 – 1 .5) × (54 – 1 .5 – 1 .5) × (48 – 1 .5 – 1 .5) = 69 × 51 × 45 = 158 355 cm3 = 158 .355 l Area of quadrant = 2. 3. 4. 5. 239 1 Revision Exercise D2 1. Area of photograph = 40 × 25 = 1000 cm2 Area of margin = (40 + 4 + 4) × (25 + 4 + 4) – 1000 = 48 × 33 – 1000 = 1584 – 1000 = 584 cm2 2. (i) Perimeter of figure 1 = 24 + 15 + (24 – 10) + × 2p(10) + (15 – 10) 4 = 24 + 15 + 14 + 5p + 5 = 58 + 5p = 73 .7 cm (to 3 s .f .) (ii) Area of figure = area of rectangle – area of quadrant 1 = 24 × 15 – × p(10)2 4 = 360 – 25p = 281 cm2 (to 3 s .f .) 3. Area of parallelogram = PQ × ST = QR × SU 10 × ST = 7 × 9 10 × ST = 63 ST = 6 .3 Length of ST = 6 .3 cm 4. Volume of prism = base area × height 1 =  × (8 + 3 + 8 + 3) × 4  × 20  2  1 =  × 22 × 4  × 20  2 = 44 × 20 = 880 cm3 Total surface area of prism = perimeter of base × height + 2 × base area = (8 + 5 + 3 + 8 + 3 + 5) × 20 + 2 × 44 = 32 × 20 + 88 = 640 + 88 = 728 cm2 5. Volume of cylinder = p(62 – 52)(2 .4 × 100) = p(36 – 25)(240) = p(11)(240) = 2640p cm3 Mass of cylinder = 7 .6 × 2640p = 20 064p = 63 000 g (to 3 s .f .) 6. (i) The attendance was the greatest in the 4th week . (ii) The Drama Club stopped its weekly meeting in the 9th week . 45 – 15 (iii) Required percentage = × 100% 45 30 = × 100% 45 2 = 66 % 3 (iv) Most of the Drama Club members were busy preparing for the school examination . 1 240 is a soft metal, diving cylinders made of aluminium are more prone to physical damage. On the other hand, as steel is a tough metal, diving cylinders made of steel are more durable and are less prone to physical damage. However, steel comprises of iron, which is more susceptible to corrosion, thus steel diving cylinders are more difficult to maintain. 3. (a) Percentage increase in the annual mean surface temperature in Singapore from 1948 to 2011 Problems in Real-World Contexts 1. (i) A suitable unit for the measurements in the floor plan is the millimetre (mm) . (ii) Length of AB = (800 + 4800 + 3200 + 1600 + 1400 + 2500) – (4400 + 1775 + 400) = 14 300 – 6575 = 7725 mm $500 000 (iii) Price per square metre = 110 = $4545 (to the nearest dollar) (iv) 1 foot ≈ 0 .3048 m 1 square foot = 0.092 90 m2 (to 4 s .f .) \ $1000 psf = $1000 ÷ 0.092 90 = $10 760 per square metre (to 4 s.f.) The condominium unit is $10 760 ÷ $4545 = 2.37 (to 3 s.f.) times as expensive as the flat that Mr Lee is interested to purchase. 2. (i) Volume of water the diving cylinder can contain = p(6 .75)2(85) = 3872 .8125p cm3 = 3 .872 812 5p l = 12 .2 l (to 3 s .f .) (ii) Volume of gas that the diving cylinder can hold volume of cylinder × pressure in cylinder = atmospheric pressure 3.872 812 5 π × 200 = 1.01 = 2409 l (to 4 s .f .) = 2410 l (to 3 s .f .) (iii) Duration the diver can stay underwater volume of gas consumed = breathing rate × ambient pressure 27.6 – 26.8 × 100% 26.8 0.8 = × 100% 26.8 = 2.99% (to 2 d.p.) (b) (i) Majority of Singapore’s emissions in 2020 is expected to come from the industry sector. Amount of emissions contributed by the industry sector = 60 .3% × 77.2 MT = 46.5516 MT (ii) Reasons for the likely increase in emissions from 2005 to 2020: • Due to rapid urbanisation, there will be an increase in the demand for expansion of petrochemicals and manufactured products from Asian countries such as Singapore. • The growth of the population and the economy results in an increased use of transportation. In addition, the expansion of port activities causes an increase in the emissions from domestic maritime transport. • There will be an increase in the demand for commercial spaces as well as a greater intensity in the usage of space. • Due to increases in population and household income, there will be an increase in the demand for electrical appliances. = 2409 15   20 ×  1.01 +  10   2409 = 20 × (1.01 + 1.5) = Teachers may wish to note that the list is not exhaustive. (c) Measures that have been put in place by the Singapore government to reduce emissions and to mitigate the effects of climate change: • The Energy Conservation Act, which was implemented in 2013, mandates companies in the industry and transport sectors to adopt energy efficient technologies and processes. • The implementation of a Carbon Emissions-based Vehicle (CEV) Scheme in 2013 seeks to encourage consumers to buy low carbon emissions cars. Moreover, in the next few years, the rail network will be increased to about 280 km. This encourages people to use public transport, which is more carbon efficient. • All new buildings and existing ones undergoing extensive renovation are required to adhere to the Green Mark standards, i.e. the buildings should be sustainable and environmentally-friendly. 2409 20 × 2.51 = 48 .0 minutes (to 3 s .f .) = Teachers may wish to ask students to state an assumption that they have made in their calculations, e.g. the volume of gas consumed by the diver as he descends to a depth of 15 m is negligible. (iv) Since the amount of time the diver can stay underwater if he uses the diving cylinder is 48.0 minutes, which is less than one hour, the diving cylinder is not suitable for the diver. Diving cylinders are usually made of aluminium or steel. Teachers may wish to ask students to find out an advantage and a disadvantage of a diving cylinder made up of aluminium and of steel. For example, aluminium cylinders are easier to maintain as aluminium is more resistant to corrosion. However, as aluminium 241 1 • • The Minimum Energy Performance Standards Scheme which was implemented in 2011 restricts the sale of energy inefficient appliances. The Singapore government is moving away from the disposal of waste in landfills to the incineration of waste. B. Solving • How do you solve the problem mathematically? • What are the different ways that you can use to present the results from your calculations? Which way should you use to present the results to your classmates so that they will be able to make an informed decision? C. Interpreting Students should be able to interpret the mathematical solution in the context of the real world, i.e. they should be able to advise their classmates to choose one of the three price plans based on the mathematical solution that they have obtained. D. Reflecting • After finding the most value-for-money plan for their classmates, students should check whether there are any other issues that their classmates may have to take into consideration before subscribing to a plan. • Students should review the chosen plan to determine whether it is the ideal plan for their classmates. • Students should also review the method that they have used and consider whether there are other methods that can be used to solve the problem. Teachers may wish to note that the list is not exhaustive. 4. The Mathematical Modelling Process consists of the following steps: A. Formulating • Students are required to understand the information given in the question and a discussion may be carried out to help them comprehend the problem. Some guiding questions are as follows: (i) Based on the information in the table given, what does your classmates’ choice of plan depend on? Should your classmates choose a plan based on the price alone, i.e. would you advise your classmates to choose Plan A because the monthly subscription fee is the lowest? (ii) Other than the monthly subscription fee, what are some other factors that may affect the amount a user has to pay each month? How can you simplify the problem so that it is easier to carry out a comparison? • Some examples of assumptions that can be made to simplify the problem are as follows: (i) The subscription contract spans a period of two years and within the two years, the monthly subscription fees, as well as the terms and conditions of the price plans, do not change . (ii) The charges for overseas incoming calls and overseas outgoing calls are the same for all three price plans. (iii) Other mobile services, e.g. caller ID, multimedia messaging service (MMS) etc., provided are set at the same price. (iv) The amount of data used each month is the same . (v) The average download speeds are the same. • For higher-ability students, teachers may wish to remove one assumption from the assumptions made, and get them to come up with another model for the problem. 5. For this problem, teachers may wish to first set the budget and the fundraising target for the students. The Mathematical Modelling Process consists of the following steps: A. Formulating • Students are required to understand the information given in the question and a discussion may be carried out to help them comprehend the problem. Some guiding questions are as follows: (i) How many cookies do you estimate your class will be able to sell? How many cookies should your class make? Does the number of cookies your class makes have to be a multiple of 48? How would this affect the ingredients required? (ii) Based on the number of cookies your class decides to make, what is the total amount of money required to purchase the ingredients? Teachers may wish to ask students to state other assumptions that may be made. In this problem, we shall consider data usage to be the only variable. Teachers may wish to ask students what is meant by ‘data usage is the only variable’ and get them to give some other examples of variables. They may also wish to guide students to classify the information into three categories, i.e. when data usage does not exceed 12 GB, when data usage exceeds 12 GB but does not reach the cap of $30 and when data usage reaches the cap of $30. 1 Teachers may wish to ask students whether they are able to buy the exact quantities of ingredients required to bake the cookies, e.g. 20 teaspoons of baking soda, and how this may affect the total cost. (iii) What is the budget for the fundraising event? Are there other costs that need to be taken into consideration? Does the total cost lie within the budget? If not, what can be done to ensure that the total cost lies within the budget? 242 • (iv) What is the fundraising target that needs to be met? Besides the fundraising target, what are some other factors which need to be considered before pricing the cookies? How should you price your cookies? Some examples of assumptions that can be made to simplify the problem are as follows: (i) Miscellaneous costs, e.g. transport costs for travelling to buy the ingredients and cost of electricity used to bake the cookies, are not taken into consideration when calculating the total cost . (ii) There is no wastage of ingredients . Teachers may wish to ask students to state other assumptions that may be made. B. Solving • How do you solve the problem mathematically? • How should you present the results from your calculations? C. Interpreting Students should be able to interpret the mathematical solution in the context of the real world, i.e. they should be able to advise the class on the number of cookies that need to be made and the price at which they should sell the cookies in order to maximise their profit based on the mathematical solution that they have obtained . D. Reflecting • After finding the number of cookies that need to be made and the price at which they should sell the cookies in order to maximise their profit, students should check whether there are any other factors that may affect the profit made, e.g. wastage at the end of the day. • Students should also review the method that they have used and consider whether there are other methods that can be used to solve the problem. For higher-ability students, teachers may wish to remove one assumption from the assumptions made, and get them to come up with another model for the problem. 243 1 NOTES CONTENTS 1 244